484
GCE Oxford Cambridge and RSA Examinations Advanced Subsidiary GCE AS H106 Advanced GCE A2 H506 History A Mark Schemes for the Units January 2009 H106/H506/MS/R/09J

History A - The Bicester Schoolthebicesterschool.org.uk/wp-content/uploads/2015/09/Mark... · 2015-09-28 · History A Mark Schemes for the Units January 2009 H106/H506/MS/R/09J

  • Upload
    others

  • View
    3

  • Download
    0

Embed Size (px)

Citation preview

Page 1: History A - The Bicester Schoolthebicesterschool.org.uk/wp-content/uploads/2015/09/Mark... · 2015-09-28 · History A Mark Schemes for the Units January 2009 H106/H506/MS/R/09J

GCE

Oxford Cambridge and RSA Examinations

Advanced Subsidiary GCE AS H106

Advanced GCE A2 H506

History A

Mark Schemes for the Units January 2009

H106/H506/MS/R/09J

Page 2: History A - The Bicester Schoolthebicesterschool.org.uk/wp-content/uploads/2015/09/Mark... · 2015-09-28 · History A Mark Schemes for the Units January 2009 H106/H506/MS/R/09J

OCR (Oxford Cambridge and RSA) is a leading UK awarding body, providing a wide range of qualifications to meet the needs of pupils of all ages and abilities. OCR qualifications include AS/A Levels, GCSEs, OCR Nationals, Key Skills, Entry Level qualifications, NVQs and vocational qualifications in areas such as IT, business, languages, teaching/training, administration and secretarial skills. It is also responsible for developing new syllabuses to meet national requirements and the needs of students and teachers. OCR is a not-for-profit organisation; any surplus made is invested back into the establishment to help towards the development of qualifications and support which keep pace with the changing needs of today’s society. This mark scheme is published as an aid to teachers and students, to indicate the requirements of the examination. It shows the basis on which marks were awarded by Examiners. It does not indicate the details of the discussions which took place at an Examiners’ meeting before marking commenced. All Examiners are instructed that alternative correct answers and unexpected approaches in candidates’ scripts must be given marks that fairly reflect the relevant knowledge and skills demonstrated. Mark schemes should be read in conjunction with the published question papers and the Report on the Examination. OCR will not enter into any discussion or correspondence in connection with this mark scheme. © OCR 2009 Any enquiries about publications should be addressed to: OCR Publications PO Box 5050 Annesley NOTTINGHAM NG15 0DL Telephone: 0870 770 6622 Facsimile: 01223 552610 E-mail: [email protected]

Page 3: History A - The Bicester Schoolthebicesterschool.org.uk/wp-content/uploads/2015/09/Mark... · 2015-09-28 · History A Mark Schemes for the Units January 2009 H106/H506/MS/R/09J

CONTENTS

Advanced GCE History (H506)

Advanced Subsidiary GCE History (H106)

MARK SCHEMES FOR THE UNITS

Unit/Content Page

F961-01 - British History Period studies 1

F961-02 - British History Period Studies Option B: Modern 1783-1994 19

F962-01 - European and World History Period Studies 37

F962-02 - European and World History Period Studies Option B: Modern 1795-2003 46

F963-01 - British History Enquiries 60

F963-02 - Option B: British History Enquiries 1815-1945 66

F964-01 - European and World History Enquiries 78

F964-02 - European and World History Period Studies Option B: Modern 1774-1975 83

Grade Thresholds 96

Page 4: History A - The Bicester Schoolthebicesterschool.org.uk/wp-content/uploads/2015/09/Mark... · 2015-09-28 · History A Mark Schemes for the Units January 2009 H106/H506/MS/R/09J

F961-01 Mark Scheme January 2009

F961-01 - British History Period studies

From Anglo-Saxon England to Norman England 1035-1087 1. How successful was Edward the Confessor as king of England? [50] Focus: An assessment of the strengths of a king. No set answer is looked for but candidates will need to address the question. Some candidates might argue that Edward had a difficult task; he had spent much time on the continent and lacked the immediate authority to rule England effectively, however he was able to secure himself on the throne. The Earls enjoyed a great deal of power and some might examine the power of the Godwin family to show how far their authority reflected a weakness in Edward. However, this could be balanced against Edward’s ability to gain sufficient support to force the Godwin’s into temporary exile. It might therefore be concluded that he managed the Earls as well as might be expected. However, Earl Godwin did exert a powerful influence from 1052 after his return from exile and Harold became very influential. There might be some focus on his failure to provide an heir, which plunged the country into the succession crisis, although how far it was his fault might also be considered. Administration continued and taxes were collected as two tests of effective government. His capabilities as a military leader might also be considered as it was an important mark of kingship and was a disadvantage when dealing with troublesome and ambitious earls. Some candidates might mention his piety and artistic interests which were admired, but for their ideals rather than for their practical relevance to kingship. Many may conclude that Edward lacked the strong qualities needed for an effective king as he was often rash and unwise in his judgements. There might also be mention of the problems that followed from his continuing patronage of Normans. Alternative explanations are possible and examiners must be open to alternative approaches. If in doubt, consult your Team Leader.

1

Page 5: History A - The Bicester Schoolthebicesterschool.org.uk/wp-content/uploads/2015/09/Mark... · 2015-09-28 · History A Mark Schemes for the Units January 2009 H106/H506/MS/R/09J

F961-01 Mark Scheme January 2009

2. To what extent was William of Normandy’s success in 1066 the result of Harold’s mistakes? [50]

Focus: An assessment of the reasons for the success of a king. No set answer is looked for but candidates will need to address the question. Candidates can offer contradictory explanations, for example that the major reason was the strength of William’s forces or his organisational abilities. However, they must include a sound paragraph on the stated factor if they are to access the highest levels on AO1b. Examiners should also not underestimate the value of answers that are organised chronologically, excellent answers that appreciate the sequence of events and their significance should not be dismissed as low-level narrative. In arguing that it was Harold’s mistakes candidates may focus on his actions after Stamford Bridge and before Hastings, although there may be some mention of mistakes made during the battle. Candidates may argue that Harold’s mistake in rushing back from Stamford Bridge and facing William without a full force and whilst still tired was a major factor in his defeat. Some may argue that Harold was simply unfortunate in that he faced two invasions in quick succession and that the wind changed direction at the moment he was in the north. They may argue that he had to deal with Harald Hardrada as he was a major threat and needed dealing with quickly and decisively. It is easy, with hindsight, to be persuaded that William would launch the more serious challenge, but Harold had to exert his authority in the north as it was a difficult area to govern. Some candidates may focus on the strengths of William and use the Bayeux Tapestry as evidence of the naval and military preparations that he made. He welded together a diverse group into a formidable fighting force, he was also an excellent commander and some might draw attention to the tactics at Hastings, particularly the feigned retreat. Alternative explanations are possible and examiners must be open to alternative approaches. If in doubt, consult your Team Leader.

2

Page 6: History A - The Bicester Schoolthebicesterschool.org.uk/wp-content/uploads/2015/09/Mark... · 2015-09-28 · History A Mark Schemes for the Units January 2009 H106/H506/MS/R/09J

F961-01 Mark Scheme January 2009

3. Assess the reasons why William I was able to defeat opposition to his rule. [50] Focus: An assessment of the failure of opposition to a king. No set answer is looked for but candidates will need to address the question. The death of Harold at Hastings deprived the Anglo Saxons of their major leader and it frequently meant that opposition was both divided and weak. The defeat at Hastings had also broken the military strength of the Anglo Saxon fyrd and earls. Important Anglo Saxon nobles from Merica and Northumbria had promised allegiance, which made his task easier. Candidates may consider the policies that William followed such as castle building and how it was used or the ‘Harrying of the North’, which would have given a clear warning to those who might oppose his rule. Many of the risings were localised, for example Kent, Northumbria, the south west and the Welsh Marches, which made their suppression much easier. Many of the risings also arose from local grievances, rather than dissatisfaction with William’s rule. The rebellions often lacked leadership. In dealing with the problem in the north in 1069 with intervention from Scotland and Scandinavia William enjoyed clear military advantage, which made his job easier as his forces were superior to anything the rebels could gather. William was able to move swiftly to put down trouble before it had a chance to develop, he acted decisively and used harsh methods which may have deterred others. The swift manner in which he took England and the armed forces on which he could rely negated the opposition. Alternative explanations are possible and examiners must be open to alternative approaches. If in doubt, consult your Team Leader.

3

Page 7: History A - The Bicester Schoolthebicesterschool.org.uk/wp-content/uploads/2015/09/Mark... · 2015-09-28 · History A Mark Schemes for the Units January 2009 H106/H506/MS/R/09J

F961-01 Mark Scheme January 2009

Lancastrians, Yorkists and Tudors 1450-1509 4. How successful was Edward IV in dealing with the nobility? [50]

Focus: An assessment of a king’s ability to deal with the nobility. No set answer is looked for but candidates will need to address the question. Some candidates may argue that Edward was not successful in his dealings with the nobility and did much to create the problems. They may point to his marriage to Elizabeth Woodville, which did much to alienate many nobles as the family was promoted to positions of importance through advantageous marriages. Their promotion was a key factor in the alienation of Warwick and his emergence as the leader of opposition as the marriage prevented a French alliance that he preferred. Some candidates may also point to the fact that the Lancastrian nobility could still mount a challenge and that some hostile nobles were supported by France. The outbreak of Civil War in 1470 may be used by candidates to show that Edward had not been successful in dealing with them. Although Edward appeared more secure in his second reign as he did not face rebellion, he did create ‘super nobles’ who, it could be argued had too much power and he was only able to control them because of his personality. However, some may argue that he gave too much leeway to potentially troublesome nobles such as Richard. He was able to curb the power of the nobility, but he did not destroy their power. It might be argued that the ability of the king to manage the nobility depended too much on his personality and that this is shown after his death. Alternative explanations are possible and examiners must be open to alternative approaches. If in doubt, consult your Team Leader.

4

Page 8: History A - The Bicester Schoolthebicesterschool.org.uk/wp-content/uploads/2015/09/Mark... · 2015-09-28 · History A Mark Schemes for the Units January 2009 H106/H506/MS/R/09J

F961-01 Mark Scheme January 2009

5. Assess the reasons why Richard III’s reign was so short. [50]

Focus: An assessment of the failings of a particular king. No set answer is looked for but candidates will need to address the question. It is likely that many answers will focus on Richard’s defeat at Bosworth as ultimately that was the reason why he lost the throne. However, without previous events and developments he would not have alienated many who either supported Henry Tudor or waited to see the outcome. The Lancastrians always opposed him and Henry Tudor was the most dangerous challenge. Candidates might go on to consider some of Henry’s strengths as they helped to remove Richard; these might include being a clever opponent, avoiding a direct confrontation with Richard until he had a large enough force. Candidates may consider Richard’s character arguing that he was untrustworthy, ambitious and even murderous. Many answers are likely to consider the illegality of his seizure of the throne and argue that this was the start of his troubles as it raised doubts about his character and alienated some. It will be relevant to discuss the probable murder of the princes in the Tower, but this must be linked to loss of support. Candidates might consider his relations with the nobility, including the execution of Buckingham. The distrust that surrounded Richard was a key factor in explaining why he was unable to create a wide section of support. His betrayal by Stanley at Bosworth might be interpreted either as justified or as the actions of an unscrupulous noble who wanted to come out on the winning side. Some answers might also argue that Richard’s promotion of northern nobles alienated a large number of southern nobles, which would be crucial. Alternative explanations are possible and examiners must be open to alternative approaches. If in doubt, consult your Team Leader.

5

Page 9: History A - The Bicester Schoolthebicesterschool.org.uk/wp-content/uploads/2015/09/Mark... · 2015-09-28 · History A Mark Schemes for the Units January 2009 H106/H506/MS/R/09J

F961-01 Mark Scheme January 2009

6. To what extent was Henry VII’s foreign policy a success? [50]

Focus: An assessment of an important aspect of policy. No set answer is looked for but candidates will need to address the question. Candidates may set out a list of criteria against which to judge success, considering Henry’s aims and how far they were achieved, security, finance and England’s position in Europe and the prevention of foreign states from undermining his kingship. Many may argue that Henry was very successful. He was able to gain recognition for the Tudors through a series of marriages to Spain and Scotland; he was able to ensure pretenders to the English throne were not given sanctuary, although this was not achieved immediately and some answers may point to the support given to Simnel and Warbeck by Margaret of Burgundy. As Henry’s position was not secure at the start of the period he was keen to avoid expensive wars and this was largely achieved; although he went to war against France over Brittany it was short-lived and Henry actually gained money through the French pension, however he was unable to prevent the French acquisition of Brittany which might be seen as a threat to English security. It might also be argued that Henry’s foreign policy was beneficial for trade with the Treaty of Medina del Campo and Magnus Intercursus. Security was improved as England was allied to Spain, the most powerful nation in Europe, and security on the northern border had also been achieved, which also weakened the Auld Alliance. Alternative explanations are possible and examiners must be open to alternative approaches. If in doubt, consult your Team Leader.

6

Page 10: History A - The Bicester Schoolthebicesterschool.org.uk/wp-content/uploads/2015/09/Mark... · 2015-09-28 · History A Mark Schemes for the Units January 2009 H106/H506/MS/R/09J

F961-01 Mark Scheme January 2009

Henry VIII to Mary I 1509-1558 7. How successful was Henry VIII in achieving his aims as king in the period from

1509 to 1529?

Focus: An assessment of the success of a king in a given period. No set answer is looked for but candidates will need to address the question. The question focuses on the aims of Henry and better answers will explain his aims and use them as the criteria against which to judge Henry’s success. The question allows candidates to consider both domestic and foreign issues, but examiners should not expect a 50:50 balance. However, in order to achieve the top levels candidates should consider both foreign and domestic issues. Some answers will focus on foreign policy as that was Henry’s main area of interest. His aim to repeat the glories of Arthur or Henry V were not repeated, but Henry did make his mark on Europe and the defeat of the French at the Battle of the Spurs did give him his glory, although it might be argued that it was exaggerated. However, some may balance this against his failure to get Ferdinand to support him. Some might argue that Henry also achieved his glory through the Peace Conferences that Wolsey organised, although others might suggest that peace was not a preferred policy. Some may also argue that he was much less successful in the 1520s and illustrate this with reference to the failure to raise money, through the Amicable Grant, for an invasion of France. It may also be concluded that he also failed at the end of the period in achieving his divorce, which was particularly crucial. This issue may also be linked to the succession and the need for a male heir; some answers may suggest that in this area he was a failure as his marriage to Catherine had produced only Mary and there was only an illegitimate son. In domestic areas some may suggest that he wanted to show that he was a very different king to his father and that this was achieved through the execution of Empson and Dudley. His desire to be a seen as a Renaissance king may also be considered. Alternative explanations are possible and examiners must be open to alternative approaches. If in doubt, consult your Team Leader.

7

Page 11: History A - The Bicester Schoolthebicesterschool.org.uk/wp-content/uploads/2015/09/Mark... · 2015-09-28 · History A Mark Schemes for the Units January 2009 H106/H506/MS/R/09J

F961-01 Mark Scheme January 2009

8. Assess the reasons for the fall of Thomas Wolsey in 1529. [50]

Focus: An assessment of the reasons for the fall of a key figure. No set answer is looked for but candidates will need to address the question. Many answers will focus on the question of the divorce as this was crucial in the minister’s fall. Candidates may explain why the divorce was so important and why Henry would abandon his minister when he failed to deliver, even though the circumstances were against him. As chief minister and with links to the Papacy Wolsey was expected to be able to bring about the divorce. Some answers might show why Wolsey’s attempts to solve the problem and satisfy Henry VIII led to his downfall. Some might consider the elements that were exposed by the divorce, such as the Boleyn faction and their ambitions, the growing distrust of Katherine of Aragaon and her supporters and others who simply saw the opportunity to bring down an over-mighty minister. They may conclude that by 1529 Henry was surrounded by those who wanted to bring down Wolsey. However, some answers will consider longer term factors that weakened Wolsey’s position, making him more vulnerable when he failed to achieve the divorce. They may consider the anti-Wolsey feeling among the nobility, who viewed him as a social upstart and resented his domination over Henry and his attempts, through the Eltham Ordinances, to exclude them. Some candidates will consider the loss of support for Wolsey following the failure of the Amicable Grant. Alternative explanations are possible and examiners must be open to alternative approaches. If in doubt, consult your Team Leader.

8

Page 12: History A - The Bicester Schoolthebicesterschool.org.uk/wp-content/uploads/2015/09/Mark... · 2015-09-28 · History A Mark Schemes for the Units January 2009 H106/H506/MS/R/09J

F961-01 Mark Scheme January 2009

9. To what extent was royal authority weak in the period from 1540 to 1547? [50]

Focus: An assessment of the strength of royal authority in a given period. No set answer is looked for but candidates will need to address the question. Although historiography is not an AS requirement some answers may make reference to the debate about Henry’s power in the 1540s, but this is not a requirement for any level. With Henry ageing in this period it is possible to argue that it was a period of weak royal authority. However, no minister replacing Cromwell some may balance this against Henry taking control of events. Some answers will consider the factional issues in the period and argue that these show that Henry had lost control, but others will argue that Henry was playing off the various groups and had ultimate control, the examples of his protection of Cranmer and Catherine Parr might be used to support the argument. Some might examine the last year of Henry’s reign and argue that the Seymour faction were able to seize control and use their power to manipulate events to their advantage when Henry died, undermining the balanced Regency Council that Henry had envisaged. They may point to the removal of Gardiner over a minor matter to show how Henry had lost control, although some may argue that this was further evidence of a strong king who demanded his will was obeyed. Some may argue that Henry actually enjoyed the sport of court politics and delighted in catching people out in order to show his power. It is therefore possible to conclude that although he may not have had overall control of events, the final decision making was his. Alternative explanations are possible and examiners must be open to alternative approaches. If in doubt, consult your Team Leader.

9

Page 13: History A - The Bicester Schoolthebicesterschool.org.uk/wp-content/uploads/2015/09/Mark... · 2015-09-28 · History A Mark Schemes for the Units January 2009 H106/H506/MS/R/09J

F961-01 Mark Scheme January 2009

Church and State 1529-1589 10. Assess the reasons for the break with Rome. [50]

Focus: An assessment of the reasons for a major historical development. No set answer is looked for but candidates will need to address the question. At the higher levels candidates should evaluate the importance of the factors. Many candidates are likely to focus on Henry’s desire for a divorce and the reasons for this. This may include a consideration of issues such as the lack of a male heir and the fact that Catherine was beyond child-bearing age. There may also be consideration of the legality of Henry’s marriage to Catherine, which had also been raised by the French during marriage negotiations over a possible marriage of Mary; this may also be linked to the issue of Henry’s conscience. However, some may choose to focus on Henry’s love for Anne, shown by his letters to her and the length of time he was willing to pursue negotiations. Some candidates might argue that Henry broke from Rome only with reluctance, shown by the attempts at pressure that were put on the papacy. There may be some candidates who argue that anti-clericalism within England and a desire to reform the church was an issue, but this may be dismissed. Ultimately, it was the foreign situation that prevented the Pope granting a divorce and with Anne pregnant resulted in the need to break with Rome. Candidates may therefore give some attention to the foreign situation with the Pope a virtual prisoner of Charles after 1527 and the fact he was Catherine’s nephew. Alternative explanations are possible and examiners must be open to alternative approaches. If in doubt, consult your Team Leader.

10

Page 14: History A - The Bicester Schoolthebicesterschool.org.uk/wp-content/uploads/2015/09/Mark... · 2015-09-28 · History A Mark Schemes for the Units January 2009 H106/H506/MS/R/09J

F961-01 Mark Scheme January 2009

11. To what extent were the religious policies of Somerset more successful than those

of Northumberland? [50]

Focus: A comparison of the religious policies of two monarchs. No set answer is looked for but candidates will need to address the question. Although examiners will not expect a 50:50 balance between the two ‘protectors’ there should be a reasonable balance for the top levels. At the higher levels candidates will need to establish criteria against which to judge the success of their policies. Candidates may examine the success in the short or long term and the different perspectives may lead to different conclusions. In considering Edward’s reign candidates may suggest that with England largely catholic in 1547 it was difficult for the nation to be converted by 1553. In support of this argument they may point to the Western Rebellion of 1549 and suggest that Somerset’s policies were not a success, but this might be balanced against the ease with which Lady Jane Grey was defeated and Mary restored. However, the issue of the unpopularity of the measures might be balanced by a consideration of the legal and doctrinal position which was fully protestant by 1553 with the Second Prayer Book, although the impact of this might be debated. Some answers might compare the legal position with the situation at the grass roots and use the evidence of wills or the failure to remove images to show the ineffectiveness of the legislation. Some might argue that the ease with which legislation repealing the Edwardian Reformation passed parliament during Mary’s reign and the lack of opposition to the burnings suggests that neither was particularly successful. It would also be possible to argue that Somerset faced a more difficult task than Northumberland, Somerset was faced with a nation where Protestantism was very limited, whereas Northumberland was able to build on the achievements of Somerset in establishing Protestantism and that despite this advantage it still took until the last months for a fully protestant church to be established. Alternative explanations are possible and examiners must be open to alternative approaches. If in doubt, consult your Team Leader.

11

Page 15: History A - The Bicester Schoolthebicesterschool.org.uk/wp-content/uploads/2015/09/Mark... · 2015-09-28 · History A Mark Schemes for the Units January 2009 H106/H506/MS/R/09J

F961-01 Mark Scheme January 2009

12. To what extent was Puritanism a serious threat to Elizabeth’s religious settlement?

[50]

Focus: An assessment of the problems posed by an important religious group No set answer is looked for but candidates will need to address the question. Although candidates will need to display an understanding of Puritanism moderate answers might spend too long describing, leaving the seriousness of the challenge to be implied rather than stated and evaluated. The term religious settlement should not be taken to mean simply the events of 1558-9, but can include later developments. Candidates might identify some aspects of the settlement of 1558-9 that puritans wanted to see changed and go on to assess whether their attempts were ever challenging or successful. This may focus on the issue of vestments, where a firm line was taken to resist puritan demands. Some candidates might argue that puritanism was a serious threat due to the numbers in parliament and mention might be made of the ‘Puritan Choir’, and the way they used parliament. However, Elizabeth was always easily able to defeat them and had considerable support from many members of parliament. The Prayer Book, the 39 Articles and the Advertisements established an Anglican orthodoxy and also resisted puritan demands. There might be some consideration of the organisations that emerged amongst extreme groups and consider links with courtiers such as Leicester. The seriousness of the challenge was assessed differently by various groups and individuals throughout the reign. Orthodox Anglicans saw Puritanism as a danger that needed to be controlled, possibly extirpated. Support among the gentry and in some towns did result in the election of some puritan MPs, who were able to put forward their proposals. Some may argue that the seriousness of the threat varied from group to group within the puritan movement; the moderate puritans were more willing to find a compromise and stay within the established church, whereas the radicals were smaller in number and not a serious threat. Alternative explanations are possible and examiners must be open to alternative approaches. If in doubt, consult your Team Leader.

12

Page 16: History A - The Bicester Schoolthebicesterschool.org.uk/wp-content/uploads/2015/09/Mark... · 2015-09-28 · History A Mark Schemes for the Units January 2009 H106/H506/MS/R/09J

F961-01 Mark Scheme January 2009

England under Elizabeth I 1558 -1603 13. ‘Cooperation rather than conflict.’ Assess this view of Elizabeth I’s relationship

with her parliaments? [50]

Focus: An assessment of the relationship between the monarch and an instrument of government. No set answer is looked for but candidates will need to address the question. Historiography is not a requirement of AS and references to the views of historians are not necessary for any mark, although credit will be given for relevant references. There are a number of areas that candidates might consider when discussing the view that conflict was more pronounced, these are likely to include the issues of parliamentary privilege, religion, especially the religious settlement and the activities of the Puritans, the problem of Mary Queen of Scots, marriage and succession, foreign policy at certain times and financial aspects, such as monopolies. Parliament was a sounding board and allowed the monarch to discover the views of the political elite and for them to take back the laws that they had to impose. However, this might be balanced by the argument that as MPs became better educated they were increasingly able and willing to challenge the control exercised on parliament through Privy Councillors and the Speaker. This may be supported by the argument that parliament was growing in assertiveness and that were anti-government views in the Commons, particularly from the puritan members. There might be discussion of the so-called Puritan choir and their impact and importance. Some may argue that they did not exist, there was no party and few were puritans pushing their religious views, but others may argue they were an active and well-organised lobby that created problems for Elizabeth. Some answers might consider the role of the House of Lords, which helped the queen in the 1590s over subsidies. The presence of Cecil in the Lords from 1572 may also have helped to ensure that there was co-operation. At the highest levels some answers might show that parliament spent longer discussing local issues and those bills were often assed with few problems. Some answers may place parliament in the wider context of the government of England that it was a only a secondary instrument in the governance of the nation, which could be ignored by the court or council or that it was still an irregular part of government. Alternative explanations are possible and examiners must be open to alternative approaches. If in doubt, consult your Team Leader.

13

Page 17: History A - The Bicester Schoolthebicesterschool.org.uk/wp-content/uploads/2015/09/Mark... · 2015-09-28 · History A Mark Schemes for the Units January 2009 H106/H506/MS/R/09J

F961-01 Mark Scheme January 2009

14. Assess the reasons why the issue of the succession caused domestic and foreign

problems for Elizabeth I. [50]

Focus: An assessment of a major problem during the reign of a monarch. No set answer is looked for but candidates will need to address the question. It was expected that Elizabeth would marry on her accession, but the problem was whom to marry? Candidates are likely to be aware of many of the suitors but each of these presented insuperable difficulties of religion, age and even mental stability. Some may argue that proposals from Philip of Spain and Charles IX of France had to be handled very carefully because of the power of their countries and therefore her foreign policy had to be careful, but candidates could balance this against the fact that Elizabeth could play off the two countries against each other, making it less of a problem. The succession was an important issue because of the claim of Mary Queen of Scots as it raised major problems concerning religion and relations with both France and Scotland. Her French links proved a threat, whilst she was also a possible centre of opposition for Catholics both at home and abroad. There was the problem of the suitability of any domestic suitors, particularly following the death in suspicious circumstances of Dudley’s wife. If Elizabeth married at home it would raise the issue of faction and the dominance and rewards that would go the family. In the same way there was concern that an overseas marriage would lead to influence and dominance from abroad, memories of Mary’s marriage to Philip and the French war still influenced many. Alternative explanations are possible and examiners must be open to alternative approaches. If in doubt, consult your Team Leader.

14

Page 18: History A - The Bicester Schoolthebicesterschool.org.uk/wp-content/uploads/2015/09/Mark... · 2015-09-28 · History A Mark Schemes for the Units January 2009 H106/H506/MS/R/09J

F961-01 Mark Scheme January 2009

15. To what extent did the popularity of Elizabeth and her government decline after

1588? [50]

Focus: An assessment of the popularity of a government in a specified period. No set answer is looked for but candidates will need to address the question. In the last years there was immense strain on the system of government. By the end of her reign Elizabeth was isolated at court and in her government because her associates had either died or retired. There were many who waiting for a new king. The patronage system, in the hands of Robert Cecil, had broken down and this caused resentment. Nevertheless, she maintained a grip on affairs and her weakness and determination should not be underestimated. The last parliament did show a willingness of MPs to criticise her over a policy of monopolies that pointed directly at the crown. On the other hand, some will argue that her Golden Speech demonstrated her continued ability to diffuse opposition. It was really only after the 1601 parliament that she failed physically. The queen herself might have lost some popularity but the prestige of the monarchy was still high. The problems in the last years should be set alongside the achievements; she had brought about comparative religious peace, a stable ministry led by William Cecil and a forced peace on Ireland. Some might argue that the lack of unrest during the period suggests that the popularity of the government had not declined, particularly as the social and economic problems of the 1590s were severe. The lack of support for Essex’s Rebellion might also be used to support this view. Elizabeth was also still popular among the populace, although the legend of ‘Good Queen Bess’ had not appeared in 1603. However, at a local level officials found it increasingly difficult to cope with the financial and administrative demands placed on it and these pressures, in the 1590s coincided with the social strains caused by harvest failures, food shortages and increasing inflation, all of which decreased the popularity of the government. It might be concluded that Elizabeth had reigned too long and the succession of James was widely welcomed. Alternative explanations are possible and examiners must be open to alternative approaches. If in doubt, consult your Team Leader.

15

Page 19: History A - The Bicester Schoolthebicesterschool.org.uk/wp-content/uploads/2015/09/Mark... · 2015-09-28 · History A Mark Schemes for the Units January 2009 H106/H506/MS/R/09J

F961-01 Mark Scheme January 2009

The Early Stuarts and the Origins of the Civil War 1603-1642 16. Assess the reasons why foreign policy caused conflict between James I and his

parliaments. [50]

Focus: Assessment of an important aspect of foreign policy No set answer is looked for but candidates will need to address the question. Some answers may claim that foreign policy was not a major cause of conflict between James and his parliaments when compared with other issues and this is a valid approach. Candidates might argue that there were two basic reasons for problems over foreign policy although they were linked. James believed that foreign policy was an essential part of the royal prerogative whereas critics resisted the claim, especially parliament as they had to vote the funds to sustain the policy. Secondly, the practicalities of his foreign policy proved unpopular. The pursuit of a peaceful policy appeared to be an appeasement of catholic powers, which was not popular with parliament. James I soon made peace with Spain, Treaty of London in 1604, and this was not popular with some who made money from the war and others who regarded Spain as the arch-enemy. James tried to maintain a balance, for example, marrying his daughter, Elizabeth to Frederick of the Palatinate and making an agreement with the German Protestant Princes. However, for many in parliament his policy did not go far enough and they wanted England to actively intervene in the Thirty Years War. They saw Protestantism as under threat and believed that James should help to defend it. The marriage of Charles became an important diplomatic tool, but it also revealed further conflict. The king’s reluctance to offend Spain led to unpopular developments, such as the execution of Raleigh and finally the embarrassment of Charles’ failed venture to Spain to agree a marriage. The intervention in the Thirty Years War in 1624 was too late to save James from criticism. He was criticised for following polices which were seen at times to be basically wrong and other times right but ineffective. Alternative explanations are possible and examiners must be open to alternative approaches. If in doubt, consult your Team Leader.

16

Page 20: History A - The Bicester Schoolthebicesterschool.org.uk/wp-content/uploads/2015/09/Mark... · 2015-09-28 · History A Mark Schemes for the Units January 2009 H106/H506/MS/R/09J

F961-01 Mark Scheme January 2009

17. How successfully did James I deal with religious problems during his reign?

[50]

Focus: An assessment of an important aspect of policy. No set answer is looked for but candidates will need to address the question. It is likely that better answers will identify the religious problems that James faced before assessing how well he dealt with them. There were some religious problems at the start of his reign but they should not be exaggerated. There were hopes of reconciliation at the start of James’ reign between orthodox Anglicans, Puritans and Catholics. The Hampton Court Conference was a failure, but its extent might have been exaggerated. The ejection of puritan clergy through Bancroft’s Canons appeared to dash the hopes of puritans and created mistrust. He failed to overcome the distrust partially as a result of the advisors he chose, such as Buckingham. The Gunpowder Plot did not help James’ preferred policy of tolerance as although it was the work of a small minority it increased antipathy towards Catholics in general and made his policy harder. By appointing Abbot as Archbishop of Canterbury in 1611 James did help to mollify some puritans, but the Book of Sports, 1618 alienated them. Foreign policy and marriage negotiations will also merit mention as that did much to exacerbate the problems. Puritans believed that he did not do enough to support Protestantism in Europe which was under threat, particularly during the Thirty Years War. His policy was seen as too conciliatory towards the Catholic powers. This was exacerbated by Charles’ marriage negotiations with Spain as England appeared to be getting closer to Catholicism. When James did finally enter the Thirty Years War it was too late to save him from criticism. Alternative explanations are possible and examiners must be open to alternative approaches. If in doubt, consult your Team Leader.

17

Page 21: History A - The Bicester Schoolthebicesterschool.org.uk/wp-content/uploads/2015/09/Mark... · 2015-09-28 · History A Mark Schemes for the Units January 2009 H106/H506/MS/R/09J

F961-01 Mark Scheme January 2009

18. ‘Religious divisions were the main reason for the outbreak of civil war in 1642.’

How far do you agree? [50]

Focus: An assessment of the causes of Civil War No set answer is looked for but candidates will need to address the question. There are many alternative arguments that candidates might put forward. However, for the higher levels candidates must write at least a good paragraph on the named factor. Other forms of alternative approach will be a focus on 1640-2, from the meeting of the Long Parliament to the outbreak of the civil war, or the longer term approach that shows that factors gradually built up. The problem with the second approach is that candidates might lose focus on the question and simply concentrate on telling the story. Developments before 1640 are relevant, but a civil war could not have been predicted then, nor was it possible. When considering religion, Charles I’s Arminianism, encouraged and enforced by Laud, was very different from the religion of many and created problems. There were suspicions that Charles was too tolerant of Catholicism, even that he had sympathies with the religion. Laud was arrested and there were criticisms of the episcopacy. The Court of High Commission was abolished. Reference could be made to events in Ireland, which were strongly linked to religion. There might also be consideration of the Root and Branch Bill. There were also suspicions about the influence of Henrietta Maria. However, candidates should also consider other factors and these might include political divisions, including Charles I tendencies towards absolutism. His prerogative powers also caused disquiet and these were attacked. The arrest and subsequent execution of Strafford was a turning point. Within parliament, the momentum gave impetus to the war party; there were divisions between those who would be reconciled to the king and those who sought to weaken his position further. Control of the army was vitally important, Charles was distrusted personally, a feeling that was confirmed by his attempt to arrest the Five Members. Alternative explanations are possible and examiners must be open to alternative approaches. If in doubt, consult your Team Leader.

18

Page 22: History A - The Bicester Schoolthebicesterschool.org.uk/wp-content/uploads/2015/09/Mark... · 2015-09-28 · History A Mark Schemes for the Units January 2009 H106/H506/MS/R/09J

F961-02 Mark Scheme January 2009

19

F961-02 - British History Period Studies Option B: Modern 1783-1994

From Pitt to Peel 1783-1846 1. How successfully did Pitt the Younger deal with the radical threats in England? [50] Focus: An assessment of a government’s policy towards a problem. No set answer is looked for but candidates will need to address the question. Many answers are likely to argue that Pitt was very successful arguing that the radical movement was weak as it was divided over aims and methods, associated with France and lacked genuine popular support. The repressive legislation in the 1790s was successful and candidates are likely to provide details of the legislation. Some answers might also argue that he was successful because he was able to exploit Whig divisions. The threat might be considered to be serious because the nation was industrialising and urbanising, if this is the line taken candidates may still conclude that Pitt was very successful, England did not face the unrest that France did in the same period. However, although the threat defeated or at least contained there were corresponding societies, combinations, riots and machine smashing. Many are likely to argue that Pitt dealt very successfully with it in the 1790s as Radicals were put on trial in 11793, Habeas Corpus was suspended, creating a new party of Order in 1794 when the Whigs split, The Treasonable Practices Act and the Seditious Meetings Act of 1795 which targeted radical methods, the regulation via taxes of newspapers, which enable freedom to be maintained in theory, and formal bans on radical societies. However, some might argue that these actions drove radical activity underground, but even then it was infiltrated by spies and by financing and fostering loyalist associations. Pitt’s success might also be challenged by reference to the danger in 1795 and in the naval mutinies of 1797, together with Foxe’s pro-revolutionary stance. Alternative explanations are possible and examiners must be open to alternative approaches. If in doubt, consult your Team Leader.

Page 23: History A - The Bicester Schoolthebicesterschool.org.uk/wp-content/uploads/2015/09/Mark... · 2015-09-28 · History A Mark Schemes for the Units January 2009 H106/H506/MS/R/09J

F961-02 Mark Scheme January 2009

2. To what extent does the Reform Act of 1832 deserve to be called ‘Great’? [50] Focus: An assessment of a major reform. No set answer is looked for but candidates will need to address the question. Better answers will establish a set of criteria against which to assess whether the Reform Act deserves to be called ‘great’. This may involve consideration of the significance of the changes that the Act brought about. A detailed knowledge of the terms of the act will allow judgements to be made about the changes. Candidates might consider the increase in the number of voters, the loss of some of the rotten boroughs and the redistribution of seats to towns and counties. The redistribution of seats went someway to correct the earlier imbalance between county and borough members, the important newer towns received their MPs and many smaller boroughs either lost their MPs or returned one instead of two. It might also be noted that this was the first major change to the franchise and set the scene for future developments. The increase in the franchise encouraged the growth of party organisation as parties needed to ensure registration of the new voters. These considerations might be balanced against the disappointment that followed for many after the act and there might be a link to the growth of other movements, such as Chartism. However, it could be argued that it was great as it prevented revolution and broadened the basis of support for the constitution. On the negative side the act did not remove corruption, although the number of very corrupt constituencies did disappear. More than 50 seats were still under the control of individual patrons. The act did strengthen the existing order, which may have been the aim and the aristocracy retained their dominance, changing little in practice. Alternative explanations are possible and examiners must be open to alternative approaches. If in doubt, consult your Team Leader.

20

Page 24: History A - The Bicester Schoolthebicesterschool.org.uk/wp-content/uploads/2015/09/Mark... · 2015-09-28 · History A Mark Schemes for the Units January 2009 H106/H506/MS/R/09J

F961-02 Mark Scheme January 2009

3. To what extent was the repeal of the Corn Laws in 1846 the most important reason for the fall of Peel’s government? [50]

Focus: An assessment of the reasons for the fall of a government. No set answer is looked for but candidates will need to address the question. Many candidates are likely to agree with the assertion in the question. The policy was divisive and Peel was unable to persuade even his own Cabinet to support repeal, Stanley resigning, nor could he persuade the bulk of his party. However, there were already tensions within the party and there had been divisions over Maynooth and Peel had frequently threatened to resign. Some might therefore argue it was the occasion, rather than the cause of the fall of the government. Some might also note that the government resigned, not over the Corn Laws, but the Coercion Bill for Ireland. Among Tory malcontents there was a long-standing dislike of Peel’s dear money and deflationary policies. They already felt he had betrayed them on Catholics and were not willing to see another Tory principle go. The Corn Laws were important to them because they guaranteed high rents and helped to uphold their status and power, which they believed were under threat form the manufacturing interest. Landlords were urged to resist their repeal by the Anti-League. Peel might well have survived had it not been for the venom and persistence of Disraeli and Bentinck. Disraeli had a personal dislike of Peel and felt snubbed by him. Peel openly despised the party that spent ‘their days in hunting and shooting and eating and drinking’ and never took it into his confidence. Disraeli was a party man, but Peel with his executive mentality starved Disraeli of hope and had to face the consequences. Alternative explanations are possible and examiners must be open to alternative approaches. If in doubt, consult your Team Leader.

21

Page 25: History A - The Bicester Schoolthebicesterschool.org.uk/wp-content/uploads/2015/09/Mark... · 2015-09-28 · History A Mark Schemes for the Units January 2009 H106/H506/MS/R/09J

F961-02 Mark Scheme January 2009

Liberals and Conservatives 1846-1895 4. Assess the reasons for the emergence of the Liberal party by 1868. [50] Focus: An assessment of the reasons for the emergence of a political party. No set answer is looked for but candidates will need to address the question. There are many reasons that candidates may consider. It is likely that many will point to the support given to the Whigs by radicals such as Cobden and Bright, unity over Italian Unification in 1859, common support for free trade, the success of Gladstone’s budgets, the fact that the public trusted the party over finance, a popular foreign policy under Palmerston and the abandonment of the aristocratic Whiggish image associated with Grey and Melbourne in the 1830s. This was important as the party had gained key Peelites, becoming a progressive party with increasingly popular ideas and a willingness to undertake reform. The popularity of its foreign policy might be considered and Palmerston ensured it was well advocated. There might also be mention of the disarray of the Conservative party and the resultant incentive to create a united party. There might be some mention of the role of the new, daily provincial press, which consistently advocated civil and religious liberty and the belief in the idea of progress. This was reflected in increased support in the boroughs. The non-conformists also wanted a party that would adopt their progressive policies in education, politics and religion. From their militant religious outlook came their adoption of ‘good causes’ that gave the liberals their dynamic quality. The men who noticed the emergence of these groups and the need to bind them to the newly found Liberal party in the Commons were Gladstone and Bright. Gladstone’s 1864 speech was important and resulted in him being hailed the ‘People’s William’, although some have seen his role as less important than Palmerston and Russell. The party was seen as the party of business sense, moral integrity and administrative efficiency. However, it is possible that some may argue it was little more than a series of fragile coalitions or ambitious men doing deals to get themselves into power. The party had come together to destroy Derby’s administration and wanted to stay in power. Alternative explanations are possible and examiners must be open to alternative approaches. If in doubt, consult your Team Leader.

22

Page 26: History A - The Bicester Schoolthebicesterschool.org.uk/wp-content/uploads/2015/09/Mark... · 2015-09-28 · History A Mark Schemes for the Units January 2009 H106/H506/MS/R/09J

F961-02 Mark Scheme January 2009

5. How far do you agree with the view that the foreign and imperial policy of Disraeli’s second ministry (1874-1880) was more successful than his domestic policy? [50]

Focus: An assessment of a ministry. No set answer is looked for but candidates will need to address the question. Examiners should not expect a 50:50 balance in the treatment of foreign and imperial policy and domestic, although both aspects do need adequate treatment. In order to assess the success, or otherwise, of foreign and imperial policy candidates may consider the objectives which might include containing Russia, maintaining prestige and the empire and protection of trade. It is likely that foreign policy consideration will be dominated by Disraeli’s handling of the Near Eastern Crisis of 1875-8 and his alleged triumph at the Congress of Berlin. In considering imperial policy they might discuss policy in Egypt, South Africa and Afghanistan. The success, or otherwise, of these policies will need to be compared with the impact of the domestic legislation that was passed. It is likely that this will focus on the issue of social reform. The conservative paternalist approach brought practical improvements for the working classes, even for the less respectable and ambitious of them through the Artisans Dwelling Act and the Sale of Food and Drugs Act. Sandon’s Education Act was prepared to force parents to send their children to school. The Conservatives were also prepared to allow peaceful picketing and in these last to acts went further than the Liberals. There was also a Public Health Act which consolidated previous statutes and the Employers and Workmen Act. It might be argued that these acts, by being compulsory, did have an impact or it could be argued that they simply tidied up legislation from Gladstone’s ministry and were therefore less successful than his foreign and imperial policies. Alternative explanations are possible and examiners must be open to alternative approaches. If in doubt, consult your Team Leader.

23

Page 27: History A - The Bicester Schoolthebicesterschool.org.uk/wp-content/uploads/2015/09/Mark... · 2015-09-28 · History A Mark Schemes for the Units January 2009 H106/H506/MS/R/09J

F961-02 Mark Scheme January 2009

6. To what extent were Gladstone’s second and third ministries a failure? [50] Focus: An assessment of governments in a given period. No set answer is looked for but candidates will need to address the question. There is a tendency to view these ministries as a failure, particularly after the achievements of the first ministry. Answers may focus on the issue of Ireland, particularly the failure to achieve Home Rule and the division within the Liberal party that followed and lead eventually to Hartington and other Whigs joining the Conservative party. In considering Ireland, the Second Land Act might be mentioned; this was too late as Parnell was now demanding Home Rule. The party appeared to be following a faddist programme and was losing relevance with many. However, the reforms to the franchise and the electoral system were significant achievements and warrant attention; this includes the Corrupt Practices Act, Parliamentary Reform and the Redistribution of Seats Act. There were also failures in both foreign and imperial policies which will merit attention. The Boers were able to defeat the British at Majuba Hill in 1881 and through the Convention of London, 1884, the Boers gained their independence. Events in the Sudan might also be considered, particularly the death of Gordon as that did a lot to discredit Gladstone with the ordinary man, who blamed him for his death. Consideration of divisions within the party did not help the Liberal cause as early as 1880 with the old-fashioned Whigs and the Radicals, this was to become a more serious issue later and hindered attempts at reform. This might lead to a discussion of Chamberlain and his ‘Unofficial Programme’ which promised that Liberal government would deal with unemployment, poverty, housing and old age; all issues that Gladstone had avoided. Alternative explanations are possible and examiners must be open to alternative approaches. If in doubt, consult your Team Leader.

24

Page 28: History A - The Bicester Schoolthebicesterschool.org.uk/wp-content/uploads/2015/09/Mark... · 2015-09-28 · History A Mark Schemes for the Units January 2009 H106/H506/MS/R/09J

F961-02 Mark Scheme January 2009

Foreign and Imperial Policies 1856-1914 7. To what extent did the principles governing British foreign and imperial policy

remain the same throughout the period from 1856 to 1902? [50] Focus: An assessment of the factors influencing foreign and imperial policy. No set answer is looked for but candidates will need to address the question. This is a wide question that encompasses much, so full or detailed coverage of every aspect is not expected. Candidates should discuss some of the main principles, such as the Balance of Power, the promotion of trade, the preservation of the Empire, concern over Russia and France and a limited preference for constitutional states. The principle of balance of power saw considerable change, this was particularly noticeable over Austria as Britain supported Italian and German nationalism at the expense of Austria. A commitment to the Treaty of Paris had failed by 1870. The balance was now one of 5 or 6 and one could argue that Britain now withdrew into splendid isolation. Challenged by the US and Germany on Trade, Britain remained committed to Free Trade and Sea Power as a means of securing markets. Candidates might argue for a change on Empire, from Free Trade Imperialism to a formal Empire, although some see this as a reluctant change, imposed by others. Russia and France remained constant curbing, Russia in the Crimea and the Balkans and France in Italy, Africa and the Far East. As for supporting constitutional states there was a Gladstonian morality that supported a nation struggling to be free but this depended upon the behaviour of any individual state or its oppressor rather than any consistent preference for constitutionalism per se. Alternative explanations are possible and examiners must be open to alternative approaches. If in doubt, consult your Team Leader.

25

Page 29: History A - The Bicester Schoolthebicesterschool.org.uk/wp-content/uploads/2015/09/Mark... · 2015-09-28 · History A Mark Schemes for the Units January 2009 H106/H506/MS/R/09J

F961-02 Mark Scheme January 2009

8. ‘The protection of trade routes was the most important reason for British interest in the Eastern question.’ How far do you agree with this view? [50]

Focus: An assessment of the relative importance of the issues involved in the Eastern Question. No set answer is looked for but candidates will need to address the question. The Balkans were important in the struggle for supremacy between Russia, Austria and Balkan nationalism; the issues of the Straits, the most pressing in 1854-6 and 1875-8 and Egypt and the Middle East. These issues should give candidates a variety of reasons to consider and make a judgement as to how far the protection of trade was the most important reason for British interest. As this was a vital area for Britain because of the route to India and trade with the east and Mediterranean trade it may be argued that this was Britain’s main reason for concern. This was particularly the case when France threatened more control over Egypt via the new Suez Canal. Russia was a threat to the Mediterranean following the invasion of the Danubian provinces and an attack on the Ottomans. Russia also championed the Bulgarian Christians following the massacre in 1875. Their growing power was seen as a menace, not just in Europe but in Asia to the British Empire. Another concern for Britain was how to deal with the declining Ottoman Empire, whether to prop it up, regardless as an economic and strategic investment. This raises the question of the balance of power and candidates might consider this in relation to the decline of Turkey and the rise of Russia. There was fear that a weak turkey might be unable to prevent a Russian advance towards the Mediterranean and Suez, with implications for trade and security. Interest in the Balkans was also stirred up by Gladstone and his pamphlet attacking the murder of Christian Slavs, although this may have had more to do with election opportunism. The growth of nationalism in the area will also need consideration and how to deal with the decline and ultimate defeat of Turkey in the Balkan Wars. Alternative explanations are possible and examiners must be open to alternative approaches. If in doubt, consult your Team Leader.

26

Page 30: History A - The Bicester Schoolthebicesterschool.org.uk/wp-content/uploads/2015/09/Mark... · 2015-09-28 · History A Mark Schemes for the Units January 2009 H106/H506/MS/R/09J

F961-02 Mark Scheme January 2009

9. How popular was the policy of imperialism in England in the period from 1880 to 1902? [50]

Focus: An evaluation of the popularity of imperialism. No set answer is looked for but candidates will need to address the question. The party that dominated politics in this period was the pro-Empire Conservatives. Candidates could support this with reference to the Khaki election of 1900, where their victory, an overall majority of 268 was, in part, the result of patriotic fervour in the earlier part of the Boer War. Even at the end of the war some might argue that imperialism was still popular; British invincibility did remain intact and some may argue that this was reinforced by music hall jingoism and added to the idea that ‘the sun never set on the Empire’. Joe Chamberlain had been able to build a new career in the Conservative party on the basis of popular imperialism, as did Curzon. However, this might be balanced by a consideration of the problems that the small force of Boers had created for the might of the British army and the brutal use of concentration camps also damaged prestige. There was questioning of Chamberlain’s imperial vision and the social and medical problems of recruitment brought demands for a concentration on domestic issues. After Gladstone, the Liberal leader Lord Rosebery realised the political importance of the Empire. However, candidates could balance this by looking at the 1880 election which Disraeli lost largely due to Gladstone’s anti-Imperial campaign. Gladstone in turn was in trouble by 1885 over Egypt and especially for his failure to send sufficient troops to save General Gordon in his attempts to defend Khartoum from the attacks by the Mahdi. After 1900 the Boer War did not aid the Conservatives. Some candidates may argue that the Empire was always politically controversial and divisive. Whether the working class were responsive to imperialism was far from certain, especially as the Empire could be portrayed as wasteful when social reform was a more pressing issue. It could be argued that the popular culture surrounding imperial expansion and policies suggests that it was a popular policy and candidates might point to some of the Music Hall songs, popular literature and also religious concerns, such as some of the late Victorian hymns or missionary activity which had captured the public imagination. Alternative explanations are possible and examiners must be open to alternative approaches. If in doubt, consult your Team Leader.

27

Page 31: History A - The Bicester Schoolthebicesterschool.org.uk/wp-content/uploads/2015/09/Mark... · 2015-09-28 · History A Mark Schemes for the Units January 2009 H106/H506/MS/R/09J

F961-02 Mark Scheme January 2009

Domestic Issues 1918-1951 10. How far was the government to blame for the General Strike of 1926? [50] Focus: An assessment of the causes of the General Strike. No set answer is looked for but candidates will need to address the question. Candidates will need to address the role of the government, even if they conclude that other parties were more to blame. The government were intransigent, combined with an economic policy that was in part mistaken by returning to an overvalued currency backed by gold in 1925 as the main cause. The government was certainly determined to stand by private ownership and lower labour costs. Candidates might also point out that it was the government that precipitated a strike on 3 May by calling off negotiations, not the Unions. Candidates are also likely to consider the role of the Trade Unions, especially the NUM and this might be set alongside the role of the mine owners with their response to adverse terms in world trade which saw British coal as increasingly uncompetitive. The role of the Daily Mail could also be considered as the trigger for the strike. The case for Union unrest as the main cause is the growing power and influence of the Unions up to the First World War and their reaction to changed economic conditions after it. Some candidates might make a distinction between the Miners, led by Smith and Cook, and the leadership of the Union movement in general who were reluctant to act and certainly unprepared to stage a General Strike. The latter were prepared to accept Samuel’s recommendation as a basis of settlement. The Miners were less prepared to compromise. Previously they had been the aristocrats of the labour market and long unionised they were especially hard hit by economic contractions, new fuels, poor geological conditions and under investment in mining technology. Employers might also be blamed as they argued that a competitive price could be achieved only at the expense of the miners. It might also be argued that having scored victories with the Sankey Commission and on Red Friday they were keen to make a stand on behalf of other well organised, but struggling industries. It might also be noted that the Unions failed to see Red Friday as simply a government tactic to buy time and stockpile and prepare for a strike. Alternative explanations are possible and examiners must be open to alternative approaches. If in doubt, consult your Team Leader.

28

Page 32: History A - The Bicester Schoolthebicesterschool.org.uk/wp-content/uploads/2015/09/Mark... · 2015-09-28 · History A Mark Schemes for the Units January 2009 H106/H506/MS/R/09J

F961-02 Mark Scheme January 2009

11. How successful were the National Governments 1931-9 in dealing with the social and economic problems they faced? [50]

Focus: An assessment of a government in dealing with social and economic problems. No set answer is looked for but candidates will need to address the question. There were a large number of policies adopted by the National Governments and examiners should not expect all of these to be covered, what is important is the quality of the analysis. In order to assess the success better candidates are likely to identify the problems that the governments faced; this might include the need for a stable currency, tackling unemployment, relieving poverty and tackling the problems of the older, traditional industries. There was success in stabilising the Pound through Snowden’s Budget and the end of the Gold Standard, even though the government had been formed to stop it. It boosted exports and reinforced confidence although better candidates might point to external pressure in forcing this. ‘Cheap Money’ proved very successful in lowering costs, although Britain was lucky that prices dropped far more than wages, enabling those in work to feel better off and sustain the new industries. Protection was more controversial in its impact; it may well have had little impact unless linked to re-organisation, as with steel. The Ottawa conference saw little gain for British industry. As for poverty relief, government continued with the Public Assistance Committees but was determined on cuts to avoid overspend, this resulted in the Means Test, later removed, which was very unpopular. The government preferred to create the conditions to enable private enterprise to prosper, with some success in the Housing Boom in new suburbs. Its one attempt to tackle regional economic problems, the Special Areas Act, was on too small a scale to achieve much. The 1936 Jarrow March was testament to token gestures on relief when faced with the near collapse of a large industry. Welfare payments did continue at a higher level than most countries. Alternative explanations are possible and examiners must be open to alternative approaches. If in doubt, consult your Team Leader.

29

Page 33: History A - The Bicester Schoolthebicesterschool.org.uk/wp-content/uploads/2015/09/Mark... · 2015-09-28 · History A Mark Schemes for the Units January 2009 H106/H506/MS/R/09J

F961-02 Mark Scheme January 2009

12. To what extent was Conservative re-organisation the main reason for the defeat of Labour in the 1951 General Election? [50]

Focus: An assessment of the reasons for the defeat of the Labour party at a specific time. No set answer is looked for but candidates will need to address the question. Candidates might point to much recovery and reorganisation of the Conservatives since their damaging 1945 defeat when they had been associated with a failure to deliver on social reform and unimaginative policies, foreign and economic, in the years preceding the war. The scale of their defeat ensured structural reorganisation. This had been run down during the war years. Key constituency staff were put in place; there were membership and money raising initiatives by Lord Woolton which made the Conservative party one of the largest organisations in the UK. Image problems were tackled with a Young Conservative movement reviving grass roots support. The party’s structure was democratised following the Maxwell-Fyfe Report in 1949 which allowed a more open selection of candidates. In policy terms the Conservatives accepted the Welfare State consensus established by Labour. An Industrial Charter was produced in 1947 acknowledging co-operation, the role of Trade Unions and a role for the state in maintaining full employment. All this led to a report in 1949 ‘The Right Road for Britain’, which became the party’s manifesto. Pledges were also made on housing, a perceived labour failure and a potential vote winner. However, candidates might argue that the main reason was the economy and continued austerity, the state of which was blamed on Labour. Devaluation benefits had yet to work through, restrictions were irritating, rationing was particularly disliked. These provided two key electoral issues in 1951. There was also the Cold War where the Conservatives could play on patriotism and defence again. It is also possible that candidates will argue that Labour lost in 1951 rather than the Conservatives winning it through active policies and reorganisation. Internal labour divisions had worsened between left and right (Bevan v Morrsion), brought to a head by Gaitskell’s proposals for rearmament to deal with Korea and Britain’s Cold War commitments, which meant prescription charges being imposed on Bevan’s Health Service, ‘free at the point of care’. Bevan, Wilson and Freeman all resigned in April 1951. Nationalisation of iron and Steel in 1951 also proved electorally damaging. The 1951 election was close, Labour registering its highest poll to date with a larger share of the vote. It could simply be a pendular swing with little to do with either Conservative revival or Labour difficulties. Alternative explanations are possible and examiners must be open to alternative approaches. If in doubt, consult your Team Leader.

30

Page 34: History A - The Bicester Schoolthebicesterschool.org.uk/wp-content/uploads/2015/09/Mark... · 2015-09-28 · History A Mark Schemes for the Units January 2009 H106/H506/MS/R/09J

F961-02 Mark Scheme January 2009

Foreign and Imperial Policies 1945-1990 13. Assess the reasons why Britain joined the EEC in 1973. [50] Focus: An assessment of the reasons for a major development in British foreign policy. No set answer is looked for but candidates will need to address the question. Candidates may place Britain’s decision to join in the context of other failed applications and suggest reasons why it was successful this time and not before. British entry had been vetoed before by De Gaulle and he had resigned as leader of France in 1969 and died in 1970, it was his vision of Europe that had stood in the ay of British entry in 1961 and 1967. He wanted France to lead Europe and wanted Europe independent from the US. Britain also had concerns as to whether her membership would be compatible with her commitments to the Commonwealth. Many in the Conservative governments of the 1960s were opposed as was the Labour party. There were economic and political reasons compelling Britain to apply, particularly the strength of sterling. Britain was also prepared to keep her conditions to a minimum, mainly involving CAP and were now willing to accept the Treaty of Rome and the supranational principles behind it. The attempt to join in 1967 was rejected by De Gaulle, but Wilson countered by leaving the application open, ready to be taken up at the earliest opportunity, therefore De Gaulle’s resignation was the opportunity. Agricultural concerns were a major issue as Britain would have to pay more into it, and would get less back, than other EEC countries. Heath’s election was crucial as he was a pro-European, he played down the special relationship with the US in order to appeal to the EEC. There were problems over the world role of the pound sterling, but Heath’s intervention and direct talks with the French President, Pompidou, were crucial as he believed he could trust Heath. The talks between those two were the ultimate reason Britain joined. Alternative explanations are possible and examiners must be open to alternative approaches. If in doubt, consult your Team Leader.

31

Page 35: History A - The Bicester Schoolthebicesterschool.org.uk/wp-content/uploads/2015/09/Mark... · 2015-09-28 · History A Mark Schemes for the Units January 2009 H106/H506/MS/R/09J

F961-02 Mark Scheme January 2009

14. How far did public opinion influence Britain’s nuclear weapons policy? [50] Focus: An assessment of factors influencing a key policy. No set answer is looked for but candidates will need to address the question. Candidates should consider a variety of reasons that influenced Britain’s nuclear weapons policy before reaching a conclusion as to how important public opinion was in influencing it. They may consider the desire to have an independent nuclear weapons policy to avoid dependence upon the USA following her marginalisation over the Manhattan project. This may be seen as giving Britain a bargaining point with the USA. The development of the Cold War and the fear generated by the fact that Britain was within range of USSR bombers added to the desire to have an independent nuclear weapon. Candidates might also consider the influence of budgets as by 1954-5 the cost of rearmament was approaching the levels at the end of the Second World War, there was also the problem of financing the production of nuclear delivery warhead systems. Cost was a vital factor in British policy, the Blue Streak Missile had to be cancelled, but this was also because there was a realisation that it was to be housed in vulnerable land silos and would not be adequate to improved soviet defence systems. The purchase of Skybolt was also made out of economic necessity as it prolonged the life of the V bombers and could be used with Britain’s own nuclear warheads, thus maintaining independence. The development of Polaris may also be considered, even under Labour only one out of the five ordered Polaris was cancelled. One lesson of Suez was the need to retain an independent nuclear deterrent capability as this would allow Britain to influence US defence and foreign policy formulation. However, in the period 1964-79 there was the need to control defence spending. Labour’s 1964 campaign had specifically mentioned this, but once in power there was little change. Developments under Thatcher saw a modernisation of weapons. In considering the importance of public opinion candidates might consider the emergence of the Aldermaston Marches and CND, but it is debatable how much influence it had on government policy. However, the new phase of nuclear rearmament under Thatcher did see the recovery of CND, a movement which had been largely dormant since the 1960s, suggesting it had had little impact. Perhaps the best-known incident was the Woman’s tent camp at Greenham Common. The Labour party made the nuclear issue central to their campaign in 1983 demanding not only the cancellation of Cruise and Trident but also a completely non-nuclear Britain, but public opinion In Britain was less hostile and Labour did badly in the election. Alternative explanations are possible and examiners must be open to alternative approaches. If in doubt, consult your Team Leader.

32

Page 36: History A - The Bicester Schoolthebicesterschool.org.uk/wp-content/uploads/2015/09/Mark... · 2015-09-28 · History A Mark Schemes for the Units January 2009 H106/H506/MS/R/09J

F961-02 Mark Scheme January 2009

15. ‘The Falkland’s War Thatcher’s most successful foreign policy achievement.’ How far do you agree with this view? [50]

Focus: An assessment of foreign policy under a British PM. No set answer is looked for but candidates will need to address the question. Candidates should write a good paragraph on the named factor before discussing other issues. The Falklands war was certainly a triumph for Thatcher as it not only restored confidence within the nation, but it was a crucial factor in her electoral success in 1983. Although the victory brought little tangible gain, it was seen as a logistical triumph and a reversal of a humiliation when the islands had been seized. This may be balanced against other areas of foreign policy, such as relations with the EU and in particular the budget negotiations and rebate that was achieved. This gain in the short term might be balanced by considering the image it created concerning Britain’s future in Europe. The development of the special relationship with the USA, particularly between Thatcher and Reagan may also merit balanced consideration. From early in her premiership she gave full support to Reagan over Afghanistan. There was support for SDI, but only after protests and voicing the concerns of other European powers. Support was given over anti-communist moves in Europe. A working relationship was established with Gorbachev and there was much Soviet-British activity in the diplomatic and economic field culminating in Thatcher’s visit to Moscow. It could be argued that Britain was a mediator between US and USSR. Thatcher was unable to halt the momentum of superpower détente. In the collapse of communism all Britain could claim was a moral victory as it had shattered the whole framework of foreign policy. There was concern about the reunification of Germany. There may also be consideration of British support for Kuwait and the subsequent involvement in the Gulf War under Major. Alternative explanations are possible and examiners must be open to alternative approaches. If in doubt, consult your Team Leader.

33

Page 37: History A - The Bicester Schoolthebicesterschool.org.uk/wp-content/uploads/2015/09/Mark... · 2015-09-28 · History A Mark Schemes for the Units January 2009 H106/H506/MS/R/09J

F961-02 Mark Scheme January 2009

Post War Britain 1951-1994 16. Assess the reasons why the Conservative party remained in power from

1951 to 1964. [50] Focus: An evaluation of the reasons for the dominance of a political party during a specific period. No set answer is looked for but candidates will need to address the question. Many candidates may see national affluence plus Conservative leadership and organisation as being more important than Labour weakness. Labour weakness was certainly important in preserving the Conservatives in power. Candidates might argue that Labour failed to modernise its policies to a more affluent Britain and the party was constantly divided. The parliamentary party was split between the Left and Right, fuelled by Bevan’s left-wing hostility to Gaitskell. The former wanted an expansion of the public sector, Gaitskell and Morrsion did not. They succeeded to the leadership and agreed on a social democracy rather than a socialist one, attempting to remove Clause IV in 1961, but failing. Only with the gloss provided by science and technology were they able to offer the electorate an alternative in 1964. This enabled the Conservatives to avoid the electoral consequences of their mistakes; particularly Suez1956 and the resignation of Eden. Their worst moments did not coincide with elections, for example the Profumo Scandal in 1961 and the failure to join the EEC in 1963. Candidates might consider Conservative leadership, which apart from an aged-Churchill, the mistakes by Eden over Suez and Home’s unsuitability was proficient with Eden pre-Suez and Macmillan. The role of Macmillan may be considered and candidates might consider his skilful exploitation of television and the media. Macmillan could also be ruthless. The economy is likely to be considered as this was an important factor, there was recovery, the end of austerity and a period of relative affluence, all of which were credited to the Conservatives. Elections were well managed by the Conservatives in 1951, 1955 and 1959. The Cold War also aided the Conservatives. The Conservatives were well organised and this also impacted on more coherent policies with the work of Butler, Maudling, Powell and MacLeod. Alternative explanations are possible and examiners must be open to alternative approaches. If in doubt, consult your Team Leader.

34

Page 38: History A - The Bicester Schoolthebicesterschool.org.uk/wp-content/uploads/2015/09/Mark... · 2015-09-28 · History A Mark Schemes for the Units January 2009 H106/H506/MS/R/09J

F961-02 Mark Scheme January 2009

17. How successful were Thatcher’s domestic policies in the period from 1979 to 1990? [50] Focus: An assessment of domestic policies under a named PM. No set answer is looked for but candidates will need to address the question. Although a very controversial PM, Thatcher did win three successive elections which might lead many to argue that her policies were successful. However, it was not always domestic policies that brought her success and therefore candidates need to be careful in using criteria against which to assess her ministries. There may be substantial consideration of her economic policies which did see unemployment rise dramatically, although it might be argued that this was essential if the substantial problem of inflation was to be brought under control. It could be argued that her first ministry had few economic achievements. It could be argued that her attack on the power of the Unions was both successful and necessary if modernisation of the economy was to occur, particularly in light of the Winter of Discontent under Callaghan. Some better answers might suggest that she was more successful in taking on the miners than Heath as she prepared for the confrontation. The denationalisation of industries brought many into share-holding, giving them a greater stake in the country, although in the long-term few kept their shares, but it was popular within the country. The availability of buying your own council house might be seen as very successful as it created a new class of property owner who would often support the party. However, in hindsight some have seen the consumerism and ‘loads of money’ culture that her time in office created as undesirable. For the ‘haves’ of the mid 1980s it was a period of prosperity as real wages outstripped inflation. It appeared in 1987 as if the government had delivered an economic miracle. However, for those who did not benefit from the economic policies it was a period of increasing social exclusion, this was shown with riots in Brixton and other cities. Unemployment in parts of the inner cities hit levels not seen since the inter-war years as a result of industrial decline. There may be an argument that the government had little regard for civil liberties. The Poll Tax and subsequent riots was not a success. The success depends upon the criteria used to measure success. It had been more difficult to roll back the state, government spending had hardly been reduced, the authority of the government had become increasingly centralised and interventionist. There were controls on local government, education and even privatisation was accompanied by the growth in quangos. This could be balanced against the reduction in Union power, the decline in the loss of the number of working days, the pattern of house ownership was changed and it might be argued that Britain’s political culture was changed as socialism as a domestic force was finished off. Alternative explanations are possible and examiners must be open to alternative approaches. If in doubt, consult your Team Leader.

35

Page 39: History A - The Bicester Schoolthebicesterschool.org.uk/wp-content/uploads/2015/09/Mark... · 2015-09-28 · History A Mark Schemes for the Units January 2009 H106/H506/MS/R/09J

F961-02 Mark Scheme January 2009

18. How effectively did British governments deal with the problem of Northern Ireland to 1994? [50]

Focus: An assessment of policy in a key area. No set answer is looked for but candidates will need to address the question. The growing unrest and disturbances, at first in Northern Ireland and then on the mainland would suggest that policy towards Ireland was not successful. By 1960 there was resentment against the Northern Ireland government by about 1/3 of the population, by 1968 there was serious rioting and by 1969 rioting by the Nationalists became so serious that RUC was finding it difficult to keep control. Armed Unionist groups were attacking Nationalist areas in Belfast and Derry. This deterioration can be supported by the need to deploy an increasing number of troops on the streets of Northern Ireland, initially to protect Catholics, who then turned on them, highlights the failings and belief among Nationalists that the army were defending Unionists. There may be reference to events such as ‘Bloody Sunday’ and its consequences, particularly Direct Rule. This can be further developed by reference to the growing support for the various paramilitary organisations and support for Sinn Fein. The use of Internment without trial may be considered as it resulted in the arrest of large numbers and resulted in increased support for the IRA. There may be reference to the attempts at talks with various Nationalist groups, for example Whitelaw in 1972 which was a failure. The Sunningdale Agreement and its failure may also be discussed. The Prevention of Terrorism Act of 1975 may also be considered as may the success of direct rule as it resulted in the reorganisation of IRA into cells, which the British army could not break. Thatcher governments faced the problem of hunger strikers, which also led to an increase in support for Sinn Fein. There were attempts at ‘rolling devolution’, but more successful were the increased links between London and Dublin and this culminated in the Hillsborough Agreement, which did have long term consequences. The increased amount of terrorist activity on the mainland during the 1980s also suggests that government policy did not work and there may be reference to events such as the murder of Airey Neave, the murder of Mountbatten, the Hyde Park bombs or the bombing of the Grand Hotel in Brighton, which came close to wiping out the Conservative leadership. There may also be reference to the murder of various ministers as evidence that the democratic process was not working. At the end of the period there was a continuation of political violence, continued political success for Sinn Fein, the existence of paramilitary groups on both sides and limited support for a lasting peace involving both sides of the community; it was only with the 1993 Downing Street Declaration that success appeared possible and this can be seen with the IRA and Loyalist declarations of ceasefire in 1994 . Therefore it might be concluded that the more successful policies were towards the end of the period. Alternative explanations are possible and examiners must be open to alternative approaches. If in doubt, consult your Team Leader.

36

Page 40: History A - The Bicester Schoolthebicesterschool.org.uk/wp-content/uploads/2015/09/Mark... · 2015-09-28 · History A Mark Schemes for the Units January 2009 H106/H506/MS/R/09J

F962-01 Mark Scheme January 2009

37

F962-01 - European and World History Period Studies

Question Number

Answer Max Mark

1

The Crusades and Crusader states 1095-1192 To what extent do the weaknesses of its enemies explain the success of the First Crusade (1096-99)? No specific answer is being looked for. However, candidates should deal with the issue of weaknesses of its enemies even if they wish to argue other factors were more important in explaining the success of the First Crusade. In relation to weaknesses candidates may discuss: the divided nature of Islam both at he general level (Seljuk/Fatimid, Sunni, Shi-ite) and more locally (rivalries between Kilij Arslan and the Danishmends, Aleppo and Damascus etc.); underestimation of the threat posed by the Crusade (e.g. Arslan away fighting the Danishmends); weaknesses compared to the Crusaders in some aspects of the military, and so on. Such discussion needs to be balanced and linked to other factors such as the relative unity and strengths of the Crusaders, their religious motivation and determination, the aid given by Alexius Comnenus, military leadership shown by Crusade leaders such as Bohemond, the fear the Crusade forces inspired after Antioch and so forth.

[50] 2 Assess the view that the ability of its rulers was the main reason

the Kingdom of Jerusalem was able to survive during the twelfth century. No specific answer is looked for, but candidates do need to ensure that they deal with the given factor adequately even if they wish to argue that other factors were more important. In relation to the ability of its rulers, candidates may point to the abilities of both Baldwin I and Baldwin II who did much to first establish the Kingdom of Jerusalem and then to hold onto it in the first thirty years of its existence, pointing both to their qualities of military leadership in the battles against the Fatimids and the Seljuks, the ways in which they ran the state, and conducted relations with barons and other princes of the Crusader states. Indeed candidates may point to the ability of all rulers up to and including Baldwin IV. Such discussion needs to be balanced against other factors. These include the relative disunity of their real and potential enemies, the occasional/limited/sporadic support from the West, the increasingly important role of the military orders, the development of defensive fortifications, good fortune and so on.

[50]

Page 41: History A - The Bicester Schoolthebicesterschool.org.uk/wp-content/uploads/2015/09/Mark... · 2015-09-28 · History A Mark Schemes for the Units January 2009 H106/H506/MS/R/09J

F962-01 Mark Scheme January 2009

Question Number

Answer Max Mark

3 Assess the reasons for Saladin’s success against the Crusader states in the 1180s. No specific answer is being looked for. Candidates need to identify, explain and assess a range of reasons. They may discuss some of the following: how Saladin operated from a strong power base and his consolidation of power in Egypt and Syria in the 1170s and earl 1180s; the use of the concept of jihad to unite Muslims in an attack on the Crusader States; the size of the forces Saladin was able to gather to attack the Crusader States in the mid 1180s (and in 1187 in particular); the succession crisis and the factional in-fighting that beleaguered the Kingdom of Jerusalem as Baldwin IV’s leprosy progressed to his death; the actions of Reynald of Chatillon in provoking Saladin; the errors made by the crusader army in 1187 when Saladin besieged Tiberias; the disaster at Hattin which left the kingdom defenceless; Saladin’s reduction of crusader castles and taking of ports and strongholds; the taking of Jerusalem. Candidates may well argue that it was a combination of Saladin’s strength and Crusader weakness that accounts for his successes.

[50]

4

The Renaissance from c.1400-c.1550 Assess the reasons why the Renaissance began in fifteenth-century Italy. No specific answer is looked for, but candidates do need to assess a range of reasons typically by evaluating relative importance and/or by analyzing linkages between different reasons. Candidates may discuss a range of reasons including: the political situation of Italy c.1400 (many city states, rivalries etc.), the nature of individual states with their city/urban base and controlling families, guilds etc, relative wealth, the existence of classical remains, the contacts with Constantinople and the Levant (and the exodus of Greek scholars as the Ottomans advanced), the development of humanism and the revival of classical learning. For example, candidates may argue that the relative independence and wealth of city states engendered a rivalry that found expression in art and architecture, that the presence of classical remains provided stimulus for artistic development, and that the interest in learning and classical literature enabled scholars and artists to draw on classical ideas.

[50]

38

Page 42: History A - The Bicester Schoolthebicesterschool.org.uk/wp-content/uploads/2015/09/Mark... · 2015-09-28 · History A Mark Schemes for the Units January 2009 H106/H506/MS/R/09J

F962-01 Mark Scheme January 2009

Question Number

Answer Max Mark

5 To what extent did the patronage of princes and nobles influence Renaissance art and architecture? No specific answer is looked for, but candidates do need to deal fully with the role of the patronage of princes and nobles even if they wish to argue other factors were more important. In relation to the patronage of nobles and princes we may get discussion of their influence on subject matter and themes, the sponsorship of art and architecture, the employment of particular artists and so on. There may be discussion of the role played by individual nobles/princes such as (Florence) Palla and Filippo Strozzi, Cosimo and Lorenzo di Medici, Francesco Sassetti, (Mantua) Ludovico and Francesco Gonzaga, Isabella d’Este and (Urbino) Federigo da Montefeltro. In relation to such figures there may discussion of the particular developments/influences they, through the artists they patronized, are associated with. Such discussion may be balanced against the role of guilds (e.g. in Florence), the civic authorities (e.g. in Florence after Savonarola’s fall), the Papacy (although individual popes may be treated as ‘princes’) and the Church, the Council of Ten in Venice. Other factors may also be brought into play, such as the influence of classical examples, the individual genius of particular artists, new techniques and so forth.

[50] 6 Assess the influence of classical literature on the ideas of Italian

writers such as Ficino, Mirandola and Machiavelli. (You may refer to other Italian writers of this period as further examples.) No specific answer is looked for, but candidates do need to deal fully with the role of classical literature even if they wish to argue that its influence has been overstated. Candidates may or may not refer to the writers included in the question. Candidates may argue that one of the key foundations of the Renaissance was the renewed and widened study of Latin and Greek texts. The study of Latin texts especially had also been a feature of Medieval scholarship and some candidates may argue for the degree of continuity here. However, the influx of Greek scholars and texts (especially after the fall of Constantinople) clearly had great influence on both what was studied, how it was studied and the results of study. In discussion of Ficino candidates may refer to the central importance of Plato and the development of Neo-Platonism and the idea of Platonic love. In relation to Mirandola, candidates may to his ‘Oration on the Dignity of Man’ and its focus on the centrality of human capacity and perspective and point to the influence of Aristotle and Plato. In relation to Macchiavelli we may have discussion of the influence of Aristotle and, especially, Livy. Candidates may argue that whilst the study of classical literature was important, there was no lock, stock and barrel importation of classical ideas. Writers and scholars were concerned to reconcile ‘philosophy’ to Christianity and the reality of the early modern world. The ‘rediscovery’ of classical literature provided them with a perspective.

[50]

39

Page 43: History A - The Bicester Schoolthebicesterschool.org.uk/wp-content/uploads/2015/09/Mark... · 2015-09-28 · History A Mark Schemes for the Units January 2009 H106/H506/MS/R/09J

F962-01 Mark Scheme January 2009

Question Number

Answer Max Mark

7

Exploration and Discovery c.1445-c. 1545 How important were Spanish monarchs to the success of Spanish overseas exploration during this period? No specific answer is looked for. Candidates must deal with the given factor even if they wish to argue that other factors were as or more important. Candidates should recognize that the focus is on overseas exploration rather than empire-building, although there is some overlap. Candidates are likely to focus on the roles of Ferdinand and Isabella, and Charles I. They may well suggest that royal patronage was vital to the success of Spanish overseas exploration and discuss the voyages of Columbus especially in relation to this. They may also discuss the acquisition of the Canaries by treaty with Portugal in 1479 (an important staging post for voyages across the Atlantic). However, candidates may balance this by noting Isabella’s rejection of Columbus’ request for support in 1486. They should also balance any discussion of royal patronage against other factors such as technological developments (caravel, ability to calculate latitude), geographic position, strong economic motivations, the role of individuals (Columbus, Balboa, Vespucci, Magellan etc.), the nature of Spanish nobility (aggressive bravery and adventurism borne of the Reconquista).

[50] 8 How important were individuals such as da Gama and Cabral in

the development of the Portuguese Empire? No specific answer is being looked for. Candidates may assess the importance of the role of individuals in relation to the role of other factors. They need to come to a reasoned judgement about ‘How important?’ in order to score well. Candidates are likely to consider both Cabral and da Gama, but may well also consider leaders of other expeditions (Diaz, Covilha etc.). In relation to Cabral, candidates are likely to consider the acquisition of Brazil (1500), whilst in relation to da Gama (voyages of 1498, 1502 and 1524) they are likely to discuss the establishment of interests along the coast of Africa and India. Such discussions are likely to be set in the context of other factors, not least royal patronage and the desire of kings like John II and Manuel to set up trading posts and capture the spice trade (there may be reference to sea battles (e.g. Diu in 1509) and seizing of various staging posts (e.g. Ormuz in 1515) . There may also be some discussion of the earlier establishment of control in the Azores, Cape Verde Islands and Madeira. Candidates may well argue that individuals like da Gama were the trailblazers that began the process of empire-building, rather than the empire-builders themselves.

[50]

40

Page 44: History A - The Bicester Schoolthebicesterschool.org.uk/wp-content/uploads/2015/09/Mark... · 2015-09-28 · History A Mark Schemes for the Units January 2009 H106/H506/MS/R/09J

F962-01 Mark Scheme January 2009

Question Number

Answer Max Mark

9 To what extent was the quest for gold and silver the main reason for the development of the Spanish Empire in America? No specific answer is looked for. Candidates must deal with the given factor even if they wish to argue that other factors were as or more important. In relation to the given factor, candidates may discuss the significance of Columbus returning from his first expedition with gold along with the Carib natives. Cortes was motivated at least in part by the rumours of a vast wealthy empire in the heart of Mexico, and Pizarro plundered the Incan empire of Peru. The hope of (easy) riches was clearly a strong motivator for those who forged the Spanish Empire in the Americas. The discovery of silver and the need to secure it was also a prime reason for the development of empire in Peru. However, such considerations need to be balanced against others such as land for colonization (from the start settlers set out from Spain and the granting/seizing of large encomiendas encouraged such settlement. There was the wealth to be made from cochineal, sugar, cocoa, cotton as well as food staples. Another reason can be found in religion. Isabella was keen that natives were converted from the start and received papal encouragement in 1493 – by 1536 there were over 5 million converts in New Spain (Mexico). Nevertheless candidates may well argue that the pursuit of gold and silver was the driving force behind the development of a Spanish Empire.

[50]

10

Spain 1469 – 1556 To what extent was Isabella’s marriage to Ferdinand of Aragon the main reason for the consolidation of her rule in Castile to 1479? No specific answer is looked for. Candidates must deal with the given factor even if they wish to argue that other factors were as or more important. Candidates may tackle this question by setting out the problems facing Isabella at the start of her reign, such as the rival claims of Joanna supported by Alfonso of Portugal, the problems posed by aristocratic factions and the cities, the parlous state of royal finances and so forth. In relation to the marriage they may discuss the significance of the terms of the marriage contract (Ferdinand had to live in Castile, acknowledge Isabella as the dominant partner, supply her with 100,000 florins and 4000 troops). The significance of Ferdinand’s contribution may be discussed in particular in relation to the defeat of Alfonso at the battle of Toro 1476. The significance of the marriage should be set in the context of other factors such as Isabella’s personal strengths and talents, relations with the nobles and cities, the use of corregidores (begun in Henry IV’s reign, the institution of the Santa Hermandad in 1476, the grant of 162 million maravedis by the Cortes in 1476 and so forth.

[50]

41

Page 45: History A - The Bicester Schoolthebicesterschool.org.uk/wp-content/uploads/2015/09/Mark... · 2015-09-28 · History A Mark Schemes for the Units January 2009 H106/H506/MS/R/09J

F962-01 Mark Scheme January 2009

Question Number

Answer Max Mark

11 How successful were Ferdinand and Isabella in dealing with the problems posed by the nobles during their reigns? No specific answer is looked for. Candidates may seek to test the degree of success, for instance, against aims, outcomes and the seriousness/nature of the problem. Candidates may discuss the power of the aristocracy and the influence of powerful families like the Mendozas before Ferdinand and Isabella came to power. They may also point to the deals done with and promises made to grandees to win their support during the war of succession. They may point to the increasing influence of letrados (lawyers) in royal councils (and lessening influence of nobles), but recognize the role of the aristocracy in provincial government (as governors and viceroys). They may also point to Isabella’s limited success in dealing with the land question, discussing, for example, the decision of the 1480 Toledo Cortes agreeing to the recovering of lands lost since 1474 (whilst accepting those lost before that date). There may also be discussion of the War v. Granada in this context (nobles could be rewarded with lands from conquered territory. There may also be discussion of royal attempts to gain control of military orders and the noble unrest accompanying the succession crisis following the death of Isabella. Candidates may well argue that the price of royal control at the centre was acceptance of aristocratic control and influence at a provincial level and that in most areas success was at best mixed.

[50]

12 To what extent were the problems Charles I faced during his reign as King of Spain of his own making? No specific answer is looked for. Candidates who have not read the question carefully may be tempted to deal only with the early years of the reign. However, to score highly candidates must give reasonable coverage to the whole reign. Candidates will need to identify a number of problems that Charles faced (such as finances, nobility, towns, rebellion, unrest and revolt, dealing with the Americas). Candidates may well argue that at least in the early part of his reign many of the problems Charles I faced were to a significant degree of his own making – the use of foreign advisers, his foreign status, his absenteeism,, his use of Spanish taxes in support of his wider ambitions (to become Holy Roman Emperor, in his wars). However, others may argue that many of the problems he faced were of a longer term or intractable nature – problems of finance, control of nobility and towns and so on.

[50]

42

Page 46: History A - The Bicester Schoolthebicesterschool.org.uk/wp-content/uploads/2015/09/Mark... · 2015-09-28 · History A Mark Schemes for the Units January 2009 H106/H506/MS/R/09J

F962-01 Mark Scheme January 2009

Question Number

Answer Max Mark

13

Charles V: International Relations and the Holy Roman Empire 1519-1559 How successful was Charles V in dealing with the princes in the Holy Roman Empire? No specific answer is looked for. Candidates may discuss a range of issues and developments. They may discuss the circumstances of his election as Holy Roman Emperor and the nature of the Empire and his authority within it, the role of Diets, the limitations placed on him by virtue of his other responsibilities (e.g. as King of Spain), and the impact of the Reformation. Candidates will also need to focus on the relationship and dealings with the princes and Electors to assess his success. They may discuss the Diet of Augsburg and Charles alienation of the protestant princes, the significance of the Schmalkaldic League and events like the Battle of Mühlberg and Charles’ failure at the Diet of Augsburg, his alienation of the Electors over the succession (Augsburg agreement) and the revolt of the princes and the Diet of Augsburg (1555).

[50]

14 Assess the reasons for the spread of Lutheranism in the Holy Roman Empire. No specific answer is looked for, but candidates do need to assess a range of reasons typically by evaluating relative importance and/or by analyzing linkages between different reasons. Candidates may discuss some of the following: the role of Martin Luther and his pamphlets, the power of the ideas associated with Lutheranism (sola scriptura, sola fide, priesthood of all believers, etc.), the Indulgences Controversy and the reputation of the Catholic Church and Papacy in Germany, the background of humanism, the role of the printing presses, the role of princely protectors, like Frederick of Saxony, the significance of the Diet of Worms, the role of popular support in towns and cities, links with peasant unrest, lack of decisive action by Charles V in 1520s, the Schmalkaldic League. Candidates may argue that there was a combination of circumstances (Papal exactions, princely concern for their privileges, weaknesses of Charles V’s actions in Germany -distractions elsewhere, lack of power-, the printing press) in the Holy Roman Empire that allowed the ideas of Luther, powerful as they were, to gain public currency and many may stress the crucial roles played by lack of effective action against Luther by Charles V combined with the protection of Frederick of Saxony.

[50]

43

Page 47: History A - The Bicester Schoolthebicesterschool.org.uk/wp-content/uploads/2015/09/Mark... · 2015-09-28 · History A Mark Schemes for the Units January 2009 H106/H506/MS/R/09J

F962-01 Mark Scheme January 2009

Question Number

Answer Max Mark

15 To what extent was Charles V’s policy towards France a failure? No specific answer is called for. The assessment of ‘failure’ may take into account aims, outcomes and context, for example. Candidates may discuss the strategic and political situation in 1519 and Charles’ aims, referring to the extent of Charles’ territories, the strategic importance of Italy to the physical linkage of these territories, the history of warfare and rivalry with France, Charles’ desire to recover Burgundy and so forth. Candidates may discuss developments over time to explain relative success and failure, pointing to the ups and downs of the Habsburg-Valois rivalry in the 1520s (Pavia, Madrid, Cognac, sack of Rome, Landriano and Cambrai) and Charles strong position at the end of the decade, the events of the 1530s and 40s to Charles’ triumph’ in the Peace of Crèpy (1544) and the renewal of war with Henry II and the failure of the siege of Metz. Candidates may legitimately refer to the Peace of Câteau-Cambrèsis (1559) as a way of discussing overall success or failure.

[50]

16

Philip II, Spain and the Netherlands, 1556-1609 Assess the strengths and weaknesses of Spain at the time of Philip II’s accession. No specific answer is called for. Strengths and weaknesses will need to be identified, explained and assessed to score highly. Assessment may take the form of analysis of relative strengths and weaknesses and/or how strengths/weaknesses combine or link together, and/or an overall judgement of strength/weakness. Candidates may discuss the relative strength/weakness of some of the following: the extent and diversity of the monarchia; finances; economy; government and administration; religion; the Americas; position of the nobility; the Cortes; communications. Candidates may point to finances and the power of the nobility as areas of relative weakness and Castile and religion as areas of relative strength, for example – but in each case the key will be the assessment of relative strength of weakness.

[50]

44

Page 48: History A - The Bicester Schoolthebicesterschool.org.uk/wp-content/uploads/2015/09/Mark... · 2015-09-28 · History A Mark Schemes for the Units January 2009 H106/H506/MS/R/09J

F962-01 Mark Scheme January 2009

Question Number

Answer Max Mark

17 How successful was Philip II in his policies towards France and England? No specific answer is looked for. Success may be assessed in terms of aims, outcomes and context; there is no need for balance of treatment between France and England, though candidates may well discuss the linkages between policies towards both. There has been dispute over Philip’s ‘aims’ although there appears to be some consensus that overall Philip’s aims were largely defensive (to hold on to what he had) and candidates may assess the success of his policies toward France and England largely in these terms. They may also assess success in the context of the security of Philip’s possessions in the Netherlands and/or in terms of religion. Candidates may discuss some of the following: war with France and the Treaty of Câteau-Cambrèsis, marriage to Mary of England and pursuit of ‘friendly’ relations with Elizabeth in the 1560s, the issues arising from the Revolt of the Netherlands and tensions over the Americas, Philip’s involvement in France via the Catholic League in the 1580s, English intervention in the Dutch Revolt and the outbreak of war with England (the Armadas), intervention in France in 1590 and 1592 and war in 1595.

[50]

18 How important was foreign support to the success of the Dutch revolt? No specific answer is looked for. Candidates must deal with the given factor even if they wish to argue that other factors were as or more important. Candidates may argue that, whilst significant and an important contributory factor in the success of the revolt in the 1580s and 90s, foreign intervention (principally by England, but also by France) was not decisive. In relation to France (Duke of Anjou) they may argue that it was of little significance, but that despite the limitations of Leicester’s actions in the 1580s, the presence of 7500 troops and annual subsidies did much to sustain the Dutch rebels in the 1590s. They may argue that other factors were more important, pointing to the leadership of William of Orange and Maurice of Nassau, the development of an effective fighting force and tactics, the diversion of Spain’s forces/priorities at crucial times (such as the Armada, 1588 and intervention in France), and Spanish financial problems.

[50]

Paper Total [100]

45

Page 49: History A - The Bicester Schoolthebicesterschool.org.uk/wp-content/uploads/2015/09/Mark... · 2015-09-28 · History A Mark Schemes for the Units January 2009 H106/H506/MS/R/09J

F962-02 Mark Scheme January 2009

46

F962-02 - European and World History Period Studies Option B: Modern 1795-2003

Question Number

Answer Max Mark

1

Napoleon, France and Europe 1795-1815 Assess the reasons for Napoleon’s rise to power in France to 1799. No specific answer is looked for. Candidates will need to identify, explain and assess a range of reasons. Candidates may argue that much of the explanation lies in the talents and opportunism of Napoleon himself, pointing to his rise as an officer and general during the 1790s, from the siege of Toulon in 1793, through his Italian campaigns in 1796-7 and his efforts in Egypt in 1798. They may also point to his taking advantage of opportunities as they arose, including his relationship with Barras and marriage to Josephine de Beauharnais. Many will seek to place such factors in the context of the developments of the revolutionary years, particularly the increasing importance of military victory to the stability of the revolution, the significance of Napoleon’s victory and peacemaking in Italy, and the weaknesses and increasing reliance on the army of the Directory. There may be some concentration on the developments of 1799 and the actions of individuals like Sieyes and Lucien Bonaparte before and during the coup of Brumaire.

[50]

2 To what extent were Napoleon’s domestic reforms during the Consulate (1799-1804) shaped by Revolutionary principles? No specific answer is looked for. Candidates may test the reforms of the Consulate against the revolutionary principles of ‘liberty’ and ‘equality’ but may also refer to ‘property’ and ‘popular sovereignty’. Candidates may use these ideas to assess the reforms of the Consulate – the Constitutions, religious changes, legal reforms, education reforms, and so on. In relation to ‘liberty’ candidates may discuss freedom of religion/religious toleration, but also controls over the press/freedom of expression and limitations on freedom of movement (such as the livret). In relation to ‘equality’ candidates may discuss legal equality and in relation to ‘property’ consider the protection afforded to those who had purchased Church and émigré lands during the 1790s. In relation popular sovereignty there may be discussion of the real degree of this in the constitutions. Candidates may well argue that Napoleon followed the principles either insofar as they contributed to wining support or insofar as they were consistent with his hold on power.

[50]

Page 50: History A - The Bicester Schoolthebicesterschool.org.uk/wp-content/uploads/2015/09/Mark... · 2015-09-28 · History A Mark Schemes for the Units January 2009 H106/H506/MS/R/09J

F962-02 Mark Scheme January 2009

Question Number

Answer Max Mark

3 To what extent did Napoleon’s treatment of conquered territory and satellite states bring benefits to them? No specific answer is looked for. This question is about the impact of Napoleon on Europe outside France that France directly or indirectly controlled. Candidates may draw distinctions between different areas and periods of influence to highlight different impacts. For example, they may point to the benefits of the changes brought about in the former Holy Roman Empire/Low Countries/Italian peninsula – the extent to which the ‘benefits’ of the French Revolution were exported to these areas (encompassed, for example, in the Code Napoleon and the reorganisation of states and government). Such benefits may be contrasted with the costs (and candidates may well argue that they outweigh them)– the subordination of these areas to the needs of France and Napoleon, taxation, conscription, the antipathy of those who lost out from government/administrative changes, the impact of the Continental blockade and system (such as the damage to the Italian silk industry to protect that of France), the use of the Empire and satellites states as a ’spoils’ system to reward Napoleon’s marshals (as in the Grand Duchy of Warsaw).

[50]

4

Monarchy, Republic and Empire: France 1814-1870 How far did Louis XVIII’s policies change during his reign? No specific answer is looked for. Candidates may well argue that the murder of the Duc de Berry in 1820 marks a watershed in the policies of Louis XVIII, contrasting the relatively ‘liberal’ policies of the period prior to 1820 under de Richelieu and to an extent Decazes with the more reactionary policies under Villèle. In relation to the question there may be discussion of the nature of The Charter, the impact of the Hundred Days, the ‘White Terror’ and Chambre Introuvable, legislation in relation to rights to vote, army reform and press freedom, the payment of the indemnity and the Congress of Aix-la-Chapelle (1818), Ultras, restoration of Ferdinand VII in Spain (1823) and so on. There may also be discussion of Louis XVIII’s personality and beliefs and their impact on policy and some may argue that despite the apparent success of the Ultras after 1820, Louis XVIII, however reluctant a ‘constitutional’ monarch kept the ultras at arms’ length.

[50]

47

Page 51: History A - The Bicester Schoolthebicesterschool.org.uk/wp-content/uploads/2015/09/Mark... · 2015-09-28 · History A Mark Schemes for the Units January 2009 H106/H506/MS/R/09J

F962-02 Mark Scheme January 2009

Question Number

Answer Max Mark

5 Assess the reasons why Charles X was overthrown in 1830. No specific answer is looked for. Candidates will need to identify and explain a number of reasons and evaluate their relative importance and/or links to access the higher bands. Candidates may identify and explain reasons in a number of ways. For example, some may distinguish between long term (such as the revolutionary heritage), short term (such as Charles X’s policies towards religion) and immediate causes (such as the Ordinances of St Cloud). Candidates may also discuss reasons such as the legacy of the revolutionary and Napoleonic era, the Charter and the attitudes of Charles X and liberals towards it, the range and nature of Charles X’s political and religious policies, the impact of economic problems, the extent and nature of opposition, the circumstances of 1830, and so on. Candidates may ague that Charles X largely brought his downfall upon himself or they may stress the difficulties of his position or that the longer term influences of the revolutionary years made further revolution/political upheaval more likely. Many may well argue that it was a combination of factors that brought about the events of 1830 – to be successful such an argument needs to be underpinned by effective analysis that may, for example, distinguish between direct and indirect causes, or contributory and necessary reasons.

[50]

6 How far did Napoleon III achieve his aims in foreign policy? No specific answer is looked for. Candidates will need to identify Napoleon’s aims in order to discuss his achievements. In relation to aims, candidates may well refer to Napoleon III’s claim ‘the Empire means Peace’, the desire to overturn the Vienna Settlement, the desire to reclaim France’s ‘natural frontiers’ along the Alps and the Rhine, sympathy for the causes of ‘Poland’ and ‘Italy’, the more general desire to achieve ‘gloire’ and national greatness. More specifically, some may suggest that Napoleon’s policy to some degree was based on a desire to break up the Holy Alliance and maintain good relations with Britain. Whatever aims are identified they need to be assessed in relation to some or all of the following: the Crimean War and its aftermath, Plombières and the Italian war of 1859 and its aftermath, the Mexican adventure, involvement in Austro-Prussian relations and the war of 1866, the attempt to secure ‘compensation’, the Franco-Prussian War. A line of argument may well be that after some initial successes (Crimea and arguably the achievement of Nice and Savoy) the general pattern was one of humiliation and failure resulting ultimately in the end of the Empire.

[50]

48

Page 52: History A - The Bicester Schoolthebicesterschool.org.uk/wp-content/uploads/2015/09/Mark... · 2015-09-28 · History A Mark Schemes for the Units January 2009 H106/H506/MS/R/09J

F962-02 Mark Scheme January 2009

Question Number

Answer Max Mark

7

The USA in the 19th Century: Westward Expansion and Civil War 1803-c.1890 To what extent was cattle-farming the most important factor in opening up the West? No specific answer is looked for. Candidates need to address the given factor adequately even if they wish to argue that other factors were more important. In relation to cattle, candidates may discuss the significance of the cattle drives from the south (Texas) to railheads such as Abilene, Dodge City and Miles City to enable cattle to be transported on to the populous north east in the 1860s.They may also discuss the development of cattle ranching from the 1870s enabled, at least in part, by the invention of barbed wire. Such discussion needs to be balanced against discussion of other factors in opening up the West, such as the role of fur-trappers and pioneers (such as those that followed the ‘Oregon Trail’, the stimulus given by Federal-sponsored explorers such as Lewis and Clark, by expeditionary forces such as Fremont’s, the discovery of gold and silver that led to various rushes, the development of railways, deals with and destruction of Native Americans, Federal encouragement such as the Homestead Act, the desire for religious freedom (the Mormons of Utah) and so on.

[50]

8 Assess the reasons why it proved impossible to secure peaceful relations between White and Native Americans for most of this period. No specific answer is looked for. Candidates will need to identify and explain a number of reasons and evaluate their relative importance and/or links to access the higher bands. Candidates may discuss reasons such as: the mutual misunderstanding arising from different cultures, the incompatibility of nomad and settler cultures, the power of interest groups in Washington in undermining agreements and the distance/poor communication/lack of knowledge between policy-makers in Washington and the situation in the West, corruption of government Indian Agencies, the constant westward pressure by a growing white population, the impact of minerals finds and the subsequent ‘rushes’, the destruction of the buffalo on the Plains as they were settled and railways pushed through, the actions of individual commanders in the field, the desperation of the Native Americans as their way of life disappeared and so forth. In discussing some of the above candidates may refer to some of the following developments: the impact of the Lousiana Purchase and the Tecumseh Confederacy, the First Seminole War, Andrew Jackson and the Indian Removal Act of 1830, the Black Hawk War, Second Seminole War, Reservations and the ‘trail of tears’, the Laramie Treaty, Sand Creek massacre, the Fetterman massacre, the Red River War, Little Big Horn, Americanisation and so on.

[50]

49

Page 53: History A - The Bicester Schoolthebicesterschool.org.uk/wp-content/uploads/2015/09/Mark... · 2015-09-28 · History A Mark Schemes for the Units January 2009 H106/H506/MS/R/09J

F962-02 Mark Scheme January 2009

Question Number

Answer Max Mark

9 Compare Lincoln and Davis as war leaders. No specific answer is looked for. Candidates, however, do need to compare the two leaders in order to score well. Candidates may argue that overall Lincoln was a better war leader than Davis and compare their qualities in various aspects of leadership, such as: appointment of ministers and management of government, appointment of and relationship with commanders, ability to inspire the people and read their mood, political judgement, the use of executive powers to pursue the war effort effectively, decision-making. For example, candidates may praise Davis’ choice of Lee and trust of him and criticize Lincoln’s early appointments such as McClellan. On the other hand, Lincoln did not interfere closely in military affairs, whilst Davis, as a military man, sometimes did. Lincoln’s oratory and judgement of the public mood may be compared favourably with Davis’ as may his relations with and appointment of ministers. Such comparisons may be set in the context that each leader found himself in. Lincoln inherited and established government and administration, whilst Davis and the Confederacy had to create one. Davis was hampered by the powers of the states (after all ‘States’ rights’ was a key reason for war).

[50]

10

Peace and War: International Relations c. 1890-1941 Assess the reasons why the stalemate on the Western Front was finally broken in 1918. No specific answer is required. Candidates should discuss a number of reasons and assess their relative significance and/or the linkages between them. Candidates should be aware of both the German Spring Offensive as well as the Entente advance after August 1918. They may discuss some of the following factors: the development of new weapons and technology, such as the tank and calibrated artillery; the development of new tactics such as whirlwind artillery bombardments and new approaches to infantry advances; the collapse of the Eastern Front with the withdrawal of Russia which enabled the Germans to concentrate on the Western Front and launch an offensive; the entry of America into the war on the Entente side; the long term effects of the war of attrition on the Western Front and the honing of the British army into the most effective armed force on the western front. For each factor discussed candidates may discuss its strengths and limitations as an explanatory factor and its links with other factors.

[50]

50

Page 54: History A - The Bicester Schoolthebicesterschool.org.uk/wp-content/uploads/2015/09/Mark... · 2015-09-28 · History A Mark Schemes for the Units January 2009 H106/H506/MS/R/09J

F962-02 Mark Scheme January 2009

Question Number

Answer Max Mark

11 To what extent was the Paris peace settlement shaped by the principle of self-determination? No specific answer is being looked for. Candidates should be aware that the Paris Peace Settlement refers to all the peace treaties concluded at the end of the First World War and not just the Treaty of Versailles. Better candidates, therefore, will seek to assess how far and in what ways the principle was applied in each of the treaties with the defeated powers and come to an overall judgement in relation to the peace settlement as a whole. They may argue that in relation to the Treaty of Versailles the principle was largely put to one side; they may argue that the forbidding of Anschluss was a specific denial of the principle as was the seizure of West Prussia for Poland and that the plebiscites held in Schleswig and Silesia were only allowed because these areas were not deemed crucial to other issues. In relation to the treaties with Austria and Hungary they may argue that more attention was paid to the principle, although the qualification may be that in reality the peacemakers were trying to rationalize a fait accompli. In relation to Turkey candidates may argue that little more than lip-service was paid to the principle as the Ottoman Empire was broken up into mandates controlled by Britain and France.

[50]

12 Assess the reasons for the outbreak of war in Asia and the Pacific to 1941. No specific answer is looked for. Candidates should discuss a number of reasons and assess their relative significance and/or the linkages between them. The focus of this question is the outbreak of war in Asia and the Pacific; events and developments in Europe are only relevant insofar as they have a bearing on the outbreak of war in Asia. Candidates may well focus on the ambitions of Japan in China and South East Asia more generally. They may point to the aggressive foreign policy pursued partly as a consequence of the Depression and the growth of nationalism. There may be discussion of Manchuria, the outbreak of the Sino-Japanese war, alliance with Germany and Italy, the Co-Prosperity sphere and the decision to attack Pearl Harbour. Such discussion needs to be balanced against other factors such as the attitudes of the USA, Britain and France, the weakness and failure of the League of Nations, the distractions of events in Europe, and the relative weakness of and internal divisions in China. Candidates must ‘assess’ the different reasons identified and come to argued judgments on relative importance and or linkages to score highly.

[50]

51

Page 55: History A - The Bicester Schoolthebicesterschool.org.uk/wp-content/uploads/2015/09/Mark... · 2015-09-28 · History A Mark Schemes for the Units January 2009 H106/H506/MS/R/09J

F962-02 Mark Scheme January 2009

Question Number

Answer Max Mark

13

From Autocracy to Communism: Russia 1894-1941 Assess the reasons for the 1905 revolution in Russia. No specific answer is looked for. Candidates should discuss a number of reasons and assess their relative significance and/or the linkages between them. Candidates may use their knowledge/understanding of the nature of the ‘revolution’ to show how the reasons identified link to the revolutionary events. They may discuss some of the following areas of reasoning: long term developments in the countryside and reasons for growing unrest to 1905; developments in industry and the problems in towns/factories etc (context of economic depression since c. 1900); Nicholas II, the nature of Tsarism, repression and the growth of opposition/political parties; the impact of the Russo-Japanese War; and the role and impact of ‘Bloody Sunday’. Candidates may relate longer term stresses and strains to the shorter term factors and the ‘immediate’ cause (‘Bloody Sunday’).

[50]

14 ‘The Bolsheviks won the Civil War mainly because of the weaknesses of the Whites.’ How far do you agree with this view? No specific answer is looked for. Candidates must deal with the given factor even if they wish to argue that other factors were as or more important. In relation to the given factor, candidates may discuss the patchwork of opposition to the Bolsheviks, the lack of coordination of efforts, the lack of clear or agreed aims, the resistance of the peasants (and Green forces), the strategic difficulties, the quality of leadership and size of armies, and the ambivalent attitude of the Entente powers despite their presence and supply of arms. Such discussion needs to be balanced against other factors which may well focus on the relative advantages and strengths of the Bolshevik forces: the central strategic position and control of key transport links and industries, undivided leadership and aims, the relative preference of the peasantry for the ‘Reds’ over the ‘Whites’, the organization and leadership of Trotsky, the quality of generalship relative to the Whites, the size of the Red Army and so on.

[50]

15 Assess the reasons why Stalin was able to rise to power in Russia after the death of Lenin. No specific answer is looked for. Candidates should discuss a number of reasons and assess their relative significance and/or the linkages between them. Candidates may discuss the factors specifically associated with Stalin: his position in the Communist Party and the use he made of his influence and power there, his opportunism in exploiting the chances afforded by Lenin’s funeral, his exploitation of the personal and ideological differences amongst rival candidates for power, his willingness to shift position to gain advantage and so forth. Candidates may also discuss the lack of a clear line of succession on Lenin’s death and the suppression of his last testament, the strengths and weaknesses of Trotsky and other possible rivals such as Kamenev, Zinoviev and Bukharin, the divisive debates within the party over policy (‘socialism in one country’).

[50]

52

Page 56: History A - The Bicester Schoolthebicesterschool.org.uk/wp-content/uploads/2015/09/Mark... · 2015-09-28 · History A Mark Schemes for the Units January 2009 H106/H506/MS/R/09J

F962-02 Mark Scheme January 2009

Question Number

Answer Max Mark

16

Democracy and Dictatorship: Italy 1896-1943 Assess the impact of the First World War on Italy from 1915 to 1920. No specific answer is called for. Impact can be discussed in a variety of ways: candidates may choose, for instance, to examine impact in terms of casualties, morale, territorial gains (or absence of them), economic and social problems, rise of extremism, impact on liberal politics and so forth. Candidates may refer to some of the following: the progress of the war (especially the reverses in 1917, Caporetto) and the 680000 dead; problems of mobilization (over 5 million) and demobilization, morale, socialist ‘pacifism’; economic dislocation, inflation (250% 1914-18), lack of raw materials, budget deficits (expenditure three times income); the ‘mutilated victory’; D’Annunzio, extreme nationalism and Fiume; total economic breakdown, strikes and violence, growth of the ‘red menace’; weak liberal government; 1919 general election and significance.

[50]

17 Assess the reasons why Mussolini consolidated his power in the 1920s after he became Prime Minister No specific answer is looked for. Candidates should discuss a number of reasons and assess their relative significance and/or the linkages between them. Candidates may discuss reasons to do with Mussolini himself: his abilities as a propagandist and orator; his leadership of/position in the Fascist Party; his political abilities. Such discussion may be related to other reasons: the circumstances of his appointment as Prime Minister; the Acerbo Law; Matteoti murder; Aventine Secession; the outlawing of other political parties; the attitude of the King; the weaknesses of potential opposition; the attitude of the Church; press censorship; OVRA; popularity and early successes.

[50]

18 How successful were Mussolini’s economic policies from 1922 to 1940? No specific answer is looked for. Candidates may establish criteria by which ‘success’ can be judged: in relation to aims, outcomes and context, for example. Candidates may assess the success of individual policies and/or analyse success over time but they should come to a reasoned overall judgement to score highly. Candidates may evaluate success of some of the following policies: the emphasis on self-sufficiency (including the raising of import tariffs to protect domestic industry, the ‘battle for grain’, the ‘battle for the lira’ etc.); improvements in the transport system (roads and railways); state subsidies to industries like steel, the ‘battle for births’ (in relation to its economic impact); the impact of the corporative system. Candidates may well point to some (superficial?) success (draining of the Pontine Marshes, trains running on time, increase in wheat production, increase in electricity supply), etc. Such ‘successes’ may be set in the context of economic recovery that had begun before Mussolini came to power, the (early) onset of the slump, the adverse impact of the inflated value of the lira on exports and tourism, the inefficiencies encouraged by protectionism and state subsidies, the distortion of the economy by emphasis on certain products (e.g. wheat).

[50]

53

Page 57: History A - The Bicester Schoolthebicesterschool.org.uk/wp-content/uploads/2015/09/Mark... · 2015-09-28 · History A Mark Schemes for the Units January 2009 H106/H506/MS/R/09J

F962-02 Mark Scheme January 2009

Question Number

Answer Max Mark

19

The Rise of China 1911-1990 To what extent was Jiang Jieshi (Chiang Kai Shek) a successful leader of China from 1928 to 1949? No specific answer is called for. Success as a leader may be assessed in terms, for example, of aims, outcomes and the historical context. Candidates may discuss some of the following aspects: the establishment of a Nationalist state symbolised by capital at Nanking (but varied extent of authority of Nationalists in areas away from key centres); his relations with Soviet Union and Japan, later reliance on USA; his failure to deal with communists and forced mutual action against Japan after 1937; comparison with warlord years; lack of democracy and corruption, lack of support; the degree of economic progress (industry, transport) and limited social reform (education, New Life Movement, women); the failure to help peasants; his eventual defeat.

[50]

20 Assess the view that popular support was the main reason for the success of the Communists in gaining power in 1949. No specific answer is looked for. Candidates must deal with the given factor even if they wish to argue that other factors were as or more important. In relation to the given factor, candidates may discuss some of the following: Mao’s and communism’s appeal to peasantry, the promise of land reform and the role of Communists in defeat of Japan. They may also balance discussion of the extent and significance of popular support against factors such as the weaknesses and unpopularity of Nationalists, the leadership and ideas of Mao, the organization and approach of Red Army and the weaknesses and mistakes of Guomindang.

[50]

21 To what extent did political and economic policy change after the death of Chairman Mao (1976) to c.1990? No specific answer is called for. In seeking to assess the extent of change, candidates may discuss some of the following: the significance of Deng Xaioping and the trial of the Gang of Four; the ‘four cardinal principles’; the ‘four modernisations’ and limited liberalization; the four ‘Special Economic Zones’ and growth of foreign trade; developments in social policy – one-child family; the limited political reforms; the significance of the ‘democracy wall’ and the ‘fifth modernisation’; Tiananmen Square. Candidates may argue that whilst there has been significant development in the economic sphere, moves towards political liberalization have been limited and that the essential political domination and control of the communist state has remained.

[50]

54

Page 58: History A - The Bicester Schoolthebicesterschool.org.uk/wp-content/uploads/2015/09/Mark... · 2015-09-28 · History A Mark Schemes for the Units January 2009 H106/H506/MS/R/09J

F962-02 Mark Scheme January 2009

Question Number

Answer Max Mark

22

Democracy and Dictatorship in Germany 1919-1963 Assess the reasons for the survival of the Weimar Republic in the 1920s. No specific answer is looked for. Candidates will need to explain a number of reasons and assess their relative significance and linkages to score well. Candidates may discuss some of the following: the early deal with the army (Groener), how the Weimar governments managed to deal with the threats to their existence from both the extreme left and the extreme right, how the government was able to survive the crisis of 1923 (invasion of the Ruhr and hyperinflation, the significance of statesmen like Ebert and (especially) Stresemann, the importance of the Dawes Plan, American and other foreign investment, the Locarno Treaties and membership of the League of Nations, the existing support for democracy (or at least rejection of extreme solutions), and so forth. Candidates may argue that early survival depended as much on the extreme left and extreme right cancelling each other out than any positive government action (pointing to the use of Freikorps to defeat the left and workers’ strikes to defeat the right), and may go on to argue for the crucial importance of Stresemann’s statesmanship, the Dawes Plan and subsequent economic recovery.

[50]

23 How successful were the Nazis in their policy of Gleichschaltung (coordination) after 1933? No specific answer is looked for. Success may be assessed in terms of aims, outcomes and context. Candidates may interpret the term Gleichschaltung as referring to the ways in which the Nazis sought to ensure control of the state by the (forcible) ‘coordination’ of all aspects of political and social life and discuss mainly the measures and methods adopted in the early years of Nazi Rule focusing on 1933-34 in the main. They may discuss: take over of the states, laws such as those for the restoration of a professional civil service and the law against the formation of new political parties and the law for the reconstruction of the state; the concordat with the Catholic Church; the German Labour Front; the Army Oath; the use of force and arrest of political opponents; the Hitler Youth; organisations such as the German Lawyers’ Front and so on. Candidates may argue for the success of the policy, but there should be some balance recognizing, for instance, the limitations of control/influence over the Church and even the army (until 1938).

[50]

55

Page 59: History A - The Bicester Schoolthebicesterschool.org.uk/wp-content/uploads/2015/09/Mark... · 2015-09-28 · History A Mark Schemes for the Units January 2009 H106/H506/MS/R/09J

F962-02 Mark Scheme January 2009

Question Number

Answer Max Mark

24 Assess the reasons why two German states emerged from 1945 to 1949. No specific answer is looked for. Candidates will need to explain a number of reasons and assess their relative significance and linkages to score well. Candidates may discuss some of the following issues: the Yalta Conference (Germany to be divided into zones of occupation), the Potsdam Conference (reparations issues), perceptions of Britain, USA, France and the Soviet Union on the future of Germany, wider context of Cold War developments (including Soviet consolidation in Eastern Europe, Truman Doctrine and Marshall Plan, Cominform and Comecon, communist coup in Czechoslovakia), the creation of a Soviet friendly ‘Socialist Unity Party’ (SED), creation of Bizonia, Trizonia, the Moscow Conference of Foreign Ministers (1947), the London Conference, new currency, the Berlin Blockade, NATO, creation of FRG and GDR. Candidates may argue that because of Cold War tensions the creation of a divided Germany was almost inevitable.

[50]

25

The Cold War in Europe from 1945 to the 1990s Assess the reasons why relations between wartime allies broke down in 1945. No specific answer is looked for. Candidates will need to explain a number of reasons and assess their relative significance and linkages to score well. Candidates may evaluate reasons through discussion of some of the following: the position in 1945 (Yalta, Potsdam and end of war, position of Allied forces), ideological differences as context, divisions over Poland, Germany etc., Soviet actions in Eastern Europe, Churchill’s ‘Iron Curtain’ speech, Truman Doctrine and Marshall Plan, Cominform, Czechoslovakia and developments in Germany. Candidates may argue that whilst conflicting political ideologies were central, this was compounded by strategic and economic concerns and mutual suspicions and fears.

[50]

56

Page 60: History A - The Bicester Schoolthebicesterschool.org.uk/wp-content/uploads/2015/09/Mark... · 2015-09-28 · History A Mark Schemes for the Units January 2009 H106/H506/MS/R/09J

F962-02 Mark Scheme January 2009

Question Number

Answer Max Mark

26 How similar were the causes and consequences of the Hungarian uprising of 1956 and the Prague Spring of 1968? No specific answer is looked for. To score well candidates should actively compare causes and consequences, seeking to assess similarities and differences; there does not need to be balance between the treatment of causes and consequences (causes may well dominate). In relation to causes of the Hungarian uprising candidates may refer to the context of destalinisation and raised expectations following the fall of Beria and release of Gomulka and Kadar compounded by Khrushchev’s recognition of the right of satellite states to find their ‘national ways to socialism’ and improved relations with Yugoslavia, protest in Poland, Tito’s encouragement, Nagy. Reference to these may be used in comparison with some of the following relative to the Prague Spring: context of retreat from Cuba, détente and Sino-Soviet split, Slovak discontent, pressure for economic links with West (also Romania), fall of Novotny, Dubček’s reforms, support of Tito. Similarly in relation to consequences candidates may discuss: (Hungary) the context of Western diversion (Suez in 1956), the decision to use force by Khrushchev in 1956 and Brezhnev in 1968, repression and the Moscow conference (1957), alienation of Tito, strengthening of Khrushchev’s position; (Czechoslovakia) context of Western diversion (and Vietnam and cracks in the NATO alliance), use of force, repression, Brezhnev Doctrine, return to Stalinist-style economic policies, slowing of détente.

[50]

57

Page 61: History A - The Bicester Schoolthebicesterschool.org.uk/wp-content/uploads/2015/09/Mark... · 2015-09-28 · History A Mark Schemes for the Units January 2009 H106/H506/MS/R/09J

F962-02 Mark Scheme January 2009

Question Number

Answer Max Mark

27 ‘Economic and political chaos in Russia and Eastern Europe was the main consequence of the collapse of the Soviet Union.’ How far do you agree with this view? No specific answer is looked for. Candidates must deal with the given factor even if they wish to argue that other factors were as or more important. Candidates may take the phrase ‘collapse of the Soviet Union’ to refer generally to the years leading to the creation of the CIS, or specifically to the end of the Soviet Union in 1991. Candidates may argue that economic and political chaos was certainly the most important immediate and short-term consequence of the Soviet Union’s collapse, pointing to the problems for the new 15 states of the CIS in creating political structures and dealing with the collapse of the Soviet economic system. Similarly in Eastern Europe new democratic structures had to be created and societies needed to adjust to multi-party democratic systems whilst coping with the economic problems of adjustment to capitalism. Some may argue that these economic problems pre-dated Soviet collapse, but certainly unemployment and dislocated trade were a feature in many states. Candidates may argue that in the longer term there were other consequences as Soviet control was released associated with nationalism (in the break up of Czechoslovakia, for example,) or in the CIS the consequences of which are still present as ethnic nationalism threatens political stability in a number of states, such as Georgia and even the Russian Federation itself (Chechnya). Candidates may also argue that many states, including Russia as well as states such as the Czech Republic, have adjusted economically relatively rapidly and living standards are rising. They may also point to the resilience and reinvention the communist parties in the new states and the trend to authoritarian and nationalist regimes. Elsewhere they may point to the closer links and economic and political cooperation with the West (request to join NATO and the EU).

[50]

28

Crisis in the Middle East 1948-2003 Assess the reasons for the creation of the state of Israel in 1948. No specific answer is looked for. Candidates will need to explain a number of reasons and assess their relative significance and linkages to score well. Candidates in assessing reasons may discuss some of the following: Zionism and Theodore Herzl; Balfour Declaration 1917; the British mandate in Palestine, British policy and post WW1 immigration; Peel Commission Report 1937; WW2 and impact of Holocaust; US and UN involvement (UNSCOP) and partition; the role of Ben Gurion, Haganah, Irgun and Stern Gang; role of Huseini, the Arab Liberation Army, the Muslim brotherhood, and the Arab League; the declaration of Israeli independence 1948. Candidates may discuss the difficulties facing Britain in the post-war years, the King David Hotel and Exodus incidents, the development of the partition proposals in the UN and the responses of the Jews and Arabs, the development of conflict within Palestine and the final withdrawal of the British and the proclamation of the state of Israel. Such short term issues should be set against the longer term context.

[50]

58

Page 62: History A - The Bicester Schoolthebicesterschool.org.uk/wp-content/uploads/2015/09/Mark... · 2015-09-28 · History A Mark Schemes for the Units January 2009 H106/H506/MS/R/09J

F962-02 Mark Scheme January 2009

Question Number

Answer Max Mark

29 To what extent was Israel responsible for the failure to resolve the Palestinian question after the Yom Kippur War (1973)? No specific answer is looked for. Candidates will need to discuss the degree of Israeli responsibility and balance this against other factors to score well. In coming to a judgement candidates may discuss some of the following: the impact of the 1967 and 1973 wars; the role the PLO and Arafat; the policies of Begin after 1977 and growth of Israeli settlement of West Bank and Gaza; the post-1985 ‘Iron Fist’ policy; the roles of Islamic Jihad, Hamas and other radical Palestinian groups; the Intifada post-1987; PLO acceptance of UN resolution 242 and renunciation of terrorism; US involvement and refusal of Israel to negotiate; post-1991 US-Soviet brokered talks in Madrid/Washington; the Oslo Accords, Oslo 2 and Arafat’s return to Gaza; the Wye River talks linking Israeli withdrawal to action against Hamas and Islamic Jihad; Camp David and renewed violence, the second Intifada; Bush and the ‘Road Map’.

[50]

30 Assess the consequences of the Iran-Iraq War (1980-88) No specific answer is looked for. Candidates will need to explain a number of consequences and assess their relative significance and linkages to score well. Candidates may distinguish between short and long term consequences, direct and indirect consequences. They may discuss some of the following: the short term costs of war – 500000 killed, $800 billion, little change (final settlement in 1990 virtually a repeat of 1975 Algiers agreement); the legacy of debt – Iraq (and Iran) bankrupt; the results for Hussein - able to tighten grip on Iraq (especially over Kurds and Shia) – and Iraq - now perhaps most powerful state in Gulf (55 divisions and 4000 tanks) – left neighbours anxious, including Israel; in longer term - crippling debt and unwillingness of Arab states to write them off despite Hussein’s ‘defence of Arabs’ would lead Saddam to look to wrest money from Kuwait and eventually invasion in 1991 and thence the first Gulf War; legacy of fear and unrest in Iraq (assassination attempts); Iraq’s isolation and seeking of friends – e.g. Arab Cooperation Council and non-aggression pacts.

[50]

Paper Total [100]

59

Page 63: History A - The Bicester Schoolthebicesterschool.org.uk/wp-content/uploads/2015/09/Mark... · 2015-09-28 · History A Mark Schemes for the Units January 2009 H106/H506/MS/R/09J

F963-01 Mark Scheme January 2009

60

F963-01 - British History Enquiries

Question Number Answer Max

Mark

1(a)

The Normans in England l066-1100

Study Sources A and B Compare these Sources as evidence for the ways in which the Norman conquerors treated the native English. Focus: comparison of two Sources No set answer is expected, but candidates need to compare the contents, evaluating such matters as authorship, dating, utility and reliability, so using the Source ‘as evidence for ...’. The headings and attributions should aid evaluation and reference to both is expected in a good answer. Both Sources focus upon the fate of the native English hierarchy. Source A, written soon after the Conquest, sees the English as treated well; no matter Norman-French gains, the native English are not treated unjustly, indeed the emphasis is upon even-handedness and justice, but Source B takes a different line. Written some time after the Conquest, it emphasises the grants made to Norman-French lords; the tone is one where ‘foreigners grew wealthy’ from ‘the spoils of England’; Englishmen have been killed or driven into exile. In A there seems to be an attempt to retain something of an English presence higher up the social scale but in B the reverse is true, with foreign influence and presence paramount. The tone of the comments in A is reasonably ameliorative, that of B harsher and tougher. The language used in both can be assessed for content and message. Comment on the provenances might refer to the different origins of the authors as well as the dates. A comes from a Frenchman closely involved in the Conquest and known to be sympathetic to William I while B comes from an Englishman, raised and educated in Normandy, but who retained some residual sympathy for his native land and its treatment. Yet both present slightly unexpected views, above all A. The context of the events described might be remembered. B might be seen as typical of opinion over time. The provenances can be engaged to help explain and evaluate the differences. So, too, the dates are important: A precedes the major tenurial-territorial changes of c.1070-2, B presents the aftermath.

[30]

Page 64: History A - The Bicester Schoolthebicesterschool.org.uk/wp-content/uploads/2015/09/Mark... · 2015-09-28 · History A Mark Schemes for the Units January 2009 H106/H506/MS/R/09J

F963-01 Mark Scheme January 2009

Question Number Answer Max

Mark

1(b)

Study all the Sources

Use your own knowledge to assess how far the Sources support the interpretation that the changes in land holding after 1066 were the result of military needs.

Focus: Judgement in context based on the set the Sources and own knowledge. Successful answers will need to make use of all five Sources, testing them against contextual knowledge and evaluating their strengths and weaknesses, including any limitations as evidence. A range of issues may be addressed in focusing upon the terms of the question but no set answer is expected. The question focuses on the reasons for changes in land holding patterns and context. The Sources offer a variety of points. All the Sources emphasise or suggest issues of security, control, defence, military needs with Sources C, D and parts of the other Sources supporting the view of the predominance of military needs and military thinking. Sources A, B and E in places offering different perspectives. The Conquest was undertaken to gain the throne but the invaders needed reward; colonisation and settlement followed; lands were used for patronage and as reward. There is much debate about William I’s intentions, given that, initially, there was no wholesale redistribution of lands, save for those lost by Anglo-Saxon landholders killed at Hastings. Land grants were rewards, forms of patronage, a means to secure control and hold on to acquired lands; they were a part of the Norman-French process of takeover, along with the use of the Church, monasteries, control of towns and castles (as in Sources A and D). The Sources, especially B and E, suggest the scales of change and upheaval. Source E does summarise some of the features of the takeover process and its stages; Sources C and D are good examples of the methods as well as military needs involved. Parts of Sources A, B and E point to the need to reward participants in the Conquest and subsequent settlement process; reference is made to the spoils of victory. A mentions grants of lands and castles, rewards for hardships and danger while B specifies some of those rewarded by the King’s patronage; E offers an idea of the scale of patronage in terms of how much was held by the new political élite. Control, settlement, rewards were all linked, above all in the period from 1066 to the mid-1070s. Topic knowledge can be used to support analysis: William I’s hold and methods of control, the issues involved in Normanisation (Sources A and B could be contrasted there), the place of Domesday Book as a source of evidence for the extent and success of changes. The better answers will see links of military and political needs and strategies and relate to the context of takeover, colonisation and control.

[70]

61

Page 65: History A - The Bicester Schoolthebicesterschool.org.uk/wp-content/uploads/2015/09/Mark... · 2015-09-28 · History A Mark Schemes for the Units January 2009 H106/H506/MS/R/09J

F963-01 Mark Scheme January 2009

Question Number Answer Max

Mark

2(a)

Mid-Tudor Crises 1536-1569

Study Sources D and E Compare these Sources as evidence for ways in which the gentry and nobility tried to win support for the rebellion.

Focus: Comparison of two Sources. No set answer is expected, but candidates need to compare the contents, evaluating such matters as authorship, dating, utility and reliability, so using the Sources ‘as evidence for …’. The headings and attributions should aid evaluation and reference to both is expected in a good answer. The Sources agree that rebel leaders of the nobility and gentry classes used xenophobia, status and appeals to religious belief to rally support from their fellow landowners. The Sources show some similarities in the approaches they adopt. Both claim to be loyal to the Queen, though Source D is less positive, specifying ‘no harm’, whereas Source E asserts their ‘true and lawful’ nature. Both Sources appeal to xenophobic feelings, Source D raising the spectre of ‘a hundred armed Spaniards’ marching on London, and Source E suggesting imminent invasion. Both refer to freedom: Source E to the ‘enslavement’ of the English people, and Source D to the ‘liberty and the commonwealth’ of England which are at stake. Source D raises the fear of loss of property – ‘our health and wealth depend on it’, and Source E talks of the ‘displacement of the ancient nobility from the Queen’s side’ with its economic repercussions. This shows the main audience of the appeal to be the landed classes themselves who are then expected to rally popular support among their tenants. Both ask for supporters to assemble, but Source D emphasises the amount of support while Source E focuses on the speed with which the rebels need to act. On the other hand, there are differences. Catholicism is an important means of rallying support in Source E, whereas religion is not mentioned in Source D, even though one of Mary’s motives in the Spanish marriage is to strengthen Catholicism in England and produce a Catholic heir. Wyatt focuses solely on xenophobic, defensive and economic persuasions. A focus on Protestantism might have lost Wyatt support, as Catholicism remained strong in some parts of Kent. In Source D Wyatt’s appeal is broadly to the men of Kent, many of whom are Protestants, after the collapse of the other planned prongs of the rising, revealed to the government by Edward Courtenay. In Source E Northumberland and Westmorland, in contrast, are Catholics appealing to their Catholic tenants and allies in the north of England, an area of with a majority of ‘church papists’. They are reacting almost spontaneously to Norfolk’s revelation of the plot, to marry him to Mary Queen of Scots, to Elizabeth’s government, and are bringing forward their plans to rise. There is no mention of Mary Queen of Scots, who had arrived in England the previous year, perhaps because of anti-Scottish sentiment in the north of England. Thus both Sources conceal some of their intentions in their rallying of support.

[30]

62

Page 66: History A - The Bicester Schoolthebicesterschool.org.uk/wp-content/uploads/2015/09/Mark... · 2015-09-28 · History A Mark Schemes for the Units January 2009 H106/H506/MS/R/09J

F963-01 Mark Scheme January 2009

Question Number Answer Max

Mark 2(b) Study all the Sources.

Use your own knowledge to assess how far the Sources support the interpretation that the main aim of rebellions between 1536 and 1569 was the restoration of the Catholic faith.

Focus: Judgement in context, based on the set of Sources and own knowledge. Successful answers will need to make use of all five Sources, testing them against contextual knowledge and evaluating their strengths and weaknesses, any limitations as evidence. A range of issues may be addressed in focusing upon the terms of the question but no set conclusion is expected. The Sources contain references to religious, economic, political, social and regional aims. Religion is central to Sources A, B and E, whereas the references to religion in Source C are less prominent and Source D implies a religious aim. Sources A, B and E aim for a restoration of Catholicism. Both Source A, the Pilgrimage of Grace, and Source B, the Western Rebellion, have traditionally been seen as having primarily religious aims, but they reveal a web of intentions. They are both supported by conservative Catholics in opposition to religious changes. Both wish to see the abbeys restored, but Source A is trying to halt the process as it is taking place, whereas Source B occurs when monastic land had long ago passed into lay hands, so the aim is less realistic. Source B is a response to the First Book of Common Prayer 1549, when Protestant doctrine and banning of Catholic practices emerged as grievances. Restoration of Catholic doctrine is also shown in Source E, and the rebels later celebrate Mass in Durham Cathedral. Sources A and B aim to punish heretics, and Source E to remove heretical councillors from influence over Elizabeth. In contrast Source C, Kett’s Rebellion accepts change and has Protestant aims: an elected, resident and well-educated ministry. Source D omits to mention the implicit religious aim of the rebels, in attempting to marry the Protestant Elizabeth to Edward Courtenay and replace the Catholic Mary. Similarly, Source E omits its other aim, to marry Mary Queen of Scots to the Duke of Norfolk and replace Elizabeth. Both of these hidden aims might be supplied from own knowledge. Political aims feature in Sources A, D and E. Source A aims to restore Mary to the succession, in contrast to the aim in Source D, to prevent Mary and Philip from establishing a Catholic succession. Factionalism is revealed in Source A, with the aim to remove Cromwell and Rich, and Source E, to remove Protestant councillors. A broader view might be taken, that the aims pursued by noble rebels differed from those of popular rebels. Thus the Pilgrimage of Grace might be seen as a hybrid rebellion with Darcy and Hussey having similar political and social aims to Northumberland and Westmorland, whereas the popular elements were merely trying to attract attention to their grievances, as their only means of making their voices heard. Economic aims feature in Source C, such as an end to enclosures

63

Page 67: History A - The Bicester Schoolthebicesterschool.org.uk/wp-content/uploads/2015/09/Mark... · 2015-09-28 · History A Mark Schemes for the Units January 2009 H106/H506/MS/R/09J

F963-01 Mark Scheme January 2009

Question Number Answer Max

Mark and rack renting. Financial aims might be inferred in Sources D, ‘health and wealth’ and E, ‘displaced the ancient nobility’. This links also to class and social aims, present in Source B, limiting the number of gentry servants, and in Source C‘s attempts to free bond men and reduce the power of capitalist landlords. Regional aims feature also in Sources A and B, which aim to reduce the control of central government. The aim to defend national interests is in Sources D and E. The style of the rebels’ presentation of their aims suggests that lower class rebels might present their aims humbly, as in Source C, or stridently, as in Source B, reflecting longer standing regional grievances. Candidates are likely to consider a range of aims taken from the wide range possible: religious, economic, social, political or regional. They are likely to see an interaction between religious aims and others. It is up to candidates to assess and decide upon relative importance here, there being no set conclusion.

[70]

3(a)

The English Civil War and Interregnum 1637-60 Study Sources A and D Compare these Sources as evidence for opposition to the First Civil War. Focus: comparison of two sources No set answer is expected, but candidates need to compare the contents, evaluating such matters as authorship, dating, utility and reliability, so using the sources ‘as evidence for…’. The headings and attributions should aid evaluation and reference to both is expected in a good answer. Both sources are contemporary accounts, although they come from different ends of the First Civil War, which affects their content. Source A is a public declaration, while source D is a letter of a local parliamentarian who offers both the facts and a judgement upon them, but his loyalties do not compromise the source’s veracity. Both sources report opposition to the war, D from the civilian population, and A from the more propertied and prominent gentry. Both refer to ‘the miseries’ (source A) or ‘misery’ (source D) of War, such as capture of prisoners (source A) and seizure of possessions and plundering (sources A and D). Here, however, the sources differ: the Cheshire gentry want to achieve peace, and to avoid taking sides, so they propose a petition to both King and Parliament urging this; source D, in contrast, sees neutralism as a reaction to military plundering and, were this redressed, then most neutralists could be converted into parliamentarians. Cheshire wishes for arms to be laid down; the west country neutralists are taking up arms to protect their property. These different concerns should be linked to the date of the two documents: in December 1642 the First Civil War was only gradually spreading across the country, with many communities not yet adjusted to war; by 1645, as source D indicates, frustration at plundering and the inability or unwillingness of parliament’s military [30]

64

Page 68: History A - The Bicester Schoolthebicesterschool.org.uk/wp-content/uploads/2015/09/Mark... · 2015-09-28 · History A Mark Schemes for the Units January 2009 H106/H506/MS/R/09J

F963-01 Mark Scheme January 2009

Question Number Answer Max

Mark commanders to do anything about it, led to militancy. Areas of similarity and difference thus should be identified and analysed.

3(b)

Study all the Sources. Use your own knowledge to assess how far the Sources support the interpretation that Parliament’s ineffective military leadership was the main reason Parliament took so long to win the First Civil War. Focus: Judgment in context, based on the set of sources and own knowledge Successful answers will need to make use of all five sources, testing them against contextual knowledge and evaluating their strengths and weaknesses, and any limitations as evidence. A range of issues may be addressed in focussing upon the terms of the question but no set conclusion is expected. Sources B and C support the proposition: that Manchester and Essex were not committed to all-out victory, squandered opportunities such as Newbury (source B), and, as candidates may add from their own knowledge, failed to build on the successes such as Fairfax and Cromwell at Marston Moor in 1644. The resolution of the tension between the ‘peace party’ and hawks such as Cromwell in the winter of 1644-45 was the Self-Denying Ordinance and the creation of the New Model Army, which quickly won the Civil War in 1645-46. But other sources point in other directions: sources A and E indicate opposition to the war, either by proposing peace (source A) or by taking up arms in protection against the parliamentary forces (source E); however, the latter suggests that were Parliament to suppress plundering, then it could recruit supporters from these militant neutralists. The wider point that these two sources raise is the extent to which opposition to war delayed Parliament’s eventual victory. Source E opens up another flank: the advantages which the royalists possessed, which Parliament had to overcome in order to achieve victory, as well as their weaknesses, which Parliament could exploit. So there are three alternative explanations here which need sorting and testing against the question.

Total for paper:

[70] [100]

65

Page 69: History A - The Bicester Schoolthebicesterschool.org.uk/wp-content/uploads/2015/09/Mark... · 2015-09-28 · History A Mark Schemes for the Units January 2009 H106/H506/MS/R/09J

F963-02 Mark Scheme January 2009

66

F963-02 - Option B: British History Enquiries 1815-1945

Question Number Answer Max

Mark

1(a)

The Condition of England 1815-53 Study Sources A and B Compare these two Sources as evidence for the arguments about state elementary education in the early 1830s. Focus: Comparison of two Sources No set answer is expected, but candidates need to compare the contents, evaluating such matters as authorship, dating, utility and reliability, so using the Source ‘as evidence for…’. The headings and attributions should aid evaluation and reference to both is expected in a good answer. Both Roebuck and the Edinburgh Review an A and B agree that the main argument in favour of education was that it was seen as the key to ensuring law and order at a time of considerable discontent. The Review stresses education’s role in promoting religion and morality whilst Roebuck differs, stressing that an understanding of problems will diffuse discontent. Peel in B takes the opposite view, that there is no problem that greater State education could solve and that the existing voluntary and private system is sufficient. It is under the control of charitable gentlemen, where it belongs. Roebuck in B and the Review in A also differ on the extent to which discontent will be diffused. ‘B’ is very optimistic, seeing education as a panacea, allowing all to put their ignorance into perspective, from the peasantry to sturdy paupers intent on demanding their doles from hard pressed parishes. Roebuck even extends this argument to future landlords and manufacturers who will see the evil of their respective monopolies. ‘A’ merely argues a view that education will serve traditional control, via religion and morality. Comment on provenance might stress that a Whig journal would be expected to support more education, especially given the context of disturbed times (the Swing Riots and the Reform Act Crisis), hence its traditional stress on religion and morality. As an upper-class ‘establishment’ journal it is concerned to spread proper social values in its arguments. In contrast Roebuck’s speech is just after the Reform Act and designed to persuade MPs that education is a liberating force, a view countered by a Conservative like Peel. Peel and the Edinburgh Review may be far more typical of the arguments that swayed MPs, with few subscribing to Roebuck’s rather ambitious claims.

[30]

Page 70: History A - The Bicester Schoolthebicesterschool.org.uk/wp-content/uploads/2015/09/Mark... · 2015-09-28 · History A Mark Schemes for the Units January 2009 H106/H506/MS/R/09J

F963-02 Mark Scheme January 2009

Question Number

Answer Max Mark

1(b)

Study all the Sources Use your own knowledge to assess how far the Sources support the interpretation that the main obstacle to educational development in the period from 1830 to 1853 was financial. Focus: Judgement in context, based on the set of Sources and own knowledge. Successful answers will need to make use of all five Sources, testing them against contextual knowledge and evaluating their strengths and weaknesses, including any limitations as evidence. A range of issues may be addressed in focusing upon the terms of the question but no set answer is expected. The question focuses on the obstacles to educational development in the period. The Sources contain a variety of useful points, either implicitly or explicitly. The sources may be grouped into C and E both of which refer to financial issues. A different set obstacles are raised in A, B and D with C and E adding to these. The view that money was an obstacle is mentioned by both C and E, although only C implies that it was the main reason. Both sources come from key moments in the educational debate, ‘C’ from the fears generated by the proposal for the first State Grant to assist elementary education, D in the wake of Graham’s attempt to use the Factory Acts to extend provision. Candidates could also stress that Roebuck’s scheme for universal state education would, inevitably, incur huge cost. Cobbett in C is a surprising source. One might expect a radical of his experience to favour educational development and ally with Roebuck, but he does not. He provides reliable evidence of the rural conservatism of some Radicals. As an enemy of ‘old corruption’ he regards taxes for education as just another excuse for a rotten state to bank- roll new jobs and placemen. Perhaps aware of the extent of the private provision for working class children, free of Church and State patronage, he opposes taxing them a second time to provide for a state education that would seek to control, although it is difficult to see how ‘the people’ will bear the brunt of taxation, unless indirectly. From a very different radical angle Engels in E is scornful of a government that will not spend more than £40,000 pa. from a total Budget of £55 million. He implies that money is there but the will is lacking. Engels is a shrewd commentator, who is likely to see cost as important to a capitalist class, yet he gives more weight to religious rivalry both as stimulus and obstacle. Own knowledge could comment on the small size of the initial grant and to concern that finance should be voluntary, not a state affair. The Monitorial system reflected the concern to keep education cheap.

[70]

67

Page 71: History A - The Bicester Schoolthebicesterschool.org.uk/wp-content/uploads/2015/09/Mark... · 2015-09-28 · History A Mark Schemes for the Units January 2009 H106/H506/MS/R/09J

F963-02 Mark Scheme January 2009

Question Number

Answer Max Mark

All the Sources mention other obstacles. One is the debate over what education was for. The stress on the benefits of education in A imply that many in the establishment feared that education might have the opposite effect, hence its stress on religion and morality. In a different context Cobbett in C, like Peel in B, stresses that there is no need for education. Cobbett’s definition of education is what most suits a man to his situation. Most working men had no need. Like Baines in D, Cobbett fears state control. He refers to teachers as a ‘new race of idlers’, useless and wasteful state servants. Baines approaches this from a powerful Nonconformist standpoint. One might expect him to champion education but his liberal Laisser-Faire outlook makes him fear education under state control. He uses the examples of military and oriental despotism (Prussia and China), both well known for their state schemes of education and thus lack of freedom. One of Baines’s motives would, undoubtedly, be his fear of the Anglican State, a point developed by Engels in E who refers to the religious obstacles to education. Using own knowledge candidates could expand upon this, as Engels sees religion as both a stimulus but also hindrance to a rational curriculum developing. His is a balanced set of points with the advantage of an outsider’s view, in every sense. Own knowledge of the religious rivalries and especially the controversy over Graham’s Bill in 1843 could develop the religious point for those seeking to argue that disagreements over who controlled education were the major obstacle. Such knowledge could also point to a lack of trained teachers, disagreement over the curriculum and to working class anti- state views as other obstacles. The better candidates will compare the obstacles to assess which was the main limitation.

68

Page 72: History A - The Bicester Schoolthebicesterschool.org.uk/wp-content/uploads/2015/09/Mark... · 2015-09-28 · History A Mark Schemes for the Units January 2009 H106/H506/MS/R/09J

F963-02 Mark Scheme January 2009

Question Number

Answer Max Mark

2(a) The Age of Gladstone and Disraeli 1865-86 Study Sources A and D Compare these two Sources as evidence for Liberal views of the Irish and their problems in the period from 1885 to 1886. Focus: Comparison of two Sources No set answer is expected, but candidates need to compare the contents, evaluating such matters as authorship, dating, utility and reliability, so using the Source ‘as evidence for…’ The headings and attributions should aid evaluation and reference to both is expected in a good answer. Both sources are reporting on Gladstone’s views and both point to a consistency on his part in acknowledging the Irish as settling for nothing less than their own parliament as the solution to their problems. Both, historically, were sympathetic to the Irish, especially on grounds of religious and land discrimination. However, both Gladstone and Bright regard the current Irish nationalist MPs and their tactics at Westminster as an increasing nuisance. Something needs to be done to tackle their obstructionism and improving organisation. They differ in that A just reports Gladstone’s views, hardly a typical liberal when it came to Irish affairs, whilst D also includes Bright’s older liberal views on Gladstone and the Irish. In A Gladstone continues to be sympathetic and thinks the Irish will be content with their own parliament. He continues to do so in D but Bright, also in D, does not believe the Irish, on past record, to be faithful. He considers them a ‘rebellious’ party intent on gaining whatever they can, up to and including independence, and the imposition of duties on British goods as a solution to their poverty. Comments on the provenance of the Sources might stress that both are private conversations, where Gladstone is sounding out colleagues on Irish issues, although Derby in A is keen to share Gladstone’s views with Granville and the context of A is significant - Derby is worried about the direction of Gladstone’s thoughts on the Union. At a later date, when Gladstone had returned to power for a third government, Bright shares these Whig fears. Although reported by Derby, Gladstone’s views are reliably conveyed. Nonetheless to a Whig leader Gladstone may have over-stressed Irish obstructionism at Westminster. Bright may be the better evidence as it goes against the grain for a former ‘friend’ of Ireland to oppose Gladstone and Home Rule. It also gives both sides, an Irish analysis but one which remains very sceptical about Irish motives and trustworthiness, both in its content and tone. Better candidates might comment on the closeness of Gladstone and Bright.

[30]

69

Page 73: History A - The Bicester Schoolthebicesterschool.org.uk/wp-content/uploads/2015/09/Mark... · 2015-09-28 · History A Mark Schemes for the Units January 2009 H106/H506/MS/R/09J

F963-02 Mark Scheme January 2009

Question Number

Answer Max Mark

2(b) Study all the Sources. Use your own knowledge to assess how far the Sources support the interpretation that the First Home Rule Bill failed because of Gladstone’s mishandling of the issue. Focus: Judgement in context, based on the set of Sources and own knowledge. Successful answers will need to make use of all five Sources, testing them against contextual knowledge and evaluating their strengths and weaknesses, including any limitations as evidence. A range of issues may be addressed in focusing upon the terms of the question but no set answer is expected. The sources can be interpreted in more than one way. All five could be used to either agree or disagree with the proposition. On balance Sources C, D and E could be said to support the view that Gladstone’s failure was because of his own mishandling of the issues. Two of these are contemporary critics of Home Rule commenting on it after its public announcement; the third is a modern historian critically assessing matters at a considerable distance. In contrast Sources A and B, before Gladstone’s conversion to Home Rule was public knowledge, suggest a much more careful and astute handling of it, although both are Gladstone’s own views and this will condition what he has to say. The historian in E could be considered to belong to either view. From hindsight Gladstone failed and disastrously split the Liberal Party, but at the time of the sources (1885-6) Home Rule could have been secured. The Sources focuses on Gladstone’s views and manoeuvrings, those of the potentially hostile Whigs whose support he was trying to secure and of his mentor and the standard bearer of older middle class Liberalism, John Bright. The evidence that Gladstone mishandled Home Rule depends on how far candidates think that his positive assessment of settling Ireland in A is correct. Clearly Bright in D thinks it mistaken and candidates might refer to Parnell’s famous hints in 1885 that independence was the real goal to suggest delusion on Gladstone’s part. In B Gladstone could be accused of converting to Home Rule simply to regain power once the November election had given Parnell the opportunity to return a Liberal government to power. Candidates could usefully deploy their knowledge on the Hawarden Kite. The Source suggests that Gladstone wished to avoid ‘bids’ for nationalist votes but clearly Home Rule would, in effect, do this. Was he unlucky that his son went public during delicate negotiations with the Whigs, or was it intended? In C, soon after the Hawarden Kite, the Whig leader Hartington clearly feels that Gladstone has kept everyone in the dark and is not to be trusted. Linking to the point in E made by Partridge about the difficulties Gladstone faced in his own party, this is reliable evidence that Gladstone had failed to keep the important Whig leadership onside. Bright in D could be said to voice wider concerns about Gladstone conceding too much to the ‘rebel’ Irish, a point not lost on English voters. Many considered Gladstone too trusting and

70

Page 74: History A - The Bicester Schoolthebicesterschool.org.uk/wp-content/uploads/2015/09/Mark... · 2015-09-28 · History A Mark Schemes for the Units January 2009 H106/H506/MS/R/09J

F963-02 Mark Scheme January 2009

Question Number

Answer Max Mark

naïve, Bright citing a very vague reply from Gladstone to his point on import duties. Partridge in E judges Gladstone to have been too reliant on oratory, too convinced of the morality of his case (‘God’). Own knowledge could add substance to E’s points about Ulster and to Gladstone’s political motives which outraged Chamberlain whose plans for local devolution were derailed. Own knowledge could then stress that Gladstone’s controversial conversion and his possible motives for it, would lead to Commons failure (where there is no evidence that he prepared Liberal MPs for it), let alone its fate in the House of Lords However a case could be made that Gladstone did the best he could and failure was not down to personal mishandling. The sources present evidence that he tried to handle the issue with some skill. He might have survived a few resignations from the likes of Chamberlain, but not the Whigs, hence his appropriate concentration on them in sources A, B and C. To Derby in A he stresses the need to overcome increasingly organised Irish obstruction. He is clearly concerned to keep the Whigs informed, possibly, as Hartington in C saw it, of gaining a ‘drift’ into acceptance of Home Rule. C thinks Gladstone stood a good chance of bouncing the Whigs into it. He reassures Hartington in B that a Dublin Parliament would only consider Irish affairs, whilst to both Bright in D and the Whigs, he is keen to stress the ‘removal’ of the Irish from Westminster. Own knowledge might point out that, in discussion with Parnell, other angles were stressed. Such detailed contacts with the Whigs in A and B would give the lie to Hartington’s accusation in C that he didn’t know what Gladstone was doing. Partridge in E also refers to the difficulties Gladstone faced from both Common and Lords. Yet B provides evidence that Gladstone is trying to get Lord Salisbury’s conservative minority government to undertake it, thereby preventing Liberal splits and getting it through the Lords. Was this a pretty forlorn hope? Gladstone is also aware of the need to prevent a bidding war between Tory and Liberal over the Irish vote, although the Hawarden Kite destroyed this. Thus he thought the right moment had come and the Irish wanted Home Rule. The Sources do not pursue in detail the other problems that led to failure – Chamberlain, Ulster and an uncooperative Conservative party (both Salisbury and Lord Randolph Churchill).

[70]

71

Page 75: History A - The Bicester Schoolthebicesterschool.org.uk/wp-content/uploads/2015/09/Mark... · 2015-09-28 · History A Mark Schemes for the Units January 2009 H106/H506/MS/R/09J

F963-02 Mark Scheme January 2009

Question Number

Answer Max Mark

3(a)

England and a New Century 1900-24 Study Sources B and D Compare these two Sources as evidence for opinions on the role of the House of Lords. Focus: Comparison of two Sources. No set answer is expected, but candidates need to compare the contents, evaluating such matters as authorship, dating, utility and reliability, so using the Sources ‘as evidence for...’. The headings and attributions should aid evaluation, and reference to both is expected in a good answer. Content: Source B presents a negative view of the House of Lords. Campbell-Bannerman accuses the Lords of damaging democracy by obstructing the legislation (an education bill) of a Government recently elected by the people with the largest majority in history. He promises that a solution will be found to this challenge facing the Constitution. Source D, on the other hand, presents a positive view of the Lords as protecting the people against a tyrannical government. According to Balfour, the second chamber exists to guard against hasty legislation. Provenance: There are clear differences of provenance. Source B. 1906. A recently-elected, radical Liberal PM, with a clear mandate for reform. But frustrated by a Conservative-dominated House of Lords. The language used supports this. Source D. 1909. Balfour is still frustrating the Liberal Government. But now, three years later, the conflict is much more serious, and the Conservatives are on the defensive. The context is now Lloyd George’s People’s Budget. Balfour (with his allies in the Lords) sees this as ‘vindictive’ legislation. The Conservatives suspect that there are wider political motives behind the bill.

[30]

72

Page 76: History A - The Bicester Schoolthebicesterschool.org.uk/wp-content/uploads/2015/09/Mark... · 2015-09-28 · History A Mark Schemes for the Units January 2009 H106/H506/MS/R/09J

F963-02 Mark Scheme January 2009

Question Number

Answer Max Mark

3(b)

Study all the Sources. Use your own knowledge to assess how far the Sources support the interpretation that it was the Conservatives who were mainly responsible for clashes between the Commons and the Lords in the period from 1906 to 1911. Successful answers will need to make use of all five Sources, testing them against contextual knowledge, and evaluating their strengths and weaknesses and any limitations as evidence. A range of issues may be addressed in focusing upon the terms of the question, but no set conclusion is expected.

Most of the constitutional problems stemmed from the Liberal’s landslide victory in the 1906 Election, and the Conservative reaction to this predicament. The sources cover the period 1906-1909. But answers will be strengthened by a knowledge of political events between 1906 and 1911, especially Lloyd George’s People’s Budget , the two General Elections of 1910, and the Parliament Act of 1911. The “Conservatives” of 3 (b) might be restricted to the opposition party led by Balfour, and its representative M.P’s in the Commons. But candidates will have more scope if they include the Conservative majority in the Lords led by Lansdowne. Sensible source grouping would put Sources B and C on the Liberal Government’s side of the argument. Thus, the House of Lords is often seen as ‘Mr Balfour’s poodle’. Sources A, D and E represent a Conservative view of the constitutional debate (the Lords as the ‘watchdog’ of the Constitution), and blame the Liberal Government for the crisis. As there is clear polarity between the two groups of Sources, one would hope that candidates in their evaluation of the Sources would spot that the authors are often extremely partisan. In Source B, Campbell-Bannerman, a reforming Prime Minister enjoying a huge majority, is frustrated by the recent destruction of the Government’s education bill in the House of Lords. He clearly blames Balfour’s Conservatives. The battle lines are being drawn. Of course, it is a matter of opinion whether Balfour and Lansdowne are acting unconstitutionally, although, it is on record that Liberals like Asquith, Lloyd George and Churchill believed the Lords were being partisan. Candidates might argue that most aristocrats would oppose new liberalism anyway, without any prompting from the Conservative Party. By 1909, Lloyd George is in no doubt (Source C) that the Lords are acting unfairly, particularly in their reaction to his budget. Candidates might wish to discuss if this budget was a deliberate trap laid by the Liberals, or rather a necessary move to raise taxes for social reforms and for the navy. Either view could be argued from own knowledge, although it is clear from the source that Lloyd George has come to regard the aristocracy as the enemy of the people. In Source A, Balfour agrees with Lansdowne the policy of conservative co-operation which was to infuriate the Liberal leadership.

73

Page 77: History A - The Bicester Schoolthebicesterschool.org.uk/wp-content/uploads/2015/09/Mark... · 2015-09-28 · History A Mark Schemes for the Units January 2009 H106/H506/MS/R/09J

F963-02 Mark Scheme January 2009

Question Number

Answer Max Mark

In Source D, the actions of the House of Lords in opposing the People’s Budget are defended by Balfour on the grounds of protecting the people against the actions of a tyrannical government. This is a different version of democracy to that presented (for example) by Campbell-Bannerman in Source B. Again, in Source E, Lansdowne defends the Lords opposition to the Budget on the grounds that this budget is more than a ‘finance bill’. Therefore, according to the Conservatives, the Lords are not acting unconstitutionally. The Conservative majority in the Lords did reject several Liberal bills between 1906 and 1909. In November 1909, they rejected the Budget. After the first Election of January 1910, they were forced to accept the Budget. Then, until August 1911 (through the accession of George the Fifth, the Liberal threat to create 500 Liberal peers, and the second Election of December 1910) they resisted the Parliament Bill, which reduced the powers of the Lords. The two elections were inconclusive. All of the major parties were weakened by them in some respect. The Liberal Government, now dependent on Irish support, found it easier to get legislation through the Lords (eg. 1911 National Insurance Act), although the Irish Home Rule Bill (1912) was lost through delay.

[70]

74

Page 78: History A - The Bicester Schoolthebicesterschool.org.uk/wp-content/uploads/2015/09/Mark... · 2015-09-28 · History A Mark Schemes for the Units January 2009 H106/H506/MS/R/09J

F963-02 Mark Scheme January 2009

Question Number

Answer Max Mark

4(a)

Churchill 1920-45 Study Sources B and C. Compare these two Sources as evidence for opinions among Conservatives about Chamberlain’s handling of the Munich Settlement. Focus: Comparison of two Sources No set answer is expected but candidates need to compare the contents, evaluating such matters as authorship, dating, utility and reliability, so using the Source ‘as evidence for…’ The headings and attributions should aid evaluation and reference to both is expected in a good answer. Content B is clearly condemnatory – an unmitigated defeat; C on the other hand thinks that the government’s case is excellent. There is no hint of ‘a disaster of the first magnitude’ with the House of Commons giving full support. C says that Churchill’s contribution lacked any substance and merely ‘enlivened the house’ casting doubt on the depth of his criticism. There is no real refutation of the points made by Churchill in B about the fate of Czechoslovakia or further German expansion eastwards. C is more concerned with the political support for Munich. In that sense Churchill may be right in saying that ‘everyone would like to forget or ignore’ the defeat. C refers to Chamberlain beaming with pleasure, whereas the tone of Churchill’s speech shows a sombre mood. Provenance: the nature of the sources is very different. Though on the same day, Churchill’s is a public speech intended to influence not only parliamentary but public opinion, so is a strong rhetorical statement. Chips’ diary is very much for his personal use and expresses feelings about Churchill which he would not say in public. It also reveals a superficial approach and a ‘toadying’ approach to the Prime Minister far removed from Churchill’s thundering. Churchill seems to be speaking in terms of high seriousness, yet this is seen as merely ‘antics’ by Channon. In a sense, Channon’s is a more typical view, as many MPs in the party were relieved that peace had been preserved and many at the top of the party felt that Britain was not ready for war. However, Churchill’s views were held by others in the party even if few ministers actually resigned and were more typical of attitudes when the euphoria of Munich died down and MPs reflected. The superficial popularity recorded by Channon came to seem hollow, while Churchill’s view came to be more accepted. In terms of usefulness, Channon’s source may help to understand Chamberlain’s policy, Churchill’s view may be less useful as evidence for Conservative views, as he was seen as an eccentric and the arguments presented are perhaps not very strong. However he and not Channon foresaw the fate of Czechoslovakia.

75

Page 79: History A - The Bicester Schoolthebicesterschool.org.uk/wp-content/uploads/2015/09/Mark... · 2015-09-28 · History A Mark Schemes for the Units January 2009 H106/H506/MS/R/09J

F963-02 Mark Scheme January 2009

Question Number

Answer Max Mark

4(b)

Study all the Sources.

Use your own knowledge to assess how far the Sources support the interpretation that Churchill’s criticisms of Munich were unjustified.

Focus: Judgement in context, based on the set of Sources and own knowledge Successful answers will need to make use of all five Sources, testing them against contextual knowledge and evaluating their strengths and weaknesses, including any limitations as evidence. A range of issues will be addressed in focusing upon the terms of the question but no set answer is expected.

The debate is whether the state of the armed forces, Britain’s lack of allies, the uncertainty about Imperial support and public opinion meant that Chamberlain had little choice in 1938 except to play for time or whether Churchill’s view that Munich merely brought about dishonour, ended British credibility, led to Germany gaining strength and encouraged her ambitions in eastern Europe is more justifiable. Grouping: by and large A, C, and E are critical of Churchill, while D offers a more balanced view. B obviously attempts to justify Churchill’s view.

The Churchill case is that the French alliance system in Eastern Europe was swept away, weakening a possible defence against Hitler. Czechoslovakia was likely to be engulfed and more resources would then be available if Hitler continued eastwards expansion. His final point in B is the subordination of Britain to Hitler Evaluation. At this point Churchill did not have wide support and was desperate to put over his points; he was not in the government and did not know Britain’s weaknesses first hand. He was speaking in the House of Commons and expressing a public view. His view is supported to an extent by Ismay in D who argues that German military preparations were not so strong in 1938 that Britain could not have resisted and also that by sacrificing Czechoslovakia Hitler gained the Skoda arms factory. However Ismay who was in a position to know does not back up Churchill’s view in the main. Own Knowledge. To support Churchill’s case, candidates could be aware that German arms manufacture outpaced that of Britain 1938-39.; that Germany gained Russia as an ally thanks to the gap between Munich and war in 1939; that the Czech army of 35 divisions was lost to any anti-Hitler coalition; that public opinion was not as anti-war as Chamberlain made it out to be. France did lose credibility in her alliances with Poland and the Little Entente. The alternative view is well expressed in D – the nation was more unified in 1939 than in 1938 and as we did depend on the empire, it was important that the dominions gave support which was not likely in 1938. Evaluation : It should be noted that Ismay is writing in hindsight, but at the time he was in a position to know about the state of Britain’s armed forces – which Churchill was not. The point about public opinion is well brought out by A which shows the royal family’s keen support for Munich and C which shows conservative support in parliament and the view that Churchill’s criticisms were merely antics.

[30]

76

Page 80: History A - The Bicester Schoolthebicesterschool.org.uk/wp-content/uploads/2015/09/Mark... · 2015-09-28 · History A Mark Schemes for the Units January 2009 H106/H506/MS/R/09J

F963-02 Mark Scheme January 2009

Question Number

Answer Max Mark

Evaluation Both these Sources are from the time of the crisis itself and both from within an establishment that feared the effects of another war on British society. Channon is writing a personal diary and there is no need for him to exaggerate a genuine affection for Chamberlain which was typical of many in the Conservative party. However a less personally involved Source, E, supports the view. This interpretation accuses Churchill of hypocrisy and opportunism and failing to understand the pressures of the government having to watch out for Germany, Italy and Japan. Evaluation This is in many ways fair – candidates could point out the Italian-German Axis and the problems caused by Japan’s renewed attacks in China. Churchill did reduce arms spending under the 10 year Rule and so was partly responsible for the situation in 1938. Own knowledge could include public revulsion against the losses of world War I ( a war that Churchill had enjoyed!) and the expressions of pacifism in the Peace Ballot of 1935 ( which Churchill despised).

Total for paper:

77

Page 81: History A - The Bicester Schoolthebicesterschool.org.uk/wp-content/uploads/2015/09/Mark... · 2015-09-28 · History A Mark Schemes for the Units January 2009 H106/H506/MS/R/09J

F964-01 Mark Scheme January 2009

78

F964-01 - European and World History Enquiries

Question Number Answer Max

Mark

1(a)

The First Crusade and the Crusader States 1073-1130 Study Sources A and C Compare these Sources as evidence for the problems faced by the new Crusader States. Focus: Comparison of two Sources. No set answer is expected, but candidates need to compare the contents, evaluating such matters as authorship, dating, utility and reliability, so using the Source ‘as evidence for ...’. The headings and attributions should aid evaluation and reference to both is expected in a good answer. Both Sources set out a range of problems but they differ as to the nature of those problems. Both point to issues surrounding location and the presence of hostile neighbours. Differences can be set out thus. In A the small numbers of crusaders, the dependence on good leadership, the needs of manpower and horses are discussed while in C the focus is upon the enmity of the Emperor of Byzantium; evidently he felt threatened and wanted to reassert control over lands he saw as his own preserve. The tone of C is significant: it records the Emperor’s machinations and his attempts to goad Muslim rulers into removing the crusader presence. The tone of A is, ultimately, more hopeful, placing much stress on the qualities of King Baldwin I and implying that problems could be overcome, not least through the fear engendered in the crusaders’ opponents. Both Sources convey, then, a range of problems, both internal and external. Comments on the provenances may well engage the authorship and dates of each as well as their typicality. A may be viewed as typical of a reliable contemporary chronicler, recording key features and problems at the inception of the new States; specifically the Kingdom of Jerusalem. C has obvious interest and importance, though composed later; its typicality lies in the reflection of a major, on-going problem faced by the States, namely the hostility and machinations of the Emperor of Byzantium.

[30]

Page 82: History A - The Bicester Schoolthebicesterschool.org.uk/wp-content/uploads/2015/09/Mark... · 2015-09-28 · History A Mark Schemes for the Units January 2009 H106/H506/MS/R/09J

F964-01 Mark Scheme January 2009

Question Number Answer Max

Mark 1(b)

Study all the Sources. Use your own knowledge to assess how far the Sources support the interpretation that the survival of the Crusader States in the period from 1100 to 1130 depended on the disunity of their Muslim enemies. Focus: Judgement in context based on the set of Sources and own knowledge. Successful answers will need to make use of all five Sources, testing them against contextual knowledge and evaluating their strengths and weaknesses, including any limitations as evidence. A range of issues may be addressed in focusing upon the terms of the question but no set answer is expected. The question focuses on a key reason for the survival of the four Crusader States in the face of much adversity. The Sources contain a range of points, explicit and implicit. Source E highlights some development of the Crusaders’ position. Divisions and disunity are mentioned in Source E and some of B and implied in parts of A (fear of crusader power is implied) and D. Other factors are set out in Sources A, C and D, plus parts of B. Source E shows the precarious nature of the crusaders’ position and its dependence upon the divided Moslem rulers but also Crusader strengths; Source B shows how the crusaders were able to use Muslim quiescence, overcome problems (there is some link with A on this point) and so expand their influence. Source A stresses crusader leadership and its qualities as well as luck and these features can be supported from own knowledge. A suggests initial crusader successes. Source C raises the problems posed by the Byzantine Emperor; again, own knowledge can support this by reference to the Emperor’s diplomacy and general unease over the presence of crusaders adjacent to his lands. There are links between Sources A, B and D: military action, good leadership and general strategic issues are raised. Again, own knowledge can supply support: examples of leaders; battles; Moslem successes and failures; incipient Muslim divisions and enmities between leaders. Source B can also be linked to examples of regional agreements made between crusader leaders and Moslem leaders (e.g. in 1109). Further factors could include the periodic arrival of new settlers (E); the value of sea power provided by, for example, the Genoese; the arrival of the Knights Templar in 1118 (E), a formidable if small elite fighting unit; the mixture of setbacks and successes (e.g.1119 Blood Field and Tell Danith); the development of castles and a feudal system (E); strategic and tactical awareness, including the better use of forces and responding to opponents’ tactics; leadership (the early leaders were all capable men). Better answers will see links and be aware of counter-argument to the interpretation, allowing for the view that, ultimately, no matter incipient strengths, so much turned on the unity or disunity of the crusaders’ enemies.

[70]

79

Page 83: History A - The Bicester Schoolthebicesterschool.org.uk/wp-content/uploads/2015/09/Mark... · 2015-09-28 · History A Mark Schemes for the Units January 2009 H106/H506/MS/R/09J

F964-01 Mark Scheme January 2009

Question Number Answer Max

Mark 2(a)

The German Reformation 1517-1555 Study Sources B and D. Compare these Sources as evidence for the attitudes of Charles V and his brother Ferdinand towards the German princes. Focus: Comparison of two Sources. No set answer is expected, but candidates need to compare the contents, evaluating such matters as authorship, dating, utility and reliability, so using the Sources ‘as evidence for …’. The headings and attributions should aid evaluation and reference to both is expected in a good answer. The Sources show some continuity in the attitudes of Charles V and Ferdinand. In both Sources they claim that their chief aim is a peaceful relationship. In both, they wish to avoid the princes gaining a one-sided settlement and imposing Lutheranism on Germany. But their attitudes have changed considerably by 1555. In Source B, the Emperor is expressing his personal views in a private letter to his half-sister. At this stage he seems in control and has consulted his brother, who, as King of the Romans, will succeed him as Emperor. He is exasperated after failing to reconcile the Protestants and Catholics at the Second Colloquy of Regensburg 1546, and is trying to divide and outmanoeuvre his princely enemies. However, Source D is an official document which gives less insight into their views. It is a two-sided settlement recognising toleration for both Lutheran and Catholic states, after the failure of Charles’s attempt to re-impose Catholicism in Germany. In Source B Charles is embarking on a war after failing to persuade the princes to join an imperial league which they feared would rob them of their power. However, in Source D Charles has lost the Schmalkaldic War and been forced into a peace, accepting princely power to impose Lutheranism, which he had resisted earlier. Source B shows his mistrust of the princes, after spies have reported their plotting. He wishes to crush princely attempts to undermine his plans to extend his and Ferdinand’s authority and restore Catholicism. Charles deliberately tries to ‘divide and rule’ the princes in Source B, using the pretext that some have taken lands from others and disturbed the public peace, whereas in Source D he is exhausted and Ferdinand takes the lead in backing down. Mistrust of individual Lutheran princes and a position of perceived imperial strength, in Source B, has given way to humiliation and defeat by the combined Catholic and Lutheran princes in the Schmalkaldic War. Thus, in Source D, the official line taken by Ferdinand lacks the imperious tone of Charles’s private views in Source B. Source D is signed by his brother Ferdinand on his behalf, as the Emperor refuses to personally acknowledge his defeat, is now a broken man and is in the process of abdicating his lands. This makes Source D much less useful as evidence for their views.

[30]

80

Page 84: History A - The Bicester Schoolthebicesterschool.org.uk/wp-content/uploads/2015/09/Mark... · 2015-09-28 · History A Mark Schemes for the Units January 2009 H106/H506/MS/R/09J

F964-01 Mark Scheme January 2009

Question Number Answer Max

Mark 2(b) Study all the Sources

Use your own knowledge to assess how far the Sources support the interpretation that the actions of the princes were the main reason for the survival of Lutheranism in Germany between 1545 and 1555. Focus: Judgement in context, based on the set of Sources and own knowledge. Successful answers will need to make use of all five Sources, testing them against contextual knowledge and evaluating their strengths and weaknesses, any limitations as evidence. A range of issues may be addressed in focusing upon the terms of the question but no set conclusion is expected. Sources A, B, D and E suggest that princely action was an important reason for the survival of Lutheranism. The actions of the princes are referred to in Sources B and E, are implied in A and may be explained using own knowledge. Electors within the Schmalkaldic League threatened imperial power, as future imperial elections might return a Protestant Emperor. Source B suggests they were intriguing to disrupt the Diet after withdrawing from the talks which sought religious compromise. Own knowledge might supply the evidence that Charles hoped to take advantage of papal promises of support and a temporary peace with the Turks, France and Denmark to outmanoeuvre his Lutheran enemies and restore Catholicism in the Empire. It might also be added that, in the ensuing war, summarised in Source E, Maurice of Saxony was rewarded for supporting the imperial side with the lands and electoral title of John Frederick of Saxony for aiding the victory at Mühlberg. Maurice’s change of sides and his part in the Emperor’s defeat might be used to evaluate Sources D and E on the victory of the combined forces of the Catholic and Protestant princes. Source D shows the result of this victory: the official recognition of Lutheranism and its survival in the Empire. Sources B, C and D suggest the Emperor’s actions, or inaction, were crucial. The emperor’s delays and failures in reconciling with Lutheranism are suggested in Source B. There may be own knowledge of his political weakness within the empire and his other distractions linked to his vast inheritance. Mühlberg and the Emperor’s consequent over-confidence might be linked to Source C. The unpopularity of Charles’s policy in imposing the Interim was temporary in areas of the Empire under his military control and only strengthened Lutheran opposition. It also provoked all princes to fear a loss of political power within their states, leading Catholic and Protestant princes to combine forces.

81

Page 85: History A - The Bicester Schoolthebicesterschool.org.uk/wp-content/uploads/2015/09/Mark... · 2015-09-28 · History A Mark Schemes for the Units January 2009 H106/H506/MS/R/09J

F964-01 Mark Scheme January 2009

Question Number Answer Max

Mark

Sources A and C suggest more personal reasons for the survival of Lutheranism. These include: papal inaction and delay in calling a General Council, also suggested in Source E, due to the fear of the conciliar movement, and genuine religious enthusiasm which stiffened opposition, disregarded by Charles, but apparent in Sources A and C. Own knowledge might be used to develop these ideas: for example, married clergy in Source C might be seen as soft-living persons in Source A. Luther himself also played a very important part in the foundations of survival, but during this period the movement passed into the hands of his successors, including Melanchthon, the author of Source A. Candidates are likely to consider a range of reasons taken from the wide range possible: princely actions, Charles’s mistakes, the strength of personal motives - including genuine religious enthusiasm - and papal inaction. They are likely to see an interaction between princely actions and Charles V’s mistakes. It is up to candidates to assess and decide upon relative importance here, there being no set conclusion.

Paper Total:

[70] [100]

82

Page 86: History A - The Bicester Schoolthebicesterschool.org.uk/wp-content/uploads/2015/09/Mark... · 2015-09-28 · History A Mark Schemes for the Units January 2009 H106/H506/MS/R/09J

F964-02 Mark Scheme January 2009

83

F964-02 - European and World History Period Studies Option B: Modern 1774-1975

Question Number Answer Max

Mark

1(a)

The Origins and the Course of the French Revolution 1774-1795. Study Sources B and C Compare these Sources as evidence for the unrest in the French provinces in the summer of 1789. Focus: Comparison of two Sources No set answer is expected, but candidates need to compare the contents, evaluating such matters as authorship, dating, utility and reliability; so using the Source ‘as evidence for….’. The headings and attributions should aid evaluation and reference to both is expected in a good answer. In terms of content, both B and C offer evidence of unrest; C refers to the destruction of chateaux and B confirms this, naming places attacked. C refers to the seizure of records of dues, which B does not – theft and banditry are the complaints here, whereas C does not refer to this – it is destruction, but as protest not for gain. There is no reference to punitive actions in C. Instead the actions are justified by the humiliations suffered by the people – barbarous remains of feudal laws. In B there is no justification but condemnation of illegal and violent acts justly punished by shooting and subsequent executions. Justice here is enforcing order; Justice in C is the ending of inhumane laws. In terms of provenance, though referring to similar events at the same period – that of the Grand’Peur of summer 1789, the two sources have different origins and purpose. Both are trying to persuade, but have different aims. C wants to use the peasant violence to persuade the assembly to end feudal obligations. Though a provincial noble like those writing in B he has a different agenda – reform. The authors of B do not call for reform but effective police support against bandits – not revolutionary peasants. Their aim is to shock the Assembly into action – but action of a different sort. The letter is from people actually in the countryside feeling the effects of disorder; C is a motion from a liberal noble in the capital, swept away by the revolutionary events in 1789 and seeking to establish a more equal society. In a sense both are typical – since the joining of many liberal nobles and clergy were seized by a vision of change; but the fears of B were typical of many who subsequently fled abroad to avoid the revolution or supported provincial counter-revolution.

[30]

Page 87: History A - The Bicester Schoolthebicesterschool.org.uk/wp-content/uploads/2015/09/Mark... · 2015-09-28 · History A Mark Schemes for the Units January 2009 H106/H506/MS/R/09J

F964-02 Mark Scheme January 2009

Question Number Answer Max

Mark 1(b)

Study all the Sources. Use your own knowledge to assess how far the Sources support the interpretation that the main motive for revolutionary activity in 1789 was economic. Focus: Judgement in context, based on the set of Sources and own knowledge. Successful answers will need to make use of all five Sources, testing them against contextual knowledge and evaluating their strengths and weaknesses, including any limitations as evidence. A range of issues may be addressed in focusing upon the terms of the question but no set answer is expected. The debate here is between explanations which focus on the economic discontents 1787-9, the rising prices, the unemployment in the swollen city of Paris; the falling income from taxes which made the crown’s financial position harder; the poor harvests and land hunger in the countryside – and explanations which see 1789 much more in political terms. These arguments focus on the political crisis arising from the deficit and the calling of the Estates general; the political failures of the Crown to manage the new assembly; the politicization of France by the discussion of cahiers and the elections of 1789; the desire for a constitution and enlightenment influences for more freedom, toleration and more efficient government. . Grouping the Sources The Sources which stress economic discontents are C and E. Economic factors may lie behind the violence as seen by B. A is much more political as is D which represents the political influence of the Enlightenment. Economic C sees economic oppression as the farmers ‘groan’ under dues and suffer a heavy burden’ the countryside is seen as desolated. However this Source has a distinct intention of modernizing and liberalizing and is justifying rural unrest. This could be assessed with knowledge of the immediate problems 1787-9 and from knowledge of taxes and dues affecting the peasants. This could also be linked to B, though the authors of the Source do not make the connection. This Source sees mere vandalism and banditry rather than an expression of economic hardship, but it is written by a member of the propertied classes directly affected by the violence who would not necessarily be analyzing the economic causes. E links political weakness of the crown in being forced back to Paris with the economic problems of the capital. This could be supported from own knowledge of discontents or might be challenged by knowledge of the growing political awareness of the Paris populace given the large number of revolutionary tracts and the influence of mob orators.

84

Page 88: History A - The Bicester Schoolthebicesterschool.org.uk/wp-content/uploads/2015/09/Mark... · 2015-09-28 · History A Mark Schemes for the Units January 2009 H106/H506/MS/R/09J

F964-02 Mark Scheme January 2009

Question Number Answer Max

Mark

Non-Economic For A the issue is of the duty of the representatives to give France a constitution, a long term political aim not motivated by economic problems. Politics is at the heart of A and the background could be explained. Some may know of Sieyès’ famous pamphlet – What is the Third Estate? And the political demands of the Third Estate in 1789 in the face of their treatment by the King in May 1789.However the author is an educated middle class intellectual not likely to have been directly affected by the economic downturns of 1787-9 eager to push a political agenda. The representative to the Estate General did not include those who suffered most from economic hardship and were more concerned with legalistic/constitutional issues. D shows that there were wider political demands based on ideas of tolerance and enlightenment not necessarily linked directly to economic discontent. Reflects the interests of an enlightened member of the elite in toleration rather than economic problems and is written at a period when the Assembly was intoxicated by its desire for all sorts of reforms. By this time the Paris crowds and the peasantry had issues which went beyond this enlightenment liberalism and were more concerned with economic discontents. Judgements Some may argue that the bad harvests and urban discontents together with the longer term economic factors such as the wealth of the bourgeoisie not being reflected in any political power or the resentment at taxation and feudal dues were more important motives purely political considerations encouraged by the enlightenment, the increasing political awareness and the rise in number of books, pamphlets and political debate. At key points popular discontent pushed along the revolution, for example 14 July, the summer violence in the countryside or the October disturbances described in D. Others may take the view that there was an interaction between political and economic motives – that awareness of economic inequalities such as shown in C leads to demands for political change. Others may argue that despite the pressure of economic events the essential motives for the key players were political change or a more enlightened society.

[70]

85

Page 89: History A - The Bicester Schoolthebicesterschool.org.uk/wp-content/uploads/2015/09/Mark... · 2015-09-28 · History A Mark Schemes for the Units January 2009 H106/H506/MS/R/09J

F964-02 Mark Scheme January 2009

Question Number Answer Max

Mark 2(a)

The Unification of Italy 1815-70 Study Sources A and B. Compare these Sources as evidence for the failure of the revolt in Piedmont in 1821. Focus: Comparison of two Sources. No set answer is expected, but candidates need to compare the contents, evaluating such matters as authorship, dating, utility and reliability, so using the Sources ‘as evidence for …’. The headings and attributions should aid evaluation and reference to both is expected in a good answer. Both accounts refer to the betrayal of the revolution which, in Source A, is attributed specifically to Charles Albert and implicitly in Source B: Mazzini highlights the conservatism of the monarchy in broader terms by emphasising the role of Charles Felix too. The intervention of the Austrians is highlighted as a key factor in Source A and Source B pinpointing the battle of Novara as the only major engagement. The more discerning may comment on the indeterminate degree of popular support. In Source A ‘popular affection’ in support of reform is mentioned with no real assessment of popular commitment and in Source B the implication of the reference to ‘local people’ is that support for the rebels may have been retrospective and limited to a few. The key difference between the sources concerns the rebels themselves. In Source A they are described as having ‘fought bravely’ but in Source B it is claimed they ‘did not resist’ and the rebel leaders refused to rise to the challenge of local people to resist in Genoa. The convergence of the accounts is explained by the historical reality of events. Charles Albert was not prepared to go as far as the rebels hoped and finally went into exile. It is also incontestable that Austrian intervention was decisive. The divergence between the sources can be explained, in part, by their authorship. In Source A Santarosa, as leader of the revolt, was anxious to deflect criticism from himself so it served his purpose to stress the weakness of Charles Albert more than is the case in Source B and for the same reason it was in his interest to claim the rebels fought bravely. Mazzini’s criticism of the rebel leadership might be questioned. He was only a teenager in 1821 (born 1805) so his understanding of the events may have been limited, (even influenced by his mother). Further, his account is written many years after the event providing him with the opportunity to write a history in line with his own political views and reputation at the time of writing. Here, candidates might stress Mazzini’s preference for a populist movement (Young Italy) to not only reform states like Piedmont (the limited ambition of Santarosa) but unite Italy.

[30]

86

Page 90: History A - The Bicester Schoolthebicesterschool.org.uk/wp-content/uploads/2015/09/Mark... · 2015-09-28 · History A Mark Schemes for the Units January 2009 H106/H506/MS/R/09J

F964-02 Mark Scheme January 2009

Question Number Answer Max

Mark 2(b)

2(b)

Study all the Sources. Use your own knowledge to assess how far the Sources support the interpretation that the revolutions in Italy in 1820-21 and 1831 left the nationalist cause without hope. Focus: Judgement in context, based on the set of Sources and own knowledge. Successful answers will need to make use of all four Sources, testing them against contextual knowledge and evaluating their strengths and weaknesses, any limitations as evidence. A range of issues may be addressed in focusing upon the terms of the question but no set conclusion is expected. Sources C and E are more likely to be seen as supportive of the interpretation whereas Sources A and B are likely to be considered as evidence against the interpretation. Source D might be used either way. In fact, there is room for argument on several of the sources. Source C might be interpreted as clear confirmation that the nationalist cause was left without hope. Describing revolution as ‘fantasy’ it claims that important sections of society were opposed to change preferring to sustain their links with Austria. The last sentence states clearly that the experience of recent disturbances caused ‘widespread disillusionment’. The prosperity of Lombardy, referred to in the source, helped generate benign attitudes to Austria and the disturbances in neighbouring Modena and the Papal States in 1831, which challenged Austrian power, were regarded with suspicion. It was only in 1848 when the economic fortunes of the area deteriorated that hostility to Austria emerged on any scale. At least a ‘few fools’ were not discouraged and the author’s claim that the higher classes backed Austria allows the possibility that the lower orders felt differently. The reliability of this account might be questioned as it was written by someone in the service of the Austrians. The author’s judgement is questionable it was the middle classes, regarded by the author as supportive of Austria, who were at the forefront of the revolution in 1848. However, the loyalty of the nobility and the clergy, stressed in the source, remained steadfast. Source E reinforces Source C. The implication is that the activities of secret societies in the early part of the century undermined the hopes of those who wanted change by creating fear and delayed independence. D’Azeglio argues that the young men of Milan were disinterested in revolutionary activity. The war of 1859 and subsequent political developments in 1860 which achieved the independence he refers to explains why ‘men are only just beginning to be less frightened of liberty’. The political views ascribed to the author in the introduction may be developed by reference to him as prime minister of Piedmont, 1849-52, when he strove to modernise the state by consensus and reform. Such an assessment of his political views could be deployed to explain his hostility to the events of the 1820s and 1830s.

87

Page 91: History A - The Bicester Schoolthebicesterschool.org.uk/wp-content/uploads/2015/09/Mark... · 2015-09-28 · History A Mark Schemes for the Units January 2009 H106/H506/MS/R/09J

F964-02 Mark Scheme January 2009

Question Number Answer Max

Mark

Source D supports the view that the nationalist cause was without hope as the masses were ‘without faith in the future’. Reference to the various revolutions in the years 1820-21 and 1831 in Naples, Piedmont and the Papal States could be made to explain how early hopes faded, as stated in the source. The identification of poor leadership as the factor to explain failure might be developed by commenting on the Carbonari and other groups. However, whilst at face value Mazzini’s comments suggest an absence of hope, in fact, they disguise a real optimism. It is clear that Mazzini was hopeful that millions could be roused ‘to the defence of the cause’ with the right leadership. Candidates might comment on Mazzini’s confidence in his own ideas and his ability to lead the nationalist cause with particular reference to Young Italy which he launched in the year in which these remarks were published. Mazzini appears to be more positive about hopes for the future in Source B. Indeed, some candidates may prefer to pair Source B with Source A which might also be regarded as revealing optimistic attitudes. Both acknowledge the reality of defeat in Piedmont in 1821 and the clear disappointment of both leaders and their supporters. The sense of betrayal is evident in both and in Source B the revolutionaries are described as ‘distressed’. Yet, the last lines of Source A present a belief in final victory even if they were the plaintive words of a leader trying to put a brave face on things. The grandiose language of his appeal and the unconvincing assessment made about Austrian power are worth analysis. Similarly, in Source B the courage of the Genoese is stressed and suggests that despite defeat in Turin hope remained. The author admits that the events he witnessed revealed to him the desire and duty of Italians to continue the struggle. Given the date of publication of Source B it is possible to argue that it was easy for Mazzini to claim hope was strong in 1821 writing at a point in history when unification had been achieved. In evaluating both Source A and Source B refer to events in Piedmont only and are confined to the year 1821 and, as such, they do not provide sufficient evidence to comment on the revolutions of 1820-21 and 1831 in their entirety. Indeed, it is possible that some candidates will place emphasis on the negative aspects of the content of these two sources and the less than reliable features of their provenance.

[70]

88

Page 92: History A - The Bicester Schoolthebicesterschool.org.uk/wp-content/uploads/2015/09/Mark... · 2015-09-28 · History A Mark Schemes for the Units January 2009 H106/H506/MS/R/09J

F964-02 Mark Scheme January 2009

Question Number Answer Max

Mark 3(a)

The Origins of the American Civil War 1820-61 Study Sources A and E. Compare these Sources as evidence for opinions on the abolition of slavery. Focus: Comparison of two Sources. No set answer is expected, but candidates need to compare the contents, evaluating such matters as authorship, dating, utility and reliability, so using the Sources ‘as evidence for …’. The headings and attributions should aid evaluation and reference to both is expected in a good answer. The sources agree that the institution of slavery is unacceptable although Source A is more strident in its denunciation of it categorically stating it ‘should no longer be tolerated’ whereas Source E merely suggests a ‘cure’ which implies that the practice of slavery needed to be addressed but with less urgency. They differ on the best way to tackle the problem and their attitude to slaves. The author of Source A implies that slaves should be freed immediately in the comments he makes about the hypothetical reversal of the situation if whites were enslaved. In stark contrast the editorial in Source E dismisses outright abolition as ‘twaddle’ expressing a preference for ‘colonization’. Source A asserts the equality of slaves with other American citizens because the majority were born in America. However, Source E indicates that slaves do not deserve equal rights by suggesting they are intellectually inferior and that they will only be free if they return to ‘the land of their forefathers’. There was common ground between the Colonization Society and other abolitionists. The fact that Garrison was invited to speak to the former indicates a friendly relationship between the two at that time. Quakers and other Christian foundations supported both movements. However, the Colonization Society had southern slave owners from the upper slave states like Kentucky and Virginia as members. Supporters of immediate abolition were almost all Northerners. This helps to explain the clear difference between the sources. Those who favoured colonization as in Source E did so because it allowed for the gradual demise of slavery. In Source A Garrison, and the Anti-Slavery Society he started three years later was impatient for change as they had less to lose. The slow progress of colonization might explain Garrison’s preference for abolition: by 1830 more slaves were born in a week in the USA than were sent to Liberia in a year. His emphasis on equality of privileges might be explained by the date within the year on which the speech was delivered. The caution of the editorial reflects the less dogmatic and practical approach of those who were aware of the racial problems that immediate abolition might create.

[30]

[30]

89

Page 93: History A - The Bicester Schoolthebicesterschool.org.uk/wp-content/uploads/2015/09/Mark... · 2015-09-28 · History A Mark Schemes for the Units January 2009 H106/H506/MS/R/09J

F964-02 Mark Scheme January 2009

Question Number Answer Max

Mark 3(b)

Study all the Sources. Use your own knowledge to assess how far the Sources support the interpretation that the campaign for the abolition of slavery failed to win popular support in America before 1850. Focus: Judgement in context, based on the set of Sources and own knowledge. Successful answers will need to make use of all four Sources, testing them against contextual knowledge and evaluating their strengths and weaknesses, any limitations as evidence. A range of issues may be addressed in focusing upon the terms of the question but no set conclusion is expected. According to Calhoun in Source C both races will benefit ‘if not disturbed by the fierce spirit of abolition’, consistent with the ‘good’ relations that existed. He warns that emancipation would lead to ‘social and political equality’ and ultimately the subservience of whites to Negroes. He claims to represent the views of those in his constituency so it could be argued that abolition clearly did not have the support of his constituents at least even if his mandate was limited to white voters only. However, candidates may also acknowledge that Calhoun was successful in his proposal which was only possible with the support of Senators from other states, including many from the North. South Carolina, which he represented, was strongly resistant to change as shown in subsequent events, notably the secession of the state in 1860. In addition, the memory of slave rebellion was acute in South Carolina after the uprising in Charleston in 1822 and the Nat Turner Rebellion in neighbouring Virginia in 1831 which were blamed by Southerners on abolitionist agitators. Sources D and E confirms that racism was a reason for opposition to abolition. Source E concedes that attitudes in both North and South were entrenched that the Negro would always be regarded as a slave and inferior to whites. Despite its sympathy for change it dismisses abolition as ‘twaddle’. Candidates should be able to comment on how typical this view was of opinion in the North. Cross reference to Source A and the mention of The Liberator would be one way of illustrating the existence of papers with a contrary viewpoint. Source D explicitly recognises racism in the opening sentence and whilst the author concedes that racial division was less in 1843 than ten years previously the fact that he says only ‘we begin to feel that we are on the same footing’ implies that there was a long way to go. The insecurity of fugitive slaves in the North and the ways in which Negroes were treated differently in the North might be mentioned. Garrison in Source A anticipates a struggle and even implies a clash with opponents. His methods were controversial and arguably counterproductive. In 1840 he formed the Liberal Party to fight the presidential election and help explain the split in the Anti-Slavery Society that year. Source B also indicates that there was considerable hostility to abolition in the town of Hartford at least. The numbers of agitators against abolition might be interpreted as reflective of substantial or limited opposition depending on the views of the candidate but the determination and strength of feeling of the opponents is not in doubt.

90

Page 94: History A - The Bicester Schoolthebicesterschool.org.uk/wp-content/uploads/2015/09/Mark... · 2015-09-28 · History A Mark Schemes for the Units January 2009 H106/H506/MS/R/09J

F964-02 Mark Scheme January 2009

Question Number Answer Max

Mark

4(a)

Yet, abolition did have considerable support. Source E advocates freedom for the slave by colonization though by the date of publication it was a largely discredited notion (only 10,000 slaves had been returned to Liberia by 1860). It is clear from Sources B and D that there was interest in abolition. Both authors refer to abolitionists who were not only active but also brave (‘decent intelligent people’ and ‘a noble band of fearless women’, according to Source B, and Quakers are singled out in Source D as well as ‘a host of American friends’). Indeed, by 1838 the Anti-Slavery Society had 38,000 members most of them drawn from the educated, wealthy and religious with women prominent. This is indicated in Sources B and D explicitly and by implication given the contrasting reference to opponents as ‘savages’ and a ‘mob’. The evidence is fairly reliable as in writing to his wife the author of Source B might have understated the danger he faced in order to allay her fears and if so it might underestimate the strength of opposition. Source D is even-handed in its assessment of opponents and supporters of abolition. Reference might be made to the experiences of other escaped slaves such as Frederick Douglass, to corroborate the evidence of Source D, who campaigned tirelessly for abolition in much the way that the author of Source B described. Even Source C provides evidence of support for abolition. Calhoun’s appeal for a ‘gag’ on petitions to Congress was a reaction to the deluge of petitions asking Congress to abolish slavery similar to the huge quantities of abolition literature being mailed to southern residents. Even the impact of the rhetoric of Source A might be assessed on the audience both immediate and afar. There is scope to expand on the role of The Liberator again and the formation of the Anti-Slavery Society in 1833. Dictatorship and Democracy in Germany 1933-63. Study Sources A and B. Compare these Sources as evidence for the effectiveness of opposition to the Third Reich. CONTENT Both sources agree that opposition was not especially effective. However whilst A is very pessimistic on the ability of the SPD to oppose, citing their illegality, the denial of plans to associate and the prevalence of spies, the Gestapo in B does acknowledge some limited opposition success in the workplace and in workers' organisations in getting their message across. For example, the mumbling of a response to a Hitler salute. It also comments on SPD training, experience and cohesion. Nevertheless, neither make any reference to more serious opposition (strikes, risings, mass disturbance etc). Interestingly, the SOPADE report in A doesn't mention the KPD whereas the gestapo in B refers to both SPD and KPD commenting that the KPD were more effective in the factory itself than the SPD with a different tradition in both factory and workers' leisure organisations.

[70]

91

Page 95: History A - The Bicester Schoolthebicesterschool.org.uk/wp-content/uploads/2015/09/Mark... · 2015-09-28 · History A Mark Schemes for the Units January 2009 H106/H506/MS/R/09J

F964-02 Mark Scheme January 2009

Question Number Answer Max

Mark

PROVENANCE Given their corroboration, both are valid pieces of evidence, coming as they do from two of the organisations pitted against each other, the Gestapo and an SPD agent. Perhaps inevitably the SPD agent on a personal level feels the inhibition of the block warden while the Gestapo, whose job it was to create a new totalitarian control and Volksgemeinschaft vision, feels the frustration of a 'word of mouth opposition'. Given this mutual frustration, both in practice concur. However, they are talking of different areas - the domestic sphere of the home in A, the factory (Dusseldorf) in B. Both are private reports, reflecting genuinely held opinion, with no motive to distort other than to give vent to a certain frustration of the effectiveness of each other's tactics. JUDGMENT Candidates might conclude that the evidence in B is better given its recognition of low level activity and its wider frame of reference, but it is possible to make a case for A on the grounds of the wider Nazi aim of indoctrination of the masses.

92

Page 96: History A - The Bicester Schoolthebicesterschool.org.uk/wp-content/uploads/2015/09/Mark... · 2015-09-28 · History A Mark Schemes for the Units January 2009 H106/H506/MS/R/09J

F964-02 Mark Scheme January 2009

Question Number Answer Max

Mark 4(b)

Study all the Sources. Use your own knowledge to assess how far the Sources support the interpretation that the Gestapo and the other security services were able to control the German population mainly because they were ‘all seeing and all knowing’. Grouping: A, C and D seem to support the assertion. The impression given in both sources is that, directly or indirectly, the Gestapo were ‘everywhere’. However, whether D belongs in this group is more questionable Maria Kraus is a private citizen, not employed by the Gestapo It is quite likely that she bears a long standing personal grudge against Ilse Totzke and sees her denunciation as an opportunity to settle old scores. She might well have been acting on the Gestapo’s exhortation to denounce ‘undesirables, but this does not necessarily mean that the Gestapo themselves were ‘all seeing’ and ‘all knowing.’ The fact that this is wartime could well have a bearing since, naturally, ‘aliens’ were suspected far more. C also is from wartime. At face value it does look as if the Gestapo were omnipresent. However, the tone and content of the source might suggest that there was not enough Gestapo presence. The SD report is asking officials to be more vigilant, expanding their watchful presence into new areas. The last sentence is relevant here. E clearly suggests that there was very little physical Gestapo presence as shown by the figures given for Essen. It points out the heavy reliance on denunciations [for whatever reason] shown in D. The author gives other reasons for the feeling of being ‘watched over’. However it refers to a large army of “spies and informers”. Own knowledge: This is a central and mainstream aspect of Nazi Germany so candidates should have no problem in drawing on examples to support their evaluation. There is the contrast between the image and the reality. Candidates can offer judgements on how successful or unsuccessful the Gestapo was the implications of rule by fear and terror in a state with totalitarian aspirations. Did the feeling that they were constantly being spied upon cause them to ‘retreat into the private sphere’? Candidates might refer to the effectiveness or otherwise of the Gestapo and other security services. One can expect other means of controlling the population to be offered: eg, propaganda and indoctrination, control of the media, elements of consent etc.. Evaluation: Relevant comments can be expected about the provenance of sources A-D. Good use of contextual knowledge can be used here, as can good cross referencing, for example D and E,, B and C etc. There is no set conclusion, candidates can either agree with the assertion. Indeed, For example, sources C and D can be interpreted either way.

[70]

93

Page 97: History A - The Bicester Schoolthebicesterschool.org.uk/wp-content/uploads/2015/09/Mark... · 2015-09-28 · History A Mark Schemes for the Units January 2009 H106/H506/MS/R/09J

F964-02 Mark Scheme January 2009

Question Number Answer Max

Mark

5(a)

5(b)

The USA and the Cold War in Asia 1945-75 Study Sources C and D. Compare these Sources as evidence for the attitudes of the USA and the USSR towards intervention in Korea. Focus: Comparison of two Sources No set answer is expected, but candidates need to compare the contents, evaluating such matters as authorship, dating, utility and reliability, so using the Source ‘as evidence for…’. The headings and attributions should aid evaluation and reference to both is expected in a good answer. The provenances and tones of the two Sources can be examined and contrasted. Both suggest the importance of the intervention of both Superpowers but there are significant differences. In C the approach is cautious but Stalin is prepared to help; there is a sense of being persuaded to that position, where he is ‘ready to help’. In D the author is clear that the USSR was indeed behind the invasion of the South and he argues that the USA has to act, citing several reasons, including the defence of Japan and the need to stop South Korea becoming a ‘Soviet puppet’. Both set out intervention as a decision area and see South Korea as an important regional issue. Comment on the provenances may include the nature of the authorship and the dates, with D being a reflection on events and decisions at the time. C is very much contemporaneous and reflects a decision at the very top, with D reflecting the response. Both can be seen as typical of attitudes and stances at that period of the Cold War. Study all the Sources. Use your own knowledge to assess how far the Sources support the interpretation that the Korean War was made possible only because of the Cold War rivalry between the USA and the USSR. Focus: Judgement in context based on the set of Sources and own knowledge. Successful answers will need to make use of all five Sources, testing them against contextual knowledge and evaluating their strengths and weaknesses, including any limitations as evidence. A range of issues may be addressed in focusing upon the terms of the question but no set answer is expected.

[30]

94

Page 98: History A - The Bicester Schoolthebicesterschool.org.uk/wp-content/uploads/2015/09/Mark... · 2015-09-28 · History A Mark Schemes for the Units January 2009 H106/H506/MS/R/09J

F964-02 Mark Scheme January 2009

Question Number Answer Max

Mark The focus of the question is the argument and counter-argument

around the key causes of the Korean War. The Sources offer a variety of useful points, explicitly or implicitly. They can be grouped accordingly. Source E provides an effective overview, raising several possible or likely causes, and Sources A on North Korean help, B on their appeal for Soviet help, and C on the firming-up of that support, all are valuable in answering the question. Source D focuses on the US defence of South Korea once attacked, while Source E does mention the ambitions of Kim II Sung in the North and Syngman Rhee in the South. Their rivalries are a key focus. But some argue that the support of the USSR and China was a decisive factor. Sources C and D are central to the view that USSR support was crucial. Source A makes no mention of the USSR while Source B is altogether very cautious about support for North Korea and Source E provides several other factors and views, effectively summarising the different opinions about the origins of the Korean War. Own knowledge can supply support here: the roles of China and the USSR and the view that Stalin acted once he thought Mao had backed North Korean action; also, the view that the South Korean governments had provoked Northern aggression, quite possibly to fit its own agenda (the ‘rollback’ thinking). The personal roles and even agendas of the Presidents of the North and South might be examined. Stalin’s attitude was indeed one of caution, even reservation (as in B), but altered within a year (as in C). The caution expressed by the North Korean regime – no action without Soviet backing – might be assessed. Source D expresses the American view of the origins of the War: North Korean aggression based on Soviet support; it puts the War into a wider context with the reference to the need to defend Japan. C and D can be linked and assessed for viewpoints. Source A can be used with own knowledge for the view that the North Koreans believed they were pursuing a true civil war and would be supported by extensive networks in the South (in fact, such had been crushed by late 1949). Candidates could cite the US arguments advanced in 1950, not least in the forum of the UN, about Soviet-backed aggression against the South, while also citing the counter-views put forward at the time by North Korea, the USSR and China, seeing the South Korean regime at fault and viewing the War as a liberation struggle. Again, Source E usefully surveys several causes without prioritising such. Better answers will see links but also argue and counter-argue from the grouped Sources and consider any factors omitted or mentioned briefly.

[70]

95

Page 99: History A - The Bicester Schoolthebicesterschool.org.uk/wp-content/uploads/2015/09/Mark... · 2015-09-28 · History A Mark Schemes for the Units January 2009 H106/H506/MS/R/09J

96

Grade Thresholds

Advanced GCE History H106 H506 January 2009 Examination Series Unit Threshold Marks

Unit Maximum Mark

A B C D E U

Raw 100 75 65 55 46 37 0 F961 UMS 100 80 70 60 50 40 0 Raw 100 75 65 55 46 37 0 F962 UMS 100 80 70 60 50 40 0 Raw 100 74 65 56 47 39 0 F963 UMS 100 80 70 60 50 40 0 Raw 100 74 65 56 47 39 0 F964 UMS 100 80 70 60 50 40 0

For a description of how UMS marks are calculated see: http://www.ocr.org.uk/learners/ums_results.html Statistics are correct at the time of publication.

Page 100: History A - The Bicester Schoolthebicesterschool.org.uk/wp-content/uploads/2015/09/Mark... · 2015-09-28 · History A Mark Schemes for the Units January 2009 H106/H506/MS/R/09J

OCR (Oxford Cambridge and RSA Examinations) 1 Hills Road Cambridge CB1 2EU OCR Customer Contact Centre 14 – 19 Qualifications (General) Telephone: 01223 553998 Facsimile: 01223 552627 Email: [email protected] www.ocr.org.uk For staff training purposes and as part of our quality assurance programme your call may be recorded or monitored

Oxford Cambridge and RSA Examinations is a Company Limited by Guarantee Registered in England Registered Office; 1 Hills Road, Cambridge, CB1 2EU Registered Company Number: 3484466 OCR is an exempt Charity OCR (Oxford Cambridge and RSA Examinations) Head office Telephone: 01223 552552 Facsimile: 01223 552553 © OCR 2009

Page 101: History A - The Bicester Schoolthebicesterschool.org.uk/wp-content/uploads/2015/09/Mark... · 2015-09-28 · History A Mark Schemes for the Units January 2009 H106/H506/MS/R/09J

Oxford Cambridge and RSA Examinations

GCE

History A Advanced GCE A2 H506

Advanced Subsidiary GCE AS H106

Mark Schemes for the Units June 2009

HX06/MS/R/09J

Page 102: History A - The Bicester Schoolthebicesterschool.org.uk/wp-content/uploads/2015/09/Mark... · 2015-09-28 · History A Mark Schemes for the Units January 2009 H106/H506/MS/R/09J

OCR (Oxford Cambridge and RSA) is a leading UK awarding body, providing a wide range of qualifications to meet the needs of pupils of all ages and abilities. OCR qualifications include AS/A Levels, GCSEs, OCR Nationals, Key Skills, Entry Level qualifications, NVQs and vocational qualifications in areas such as IT, business, languages, teaching/training, administration and secretarial skills. It is also responsible for developing new syllabuses to meet national requirements and the needs of students and teachers. OCR is a not-for-profit organisation; any surplus made is invested back into the establishment to help towards the development of qualifications and support which keep pace with the changing needs of today’s society. This mark scheme is published as an aid to teachers and students, to indicate the requirements of the examination. It shows the basis on which marks were awarded by Examiners. It does not indicate the details of the discussions which took place at an Examiners’ meeting before marking commenced. All Examiners are instructed that alternative correct answers and unexpected approaches in candidates’ scripts must be given marks that fairly reflect the relevant knowledge and skills demonstrated. Mark schemes should be read in conjunction with the published question papers and the Report on the Examination. OCR will not enter into any discussion or correspondence in connection with this mark scheme. © OCR 2009 Any enquiries about publications should be addressed to: OCR Publications PO Box 5050 Annesley NOTTINGHAM NG15 0DL Telephone: 0870 770 6622 Facsimile: 01223 552610 E-mail: [email protected]

Page 103: History A - The Bicester Schoolthebicesterschool.org.uk/wp-content/uploads/2015/09/Mark... · 2015-09-28 · History A Mark Schemes for the Units January 2009 H106/H506/MS/R/09J

CONTENTS

Advanced GCE History (H506)

Advanced Subsidiary GCE History (H106)

MARK SCHEMES FOR THE UNITS

Unit/Content Page

AS/A2 HISTORY SYLLABUS-SPECIFIC MARKING INSTRUCTIONS 1

F961 British History Period studies Option A : Medieval and Early Modern 1035-1642 10

F961-02 - British History Period Studies Option B: Modern 1783-1994 21

F962 European and World History Period Studies Option A: Medieval and Early Modern 1095-1609 33

F962 European and World History Period Studies Option B: Modern 1795-2003 40

F963 British History Enquiries Option A: British History Enquiries 1066-1660 50

F963 British History Enquiries Option B: British History Enquiries 1815-1945 55

F964 European and World History Enquiries Option A: Medieval and Early Modern 1073-1555 63

F964 European and World History Enquiries Option B: Modern 1774-1975 67

Grade Thresholds 78

Page 104: History A - The Bicester Schoolthebicesterschool.org.uk/wp-content/uploads/2015/09/Mark... · 2015-09-28 · History A Mark Schemes for the Units January 2009 H106/H506/MS/R/09J

Marking Instructions

AS/A2 HISTORY SYLLABUS-SPECIFIC MARKING INSTRUCTIONS

AS UNIT F961 & UNIT F962 – PERIOD STUDIES Distribution of marks for each level that reflects the Unit’s AOs and corresponds to the UMS 2 answers: each maximum mark 50.

A01a A01b IA 21-24 24-26

IB 18-20 22-23

II 16-17 19-21

III 14-15 16-18

IV 12-13 13-15

V 9-11 11-12

VI 4-8 6-10

VII 0-3 0-5

Notes:

(i) Allocate marks to the most appropriate level for each AO.

(ii) If several marks are available in a box, work from the top mark down until the best fit has been found.

(iii) Many answers will not fall at the same level for each AO.

(iv) Analysis refers to developed explanations; evaluation refers to the argued weighing up/assessment of factors in relation to their significance in explaining an issue or in explaining linkages between different factors.

1

Page 105: History A - The Bicester Schoolthebicesterschool.org.uk/wp-content/uploads/2015/09/Mark... · 2015-09-28 · History A Mark Schemes for the Units January 2009 H106/H506/MS/R/09J

Marking Instructions

AOs AO1a AO1b Total mark for each question = 50

Recall, select and deploy historical Demonstrate understanding of the past through knowledge appropriately, and explanation, analysis and arriving at substantiated communicate knowledge and judgements of: understanding of history in a clear - key concepts such as causation, consequence, and effective manner. continuity, change and significance within an historical

context; - the relationships between key features and characteristics of the periods studied

Level IA

Uses a wide range of accurate, detailed and relevant evidence

Accurate and confident use of appropriate historical terminology

Answer is clearly structured and coherent; communicates accurately and legibly

21-24

Clear and accurate understanding of key concepts relevant to analysis and to the topic

Clear and accurate understanding of the significance of issues in their historical context

Answer is consistently and relevantly analytical with developed and substantiated explanations, some of which may be unexpected

The argument evaluates a range of relevant factors and reaches clearly substantiated judgements about relative importance and/or links.

24-26

Level IB

Uses accurate, detailed and relevant evidence Accurate use of a range of

appropriate historical terminology Answer is clearly structured and

mostly coherent; writes accurately and legibly

18-20

Clear and accurate understanding of most key concepts relevant to analysis and to the topic

Answer is mostly consistently and relevantly analytical with mostly developed and substantiated explanations

Clear understanding of the significance of issues in their historical context.

Substantiated judgements about relative importance of and/or links between factors will be made but quality of explanation in support may not be consistently high.

22-23

Level II

Uses mostly accurate, detailed and relevant evidence which demonstrates a competent command of the topic Generally accurate use of historical terminology Answer is structured and mostly

coherent; writing is legible and communication is generally clear

16-17

Mostly clear and accurate understanding of many key concepts relevant to analysis and to the topic

Clear understanding of the significance of most relevant issues in their historical context

Much of the answer is relevantly analytical and substantiated with detailed evidence but there may be some description

The analysis of factors and/ or issues provides some judgements about relative importance and/or linkages.

19-21

Level III

Uses accurate and relevant evidence which demonstrates some command of the topic but there may be some inaccuracy

Answer includes relevant historical terminology but this may not be extensive or always accurately used

Most of the answer is organised and structured; the answer is mostly legible and clearly communicated

14-15

Some/uneven understanding of key concepts relevant to analysis and of concepts relevant to their historical context

Answers may be a mixture of analysis and explanation but also simple description of relevant material and narrative of relevant events OR answers may provide more consistent analysis but the quality will be uneven and its support often general or thin.

Answer considers a number of factors but with very little evaluation of importance or linkages between factors/issues

Points made about importance or about developments in the context of the period will often be little more than assertions and descriptions

16-18

2

Page 106: History A - The Bicester Schoolthebicesterschool.org.uk/wp-content/uploads/2015/09/Mark... · 2015-09-28 · History A Mark Schemes for the Units January 2009 H106/H506/MS/R/09J

Marking Instructions

AOs AO1a AO1b Level IV

There is deployment of relevant knowledge but level/accuracy of detail will vary; there may be some evidence that is tangential or irrelevant.

Some unclear and/or under-developed and/or disorganised sections; mostly satisfactory level of communication.

12-13

Understanding of key concepts relevant to analysis and the topic is variable but in general is satisfactory.

Limited and patchy understanding of a few relevant issues in their historical context.

Answer may be largely descriptive/ narratives of events and links between this and analytical comments will typically be weak or unexplained OR answers will mix passages of descriptive material with occasional explained analysis.

Limited points made about importance/links or about developments in the context of the period will be little more than assertions and descriptions

13-15

Level V

There is some relevant accurate historical knowledge deployed: this may be generalised and patchy. There may be inaccuracies and irrelevant material also

Some accurate use of relevant historical terminology but often inaccurate/ inappropriate use

Often unclear and disorganised sections; writing will often be clear if basic but there may be some illegibility and weak prose where the sense is not clear or obvious

9-11

General and sometimes inaccurate understanding of key concepts relevant to analysis and of concepts relevant to the topic

General or weak understanding of the significance of most relevant issues in their historical context

Attempts at analysis will be weak or generalised, based on plausible but unsubstantiated points or points with very general or inappropriate substantiation OR there may be a relevant but patchy description of events/developments coupled with judgements that are no more than assertions

There will be some understanding of the question but answers may focus on the topic not address the focus of the question

11-12

Level VI Use of relevant evidence will be limited; there will be much irrelevance and inaccuracy Answer may have little

organisation or structure; weak use of English and poor organisation

4-8

Very little understanding of key concepts Very limited understanding of the topic or of the

question’s requirements Limited explanation will be very brief/ fragmentary The answer will be characterised by generalised

assertion and/or description/ narratives, often brief

6-10 Level VII No understanding of the topic or of

the question’s requirements; little relevant and accurate knowledge Very fragmentary and disorganised

response; very poor use of English and some incoherence

0-3

No understanding of key concepts or historical developments. No valid explanations Typically very brief and very descriptive answer

0-5

3

Page 107: History A - The Bicester Schoolthebicesterschool.org.uk/wp-content/uploads/2015/09/Mark... · 2015-09-28 · History A Mark Schemes for the Units January 2009 H106/H506/MS/R/09J

Marking Instructions

AS UNIT F963 & UNIT F964 – Historical Enquiries Maximum mark 100. 1 answer: 2 parts. Question (a) Maximum mark 30 A01a A01b AO2a

IA 6 8 16

IB 6 7 13-15

II 5 6 11-12

III 4 5 9-10

IV 3 4 7-8

V 2 3 5-6

VI 1 2 3-4

VII 0 0-1 0-2

Notes related to Question (a) (i) Allocate marks to the most appropriate level for each AO (ii) If several marks are available in a box, work from the top mark down until the best fit has

been found (iii) Many answers will not fall at the same level for each AO Question (b) Maximum mark 70 A01a A01b AO2a AO2b

IA 9-10 11-12 26-28 20

IB 8 9-10 23-25 17-19

II 7 8 20-22 14-16

III 6 6-7 17-19 11-13

IV 4-5 4-5 14-16 8-10

V 3 3 11-13 6-7

VI 2 2 5-10 3-5

VII 0-1 0-1 0-4 0-2

Notes related to Part B: (i) Allocate marks to the most appropriate level for each AO (ii) If several marks are available in a box, work from the top mark down until the best fit has

been found (iii) Many answers will not be at the same level for each AO

4

Page 108: History A - The Bicester Schoolthebicesterschool.org.uk/wp-content/uploads/2015/09/Mark... · 2015-09-28 · History A Mark Schemes for the Units January 2009 H106/H506/MS/R/09J

Marking Instructions

Marking Grid for Question (a)

AOs AO1a AO1b AO2a Total for each question = 30

Recall, select and deploy historical knowledge appropriately, and communicate knowledge and understanding of history in a clear and effective manner.

Demonstrate understanding of As part of an historical enquiry, the past through explanation, analyse and evaluate a range analysis and arriving at of appropriate source material substantiated judgements of: with discrimination. - key concepts such as causation, consequence, continuity, change and significance within an historical context; - the relationships between key features and characteristics of the periods studied.

Level IA

Accurate use of a range of appropriate historical terminology

Answer is clearly structured and coherent; communicates accurately and legibly

6

Answer is consistently and relevantly analytical with developed comparison and judgement

Clear and accurate understanding of key concepts relevant to analysis and to the topic

Clear and accurate understanding of the significance of issues in their historical context

8

Response provides a focused comparison and/or contrast of both content and provenance

Evaluates qualities such as reliability, completeness, consistency, typicality, and especially utility, in relation to the question

16 Level IB

Accurate use of a range of appropriate historical terminology

Answer is clearly structured and coherent; communicates accurately and legibly

6

Judgements are supported by appropriate references to both content and provenance

Very good level of understanding of key concepts

Clear and accurate understanding of the significance of issues in their historical context

7

Response provides an effective comparison and/or contrast of both content and provenance

Evaluates a range of qualities of authenticity, completeness, consistency, typicality and usefulness in relation to the question

13-15 Level II

Generally accurate use of historical terminology Answer is structured and mostly coherent; writing is legible and communication is generally clear

5

Good attempt at explanation/ analysis but uneven overall judgements

Mostly clear and accurate understanding of key concepts

Clear understanding of the significance of most relevant issues in their historical context

6

Provides a relevant comparison and/ or contrast of both content and provenance

Answer lacks completeness in evaluating most of the range of available criteria (eg. limited use of the introductions and/ or attributions)

11-12

5

Page 109: History A - The Bicester Schoolthebicesterschool.org.uk/wp-content/uploads/2015/09/Mark... · 2015-09-28 · History A Mark Schemes for the Units January 2009 H106/H506/MS/R/09J

Marking Instructions

AOs AO1a AO1b AO2a

Level III

Answer includes relevant historical terminology but this may not be extensive or always accurately used

Most of the answer is organised and structured; the answer is mostly legible and clearly communicated

4

A mixture of internal analysis and discussion of similarities and/or differences. A judgement is unlikely Some/uneven understanding

of many key concepts relevant to analysis and of many concepts relevant to the topic

Uneven understanding of the significance of most relevant issues in their historical context

5

Provides a comparison and/ or contrast

Makes limited links with the sources by focusing too much on content or on provenance

The organisation is uneven, confining the comparison to the second half of the answer or simply to a concluding paragraph

9-10

Level IV

There may be some evidence that is tangential or irrelevant

Some unclear and/or under-developed and/or disorganised sections; mostly satisfactory level of communication

3

Mostly satisfactory understanding of key concepts

Mostly satisfactory explanation but some unlinked though relevant assertions, description / narrative

There is no judgement

4

Response attempts a comparison and/or contrast but the comment is largely sequential

Few points of comparative provenance or discussion of similarity/difference of content

7-8 Level V

There may be inaccuracies and irrelevant material.

Some accurate use of relevant historical terminology but often inaccurate/ inappropriate use

Often unclear and disorganised sections; writing will often be clear if basic but there may be some illegibility and weak prose where the sense is not clear or obvious

2

General and sometimes inaccurate understanding of key concepts relevant to analysis and of concepts relevant to the topic

General or weak understanding of the significance of most relevant issues in their historical context

3

Identifies some points of agreement and/or disagreement

The comparison and/or contrast is implicit

There is no judgement

5-6 Level VI

There will be much irrelevance and inaccuracy Answer may have little organisation or structure; weak use of English and poor organisation

1

Limited explanation but mainly description / narrative

Very little understanding of key concepts

2

Very weak commentary on one point of agreement/ disagreement

Sources may be paraphrased with no real attempt to compare and/or contrast

3-4

Level VII

No understanding of the topic or of the question’s requirements Totally irrelevant answer Very poor use of English

0

Weak explanation, and descriptive / narrative commentary on the sources

No understanding of key concepts

0-1

No attempt to provide a comparison and/or contrast

Sources are paraphrased or copied out

0-2

6

Page 110: History A - The Bicester Schoolthebicesterschool.org.uk/wp-content/uploads/2015/09/Mark... · 2015-09-28 · History A Mark Schemes for the Units January 2009 H106/H506/MS/R/09J

Marking Instructions

Marking Grid for Question (b)

AOs AO1a AO1b AO2a AO2b Total mark for the question = 70

Recall, select and deploy historical knowledge appropriately, and communicate knowledge and understanding of history in a clear and effective manner.

Demonstrate understanding of the past through explanation, analysis and arriving at substantiated judgements of: - key concepts such as causation, consequence, continuity, change and significance within an historical context; - the relationships between key features and characteristics of the periods studied.

As part of an historical Analyse and evaluate, enquiry, analyse and in relation to the evaluate a range of historical context, how appropriate source aspects of the past material with have been interpreted discrimination. and represented in

different ways.

Level IA

Uses a wide range of accurate, detailed and relevant evidence

Accurate and confident use of appropriate historical terminology

Answer is clearly structured and coherent; communicates accurately and legibly

9-10

Clear and accurate understanding of key concepts relevant to analysis and to the topic

Clear and accurate understanding of the significance of issues in their historical context

Answer is consistently and relevantly analytical with developed explanations leading to careful judgements

11-12

Excellent analysis and evaluation of all sources with high levels of discrimination

Analyses and evaluates the limitations of the sources and what is required to add to their completeness as a set

26-28

Excellent analysis and evaluation of the historical interpretation using all sources and own knowledge to reach a clear conclusion

Fully understands that the sources may either support or refute the interpretation

20

Level IB

Uses accurate, detailed and relevant evidence Accurate use of a

range of appropriate historical terminology

Answer is clearly structured and mostly coherent; writes accurately and legibly

8

Clear and accurate understanding of most key concepts relevant to analysis and to the topic

Clear understanding of the significance of issues in their historical context

Judgements are supported by appropriate references to both content and provenance

9-10

Focussed analysis and evaluation of all sources with high levels of discrimination

Analyses and evaluates the limitations of the sources and what is required to add to their completeness as a set

23-25

Focussed analysis and evaluation of the historical interpretation using all sources and own knowledge to reach a clear conclusion

Understands that the sources may either support or refute the interpretation

17-19

7

Page 111: History A - The Bicester Schoolthebicesterschool.org.uk/wp-content/uploads/2015/09/Mark... · 2015-09-28 · History A Mark Schemes for the Units January 2009 H106/H506/MS/R/09J

Marking Instructions

AOs AO1a AO1b AO2a AO2b

Level II

Uses mostly accurate, detailed and relevant evidence which demonstrates a competent command of the topic Generally accurate use of historical terminology Answer is structured and mostly coherent; writing is legible and communication is generally clear

7

Mostly clear and accurate understanding of key concepts

Clear understanding of the significance of most relevant issues in their historical context.

Good attempt at explanation/ analysis but uneven overall judgements

8

Focussed analysis and evaluation of most of the sources with good levels of discrimination

Analyses and evaluates some of the limitations of the sources and what is required to add to their completeness as a set

20-22

Focussed analysis and evaluation of the historical interpretation using most of the sources and appropriate own knowledge to reach a clear conclusion

There may be some imbalance between discussion of the sources and use of external knowledge in evaluating the interpretation

14-16 Level III

Uses accurate and relevant evidence which demonstrates some command of the topic but there may be some inaccuracy

Answer includes relevant historical terminology but this may not be extensive or always accurately used

Most of the answer is organised and structured; the answer is mostly legible and clearly communicated

6

Shows a sound understanding of key concepts. Sound awareness of

the significance of issues in their historical context

Attempts an explanation/ analysis but overall judgement may be incomplete

6-7

Refers to most of the sources to illustrate an argument rather than analysing and evaluating their evidence

Aware of some of the sources’ limitations either individually or as a set

17-19

Sound analysis and evaluation of the historical interpretation.

There may be some description and unevenness between use of own knowledge and use of sources

Answers which use the sources but no own knowledge in assessing the interpretation have a Level III ceiling

11-13 Level IV

There is deployment of relevant knowledge but level/ accuracy of detail will vary; there may be some evidence that is tangential or irrelevant

Some unclear and/or under-developed and/or disorganised sections; mostly satisfactory level of communication

4-5

Mostly satisfactory understanding of key concepts

Some explanation but not always linked to the question

Assertions, description / narrative will characterise part of the answer

4-5

Sources are discussed sequentially

Considers some of the limitations of the sources; but may not establish a sense of different views

14-16

Some analysis and evaluation of the historical interpretation with increasing amounts of description

Response is more imbalanced than Level III in using sources and own knowledge

Answers that use own knowledge but make no use of the sources in assessing the interpretation have a Level IV ceiling

8-10

8

Page 112: History A - The Bicester Schoolthebicesterschool.org.uk/wp-content/uploads/2015/09/Mark... · 2015-09-28 · History A Mark Schemes for the Units January 2009 H106/H506/MS/R/09J

Marking Instructions

AOs AO1a AO1b AO2a AO2b

Level V There is some relevant historical knowledge deployed: this may be generalised and patchy. There may be inaccuracies and irrelevant material Some accurate use of relevant historical terminology but often inaccurate/ inappropriate use Often unclear and disorganized sections; writing will often be basic and there may be some illegibility and weak prose where the sense is not clear or obvious

3

General and sometimes inaccurate understanding of key concepts relevant to analysis and of concepts relevant to the topic General or weak understanding of the significance of most relevant issues in their historical context

3

Limited attempt to use the sources or discriminate between them; they are discussed sequentially Sources will be used for reference and illustration of an argument

11-13

Mainly description with limited comment on the context of the question Little effective analysis of how far the sources support the interpretation

6-7 Level VI

Use of relevant evidence will be limited; there will be much irrelevance and inaccuracy Answer may have

little organisation or structure

Weak use of English and poor organisation

2

Very little understanding of key concepts.

No explanation. Assertion, description

/ narrative predominate

2

Weak application of the sources to the question

Weak attempt at analysis

5-10

Weak contextual knowledge

Mainly description with weak evaluation of the historical interpretation

3-5 Level VII

No understanding of the topic or of the question’s requirements; little relevant and accurate knowledge Very fragmentary and disorganised response; very poor use of English and some incoherence

0-1

No understanding of key concepts Weak explanation,

assertion, description / narrative

0-1

Very weak application of the sources to the question

No attempt at analysis

0-4

Very weak attempt at evaluating the historical interpretation

Heavily descriptive No contextual

knowledge

0-2

9

Page 113: History A - The Bicester Schoolthebicesterschool.org.uk/wp-content/uploads/2015/09/Mark... · 2015-09-28 · History A Mark Schemes for the Units January 2009 H106/H506/MS/R/09J

F961 Mark Scheme June 2009

10

F961/01 British History Period studies Option A: Medieval and Early Modern 1035-1642

From Anglo-Saxon England to Norman England 1035-1087 1 Assess the reasons for the importance of the Godwin family in the reign of Edward

the Confessor. No set answer is looked for but candidates will need to address the question. Candidates might consider both the positive and negative reasons for their importance. Earls occupied an important place in Anglo Saxon society and the most powerful family in the period was the Godwins. They had a power base in Wessex, at the heart of the country. Earl Godwin probably played an important role in the accession of Edward the Confessor. Edith, Godwin’s daughter, married Edward and this might have cemented an alliance. The strength of his position meant that he could pose powerful problems to the king, for example over the influence of Normans in England. It might be argued that Godwin was jealous of the influence foreigners exerted at court and over the king and this created further problems. Harold succeeded to his position and the role of the family became even more important when Harold emerged as the strongest Anglo Saxon claimant to the throne. Among the negative aspects that candidates might consider might be the personal role of Edward the Confessor. Although not a cipher, he was not a dominant head of state and this enabled the Godwins to play their part to the full.

2 Assess the reasons for the succession crisis at the end of Edward the Confessor’s

reign. No set answer is looked for but candidates will need to address the question. It is likely that candidates will focus on Edward’s failure to produce an heir, but nor was there anybody close enough in his family to exert an unchallenged claim. Harold became head of the Godwin family in 1053 but the period to 1066 showed his problems in maintaining his primacy among the nobility. However, some candidates might argue that Harold was widely accepted as king in England but there was not universal acceptance. Harold would face problems establishing himself on the throne. The near simultaneous challenges from William and from Harold Hadrada and Tostig did not give Harold time to secure himself on the throne. Candidates may suggest that there were others with claims and point to Tostig and Harold Hadrada, who mounted a powerful challenge. The reasons for the rival claims will probably be examined with the better answers explaining why each of these did not represent an unchallenged claim. William’s claim, allegedly substantiated by Harold’s promise, Edward’s nomination and papal approval, was rejected by the Anglo Saxons who preferred Harold. Edward might have changed his preference on his deathbed and this added to the dispute. There is no need for candidates to look at the outcome of the dispute but it will be possible to take the argument to Hastings because this effectively ended the dispute over the succession. Some answers might consider Edgar the Aetheling, but this would be a bonus and his omission should not be regarded as a gap.

Page 114: History A - The Bicester Schoolthebicesterschool.org.uk/wp-content/uploads/2015/09/Mark... · 2015-09-28 · History A Mark Schemes for the Units January 2009 H106/H506/MS/R/09J

F961 Mark Scheme June 2009

3 How serious were the rebellions William I faced as king of England? No set answer is looked for but candidates will need to address the question. Candidates might argue that Harold’s death at Hastings removed the greatest danger; the defeat had also broken the military strength of the Anglo Saxon fyrd and earls. Some answers might refer to the opposition to William immediately after his victory at Hastings until he was crowned but it hardly amounted to a rebellion. The new king could rely on the fact that his main rivals were dead and that other important Anglo Saxon nobles from Northumbria and Mercia had promised allegiance. There was disorder until 1071 in Kent, Northumbria, the south west and the Welsh Marches. However, the risings were usually localised and were the result of local grievances rather than dissatisfaction with William’s rule. These risings lacked leadership and were therefore less serious. The situation at Exeter, it might be argued, was more serious, but order was restored after a siege. It might be argued that with the support of the Danes the rising in Northumbria was more serious, particularly as it also involved Edwin and Morcar. Candidates might note that William always had a clear military advantage and the ability to use ruthless devastation to put down a rebellion. Castles also allowed William to deal with rebellion effectively. The guerrilla type resistance of Hereward was more of a nuisance than serious. This shows that there was continued resistance to William’s rule.

11

Page 115: History A - The Bicester Schoolthebicesterschool.org.uk/wp-content/uploads/2015/09/Mark... · 2015-09-28 · History A Mark Schemes for the Units January 2009 H106/H506/MS/R/09J

F961 Mark Scheme June 2009

Lancastrians, Yorkists and Tudors 1450-1509

4 How important was the weakness of Henry VI in causing the civil war in England in the period from 1450 to 1470? No set answer is looked for but candidates will need to address the question. The king’s personality was not one that enabled him to control the nobility. He was open to influence, often to unsound advice, but was also obstinate. Candidates might also consider the state of his mental health, which was variable, representing considerable weakness at the head of state. However, his weakness (an undermighty king) should be balanced by a consideration of the problem of ambitious nobles (overmighty nobles), such as Warwick, Somerset and York. However, it should be noted that the number of alienated nobility should not be exaggerated; most of the aristocracy continued to support Henry VI, suggesting an unwillingness to go to war. However, as a weak king he was unable to control, even less resolve, the rivalry between the ambitious nobles. Candidates might also discuss the role of Margaret of Anjou. The loss of land in France might be seen as an issue as some nobles had dual-landholding, but found that with defeats in France they were subject to the King of France and this placed a strain on their relationship with Henry.

5 ‘Edward IV was more successful as a king in his second reign than his first’. How far

do you agree with this view? No set answer is looked for but candidates will need to address the question. Many candidates are likely to agree with the proposition as Edward was deposed at the end of his first reign, but died a natural death at the end of the second. However, some might challenge this view and argue that his failure to ensure a peaceful succession to his son means that the second period was no more successful. Candidates are likely to consider his relations with the nobility; in the first period of his rule he was dependent upon some powerful nobles such as Warwick and that created problems. He also tried to win over opponents by conciliation, for example Somerset and Percy, but that failed and both continued in rebellion. The problem created by Warwick was the most serious as it resulted in Edward’s deposition. Some may argue that this was balanced by the support he did win from some group of nobles. However, some will argue that he was no more successful in his second period of rule as he created super-nobles who were only controlled by the sheer force of Edward’s personality. Some candidates will consider how successfully Edward dealt with the problem of finance during his two periods of rule. In the first period he faced a lack of money, which he tried to solve by means of benevolences and forced loans. This was reasonably successful as most paid without complaint. In the second period he was financially very successful and actually died leaving the crown solvent.

12

Page 116: History A - The Bicester Schoolthebicesterschool.org.uk/wp-content/uploads/2015/09/Mark... · 2015-09-28 · History A Mark Schemes for the Units January 2009 H106/H506/MS/R/09J

F961 Mark Scheme June 2009

6 How effective was Henry VII in dealing with the Yorkist challenges to his throne? No set answer is looked for but candidates will need to address the question. It is likely that most answers will focus on the Simnel and Warbeck rebellions, but better answers might also consider the de la Pole threat, the Yorkshire tax rebellion and Suffolk. There might also be mention of the situation in Ireland and the general threat from Yorkist nobility who often found themselves under heavy financial burdens. Many candidates are likely to argue that Henry was successful as none of the challenges were able to remove him from the throne. They may also argue that Henry was successful as the Yorkist cause was reduced to using pretenders. However, the Simnel rebellion did force Henry into battle only two years after Bosworth and it was not impossible that a similar result might have occurred. Having defeated Simnel, Henry was able to show mercy and defuse the situation by parading the real Earl, the success of his policy can be seen when Warbeck rose and failed to gain support in Ireland, unlike the Simnel rising. Some answers may also suggest that foreign support for the Pretenders, particularly from Margaret of Burgundy, made it more difficult for Henry as he was unable to deal with her. Henry, it can be argued, did deal very successfully with Warbeck as his attempts at invasion were pathetic failures, and he was also able to keep him on the move between European courts. This might be balanced against the difficulty Henry had in capturing Warbeck, Scotland’s willingness to give him aid and marry him to the king’s cousin. Some candidates might consider some of the other steps that Henry took to limit Yorkist danger; there might be consideration of his marriage to Elizabeth of York and the imprisonment of the Earl of Warwick, son of Clarence, until the conspiracies of other plots provided an excuse for his execution. The other rebellions were handled quite well, although tax concessions had to be granted.

13

Page 117: History A - The Bicester Schoolthebicesterschool.org.uk/wp-content/uploads/2015/09/Mark... · 2015-09-28 · History A Mark Schemes for the Units January 2009 H106/H506/MS/R/09J

F961 Mark Scheme June 2009

Henry VIII to Mary I 1509-1558 7 How successful was English foreign policy in the period from 1509 to 1529?

No set answer is looked for but candidates will need to address the question. Candidates may argue that Henry was more or less successful. Some might argue that it was Wolsey who was more or less successful. This approach is satisfactory provided the candidate demonstrates enough understanding of Henry’s role. Henry was successful in making England an active participant in foreign affairs, particularly given her relative power, but it might be judged that his achievements were limited. From 1511-14 he fought France and won the Battle of the Spurs in alliance with the Emperor Maximilian. He did capture both Therouanne and Tournai. The Battle of Flodden was also an English victory, but not for Henry himself although he did take the credit. A treaty was made with France that included the marriage of Mary to Louis XII. Candidates might assess the ultimate benefit of these early gains and conclude that they achieved very little of importance. Some may argue that in the short term they had diplomatic success as was evident in Treaty of London, 1518, and the Field of the Cloth of Gold, 1520. The 1520s saw Henry switching his alliance between France and Spain in order to maximise his influence, however there was little success and the failure of the Amicable Grant robbed him of his major opportunity. Charles’ victory at Pavia lessened Henry’s influence and also led to his final failure over his divorce. It might also be noted that Henry was left out of the peace at Cambrai, 1529, suggesting that his influence was not important. In conclusion candidates might suggest that England was isolated and lacking in influence in 1529.

8 To what extent was Northumberland more successful than Somerset in ruling

England during the reign of Edward VI? No set answer is looked for but candidates will need to address the question. There are a wide range of issues available for candidates to discuss and it is unlikely that they will be able to consider all the areas, what matters is the quality of analysis. Many candidates might argue that Northumberland was more successful as he did not face the series of rebellions that Somerset did in 1549. However, some might argue that the one rebellion he did face, from Mary Tudor, resulted in his downfall. Some might even suggest that neither ruler was successful in dealing with rebellion. In the area of social policy candidates might argue that Somerset was more benevolent, although his Vagrancy Act does not support this view and the failure to enforce it could be used to show he was not successful. At the same time his establishment of the Enclosure Commission helped to provoke unrest in 1548-9 and could be used to show his policies were misguided. In financial terms many are likely to argue that Northumberland was more successful, Somerset spent a great deal of money on the Scottish War for little gain and was forced into debasement on a large scale, which might be linked to the unrest of 1549. However, Northumberland was able to bring some financial stability. In governing the realm Somerset’s approach was often personal and this alienated him from many and may help to explain his downfall. His frequent use of Proclamations suggests that there were frequent emergencies, whereas Northumberland was careful to ensure a wider base of support. Some candidates might point to the work of Dale Hoak to show that Northumberland was a successful ruler who brought stability back to the country.

14

Page 118: History A - The Bicester Schoolthebicesterschool.org.uk/wp-content/uploads/2015/09/Mark... · 2015-09-28 · History A Mark Schemes for the Units January 2009 H106/H506/MS/R/09J

F961 Mark Scheme June 2009

9 How successful was Mary I in dealing with the problems she faced? Explain your answer. No set answer is looked for but candidates will need to address the question. Although historiography is not a requirement at AS, candidates who use the views of historians to support their arguments should be credited. Traditional accounts of Mary’s reign have been negative, but more recent work Loades, Loach and Williams has portrayed her in a more positive light. Mary faced a number of problems on her accession and many will identify these and then assess how successful she was in dealing with them. Her first concern was the succession, she had to convince the Privy Council of her right to be Queen and then deal with Northumberland and his supporters. This was achieved relatively easily as Mary’s accession was generally greeted with enthusiasm; she was able to overcome the problem of Lady Jane Grey quite easily as the latter had little popular support. However, there was still the problem of her legitimacy as she and her half sister, Elizabeth had been declared illegitimate by Royal Patent on 21st June 1553, but this issue was resolved. In order to bolster her security and ensure the long-term survival of the regime she needed to marry and produce an heir, particularly given her religious views and those of Elizabeth. The significance of her marriage can be assessed as it failed to solve the problem of providing an heir to the throne and also resulted in Wyatt’s rebellion. There may be an assessment of how well the government dealt with the rebellion, some may conclude that its defeat shows success, but others may argue that it was a serious challenge that was only defeated through the last minute actions and defiance of Mary. The marriage may be taken further as the government did not solve the problem of being dragged into an unpopular war against France, which many perceived was for Habsburg ends and not English. The modest attempts at economic and financial reforms could also be considered. Although Mary left a legacy of debt she managed her finances well. The government cannot be blamed for the terrible harvests and devastating epidemic it faced. Some may even conclude that the issue of the succession was resolved satisfactorily as Elizabeth the legitimate heir, did inherit the throne. However, others may conclude that her failure to secure a catholic succession, her priority, was not achieved and therefore she did not solve the problems she faced.

15

Page 119: History A - The Bicester Schoolthebicesterschool.org.uk/wp-content/uploads/2015/09/Mark... · 2015-09-28 · History A Mark Schemes for the Units January 2009 H106/H506/MS/R/09J

F961 Mark Scheme June 2009

Church and State 1529-1589 10 How far was the Church in England in need of reform in 1529?

No set answer is looked for but candidates will need to address the question. Historiography is not a requirement at AS, but candidates who use the views of historians to support their argument should be credited; however descriptive accounts of historians views should not be unduly credited. In arguing that there were positive elements to the church candidates might consider the evidence that people still volunteered money to the Church, for example in wills, and this allowed church building to continue up to the eve of the Reformation. They might also show that the payment of tithes was not unpopular, nor was attendance at church courts, despite events such as the Hunne Case. The church was satisfying the needs of most as Lutheranism and Lollardy found little support, whilst the agricultural year was supported by the church calendar through events such as Rogationtide or Harvest Festival, meanwhile in urban communities the number of guilds and the production of devotional literature shows that the church was still flourishing. Ordination rates were high until the Reformation and complaints about the behaviour of the clergy were few. Some may argue that the situation was no worse than it had been and support this by reference to comments by Chaucer or Langland. Anti-clericalism was a problem, but it was not new and anti-papalism was not strong. There were calls for reform from Colet and Melton, but these were limited. Candidates might also use the example of Wolsey to show that there were individuals who were guilty of abuses, but even where there was absenteeism this was often overcome by the use of curates. Some answers might point to the anti-clerical legislation of the 1529 parliament, but this might be seen as an attack on Wolsey, rather than a general attack on the Church.

11 Assess the reasons for opposition to the religious changes during the reign of Henry VIII. No set answer is looked for but candidates will need to address the question. Some candidates may consider the reasons for the individuals or groups who opposed the religious changes; for example they may write about Fisher’s opposition to the divorce and More’s attitude to the Royal Supremacy. There may be consideration of groups, such as the Carthusians who opposed the attack on catholic practices or the clergy and monks who disliked the attack on monasteries and the rumour of changes to the parish church. This approach is acceptable, what matters is the quality of evaluation. However, some answers may take a more thematic approach or argue that until the Dissolution of the Monasteries opposition was very limited. Some may argue that people saw it as an attack on fundamental and traditional religious practices; they may use the attacks on saint’s days as evidence, or disliked the Royal Supremacy as it attacked papal authority, both of which were traditional. Some may focus on the Dissolution as the key as this made the religious changes apparent to everyone, but also had a social and economic impact and use evidence from the Pilgrimage of Grace to support their argument. The attack on purgatory may have worried many as it threatened people’s ability to determine their after-life. There were also many rumours about new taxes, the loss of church goods and why parish registers were being introduced, again evidence from the Pilgrimage of Grace could be used to support this line of argument. Many people associated the changes with the divorce from Catherine and the influence of Anne Boleyn, who was disliked. For many simply the fear of the unknown in a world where there was little certainty may have been enough to cause uncertainty and even resistance.

16

Page 120: History A - The Bicester Schoolthebicesterschool.org.uk/wp-content/uploads/2015/09/Mark... · 2015-09-28 · History A Mark Schemes for the Units January 2009 H106/H506/MS/R/09J

F961 Mark Scheme June 2009

12 To what extent were Elizabeth’s religious views the most important factor in shaping the Elizabethan religious settlement of 1559? No set answer is looked for but candidates will need to address the question. Candidates have studied the topic from 1529 and should be expected to know the background to Elizabeth’s problems at her accession, but candidates should avoid the temptation to write long surveys that deal with settlement only in the conclusion. In the same way candidates should not take their answers much beyond 1560 as the question does not ask about her religious aims throughout her reign. Candidates might focus exclusively on the situation in 1558-1560 and this is an acceptable approach. They are likely to analyse her aims, problems and the resulting settlement. In considering Elizabeth’s views candidates might point out that she was personally religious but not bound to a particular form of belief and practice beyond what was most advantageous politically. However, her background meant that she had protestant sympathies and she was orthodox, preferring the formality of Anglicanism to the radicalism of Puritanism. Although Elizabeth normally preferred to delay decisions she saw a settlement as important in view of the divisions at home and the situation abroad. Domestic issues that influenced the settlement included her legitimacy as queen, the presence of Catholics in high Church offices and determined Puritans who were reinforced by returning exiles. Some may argue that the Queen was forced into a more radical settlement by a group of radical Protestants in the House of Commons than she wanted. This may be supported by a consideration of the international situation which, it can be argued, suggested a more moderate approach as England was still at war and did not possess the revenue to keep fighting. However, this view might be challenged by those who argue that the Commons did not have sufficient influence to bring about these changes. The majority of the population was probably Catholic in a broad sense, not necessarily papist. Many may argue that the most important consideration was to bring about a widely acceptable settlement. There may be some consideration of the difficulty in getting the settlement passed and the concessions and compromises that had to be made. This may lead some to conclude that the Catholics did have some influence and use the evidence of the Catholic Bishops in the House of Lords to support their argument as Elizabeth had to imprison some Catholic Bishops to get the Act of Uniformity passed. It might be concluded that as Elizabeth was unwilling to allow Parliament to discuss the Religious Settlement for the rest of her reign that she was generally satisfied with it.

17

Page 121: History A - The Bicester Schoolthebicesterschool.org.uk/wp-content/uploads/2015/09/Mark... · 2015-09-28 · History A Mark Schemes for the Units January 2009 H106/H506/MS/R/09J

F961 Mark Scheme June 2009

England under Elizabeth I 1558 -1603. 13 How serious were the problems Elizabeth I faced in 1558?

No set answer is looked for but candidates will need to address the question. There are a wide range of issues that candidates might consider and it is unlikely that they will be able to cover them all, what matters is the quality of analysis. Elizabeth’s position in 1558 was very vulnerable. She was not seen as the legitimate ruler by Catholics both at home and overseas. As a result Mary Queen of Scots was seen by many as the rightful ruler and her links with France further weakened Elizabeth’s position. England was also at war with France in 1558 and this added to the danger and was made worse when Spain made peace with France as this isolated England and left them open to the possibility of a Catholic crusade. However, this problem should not be exaggerated as it was unlikely that Philip would help to put a pro-French ruler on the throne of England. Philip even offered his hand in marriage to Elizabeth, so provided she did not alienate him her position was more secure. It was possible that there might be a Catholic rebellion at home, but without foreign leadership this was less likely and the Papacy failed to give a lead, hoping that Elizabeth might rejoin the catholic fold. It was expected that Elizabeth would marry and that raised a number of issues over the choice, foreign influence and faction if she chose to marry an Englishman. However, she was also able to use the prospect of marriage as a tool to strengthen her position and buy time to secure her position at the start of her reign. The problem of the succession was crucial and parliament asked her in 1559 to name a successor. She also had to overcome the prejudices against female rulers following the reign of Mary, which had been reinforced by the disasters of the latter years of her reign. She also had to overcome the popular stereotype of women, which emphasised their physical, emotional and intellectual inferiority to men. There were social and economic problems to be dealt with in the aftermath of Mary’s reign. The reign also began with war against France and this created major financial problems.

14 How successful were Elizabeth and her ministers in managing parliament? No set answer is looked for but candidates will need to address the question. Candidates may argue that relations with parliament were usually good; however there were disputes over marriage and the succession, but this was when parliamentary opinion was closer to that of the Privy Council than the Privy Council was to the Queen. These harmonious relations were reinforced by good parliamentary managers such as Mildmay and Norton. Most MPs knew that there were certain boundaries which they could not cross by expressing their opinions. It could be argued that such limitations on the manner of debate caused far less trouble than the Queen’s use of prerogative to prevent debate on issues she considered too sensitive. This point of view might lead some to conclude that parliament was a subordinate body, but this might be balanced by the view that parliament had evolved and grown in political importance and was able to seize the political initiative at the expense of a tired and increasingly incompetent government – to sustain this view answers might focus on the 1590s. Some candidates might draw attention to the times there were conflicts, such as the succession and the execution of Mary Queen of Scots, however this could be balanced by noting that most legislation was local and passed with little debate. It is possible that some answers will consider the methods used to manage parliament, such as starting debate in the Lords, where after Cecil’s appointment, there was a minister to lead debate. Answers might argue that MPs were becoming more educated and therefore were increasingly able and willing to challenge the control exercised on parliament through Privy Councillors and the Speaker. In addition they might argue that there were a growing number of Puritans in parliament who used it to try and bring about changes to the religious settlement. It should be remembered that Elizabeth still had the power to summon, prorogue and dissolve parliament when she wanted and that her ability as a speaker, as with the Golden Speech of 1601 was sufficient to win

18

Page 122: History A - The Bicester Schoolthebicesterschool.org.uk/wp-content/uploads/2015/09/Mark... · 2015-09-28 · History A Mark Schemes for the Units January 2009 H106/H506/MS/R/09J

F961 Mark Scheme June 2009

support. Most parliamentary business was completed without fuss, but at other times there were crises, but they were usually worked through without too much trouble.

15 How effectively did Elizabeth I and her government deal with the problems they faced in the period from 1588 to 1603? No set answer is looked for but candidates will need to address the question. There are a large number of problems faced by Elizabeth and her governments in this period and examiners should not expect candidates to consider all of them, although a reasonable range is expected for the highest levels, what matters is the quality of analysis. The specification mentions a number of issues that might be found in answers: the defence of the royal prerogative, relations with parliament, the effects of the war with Spain, the Irish Rebellion and the Essex Rebellion. The Irish Rebellion was the heaviest expense of the reign and this and its length might lead candidates to argue it was not handled well, but it was eventually defeated and order was restored under Mountjoy. The Essex Rebellion was crushed easily, but some might argue that its occurrence ended the possibility of maintaining a balance between factions and that Robert Cecil would now dominate. There were conflicts between the Queen and her parliaments about monopolies and the crown’s request for supplies, but neither weakened the Queen significantly and the Golden Speech might be used to show that she was still able to handle parliament well at the end of her reign. Financially it might be argued the Crown did not deal successfully with problems and income did not keep up with inflation. Some candidates might point to the social problems of the 1590s, which were severe, yet the government never faced the unrest of the 1540s and it appeared as if their legislation had been successful.

19

Page 123: History A - The Bicester Schoolthebicesterschool.org.uk/wp-content/uploads/2015/09/Mark... · 2015-09-28 · History A Mark Schemes for the Units January 2009 H106/H506/MS/R/09J

F961 Mark Scheme June 2009

The Early Stuarts and the Origins of the Civil War 1603-1642 16 How serious were religious problems during the reign of James I?

No set answer is looked for but candidates will need to address the question. There were religious differences when James came to the throne, but they were moderated in the first instance because of the varied expectations of the new monarch. There might be reference to the diverse views and strengths of orthodox Anglicans, Puritans and Catholics. There were hopes of conciliation between the King, orthodox Anglicans and Puritans at the Hampton Court Conference, suggesting that divisions and problems were not that great. Bancroft’s Canons of 1604 resulted in the ejection of puritan clergy, but it caused few serious problems, although it might be argued that it created distrust. This distrust was further developed by the King’s choice of ministers, particularly Buckingham. The Gunpowder Plot had the potential to be serious and although it was the work of a small minority did increase antipathy and hatred towards Catholics. The puritan problem was, to some extent, mollified by the appointment of Abbot as Archbishop of Canterbury. However, the 1618 Book of Sports alienated them. The religious problems were exacerbated by the issues of foreign policy and marriage. Parliament wanted a more active pro-protestant foreign policy and this highlighted divisions between King and parliament. When James finally came to the aid of co-religionists in the Thirty Years War in 1624 it was seen by most Protestants as too late. James was also seen as being too close to Catholic powers and this was given credence by the marriage negotiations with Spain over his son Charles. However, some answers might conclude that the problems were nothing like as severe as they would become under Charles.

17 How far were James I’s financial problems of his own making? No set answer is looked for but candidates will need to address the question. Some candidates may argue that James inherited a difficult financial situation, revenue from taxation and assessment rates had not kept pace with inflation and the demands of war from the last years of Elizabeth’s reign made the situation worse. These considerations might be balanced against the cost of James’ court and his attempts to revise customs duties though the Book of Rates, which was unpopular. James’ belief in Divine Right did little to help the situation and made parliament more reluctant to grant supply and use finance as a way to defend what they saw as just privileges to influence royal policy.

18 How successful was Charles I’s personal rule? No set answer is looked for but candidates will need to address the question. Although Charles’ personal rule was brought to an end by the financial needs created by the Scots War and was ultimately a failure, it is possible to argue that for much of the period it was relatively successful, bringing order and stability. It may have been unpopular with some groups, but Charles was able to rule for eleven years without calling parliament. At first the returns from Ship Money were high and other methods of taxation, although not always popular, did allow Charles to raise sufficient money, provided he followed a pacific foreign policy. However, this might be balanced by a consideration of the unpopularity of the rule of ‘Thorough’, although it did bring stability and order. There is little to suggest that if Charles had not tried to introduce the Prayer Book to Scotland and embarked on imposing his religious policy there that personal rule would have come to an end.

20

Page 124: History A - The Bicester Schoolthebicesterschool.org.uk/wp-content/uploads/2015/09/Mark... · 2015-09-28 · History A Mark Schemes for the Units January 2009 H106/H506/MS/R/09J

F961 Mark Scheme June 2009

21

F961/02 - British History Period Studies Option B: Modern 1783-1994

From Pitt to Peel 1783-1846 1 How serious were the radical challenges to Lord Liverpool’s government in the

period from 1812 to 1822?

No set answer is looked for but candidates will need to address the question. Candidates will need to assess the various radical challenges facing Liverpool’s government before reaching an overall conclusion about their seriousness. Candidates might establish the nature of the challenge and the extent to which it posed a threat to Liverpool’s government. The radical challenge fed off the economic crises and agricultural difficulties that followed the end of the Napoleonic Wars and, although they failed, in hindsight the 1810s were difficult years. 1812 saw bread prices reach their highest in the nineteenth century and the 1815 Corn Law was seen as the hated bread tax to keep prices artificially high. The Continental System was followed by depression in 1815 with Poor Relief reaching its peak in 1818. At various stages the government was faced with seditious meetings, urban discontent and various plots (Luddism, the March of the Blanketeers, Pentrich Rising, Peterloo and the Cato St. Conspiracy). Many may argue that the government was not seriously challenged and that they handled the challenges with relative ease. However, Luddism in 1812-13 and rick burning in 1816 threatened both town and country. Such economic distress stimulated the revival of radical politics and the spread of revolutionary ideas continued via frequent protest meetings spreading democratic and republican ideas under the banner of Parliamentary Reform and old corruption. However, the challenge was undermined by an experienced government, a lack of cohesive leadership which was often impractical, divisions over aims and tactics, the fragmentation of a very regionalised response and moments of economic recovery. The government certainly took the radical challenge seriously. Candidates might find a route through the material by examining the government response or examining the activities of a national leader like Cartwright and his Hampden Clubs and Political Union Societies, or Hunt or revolutionaries.

2 How far was Peel able to reconstruct the Tory party in the period after 1834?

No set answer is looked for but candidates will need to address the question. Candidates should able to examine the two sides of the debate before reaching a conclusion. During his leadership there were some positive achievements that candidates might be expected to discuss. The party was reorganised after the Great Reform Act and the 1832 election. The widening of the base of party support might be considered through the publication of the Tamworth Manifesto, which might be seen as visionary and more inclusive of a wider property-based electorate, although this is open to debate as a close examination of the 1841 election result reveals. Candidates might also consider the creation of the Carlton Club, but here they may argue that this was not the work of Peel. Peel was also able to defeat the repeal movement in Ireland, win the 1841 election (367 seats, compared with 175 in 1832) and pursue popular policies for much of the 1841-6 ministry, although this is also open to debate. There may also be consideration of issues such as Registration, leading from the front from 1834 and the 100 days, carefully choosing the right moment to challenge the Whigs. On the negative side Peel was not very successful at widening the base of support or overcoming the mistrust that many had in him after the Catholic Emancipation and the Reform Act. The two areas that many are likely to focus on when arguing that he was not successful are the 1841 election, this showed that most Conservative support still came from the rural areas of the south and Peel’s attempts to broaden the party appeal had failed, and the Repeal of the Corn Laws, as far from

Page 125: History A - The Bicester Schoolthebicesterschool.org.uk/wp-content/uploads/2015/09/Mark... · 2015-09-28 · History A Mark Schemes for the Units January 2009 H106/H506/MS/R/09J

F961 Mark Scheme June 2009

reconstructing the party he destroyed its unity. Some might also argue that it was not Peel, but simply the weakness of the Whigs that allowed him to reconstruct the party. It is also possible that some will argue Peel was not interested in reconstructing the party as he put the needs of the nation before party.

3 How successful were Peel’s economic and financial reforms in the period from 1841

to 1846? No set answer is looked for but candidates will need to address the question. Candidates will need to establish criteria for success and could use Peel’s own cost-effective and carefully thought out reform schemes; economy and care with finance; stabilising measures in finance; recognising and controlling industrialisation and its worst effects and feeding a growing urban population. It is likely that many candidates will argue that the reforms were successful, but in the higher levels discrimination between areas may be expected before an overall conclusion is reached. Some candidates might also use those of contemporaries: his ability to push the measures through, his concern for working class budgets and the acceptability of much of what he did. Some may argue that the Budgets were particularly successful, linking the lowering of duties and a cheapening of the manufacturing base. At the same time they may point to the successful introduction of Income Tax to deal with the deficit inherited from the Whigs. The combination of sound government finance, a boost for manufacturing and broadening of consumption meant that it could be argued he was very successful. Peel could also be credited with the improvement in the economy in 1845 and this allowed him to bring in further free trade. However, the success can also be questioned; recovery occurring through a combination of factors, including the development of the railways. Some may argue that the Corn Laws, although controversial, were particularly successful as it made cheap food available to the ordinary people and therefore helped achieve social stability. It could be argued, by Peel’s criteria, that Factory Reform was also successful as it preserved a free labour market, although this might be balanced against the failure of educational aspects. Bank and Company Reform may also be seen as very successful as a stabilised gold standard in the Bank Charter Act was the key to sterling’s 19th century supremacy and to fiscal orthodoxy well into the 20th century and at the same time the Company Act and Railways showed the need for, and difficulties of, regulating industrial capitalism.

22

Page 126: History A - The Bicester Schoolthebicesterschool.org.uk/wp-content/uploads/2015/09/Mark... · 2015-09-28 · History A Mark Schemes for the Units January 2009 H106/H506/MS/R/09J

F961 Mark Scheme June 2009

Liberals and Conservatives 1846-1895 4 To what extent were the reforms of Gladstone’s first ministry limited in their

achievements? No set answer is looked for but candidates will need to address the question. Many candidates might argue that the reforms were not limited as it is usual to see this ministry as the great reforming ministry; however at the top levels candidates should produce a balanced answer. The Education Act, although it played an important role in providing education for working class children, was an uneasy compromise between Anglicans and non-conformists and it created class divisions. Trade Union reform may have equalised the law between worker and employer but stopped short of what the skilled workers wanted-peaceful picketing and immunity from prosecution for strikes. Administrative reforms, such as the Civil Service and Universities, may have brought equality of opportunity, but there were still few who could take advantage of the changes. The Secret Ballot Act had a large impact, especially within Ireland. Irish legislation failed to have the required impact as the Land Act did not go far enough. The Licensing Acts annoyed temperance groups and the working class. Some candidates might approach the question by looking at the success or failure of the reforms in satisfying Liberal support and conclude that most of the reforms alienated some group of supporters; for example, they might argue that the Education Act failed to please the non-conformists.

5 ‘Disraeli became leader of the Conservative party because there was no-one else.’

How far do you agree with this view? No set answer is looked for but candidates will need to address the question. Candidates might look to weigh up the strengths of Disraeli as a leader against his weaknesses and other possible contenders before reaching a balanced conclusion. In agreeing with the statement they might argue that Derby had not been interested in leading the party and was more at home horse-racing, whilst other possible contenders, such as Gladstone, had left the party. It might be noted that Derby had led the government in 1852 and 1858-9 and it was only ill-health that saw him resign. This argument may be further developed by looking at the weaknesses of Disraeli; he was after all an outsider for the Conservative party, not an Anglican. However, some might argue that he was their best speaker, had managed to pilot the Second Reform Act through parliament after Gladstone’s failure and shown himself to have appeal. It might be argued that his successful guidance of the Reform Act through parliament was what guaranteed him the succession. Disraeli was responsible for many of the changes that helped bring the Conservatives back to power; he taught them that it would be pointless to try to win support for the reversal of the Free Trade policy, promoted reform when in office in the minority government, developed ideas on the need for social reform, appointed Gorst to reorganise the party machinery and appealed to the electorate as the spokesman for a party that would offer a strong foreign and imperial policy. These positive qualities should be balanced against the claim that there were no alternatives once Derby resigned in February 1868.

6 ‘Tory Democracy was the most important element of Disraelian Conservatism.’ How

far do you agree with this view? No set answer is looked for but candidates will need to address the question. They do not need to know the details of the historical debate between a stress on traditional conservatism and newer Disraelian elements; however they will need to address the issue of Tory Democracy. Candidates may range widely across the whole period, although it is probable that they will focus on the period after 1867. The social reforms of the Second Ministry are likely to play a large role in many answers. Candidates might argue that Disraelian Conservatism was essentially a traditional English view of one-nation, supporting the traditional institutions of State: Monarchy, Aristocracy and the Church of

23

Page 127: History A - The Bicester Schoolthebicesterschool.org.uk/wp-content/uploads/2015/09/Mark... · 2015-09-28 · History A Mark Schemes for the Units January 2009 H106/H506/MS/R/09J

F961 Mark Scheme June 2009

England. Earlier evidence would suggest that this was the key to fend off Whig/Liberal cosmopolitanism. Some candidates might point to Disraeli’s novels, such as Sybil, to support their argument. The Second Reform Act could be used to show Disraeli restoring artisans to a system that had rejected them in 1832. In his second ministry he is concerned with the rural areas and clearing the church of Romish practices; this can be seen in his speeches at Crystal Palace and Manchester. Before 1872 it can also be argued that Conservatism saw foreign interests and the Empire as an extension of traditional values. Candidates could challenge the domination of conservatism by traditional English values and British interests by discussing whether Tory Democracy was ever the predominant force - an alliance between workers and Tory aristocrats against their common middle class liberal enemies, using worker numbers to gain power for a government who then delivered on social reform. Candidates might suggest other areas were the most important element. They might suggest the upholding of British interests abroad and in the Empire and this could be supported by considering the period from 1872-1880, with Disraeli seizing on Gladstone’s weakness over the Alabama Incident and the Black Sea Clauses and focusing on Empire, with South Africa, Egypt, India and Afghanistan. Upholding British interests was a key in his speeches of 1872 and was much condemned by Gladstone in his anti-Beaconsfieldism campaigns of 1876 and 1879-80. Candidates may conclude that all elements were important, or that one was more important at a particular moment.

24

Page 128: History A - The Bicester Schoolthebicesterschool.org.uk/wp-content/uploads/2015/09/Mark... · 2015-09-28 · History A Mark Schemes for the Units January 2009 H106/H506/MS/R/09J

F961 Mark Scheme June 2009

Foreign and Imperial Policies 1856-1914 7 To what extent were economic motives the most important reason for Britain

obtaining influence and possessions in Africa from 1868 to 1902? No set answer is looked for but candidates will need to address the question. Many candidates are likely to argue that trading developments were at the forefront of imperial growth in this period and use examples from South Africa and Egypt to support their arguments. It would also be valid to argue that the protection of the Indian trade routes would explain the involvement in South Africa, the East African coast and in protecting the Suez Canal post-1875, controlling the shares the acquisition of Egypt from 1882, involvement in the Sudan in 1885 and 1898, Zanzibar 1899 and the Fashoda incident with France in 1898 over the White Nile. Trade also had a large impact on individuals and they might point to Cecil Rhodes with the Cape to Cairo railway, which was clearly for profit. The discovery of gold and diamonds, and clear economic interests, in the Transvaal were important. Coffee and Tea plantations in East Africa also became economically important in the later period. However, this should be balanced against other factors such as strategic advantage, religious and humanitarian motives, political and jingoistic considerations and personal gain. The importance of strategic factors could be stressed in the need to limit German involvement in East Africa and French involvement in North and West Africa. Pre-1886 the strategic interests of Lord Carnarvon and Bartle Frere were important. Some may argue that it is impossible to differentiate between economic and strategic motives where North East, East and Southern Africa are concerned. Some may conclude that strategic issues were often determined by economic motives, although there were exceptions.

8 To what extent did the Anglo-Japanese alliance of 1902 mark a significant change in British foreign policy? No set answer is looked for but candidates will need to address the question. The question focuses on the view that after 1902 and the Anglo Japanese alliance British foreign policy changed from one of splendid isolation to full involvement in European affairs, which culminated in the decision to go to war in 1914. There are arguments that foreign policy did not change as Britain had always been involved in European affairs given her Empire and the Mediterranean Agreements of the 1880s and 1890s and issues with the USA. At the same time the Anglo Japanese agreement did not deal with affairs in Europe, but with imperial concerns in Asia where the fear of Russian growth remained. The treaty did not mark a watershed as it was designed to deal with a specific imperial issue and protect India. On the other hand it could be argued that this was the first formal alliance that Britain had entered into for a long time. There is also likely to be discussion of the 1903 Entente Cordiale, some will argue that this also drew Britain further into European affairs, although this could be balanced by a consideration that all it did was resolve colonial problems in Africa and was simply an understanding rather than a military alliance. However, candidates might go on to argue that in the longer term it did draw England further into Europe as there were military talks with the French, particularly over the stationing of the navies and support for them over Morocco. At the same time Britain did try to maintain good relations with Germany and it was the Kaiser who appeared to push England to choose, taking the view that friendship with France meant enmity with Germany, which was not Britain’s position. It should also be noted that throughout the period the main concern of Britain was to protect her Empire.

25

Page 129: History A - The Bicester Schoolthebicesterschool.org.uk/wp-content/uploads/2015/09/Mark... · 2015-09-28 · History A Mark Schemes for the Units January 2009 H106/H506/MS/R/09J

F961 Mark Scheme June 2009

9 How far was the German invasion of Belgium the main reason for Britain going to war in 1914? No set answer is looked for but candidates will need to address the question. Many candidates are likely to focus on the long-term problems in Anglo-German relations, but there must be some consideration of the importance of the invasion of Belgium in the decision to go to war. It was the invasion that triggered the declaration, although some may argue it was the occasion not the cause and that there were other reasons that caused it. Some may argue that the invasion provided a divided Liberal government with the excuse and justification it needed, to uphold the Treaty of 1830. There had already been resignations from the government and without the invasion it is unlikely that the government would have held together. War was seen by many as an illiberal act and therefore they needed the justification of the defence of a small nation to uphold the decision. However, answers may focus on the longer term problems of naval rivalry, the economic growth of Germany and imperial rivalry. Candidates might also consider the wish or need to support France following the Entente; if Britain did not support her following the military discussions of the previous years then her moral worth would be questioned. There may also have been concern that Britain would be isolated if Germany should win and it was therefore in her interests to ensure that this did not happen. Some answers might suggest that domestic problems also encouraged Britain to declare war as a short and glorious war would detract from the problems of Ireland, industrial unrest and the suffragettes. It might also be mentioned that declaring war was popular and most expected that it would be over by Christmas at the latest.

26

Page 130: History A - The Bicester Schoolthebicesterschool.org.uk/wp-content/uploads/2015/09/Mark... · 2015-09-28 · History A Mark Schemes for the Units January 2009 H106/H506/MS/R/09J

F961 Mark Scheme June 2009

Domestic Issues 1918-1951 10 How important was the First World War in the growth of the Labour party in the

period from 1918 to 1924? No set answer is looked for but candidates will need to address the question. The focus of this question requires a relative evaluation of the War among the various reasons contributing to the development of the Labour party. In considering the war as important candidates might consider the role of Arthur Henderson and Cabinet responsibility, new organisation at grass roots level, the importance of changes in the Franchise in 1918, the increased number of candidates in 1918 as opposed to 1910, the unity of the party during the war on issues that affected the working classes, such as the war emergency workers national committee, and the new constitution of the party in 1918. These factors might be compared with the importance of the Liberal split during the war and its subsequent consequences. The 1922 election was also important in the emergence of the Labour party. Candidates may also consider the importance of the formation of the First labour government as this established the Labour party as the genuine alternative and gave them credibility. The role of Macdonald may also be considered as he succeeded Clynes, his qualities of leadership and his socialist philosophy which was of the organic evolutionary type. The 1923 election was also important as Asquith decided to back Labour because of the danger of Liberal absorption by the Conservatives.

11 ‘Baldwin was the most important reason why the Conservative party dominated

British politics in the period from 1922 to 1939.’ How far do you agree with this view? No set answer is looked for but candidates will need to address the question. There are many reasons for Conservative dominance in this period and examiners should not expect candidates to cover all areas, what matters is the quality of the argument. The divisions within the liberal party after the First World War meant that Labour emerged as the main opposition. There were some fears about their links with Soviet Russia and communism which worried many of the middle class. Labour were also seen as inexperienced and their subsequent division in 1931 left the field open to the Conservative party. The extension of the franchise to women benefited the Conservative party. Legislation passed by Conservative governments was often appealing and mention might be made of the work of Neville Chamberlain at the Ministry of Health. The leadership of Baldwin might be seen as a positive reason; he seemed to reflect the national mood and his brand of new Conservatism with its appeal to Englishness and morality was popular. He was seen as being moderate and non-confrontational, apart from the General Strike, following a policy of one-nation Conservatism. It might be argued that the Conservatives gained from the creation of a National Government as some of the unpopular policies were not associated with them. As the policies were introduced under the umbrella of a National Government it appeared to many as if there was no alternative, particularly to orthodox economic policies. The National Government did appear to handle some issues well; the Abdication Crisis of 1936 might be used as an example. Some might even claim that the social and economic measures undertaken to cope with the Depression were beneficial, particularly for those who were in work as their standard of living rose.

27

Page 131: History A - The Bicester Schoolthebicesterschool.org.uk/wp-content/uploads/2015/09/Mark... · 2015-09-28 · History A Mark Schemes for the Units January 2009 H106/H506/MS/R/09J

F961 Mark Scheme June 2009

12 ‘The Second Labour government (1929-1931) was a complete failure.’ Assess this view. No set answer is looked for but candidates will need to address the question. Some candidates may argue that the short length of time spent in office by the government is a clear sign of its failure and this is reinforced by the divisions within the party that followed. However, this should be balanced against the fact that they were a minority government. Candidates might also consider the financial policies of Snowden at the Treasury, which were orthodox. The government was unable to deal with the economic problems created by the Depression and it is likely that this will form a central part of many answers. However, it is unlikely that any government would have been able to deal with the problem. The situation was worse for the Labour party as many of those who were made unemployed and would suffer from the proposed benefit cuts were their natural supporters and the people who they were supposed to represent. As they were a minority government they would be unable to pursue a socialist or radical policy on either unemployment or housing and this might therefore be considered a failure. Labour’s constitution, through Clause IV, committed them to this but as a minority government and faced with Liberal and Conservative hostility they had little chance. The moderation of both Macdonald and Snowden might be argued to have been a success as it convinced some that Labour were safe and moderate, which was vital to win votes. It might therefore be argued that Macdonald’s solution to the crisis was correct and that failure was the result of those who stuck to their trade union roots.

28

Page 132: History A - The Bicester Schoolthebicesterschool.org.uk/wp-content/uploads/2015/09/Mark... · 2015-09-28 · History A Mark Schemes for the Units January 2009 H106/H506/MS/R/09J

F961 Mark Scheme June 2009

Foreign and Imperial Policies 1945-1990 13 To what extent was the Second World War the main reason for Britain’s decision to

decolonise? No set answer is looked for but candidates will need to address the question. Many candidates are likely to agree with the assertion and may point to the economic difficulties created by the war and the political changes in Britain that the war had brought about as evidence, they may even suggest that dependence on the USA financially, that resulted from the war, was a further reason. Economic difficulties were serious as a result of the war, military and defence costs were crippling, but this was complicated by the belief that the Empire was less the problem than part of the solution to her economic difficulties as a closed imperial economy could guarantee markets, cheap food and raw materials. In order to overcome the financial problems at the end of the war Britain was heavily dependent upon the USA and they were opposed to colonialism and put pressure on Britain to abandon her Empire, in India in 1947, Palestine in 1948 and in 1956 to abandon Egypt and Suez. It was difficult for Britain to resist this unless they played the Cold War card, as they did in Malaysia. Candidates may also consider the pressure from colonial national movements such as the Gold Coast Riots in 1950, the Quit India Movement and the Mau Mau in Kenya. This also had an economic impact, that Britain could not handle after the war, and will provide candidates with opportunities to link together factors. Although there were many attempts to foster local elites and economies this strategy usually involved the acceptance of independence as a goal that tended to come sooner rather than later. This could be seen as the main reason for decolonisation by the 1950s. Some candidates may also argue that political change in the United Kingdom was particularly important; especially the attitudes of the political parties, originally united in resisting decolonisation this outlook had changed. Talk centred on managing decolonisation as part of trying to retain influence, this was certainly the view of Macmillan after 1957. Racism at home may also be mentioned.

14 To what extent did Britain lose its position as a great power in the period after 1945? No set answer is looked for but candidates will need to address the question. Answers may start by considering what is needed for great power status, but this then needs to be linked to developments after 1945. The economic consequences of the Second World War for Britain meant that she became heavily dependent upon US aid, it forced Britain to hand over the initiative to the US in Greece, Turkey, Persia and the Mediterranean. There is evidence for candidates to use to show that Britain was unable to stand up to the USSR. However, there were still signs that Britain was a major power; candidates might point to Britain’s place as a permanent member of the Security Council at the UN and the development of a British bomb, which was detonated in 1952. The loss of Empire might be used to show that British influence had declined, although some might suggest that the Commonwealth has kept some role for Britain. Most significantly many may consider the Suez crisis to show that Britain was no longer a great power and this lead ultimately to a withdrawal from east of Suez with the Gulf, Singapore and Malaysia by 1971. In some instances this was due to costs, again suggesting that Britain was unable to sustain its commitments. There may be consideration of an increased dependence upon the US and what that reveals about British power, this may be over nuclear weapons or détente. The development of NATO might be used to show that Britain was unable to defend herself, however this might be countered by reference to Britain’s role in countering the USSR as unlike France she did not commit to a purely European role; Britain was involved in Korea. There may also be a consideration of Britain’s involvement in resolving other conflicts in Asia. She was not involved in Vietnam as there were many other concerns and forces were overstretched. Involvement in Europe might be used to show that Britain needed closer European involvement and that her success, as in the Falklands, was limited to minor issues.

29

Page 133: History A - The Bicester Schoolthebicesterschool.org.uk/wp-content/uploads/2015/09/Mark... · 2015-09-28 · History A Mark Schemes for the Units January 2009 H106/H506/MS/R/09J

F961 Mark Scheme June 2009

15 How successful was the foreign policy of Thatcher?

No set answer is looked for but candidates will need to address the question. Candidates at the higher levels will probably identify the aims of her foreign policy, she had strong ideological convictions and a forceful style and use these as criteria against which to judge success or failure, rather than simply look at each event and assess it, however either approach is acceptable. It might be argued that her anti-communist views were very successful as she was in office as communism collapsed in Europe, but this was largely due to factors outside her influence. The consequence was the reunification of Germany, which she did not want. However, she was able to build up a working relationship with Gorbachev and initially use it to mediate between USSR and USA, but this was later undermined as the two powers reached an agreement at Reykjavik which would mean Europe would be exposed to the larger conventional forces of the USSR. Thatcher was able to limit the impact of this. This might be used as an example of occasions when Britain struggled to control US independence despite the special relationship between Thatcher and Reagan; other examples might include SDI and the invasion of Grenada. Thatcher did try to uphold British power and independence and this could be illustrated through reference to the Falkland’s War and negotiations with the EU. Thatcher was able to get back some of Britain’s contribution to the EU budget and it might be argued that this helped Britain’s standing in Europe, but this was at the expense of a good relationship with France and Germany and would cause problems in the future. Britain contributed to the SEA as it was consistent with her belief in free market economics. However, this might be balanced by considering the nuclear dependence Britain had on the US and mention might be made of the presence of US weapons in Britain. British willingness to work with the US in support for Kuwait and the Gulf War might also be used to show that Britain was a major power.

30

Page 134: History A - The Bicester Schoolthebicesterschool.org.uk/wp-content/uploads/2015/09/Mark... · 2015-09-28 · History A Mark Schemes for the Units January 2009 H106/H506/MS/R/09J

F961 Mark Scheme June 2009

Post War Britain 1951-1994 16 How far were Conservative scandals the most important reason for their defeat in

the 1964 election? No set answer is looked for but candidates will need to address the question. Labour won in 1964 by the large swing in the popular vote but only by a narrow margin of seats-317 to the Conservatives’ 304 and only a 5 seat majority in the Commons. Candidates should weigh up the importance of scandals in causing defeat against other factors. In considering the question of scandal most will consider the Profumo affair and its impact. Some answers will point to the youthful leadership of Wilson and his identification with all things modern, this contrasted well with the new aristocratic Conservative leader Sir Alec Douglas Home, who ran an ineffective campaign and did not handle television well. Wilson promoted his image with much talk of planning and the opportunities offered by the white heat of technology. The grey years of Gaitskell were over and Wilson exploited Conservative weakness, especially economic, with skill. However, many candidates will conclude that it was the Conservatives who lost the election in the years after 1959, rather than Labour or Wilson who won it. The Conservatives appeared too ‘Establishment’, the promotion of a peer to the leadership was a mistake given the satirists of the day. Party organisation lost its way after 1959; Butler replaced Hailsham and was in turn replaced by MacLeod. The affluence of the 1950s now appeared to be sluggish by comparison with elsewhere and the Conservative Chancellor imposed unpopular deflationary policies in 1961. Decolonisation and immigration unsettled some whilst a new economic policy, the New Approach, involving controlled expansion was undermined by De Gaulle’s veto of joining the EEC. A radical Cabinet reshuffle, the Night of the Long Knives, unsettled his ministerial colleagues when it was intended to create a fresh and dynamic government. Macmillan’s choice of replacement was botched; middle and working class voters were lost. Labour won on a modernising agenda.

17 How successful was the Heath government of 1970 to 1974? No set answer is looked for but candidates will need to address the question. Many will see this period as one of failure and will point to the Miners Strike and the Three Day Week, which appeared to epitomise the failed economic policy of the government. Heath has been criticised for failing to carry through the promised tough programme of economic and industrial reform on which the party had won the election of 1970. They started out determined to carry through a ‘quiet revolution’ by reducing the scale of the public sector and government intervention in the economy. The government was beset by a series of problems, but also made tactical errors of judgement. The Trade Union legislation was brought in very quickly and without sufficient consultation. The Industrial Relations Act was so broad in scope that it became a target for labour hostility; the good aspects were lost in the general bitterness about the method of its passage. The general refusal to comply with the terms meant it never became credible. The Ugandan Crisis inflamed hostility towards immigration, Ireland was another problem as Heath relied on the support of Unionist MPs. There were some achievements: taxes were cut, radical reform of tax and benefits system was well advanced when the government fell. However, it was the reversals of 1972 that the government is best remembered for. The decision to bail out Upper Clyde Shipbuilders, a year after refusal, was seen as a humiliating climb down. There was rising unemployment, combined with the determination to speed the rate of growth before entry to the EEC led to deliberate economic expansion, which flew in the face of the previous commitment to solve the problem of inflation. Voluntary wage control was impossible and Heath had to do this by law, the ultimate U turn.

31

Page 135: History A - The Bicester Schoolthebicesterschool.org.uk/wp-content/uploads/2015/09/Mark... · 2015-09-28 · History A Mark Schemes for the Units January 2009 H106/H506/MS/R/09J

F961 Mark Scheme June 2009

18 To what extent were Labour weaknesses the main reason for Conservative electoral victories under Thatcher? No set answer is looked for but candidates will need to address the question. There are a variety of reasons that candidates might offer for Conservative electoral success, however to achieve the higher levels candidates must write at least a good paragraph on the named factor even if they then argue that it was less important. When considering the weakness of Labour they might consider the weak leadership of Foot and Kinnock as major factors or they might look at areas of policy that were not popular with the electorate, particularly defence. Candidates might also consider the ‘Looney Left’ as a factor in discrediting Labour with the electorate. Labour were also closely associated with the Trade Unions and the question of too much union power, following the ‘Winter of Discontent’ might be seen as an issue. In the first period in office Labour weakness was an issue as with the economic problems of rising unemployment it should have been possible for Labour to have been a strong alternative. Against this candidates should consider the strength of the Conservative party. This might include the appeal of Thatcher as a strong leader, in contrast to the Labour party. Although she was controversial she appeared strong and willing to stand up to the Unions. The recovery of the economy helped later on, but crucial for the second term was the success of the Falkland’s War and restoration of pride that followed; the Conservative party were able to take full advantage of it as opinion polls beforehand were not good. Some answers might suggest that Thatcher had a strong set of ministers around her, others might comment on the reforms, particularly the denationalisation and selling of council homes which helped to create a new class to which Thatcherism appealed. There might be some consideration of changes in voting behaviour.

32

Page 136: History A - The Bicester Schoolthebicesterschool.org.uk/wp-content/uploads/2015/09/Mark... · 2015-09-28 · History A Mark Schemes for the Units January 2009 H106/H506/MS/R/09J

F962 Mark Scheme June 2009

33

F962/01 European and World History Period Studies Option A: Medieval and Early Modern 1095-1609

The Crusades and Crusader states 1095-1192 1 Assess the reasons why Pope Urban II called for a crusade in 1095. [50]

No specific answer is looked for. Candidates may address two main areas of reasons: those to do with developments in Asia Minor and the Holy Land and those to do with Western Christendom and the papacy. There should be coverage of both areas for candidates to score in the higher bands. In relation to the developments in Asia Minor and the Holy Land candidates may refer to the appeal from Alexius Comnenus which reached Italy in 1095, the advance of the Seljuk Turks towards Constantinople, the supposed atrocities of the Turks and the difficulties facing both Christians under Muslim rule and Christian pilgrims. There may be reference to longer term developments such as the defeat of the Byzantine Empire at Manzikert. In relation to reasons to do more directly with Western Christendom and the papacy, candidates may refer to Urban II’s desire to aid fellow Christians, to heal the rift between the Latin and Greek Churches and enhance the authority of the papacy both in the Byzantine Empire and in Western Christendom. In relation to the latter there may be discussion of the 11th century papacy’s attempts to establish its authority over the western church (uniting the Church under the papal banner in an armed pilgrimage against the infidel would help this aim). There may also be reference to the desire of the Church to secure peace in western Christendom by diverting its warrior class to war against the infidel. In discussing all the above candidates may refer to Urban’s appeal at Clermont in November 1095.

2 ‘The Second Crusade failed because there was no clear aim.’ How far do you agree

with this view? [50]

No specific answer is looked for, but candidates should address the factor in the question even if they wish to argue that other factors were more important. Candidates in dealing with the given factor may discuss the context of the calling of the crusade (fall of Edessa) and the dilution of the specific aim to recover it, both in the Crusade appeal and in the preaching of Bernard of Clairvaux and in the way the crusade developed from the authorization of a crusade against the Wends and the diversion of crusading effort in the Iberian peninsula. In addition once the crusade reached the Levant, it became clear that to re-take Edessa was impractical, Antioch had failed to persuade Louis to attack Aleppo and the council at Acre after much discussion agreed to attack Damascus. However, dilution of aims was just one reason for failure and candidates may set discussion of this against other factors such as the divisions amongst the crusader leadership (both between Louis and Conrad and in the Holy Land), the defeat of Conrad in Asia Minor, the relative unity of the enemy forces, rivalries in the Holy Land, strategic errors and the strength of Nur ed Din.

3 Assess the view that the Third Crusade was a failure. [50]

No specific answer is looked for but candidates will need to assess the degree of success or failure of the Third Crusade. In considering the arguments for failure, candidates may address the aims of the crusade, the outcomes of it and the historical context. They may well consider the failure to take Jerusalem or decisively defeat Saladin as well as the divisions between Crusade leaders (Richard and Philip), the rivalries over who should be King of Jerusalem and the disintegration of the German effort after the death of Frederick

Page 137: History A - The Bicester Schoolthebicesterschool.org.uk/wp-content/uploads/2015/09/Mark... · 2015-09-28 · History A Mark Schemes for the Units January 2009 H106/H506/MS/R/09J

F962 Mark Scheme June 2009

Barbarossa. On the other hand candidates may stress the historical context – the relative strengths of the Saladin, the weak position of the remaining crusader forces in the Holy Land, the lack of support from the Byzantine Empire – and the achievements of the campaign – the taking of Cyprus, the taking of Acre, the defeat of Saladin at Arsuf, and the negotiated truce which guaranteed the continued survival of the rump of the Kingdom of Jerusalem and the rights of pilgrimage.

34

Page 138: History A - The Bicester Schoolthebicesterschool.org.uk/wp-content/uploads/2015/09/Mark... · 2015-09-28 · History A Mark Schemes for the Units January 2009 H106/H506/MS/R/09J

F962 Mark Scheme June 2009

The Renaissance from c. 1400- c. 1550 4 How important was the papacy and the Church in the development of the

Renaissance in Italy? [50]

No specific answer is looked for, but candidates do need to deal fully with the role of the papacy and the Church even if they wish to argue that its importance was not as significant as other factors. In relation to the Church and the papacy candidates may discuss some of the following (in no particular order): patronage of Church towards humanists (like Bruni and Bembo) and artists (like Masaccio, Raphael and Michelangelo), sculptors (like Brunelleschi); influence of the Church more generally in shaping the context of Renaissance art/sculpture and writing – the importance of religious themes and scenes (candidates may refer to particular examples (Leonardo’s ‘Last Supper’, Michelangelo’s ‘Pieta’ etc.)); the role of specific popes – such as Julius II’s employment of Bramante and Raphael and Pius II who was himself a humanist scholar, or Nicholas V and Sixtus IV who created and developed the Vatican Library and the popes Leo X and Clement VII who made Rome the centre of the Renaissance. There may be some discussion of the later period where the influence of Rome gave way to Venice, although religious themes remained dominant. Such discussion needs to be set in the wider context of developments in the Renaissance to answer ‘How important?’ – the role of the nobles, princes, guilds and other patrons who commissioned and influenced the subject matter of the Renaissance, the influence of classical ideas and literature, the individual genius of particular artists and writers and so on.

5 To what extent did the Medici family influence the development of the Renaissance

in Florence? [50]

No specific answer is looked for, but candidates do need to deal fully with the role of the Medici in Florence even if they wish to argue that other factors were as or more important. Discussion of the role of the Medici in Florence may include reference to Giovanni di Bicci, Cosimo (Medici palace, church of San Lorenzo (Brunelleschi), monastery of San Marco (Fra Angelico), Gozzoli’s ‘The Procession of the Magi’, Piero, Lorenzo (patron of Ficino and other Neo-Platonists)). Candidates may discuss the influence of the Medici in terms of both patronage and their influence over the type and style of art. Such discussion needs to be balanced against other influences, such as that of the guilds (wool, cloth, silk merchants in particular), other prominent individuals (the struggle between the Strozzi family and the Medici for example), the reaction against the lavish patronage and subject matter that came with Savonarola after the fall of the Medici in 1494 (eg Savonarola’s influence on Botticelli, and the influence of the Florentine Republic after 1498 with its emphasis on civic virtues).

6 Assess the view that the Italian Renaissance was the main influence on the

development of the Northern Renaissance. [50]

No specific answer is looked for, but candidates do need to deal fully with the role of the Italian Renaissance even if they wish to argue that other influences predominated. On the influence of the Italian Renaissance, candidates may discuss the influence of Rome and the Church, attendance of northern students at Italian universities such as Bologna and Padua, the exposure of foreign merchants to Italian Renaissance art/sculpture etc. in the major centres of trade (Florence, Milan and Venice). Candidates may discuss the influence of Italian artists in the work of Holbein and Dürer and the influence of Italian humanism on northern humanism. Candidates may balance such discussion against native and distinctively northern characteristics/developments including, for example, the Christian focus of northern humanism and its concern for true reading of the scriptures (albeit by using the methods of Italian scholars) – there may be reference to Reuchlin, Erasmus, and others. Similarly there may be reference to the distinctiveness and realism of northern

35

Page 139: History A - The Bicester Schoolthebicesterschool.org.uk/wp-content/uploads/2015/09/Mark... · 2015-09-28 · History A Mark Schemes for the Units January 2009 H106/H506/MS/R/09J

F962 Mark Scheme June 2009

artistic developments, particularly in relation to Dutch art and the concerns of northern patrons and the influence of Lutheran and Calvinist ideas.

Exploration and Discovery c.1445-c.1545 7 To what extent were economic factors the main reason for undertaking overseas

exploration in this period? [50]

No specific answer is looked for, but candidates must deal with the given factor adequately even if they wish to argue that other factors were more important. Candidates are likely to argue that economic factors were the most important motivation although this will need analyzing and supporting. Candidates may discuss the desire to break into the spice trade – pepper, cinnamon, cloves – and the trade in luxury goods (silks and jewels) both of which offered the prospect of high profits – so the search was on for a new route to the east. This search was also motivated by the growing insecurity of supply (partly as a result of Ottoman advance). This desire helps to explain Portuguese search for a route round Africa and Columbus’ attempt to find an ocean route across the Atlantic. A further economic motive was gold – a factor in Portuguese exploration of the African coast (‘Gold Coast’) and Spanish exploration of Latin America. A further economic motive was the search for labour (slaves) – especially for Portugal. Broadly economic still was the motive to find land to settle (Columbus’ 1493 voyage included 1200 settlers). Consideration of economic motives needs to be balanced against other reasons. Candidates may consider the role of religion – to spread Christianity and find Prester John and other Christians – explorers testified to the dual motivation of religion and gold/spices. They may also consider the influence of the Renaissance (and its emphasis on human endeavour), issues of individual and national prestige/rivalry and the desire for knowledge.

8 Assess the reasons why Portugal was able to develop an overseas Empire in this

period. [50]

No specific answer is being looked for but candidates will need to discuss and evaluate a range of reasons to score well. Candidates may discuss some of the following reasons, some specific to Portugal, some more general. Candidates may consider the technological developments such as that of a suitable ocean-going vessel – the caravel, with its shallow draught, seaworthy construction and lateen sails, enabling effective handling in all winds – and the larger cargo vessel – the carrack. They may also discuss the development of the compass, astrolabe and Zacuto’s method for calculating latitude. The development of gunpowder technology also enabled ships to be defended. Candidates may also consider Portugal’s geographic position, its maritime experience, the significance of the capture of Cueta, and development of Madeira, the Azores and Cape Verde islands, the establishment of forts and trading posts on the African coast, military superiority and destruction of rival fleets, exploitation of divisions in the political situation in Asia. Candidates may also stress the roles of individuals from kings like Henry the Navigator and John II to explorers like Diaz, da Gama and Cabral as well as crucial agreements with the Spanish (such as that over the Canaries in 1479 and the deal with Charles V in 1529 over the spice islands).

9 ‘The Spanish Empire brought Spain as many problems as benefits.’ How far do you

agree with this view? [50]

No specific answer is being looked for and candidates may agree or disagree to an extent with the view expressed. What is important is that candidates do not simply describe problems and benefits, but also evaluate their significance to reach a judgement on the balance between problems and benefits to Spain. Among the benefits we can expect candidates to consider the import of specie (mainly silver), the prestige for Spain and the Spanish Crown, the acquisition of land and territories for the Spanish Crown, the

36

Page 140: History A - The Bicester Schoolthebicesterschool.org.uk/wp-content/uploads/2015/09/Mark... · 2015-09-28 · History A Mark Schemes for the Units January 2009 H106/H506/MS/R/09J

F962 Mark Scheme June 2009

opportunities for ambitious Spaniards for adventure and settlement in the New World and the growth of trade between Spain and the Americas. Amongst the problems, candidates may consider problems of control and organization both of conquistadors and the colonies established (the tensions between royal governors and the established colonists), the distorting effects that the import of gold and silver began to have on the economy, the security of the routes to and from the New World, the problem of the treatment of native populations and so on.

Spain 1469-1556 10 To what extent were Ferdinand and Isabella’s domestic policies influenced by their

religion? [50]

No specific answer is looked for. Candidates need to discuss the influence of religion on domestic policy and need to balance this discussion against the influence of other factors to address ‘To what extent?’. Candidates may discuss the role of religion for both Ferdinand and Isabella and may discuss the role of religion in their specifically religious policies (such as Church reform, the Inquisition and policies towards convivencia, Jews and Moors). Though not strictly domestic policy, there may be discussion of the conquest of Granada. There may be discussion of other domestic policies where religion plays little or no role: policy towards nobles, towns, administration, finance etc. Candidates may well argue that the influence of religion was stronger for Isabella than Ferdinand and that the motivation for most/all religious policies was a mixture of religious and other motives.

11 How successful were the foreign policies of Ferdinand and Isabella? [50]

No specific answer is looked for. Success may be assessed by reference to aims, outcomes and context. Candidates may or may not include the conquest of Granada in foreign policy. Clearly the conquest of Granada was successful in terms of its main aim, but candidates may also discuss its consequences for Ferdinand and Isabella’s position in Spain and their international reputation. Candidates may argue that because of the civil war and then the war with Granada, Ferdinand and Isabella were not able to pursue and active foreign policy until1492. They may argue also that thereafter their foreign policy was largely opportunistic. They may discuss the need to check France and the attempts to construct anti-French coalitions and marriage alliances, the attempts to intervene in Italy (to prevent French domination) and the attempts to expand influence in North Africa. Candidates may argue that success was considerable: with success in northern Spain (acquisition of Cerdagne, Rousillon and Navarre) and success in containing French influence in Italy through diplomacy and arms (Cordoba’s generalship may be referred to) and cementing Aragonese interests in Naples and Sicily. They may also refer to North African conquests such as that of Tripoli (1510) and to the beginnings of a Spanish Empire in the Americas.

12 How successful was the domestic policy of Charles I? [50]

No specific answer is looked for. Candidates may assess success by, for example, testing polices against aims, results and historical context; analysis may also consider success at different times or in different areas but there needs to be some overall judgement as well about the reign as a whole. This is a question about domestic policy (in Spain) and discussion of foreign policy/ other elements of Charles’ monarchia should not be credited unless it is in terms of its impact on the success of domestic policy. Candidates may consider: how well Charles dealt with his initial difficulties; his relations with the Cortes of Castile; policy towards Aragon; relations with the nobility; administrative reform; financial and economic policy; religion; the impact of absence, costs of foreign policy, Americas etc.

37

Page 141: History A - The Bicester Schoolthebicesterschool.org.uk/wp-content/uploads/2015/09/Mark... · 2015-09-28 · History A Mark Schemes for the Units January 2009 H106/H506/MS/R/09J

F962 Mark Scheme June 2009

Charles V: International Relations and the Holy Roman Empire 1519-1559 13 To what extent were Luther’s ideas between 1517 and 1530 simply a reaction against

abuses in the Catholic Church? [50]

No specific answer is looked for. Candidates may well argue that although abuses in the Catholic Church played a part, other factors played a great role in shaping Luther’s ideas. In relation to the abuses in the Catholic Church, candidates may discuss the indulgences controversy and the 95 Theses, the Address to the Christian Nobility of the German Nation (with its attack on Church corruption/abuse) … Such discussion may be balanced against other influences, such as his theological studies and his belief in the efficacy of faith (they may refer to the so-called ‘tower experience’) and its development into the doctrine of ‘sola fide’, the development of the idea of sola scriptura in response to the debate with Eck, the idea of the priesthood of all believers, the reliance on scripture led him to question the sacraments (The Babylonish Captivity of the Church). Candidates may argue that Luther’s ideas were developed out of his own study of theology and concern for his own salvation and then in response to the debate and pressures his ideas unleashed rather than a reaction to the abuses of the Church – the latter provided more the occasion for the exposition and development of his ideas than the cause of them.

14 ‘The power of the princes was the main reason Charles V was unable to crush

Lutheranism.’ How far do you agree with this view? [50]

No specific answer is looked for, but candidates must deal with the given factor adequately even if they wish to argue that other factors were more important. Candidates may well argue that the power of the princes was the main factor and discuss, for example, the limited ability Charles had to act without princely cooperation, the role of Frederick of Saxony, the Diet of Augsburg, the Schmalkaldic League, Charles’ need to appease the princes to pursue his wars with France, the unwillingness of the Catholic princes to enhance Charles’ power after the victory at Mühlberg, the revolt of the princes and the Peace of Augsburg. Candidates may also discuss the impact of Charles’ absences (especially in the 1520s), the Diet of Worms, his initially conciliatory policy towards the princes, the nature of his authority within the Holy Roman Empire, the attractions of Lutheranism to princes, peasants, towns and so on, the variable attitudes of popes, and so on.

15 How successful was Charles V in dealing with the Ottoman threat? [50]

No specific answer is looked for. Candidates may assess success by, for example, testing polices against aims, results and historical context; analysis may also consider success at different times but there needs to be some overall judgement as well about success as a whole. Candidates may discuss the situation near the start of Charles’ reign and the threat posed by the Ottomans in the Mediterranean (fall of Rhodes, alliance with Barbary pirates) and the Balkans (conquest of Belgrade). In relation to the former candidates may discuss, for example, the capture of Tunis and its impact and the failure against Algiers; in relation to the latter, the defeat at Mohacs and the siege of Vienna and its impact. Judgements on success or failure may involve discussion of the role of factors such as the demands on Charles of other problems (e.g. war with France or the princes in Germany), available resources (e.g. no navy able to take on the Turks), the limitations on the Ottomans (e.g. their other problems, such as Persia, and the constraints of distance from Constantinople).

38

Page 142: History A - The Bicester Schoolthebicesterschool.org.uk/wp-content/uploads/2015/09/Mark... · 2015-09-28 · History A Mark Schemes for the Units January 2009 H106/H506/MS/R/09J

F962 Mark Scheme June 2009

Philip II, Spain and the Netherlands, 1556-1609 16 How successful were Philip II’s religious policies in Spain? [50]

No specific answer is looked for. Success may be assessed by reference to aims, outcomes and context. Candidates may discuss Philip II’s faith, relations with the papacy, and the position of the Church and royal powers over it at the start of the reign by way of context. There may be discussion of: the differences with the papacy over matters such as Church jurisdiction; the need for reform and Philip’s success in promoting it; the work of the Inquisition; policy towards conversos and ‘heretics’. Candidates may argue that overall the success of Philip II’s religious policies was variable by arguing, for example, that whilst Philip maintained control of the Church, relations with the papacy were uneasy; that there was some success in Church Reform and some evidence of a revival in religious fervour.

17 To what extent was Philip II responsible for the outbreak of rebellion in the

Netherlands to 1572? [50]

No specific answer is looked for. Candidates must deal with the given factor adequately even if they wish to argue that other factors were more important. In relation to Philip II, candidates may discuss Philip’s absence from 1559, his lack of understanding of the depth of feeling in the Netherlands, his religious policies (reform of bishoprics, the Segovia letters), his decision to send a Spanish army to the Netherlands, his appointments such as Granvelle and Alba and their policies. Such discussion may be balanced against the role of other factors, such as the actions of Margaret of Pama, Granvelle and Alba, the longer term context of regional, States and noble privileges, the burdens of taxation and the spread of heresy, the Iconoclastic Fury, hedge preaching, the roles of Egmont, Horne, William of Orange and Brederode, the Tenth Penny tax, the Sea Beggars and so on. Candidates may well argue that the outbreak of rebellion was caused by a combination of Philip’s uncompromising rule from Spain and the actions of his lieutenants in the Netherlands in the context of a Netherlands and nobility jealous of their privileges/semi-independence, economic hardship and the spread of Protestantism.

18 How important was William of Orange to the success of the Dutch revolt? [50]

No specific answer is looked for. Candidates must deal with the given factor adequately: the focus must be on the assessment of the role of William of Orange, an assessment that may well involve evaluating his role against that of other factors. In relation to William of Orange candidates may discuss his abilities and contribution as a figurehead, statesman and military leader in the years up to his assassination and his importance as a symbol of national resistance thereafter. They may refer to his role in the initial stages of the Revolt, his importance in Holland and Zeeland in the early 1570s, his role in the Union of Utrecht, and his role in securing foreign support. Such discussion may be set in the context of the importance of other factors, such as the impact of Spanish policy, diversions and mistakes, the roles of other leaders such as Maurice of Nassau, the significance of foreign aid (especially from England), the significance of religion and the resistance of Holland and Zeeland and so on.

Paper Total [100]

39

Page 143: History A - The Bicester Schoolthebicesterschool.org.uk/wp-content/uploads/2015/09/Mark... · 2015-09-28 · History A Mark Schemes for the Units January 2009 H106/H506/MS/R/09J

F962 Mark Scheme June 2009

40

F962/02 European and World History Period Studies Option B: Modern 1795-2003

Answer any two questions from either one or two of the Study Topics. Napoleon, France and Europe 1795-1815 1 To what extent did the creation of the Empire in 1804 mark a significant change in

Napoleon’s rule of France? [50]

No specific answer is looked for. To score highly candidates should discuss elements of continuity and change before and after 1804. Candidates may point to the change from Republic to Empire and stress the increasing authoritarianism of Napoleonic rule (eg institution of hereditary principle, increasing use of senatus consultum and arbitrary arrest and the creation of an imperial nobility) and the lack of significant reform after 1804. Others may stress the elements of continuity and argue that authoritarian rule was implicit form the start (Napoleon’s powers as First Consul, the changes to the Constitution, the ‘sham’ of popular sovereignty (there may be reference to ‘plebiscitory dictatorship’, the elements of the police state and so forth. They may argue that the story of Napoleon’s rule was one of increasing dictatorship from the start and that 1804 was just one stage in this process.

2 How far can Napoleon’s military success in Europe from 1796 to 1809 be explained

by the weaknesses of his opponents? [50]

No specific answer is looked for. Candidates must deal with the given factor adequately even if they wish to argue that other factors were more important. In relation to the weaknesses of his opponents, candidates may discuss some of the following: the size, membership and organisation of their armies and their strategies/tactics, the competence of their generals, the divisions between political and military leadership, the weaknesses of coalitions and alliances and so forth. Such discussion may be contrasted with the size, membership and organisation of the French army, the battlefield tactics and campaign strategies reflecting both the legacy of pre-Napoleonic reforms and Napoleon’s developments, the competence of the officer corps, Napoleon’s generalship, Napoleon’s combination of political and military leadership, the resources of France, and so on. Candidates may well support their arguments by reference to Napoleon’s Italian campaigns, the Marengo campaign, Ulm and Austerlitz, Jena and Auerstadt, Eylau and Freidland, Wagram.

3 To what extent does British opposition explain Napoleon’s eventual defeat in 1814

and 1815? [50]

No specific answer is looked for. Candidates must deal with the given factor adequately even if they wish to argue that other factors were more important. In relation to British opposition, candidates may discuss British naval supremacy and the use of Orders in Council, British diplomacy in organizing and subsidizing anti-Napoleon coalitions, British military action, especially in support of the Peninsular War, Wellington and Waterloo and so forth. Such discussion may be balanced against discussion of the impact of the Continental System, the growth of opposition to Napoleon, the Peninsular War and Russian campaign (1812), the arguable decline of Napoleon’s generalship and armies, the improvements in his opponents’ armed forces and officer corps, the adaptation to Napoleonic tactics, the efforts of the Quadruple Alliance and so on.

Page 144: History A - The Bicester Schoolthebicesterschool.org.uk/wp-content/uploads/2015/09/Mark... · 2015-09-28 · History A Mark Schemes for the Units January 2009 H106/H506/MS/R/09J

F962 Mark Scheme June 2009

Monarchy, Republic and Empire: France1814-1870 4 To what extent should Louis Philippe’s foreign policy be considered a failure? [50]

No specific answer is looked for. Failure may be assessed by reference to aims, outcomes and context. In assessing Louis Philippe’s foreign policy, candidates may discuss some of the following: the Belgian question, events in Poland and Italy, the Mehmet Ali crisis, Tahiti and the Spanish marriage. Candidates may seek to compare the general approach of Louis Philippe (avoid war, cooperate with Britain) with the clamour in the French press for glory and a more active foreign policy. By the former the foreign policy may be judged a success, by the latter a failure (‘France is bored’). Candidates may see failure also in the role apparently weak foreign policy had in undermining support for the Orleanist monarchy and contributing to the Revolution in 1848. Candidates may also point to the ‘missed opportunity’ of the Belgian issue and the humiliation of the outcome of the Mehmet Ali Crisis.

5 Assess the reasons why the Second Republic was short-lived. [50]

No specific answer is being looked for but candidates will need to discuss and evaluate a range of reasons to score well. Candidates may discuss some of the following: the tension between the Parisian/left wing revolution and the provinces reflected in the elections to the Constituent Assembly; the June Days and its repression and its legacy of bitter division; the weaknesses of the constitution (especially with regard to the roles of President and Assembly); the election of Louis Napoleon; the misjudgement of politicians who believed Louis Napoleon could be managed; changes to the franchise; Louis Napoleon’s ambitions and exploitation of divisions; the carefully managed coup of December 1851 and the plebiscite of 1852. Candidates may argue that the key factors were the tensions between left and right, the popularity of Louis Napoleon and his careful propaganda, the misjudgement of the politicians like Thiers and the lack of support for the Republic from the workers after the June Days.

6 How successful was Napoleon III’s domestic policy? [50]

No specific answer is looked for. Success may be assessed by reference to aims, outcomes and context. In relation to aims, candidates may claim that Napoleon had no clear aims beyond staying in power or may focus on his social and economic aims as outline, for example, in his widely distributed booklet, The Extinction of Pauperism. In relation to social and economic policy, candidates may point to some successes, for example, state stimulus to railway building (fivefold increase in extent) through operating leases and the considerable knock-on effects to other industries (6% p.a. growth in iron, steal and coal) and agriculture (extension of railway network stimulated production for urban markets). They may also point to promotion of banking and free trade (Chevalier Treaty with Britain) and the work of Haussman in Paris. There are qualifications to this success (such as the end of the railway boom in the 1860s and the need to rescue Crédit Moblier in 1867). In relation to political survival, candidates may argue that staying in power for 18 years was a considerable achievement achieved by retaining popular (at least peasant) support and modifying the constitution. Candidates may also argue that ultimately Napoleon III failed as opposition increased and economic woes increased in the late 1860s.

41

Page 145: History A - The Bicester Schoolthebicesterschool.org.uk/wp-content/uploads/2015/09/Mark... · 2015-09-28 · History A Mark Schemes for the Units January 2009 H106/H506/MS/R/09J

F962 Mark Scheme June 2009

The USA in the 19th Century: Westward Expansion and Civil War 1803-c.1890 7 Assess the reasons for the acquisition of territory by the United States from the

Louisiana Purchase in 1803 to the Gadsden Purchase in1853. [50]

No specific answer is being looked for but candidates will need to discuss and evaluate a range of reasons to score well. Candidates may refer to the Louisiana Purchase, the acquisition of Florida, Texas, New Mexico and California, Oregon and the Gadsden Purchase. Candidates may also refer to the acquisition of territory from the Indians. They may discuss the reasons for particular acquisitions and/or assess more general reasons for the acquisition of territory. Candidates may argue that one major reason for acquisition of territory was to prevent it falling into other hands and point to Jefferson’s fears over New Orleans (seen as vital to US economic interests) being in French hands under Napoleon. They may point to the actions of individuals such as General Jackson in Florida. They may point to rivalry with Britain over Oregon and the need for land for settlement and the appeal of American settlers over Texas and California. They may also discuss the ambitions of different presidents (for example, Polk over Texas) and the propaganda associated with ‘Manifest Destiny’.

8 How important was the issue of slavery in causing the American Civil War? [50]

No specific answer is looked for. Candidates must deal with the given factor adequately: the focus must be on the assessment of the role of slavery, an assessment that may well involve evaluating his role against that of other factors. In relation to slavery, candidates may argue its central importance as the defining feature of the South and its way of life and argue that the development of the United States and the implications of westward expansion if slavery was limited threatened the South’s future. They may argue that it was the issue of slavery expansion in new territories rather than slavery itself that polarized the nation. Such discussion may be linked to other developments and factors, such as the emergence of a ‘northern’ Republican Party, the preservation of the southern economy in the face of the industrial north, the issue of States’ rights (including the right to secede from the Union), the tensions roused by abolitionists and fire-eaters, violent incidents (‘Bleeding Kansas’ and John Brown’s raid) and the consequences of the election of a Republican president.

9 To what extent were superior resources the main reason for the Union victory in the

American Civil War? [50]

No specific answer is looked for. Candidates must deal with the given factor adequately even if they wish to argue that other factors were more important. In relation to superior resources, candidates may compare North and South in terms of the relative size of populations, the degree and extent of economic development, economic resources and railways, merchant navy and trade and so forth, arguing that in the long term the North’s superiority would tell in any war of attrition. Such discussion need to be balanced against other factors that shaped the wars course and influenced its outcome, such as war aims, strategies, army sizes, generalship, morale, battles and campaigns, political leadership, pubic opinion, international opinion and support. Candidates may argue that at first the two sides were evenly matched and that, if anything, the South had advantages over the North and that resources only became significant as the war dragged on without decision.

42

Page 146: History A - The Bicester Schoolthebicesterschool.org.uk/wp-content/uploads/2015/09/Mark... · 2015-09-28 · History A Mark Schemes for the Units January 2009 H106/H506/MS/R/09J

F962 Mark Scheme June 2009

Peace and War: International Relations c.1890-1941 10 To what extent was Germany responsible for the outbreak of the First World War?

[50]

No specific answer is looked for. Candidates must deal with the given factor adequately even if they wish to argue that other factors were more important. In relation to German responsibility, candidates may discuss the aims and nature of Wilhelmine foreign policy, especially in relation to elements such as the Moroccan crises, the Dual Alliance and the ‘blank cheque’, and German actions in the arms and naval races. Such discussion may be balanced against the degree of culpability of other states (Russia, Austria, France and Britain), the role of militarism, imperialism, the Alliance system, domestic pressures, the Balkan crises and the decisions taken during the July crisis. Candidates may show awareness of the Fischer thesis, but this is not required.

11 How successful was the League of Nations in resolving international disputes in the

1920s? [50]

No specific answer is looked for. Success may be assessed by reference to aims, outcomes and context. Candidates may discuss the role of the League of Nations in some of the following disputes: Poland-Lithuania dispute, Aaland Islands, Corfu, Greece- Bulgaria dispute, Iraq-Turkey dispute. Candidates may argue that the League was successful in many of these but qualify this by recognizing their minor nature; they may also suggest that the limitations of the League’s success was demonstrated in particular by the Corfu Incident when a great power was involved. Candidates may also suggest that often international disputes were settled in other ways: by the Conference of Ambassadors or by international treaties (such as the Locarno and Kellogg-Briand treaties). They may also claim that the League’s success can be exaggerated because of post-war desire for peace. Candidates may also explain the limitations of League success by reference to the weaknesses of the League as a peacekeeping organization.

12 Assess the reasons why Britain followed a policy of appeasement in the 1930s. [50]

No specific answer is being looked for but candidates will need to discuss and evaluate a range of reasons to score well. Candidates may set the policy in the context of the impact of the Great Depression, the weakness of the League of Nations, American isolationism and the growth of aggressive nationalism both in Asia and Europe in the 1930s. Candidates are likely to focus their attention on British policy towards Germany and may focus on the late 1930s. They may discuss some of the following reasons for the policy: feelings of justice and morality (e.g. Germany had some legitimate claims in relation to the Treaty of Versailles); economic necessity and the limited resources available to finance military expenditure; the time required for military rearmament; Britain’s global responsibilities and the diplomatic realities (e.g. a weak/politically unstable France with its Maginot mentality, alienation of Italy over Abyssinia and US isolationism); and the influence of public opinion. Candidates may also point to the beliefs of some politicians that Germany could be reasoned with and that a strong Germany was important as a bulwark against Communism.

43

Page 147: History A - The Bicester Schoolthebicesterschool.org.uk/wp-content/uploads/2015/09/Mark... · 2015-09-28 · History A Mark Schemes for the Units January 2009 H106/H506/MS/R/09J

F962 Mark Scheme June 2009

From Autocracy to Communism: Russia 1894-1941 13 How successful were the economic and social policies of the Tsar’s government

from 1894 to 1914? [50]

No specific answer is looked for. Success may be assessed by reference to aims, outcomes and context. Candidates may focus their attention on the policies of Witte and Stolypin and may not distinguish clearly between economic and social policies. Candidates may consider policies related to agriculture and the peasantry and those related to industry and urban living and working conditions. In relation to agriculture and the peasantry candidates may point to the context of land hunger, rising population and harvest failures, subsistence farming dominated by the mir, and lack of mechanisation. It may be argued that government policy was motivated by government concern for its own income, the interests of the nobility and the desire to contain peasant unrest, and discuss measures such as peasant resettlement to Siberia, and Stolypin’s measures to encourage peasant independence and release of state lands to the peasants. They may argue that by the time of his assassination in 1911 success was limited, with more success in encouraging peasant landownership than in creating viable consolidated farms. In relation to industry, candidates may argue that Witte’s reforms focused on the encouragement of industry through protectionism and foreign investment, and argue that whilst there were some impressive percentage improvements in heavy industrial production and railway extension the levels achieved by 1914 were well below other great powers. There was also a price in terms of urban living and working conditions and here the government did little to regulate factories. !905 and the strikes of 1912-14 may be used as evidence of the limited effectiveness of reforms and policies to address the fundamental conditions in Russia.

14 ‘The Bolsheviks were able to seize power in October 1917 mainly because of the

weaknesses of the Provisional Government.’ How far do you agree with this view? [50] No specific answer is looked for. Candidates must deal with the given factor adequately even if they wish to argue that other factors were more important. The focus of the question is on 1917 and specifically reasons for the October Revolution. In relation to the Provisional Government, candidates may refer to the circumstances of its creation, its legitimacy and relationship with the Petrograd Soviet, its policies towards the war, the land question, the economy and its ability to deal with opposition. Candidates may argue that its fate was not inevitable and suggest the failure of the Kerensky offensive, of Kornilov’s attempted coup and to deal with the land question were crucial to its fate. Such discussion may also discuss the exploitation of unrest by the Bolsheviks and the effectiveness of their propaganda and organisation after the return of Lenin, and the roles of Lenin and Trotsky in orchestrating the coup of October.

15 How successful were Stalin’s economic policies in the 1930s? [50]

No specific answer is looked for. Success may be assessed by reference to aims, outcomes and context. Candidates may focus their discussion on collectivization and the five year plans. They may refer to Stalin’s desire to catch up with the West and destroy capitalist elements in industry and agriculture (and his targeting of Kulaks). In relation to the Five Year Plans candidates may discuss the differences between targets, propaganda claims and achievements, but may well still argue that results in terms of production were impressive. They may also suggest the second Five Year Plan learnt some lessons from the mistakes of the first (more reasonable targets and concern for infrastructure), whilst the third was dominated by rearmament. Candidates may also argue that whilst the economic results were impressive the social costs were high with highly controlled and disciplined workers and decline in living standards (at least in the early 1930s). In relation to agriculture, candidates may argue that the forced collectivization was successful insofar as farms were collectivized into Sovkhoz and Kolkhoz, but had a disastrous impact, at least in the short term, on agricultural production and led to famine in the countryside. They may also stress the social costs of the policy as the Kulaks were eliminated.

44

Page 148: History A - The Bicester Schoolthebicesterschool.org.uk/wp-content/uploads/2015/09/Mark... · 2015-09-28 · History A Mark Schemes for the Units January 2009 H106/H506/MS/R/09J

F962 Mark Scheme June 2009

Democracy and Dictatorship: Italy 1896-1943 16 How successfully did Italian governments deal with the problems they faced from

1896 to 1915? [50]

No specific answer is looked for. Success may be assessed by reference to aims, outcomes and context. Candidates may identify a number of problems and assess the policies adopted by Italian governments to them, but there also needs to be some overall assessment of success. Candidates may point to the problems posed by the development of socialism and trades unions, relations with Catholic opinion, economic problems, questions of social welfare, unrest, the poverty of the South, the widening of the franchise, the legacy of the humiliation of Adowa, nationalist and imperial aspirations, violence, assassinations and the disorders of 1898 and 1914, emigration and so on. In relation to these candidates may focus on the leadership of Giolitti and his trasformismo approach, the use of repression and censorship, limited social reforms, taxation policy, the Libyan issue, the decision to widen the franchise and its results, and so on. Candidates may argue that for all Giolitti’s ability to manage parliament, balance interests and introduce some limited reforms, his success was limited and many problems remained.

17 To what extent do economic problems explain Mussolini’s rise to power in 1922? [50]

No specific answer is looked for. Candidates must deal with the given factor adequately even if they wish to argue that other factors were more important. In relation to the significance of economic problems, candidates may discuss the impact of unemployment, inflation, economic restructuring, problems in the countryside and the north-south divide and link these to the growth of socialism and the biennio rosso and the failure of the liberal governments of Nitti and Giolitti to deal with the problems effectively. They may also link the economic problems more directly to the rise of the fascist party and the direct action it was willing to take against strikers and communists (albeit after the main crisis had passed). Candidates may also discuss the other weaknesses of the liberal governments (for example, the failure to gain a creditable peace settlement, the failure of trasformismo), the fears aroused by the ‘red menace’, the legacy of nationalism, the ability and opportunism of Mussolini and the fascists, the attitude of the King and the establishment and the fateful decisions of 1922.

18 How successful was Mussolini’s foreign policy from 1922 to 1940? [50]

No specific answer is looked for. Success may be assessed by reference to aims, outcomes and context. Candidates may argue that Mussolini’s foreign policy had no clear aims or direction until the mid 1930s beyond some grand aim of restoring Italian prestige. Discussion in relation to the 1920s may refer to the Corfu Incident, the acquisition of Fiume and the Locarno Treaties as evidence of some limited success. Candidates may argue that in the 1930s Mussolini’s foreign policy became more assertive and defined, looking for concessions from Britain and France, supporting Austrian independence, and a drive for Empire (Abyssinia). The Abyssinian crisis may be viewed as a turning point – ‘success’ was qualified by the costs and the loss of British and French friendship. After 1935, increased cooperation (e.g. over involvement in the Spanish Civil War) and alliance with Germany undermined Italy’s international prestige and led Italy into a war for which it was unprepared. Candidates may argue therefore that whilst there were victories and successes, these came at great cost and in the end served to undermine Mussolini’s reputation.

45

Page 149: History A - The Bicester Schoolthebicesterschool.org.uk/wp-content/uploads/2015/09/Mark... · 2015-09-28 · History A Mark Schemes for the Units January 2009 H106/H506/MS/R/09J

F962 Mark Scheme June 2009

The Rise of China 1911-1990 19 Assess the reasons why it took the Nationalists (Guomindang) so long to establish

their authority over China after the 1911 revolution. [50]

No specific answer is being looked for but candidates will need to discuss and evaluate a range of reasons to score well. In assessing reasons, candidates may discuss some of the following: the state of China in 1911; ‘sudden’ nature of the revolution in 1911 and resulting power vacuum; ambitions of Yuan Shikai; the limited authority of government and local power/rivalries of warlords (the significance of the warlords may be stressed); the extent and nature of support for Sun Zhongshan (Sun Yat-sen) and the Nationalists (party formed only in 1912); the significance of the 4 May Movement; Sun Zhongshan (Sun Yat-sen) and the reorganization of the Guomindang; Foundation of CCP and links with Guomindang; the significance of the Northern Expedition and ‘reunification’ of China. Candidates may argue that not until the 1920s were the nationalists in a position to establish their authority and this depended in the end on military power and cooperation from sympathetic warlords and the communists.

20 How successful was the Great Leap Forward? [50]

No specific answer is looked for. Success may be assessed by reference to aims, outcomes and context. Candidates may consider economic, social and propaganda aims and may point to some impressive results and the lasting legacy of self-reliance, communes etc. However, they may also argue that the Great Leap Forward was a failure because its approach to economic policy was misconceived and targets were not rooted in sound economic analysis. Candidate’s may argue that Mao’s belief in mass effort by the peasantry to revolutionise China’s industrial and agricultural production was hopelessly unrealistic, emphasizing, for example, the weaknesses of ‘backyard furnaces’ (the steel produced was practically useless and large areas were deforested to feed the furnaces), the limited success of the State Owned Enterprises, the weaknesses of Lysenkoism, ‘sparrowcide’, the collapse in food production and the ‘three hard years’ (50 million dead), economic crisis and the changes made in early 60s. On the other hand ‘the blue ants’ did achieve some impressive feats in creating canals, bridges, dams etc. through manual labour.

21 To what extent did Mao achieve his aims in the Cultural Revolution? [50]

No specific answer is looked for. Candidates need to explain Mao’s aims in the Cultural Revolution and then assess its consequences in terms of those aims. Candidates may suggest that Mao’s aims were to reassert his authority over the Communist Party and China and reverse the trend to the ‘right’ and the ‘capitalist road’ that had occurred after the Great Leap Forward. This may be coupled with a second linked aim to ‘revolutionise’ population, especially youth to ensure a ‘Socialist road’ and the peasant character of China’s communist revolution. In assessing success candidates may discuss extent of Mao’s authority, the role of Jiang Qing, the Gang of Four and the Central Cultural Revolution Group, the significance of the Mao personality cult (swimming in the Yangtse), Red Guards and the Little Red Book, attack on the ‘four olds’, removal of rightists (such as Deng Xiaoping and Liu Shaoqi), three in one committees, changes in education, medicine, agriculture, industry, culture, ‘down to the countryside’, self-criticism and struggle sessions, the ‘cleansing the class ranks’ campaign. Candidates may also discuss the fate of Mao’s erstwhile ally Lin Biao, growing criticism of the Cultural Revolution in the 1970s and the return of Deng Xiaoping. Candidates may argue that whilst the Cultural Revolution reaffirmed Mao’s dominance, in the longer term it did not ensure his vision of Chinese Communism as after his death Deng became dominant and the Gang of Four fell.

46

Page 150: History A - The Bicester Schoolthebicesterschool.org.uk/wp-content/uploads/2015/09/Mark... · 2015-09-28 · History A Mark Schemes for the Units January 2009 H106/H506/MS/R/09J

F962 Mark Scheme June 2009

Democracy and Dictatorship in Germany 1919-1963 22 To what extent do the weaknesses of Weimar democracy explain Hitler’s rise to

power in 1933? [50]

No specific answer is looked for. Candidates must deal with the given factor adequately even if they wish to argue that other factors were more important. In relation to the weaknesses of Weimar democracy, candidates may discuss some of the following: features of the Weimar Constitution such as proportional representation and article 48 and the difficulties arising from coalition governments. This discussion needs to be related directly to events in the 1920s and especially 1929-33 to link to Hitler’s rise to power. Such discussion may be balanced against discussion of other relevant factors such as the context of the economic depression, fear of communism, Hitler, the organization and propaganda of the Nazi Party, the ‘backstairs intrigue’ and so on.

23 How successful were Hitler’s economic policies to 1945? [50]

No specific answer is looked for. Success may be assessed by reference to aims, outcomes and context. Candidates may suggest that Hitler’s main aim was to create an economy which could support sustained rearmament and war, but he needed also to deliver on promises to reduce unemployment. Candidates may argue that Hitler benefited from the work schemes introduced by Papen and Schliecher in 1932 and that he simply extended them and refer to the Arbeitdienst and government loans to private companies. They may also discuss the degree of success of Schacht’s New Plan and his system of Mefo Bills. There should also be discussion of the Four Year Plan and then the war economy and the move to total war. There may be reference to the ‘Guns v butter’ debate and the evidence of looming economic crisis in the late 30s. Candidates may argue that there was success (e.g. reduction in unemployment and the impressive figures for war production) but also limitations (such as those related to autarky – by 1938 the trade deficit was 432 million RM).

24 Assess the reasons for West Germany’s ‘economic miracle’ in the 1950s. [50]

No specific answer is looked for. Candidates will need to explain a number of reasons and assess their relative significance and linkages to score well. Candidates may discuss the significance of some of the following in developing their assessment: the rebuilding of Germany’s industrial base after the destruction of WW2, the importance of Marshall Aid, relative political stability following the creation of the Federal Republic of Germany in 1949, the economic policies of Erhard, the recovery in world trade in the 1950s, good labour relations based on worker participation (laws of 1951 and 52), the ECSC and Germany’s joining of the EEC, the consolidation and mechanization of agriculture, low inflation, the significance of immigration (from East Germany, Yugoslavia and Turkey) providing a pool of cheap labour and so on.

47

Page 151: History A - The Bicester Schoolthebicesterschool.org.uk/wp-content/uploads/2015/09/Mark... · 2015-09-28 · History A Mark Schemes for the Units January 2009 H106/H506/MS/R/09J

F962 Mark Scheme June 2009

The Cold War in Europe from 1945 to the 1990s 25 Assess the reasons for the development of the Cold War in Europe from 1945 to

1948. [50]

No specific answer is looked for. Candidates will need to explain a number of reasons and assess their relative significance and linkages to score well. In assessing reasons candidates may discuss some of the following: the significance of long term tensions and differences over ideology; the tensions in the wartime alliance over the defeat of Nazi Germany; the role of individual leaders (Churchill, Stalin, Roosevelt, Truman and Attlee); the military situation at the end of the war; differences and tensions at Yalta and Potsdam; the atom bomb; Soviet and Western fears and suspicions, strategies and policies.

26 Assess the reasons why Berlin was a major source of tension in the Cold War from

1948 to 1961. [50]

No specific answer is looked for. Candidates will need to explain a number of reasons and assess their relative significance and linkages to score well. Candidates may focus on the two crises surrounding the Berlin Blockade and the building of the Berlin Wall, but there needs to be some appreciation of the whole period to score well. In assessing reasons, candidates may discuss some of the following: the significance of the decision to dived Berlin into four zones; the strategic position of Berlin in the Soviet zone; the reasons for and consequences of the Berlin Blockade in 1948; the significance of West Berlin as an island of capitalism/western shop window and as the front-line in the Cold War; the stream of refugees to the west; the decision to build the Berlin Wall and its impact in the context of the failure of the Paris summit and the U2 spy plane incident.

27 To what extent was Gorbachev responsible for the collapse of Soviet power in

Eastern Europe? [50]

No specific answer is looked for. Candidates will need to discuss the degree of Gorbachev’s responsibility and balance this against other factors to score well. In relation to Gorbachev, candidates may discuss the repudiation of the Brezhnev Doctrine and policies of perestroika and glasnost, the beginnings of the break-up of the Soviet Union and their implications for/impact on Eastern Europe, pointing to, perhaps, the legalization of Solidarity in Poland along with democratic elections there and in Hungary, followed by the collapse of communist regimes elsewhere. Such discussion needs to be balanced with consideration of other factors such as the longer term problems in the Soviet Union and Eastern Europe that perhaps caused Gorbachev’s changes in policy and stimulated popular protest – such as the growth of economic problems and the contrast with the wealth and freedom of the West, the growth of dissidence in Eastern Europe and the impact of the Soviet Union’s inability to compete militarily with the USA.

48

Page 152: History A - The Bicester Schoolthebicesterschool.org.uk/wp-content/uploads/2015/09/Mark... · 2015-09-28 · History A Mark Schemes for the Units January 2009 H106/H506/MS/R/09J

F962 Mark Scheme June 2009

Crisis in the Middle East 1948-2003 28 Assess the consequences of the Suez Crisis (1956) for Egypt and the Middle East. [50]

No specific answer is looked for. Candidates will need to explain a number of consequences and assess their relative significance and linkages to score well. Candidates may distinguish between immediate and longer term consequences and/or direct and indirect consequences. They may discuss some of the following with regard to the degree to which: Suez secured Nasser’s control of Suez and his popularity in Egypt; Suez established Nasser as the leader of the Arab world and facilitated his ambitions to create a single Arab state (e.g. UAR formed with Syria in 1958) but Arab world split as many leaders against Nasserism; Suez gave encouragement to Arab nationalism (e.g. Nasserites were successful in seizing powering Iraq); Suez marked the effective end of British and French influence in the Middle East; Suez increased influence of Soviet Union in Middle East (in Egypt and Syria); Israel could claim a victory – UNEF patrolled Sinai and Eilat able to develop and Israel was confirmed in its policy of aggressive defence; US aid continued to flow; Suez did little to end the Arab-Israeli dispute.

29 Assess the reasons why the Six Day War (1967) was followed by another conflict

(the Yom Kippur War) just six years later. [50]

No specific answer is looked for. Candidates will need to explain a number of reasons and assess their relative significance and linkages to score well. In assessing reasons, candidates may discuss some of the following: Israel’s acquisition of territory was both a deep humiliation for Arab World and solved nothing; Arab states were able to recover from military destruction (Soviet aid); the absence of a peace treaty – the war only gave Israel more defensible frontiers; Israel’s annexation of East Jerusalem caused further resentment; the failure of peace initiatives based on new UN Resolution 242 and the Rogers Plan; the development of a more radical PLO under Arafat – terrorism stiffened Israeli opposition to compromise; Sadat’s aim to recover Sinai and remove Israel from Suez.

30 Assess the view that US action against Iraq in 1991 and 2003 was mainly motivated

by the threat Saddam Hussein posed to Middle East stability. [50]

No specific answer is looked for. Candidates must deal with the given factor adequately even if they wish to argue that other factors were more important. Candidates may treat both actions together or separately (there is no need for strict balance of treatment). Candidates may discuss some of the following in relation to1991: US distancing from Iraq after Iraq-Iran war because of use of WMD against Iranians and Kurds and sympathy with Israeli fears over development of missiles (Scuds) capable of using a nuclear warhead (but strong trade links between West and Iraq); Western press vilification of ‘Butcher of Baghdad’ and the ‘super gun’ also cooled relations; Saddam’s rhetoric spoke of an anti-imperialist and anti-Zionist campaign; Saddam’s attempts to bully Kuwait into giving aid (Iraq had huge debts) unsettled Arab world; Invasion took West and Arab world by surprise, but was a naked act of aggression; Thatcher was key in persuading Bush Senior to act under auspices of UN with many Arab states in support; and, of course, western concerns over oil. In relation to 2003, candidates may discuss: the fact that 1991 cleared Kuwait but did not topple Saddam; US close involvement in Iraq after the failed uprisings by Kurds and Shias – no fly-zones etc – was a constant source of tension; security of oil; fear of WMD’s and missile technology that could be used against Israel; Iraq’s alleged links to militant Islam and Al Qaeda and the mentality of the ‘war on terror’ after 2001 (invasion of Afghanistan, identification of Iran, Iraq, Syria (in Middle East) as sponsors of terror and militant Palestinian organizations); Iraq’s ‘refusal’ to cooperate with UN weapons inspections and US/British bombing raids; aim of regime change as way of achieving Middle East and world stability. Paper Total [100]

49

Page 153: History A - The Bicester Schoolthebicesterschool.org.uk/wp-content/uploads/2015/09/Mark... · 2015-09-28 · History A Mark Schemes for the Units January 2009 H106/H506/MS/R/09J

F963 Mark Scheme June 2009

50

F963/01 British History Enquiries Option A: British History Enquiries 1066-1660

1 The Normans in England 1066-1100

(a) Study Sources A and B Compare these Sources as evidence for the Papal relations with the Church in England. [30]

No set answer is expected, but candidates need to compare the contents, evaluating such matters as authorship, dating, utility and reliability, so using the sources ‘as evidence for…’. The headings and attributions should aid evaluation and reference to both is expected in a good answer. The differences in authorship and dates are very important here: Pope Alexander II, a known supporter of Kings, not least William I, in 1071 five years after Hastings but just after the Conquest period; Pope Gregory VII, known for his confrontational style (the Investiture Contest), in 1079. Sources A and B reflect these features. In Source A the Pope supports the new archbishop and invest him with many powers, not least authority to determine cases without reference to Rome, while in B the language suggests problems and demands ultimate obedience to the Pope over the King of England. Both have bearing on the relationships between the Papacy and the Archbishopric of Canterbury (effectively A is full of rapport, B full of tension) and between the Papacy and the King of England (A implies amity, B sets out problems). In a short space of time, much had changed, at least in terms of attitudes and requirements. Nonetheless even Alexander in A is careful to set limits – that decisions must be in accordance with Church law; they must be just. Contextual knowledge would point, by 1079, to changes both in terms of royal power over the English Church and to the question of Lanfranc’s complicity in this. In terms of judgement both are valid, particularly as evidence of a changing relationship.

(b) Study all the Sources

Use your own knowledge to assess how far the Sources support the interpretation that Lanfranc’s “harmonious relationship with William I” helped rather than obstructed reform of the Church in England. [70]

Successful answers will need to make use of all five Sources, testing them against contextual knowledge and evaluating their strengths and weaknesses, any limitations as evidence. A range of issues may be addressed in focusing upon the terms of the question but no set conclusion is expected. The argument here is whether Lanfranc’s relationship with William assisted or obstructed Church reform. It assumes, as the basis of the historian Bates’ assertion in source E, that there was a harmonious relationship, which B (Pope Gregory) contests, suggesting that Lanfranc may fear William, although the evidence points to complicity with William, a factor Pope Gregory tends to prefer (‘his own fault’) in his balanced judgement as to what is going on. All the sources except A and B give mixed messages but the argument that their relationship helped reform is to be found in A, C, D and E. Source A, Pope Alexander, sets out a framework of considerable authority to Lanfranc within the English Church, and own knowledge could point out that William had had papal support for his conquest partly on the grounds that he would

Page 154: History A - The Bicester Schoolthebicesterschool.org.uk/wp-content/uploads/2015/09/Mark... · 2015-09-28 · History A Mark Schemes for the Units January 2009 H106/H506/MS/R/09J

F963 Mark Scheme June 2009

support a continental reform agenda. Eadmer in source C, who is critical of William, does grudgingly imply that if William was happy with reform and it furthered his political authority, then his support for Lanfranc helped. This is supported when Orderic Vitalis in source D comments on the areas both agreed upon, the competency of bishops, simony and monastic discipline. Source D is effective evidence, coming from a Norman monk with English origins. Although written at least 50 years later when such ‘Norman reform’ attitudes had become commonplace it does comment on William’s commitment to reform and of his efforts to find churchmen ‘distinguished in life and doctrine.’ The comments on monasticism are supported by Bates in source E. However source D is effectively a Norman monk and is likely to approve of such an agenda. His is a very rosy view of William and it is interesting and instructive that his focus is here. There is no mention of Lanfranc. The evidence that most effectively argues that a harmonious relationship is good for church reform is source E. Bates comments that Lanfranc could be effective because of his close relationship with the King, effectively Normanising the Church, centralising authority at Canterbury (over York), something which chimes with William’s own destruction and re-ordering of the North in 1070, and providing synods with real power to drive an authoritative canon law and a continental monasticism. Own knowledge would suggest that European practice was imported retaining only some lower level English religious practice and personnel. New sees and sites were created and new structures put in place, diocesan and diaconal. The alternative view, that reform was obstructed by the close relationship of William and Lanfranc, can be found in source B and by a different reading of sources C, D and E. Source B, Pope Gregory, is unambiguous in stating that William and Lanfranc hindered the relationship with the papacy and possibly Lanfranc may be in private conflict with the King (‘fear’). This did not bode well for the Church but candidates might consider that this gave only one side - that of a centralising Pope, who like William, insisted on the central control of office (the investiture contest) - something which necessarily prevented internal reform. One could argue it was a clash of personality and authority rather than a reform aiming to affect the day-to-day running of the Church. Eadmer in source C is also negative about Lafranc’s relationship, arguing that William insisted on reform loopholes over issues of justice and canon law when it affected his barons and officers, even when blatantly guilty. This can be supported from own knowledge and the tone of Eadmer is revealing. As a Canterbury monk writing later and who might be expected to approve of the primacy of Canterbury, this is telling evidence of the limitations to reform, albeit from a central, Canterbury, perspective. More effective answers might point out that all except E are religious sources, monastic and papal, yet are divided on the issue. Orderic Vitalis in source D, although generally approving of the reforms, can also be used to suggest that William, beneath the surface, was merely using Lanfranc. Like the pope in B, who mentions the possibility of ‘fear’ as the basis of the relationship, Orderic Vitalis informs us that William’s first interest was in recording “church property” when a vacancy occurred. One might ask, as Pope Gregory did elsewhere, whether it was the King’s function to appoint. It is also interesting that the criterion Orderic Vitalis mentions for religious office was ‘a man most capable of governing the bishopric or abbey’: i.e. William’s priority was government and authority rather than reform as such. Candidates could also point to Bates in source E whose comment on the ‘close – harmonious relationship’ of Lanfranc and William is to do with assisting the Norman settlement of England rather than reform per se. If one views Norman reform as good for the church then one will argue their relationship was positive. If there was another agenda, with Lanfranc as a political regent with blatant legal loopholes and political control as Pope Gregory alleged, and the Anglo-Saxon church lost its spiritual features, then one would view it as negative.

51

Page 155: History A - The Bicester Schoolthebicesterschool.org.uk/wp-content/uploads/2015/09/Mark... · 2015-09-28 · History A Mark Schemes for the Units January 2009 H106/H506/MS/R/09J

F963 Mark Scheme June 2009

Mid-Tudor Crises 1536-1569 Attitudes to Religious Change 1552-1559 2(a) Study Sources C and D

Compare these Sources as evidence for the attitudes of people in London concerning religious change. [30]

Focus: Comparison of two Sources. No set answer is expected, but candidates need to compare the contents, evaluating such matters as authorship, dating, utility and reliability, so using the Sources ‘as evidence for …’. The headings and attributions should aid evaluation and reference to both is expected in a good answer. The Sources indicate that attitudes to religious change in London remained similar in some ways, but that there were also changes due to the altered context. Source C reports the reaction of the Protestant London audience to the condemnation and public burnings of martyrs in the last months of Mary I’s reign. They seem calm, sympathetic and comforting, yet brave in their defiance of the authorities. In contrast, Source D, written at the end of the same year, under the new Queen Elizabeth, suggests that lower class Protestants are unruly, irreverent and violent, roaming the city in mobs, attacking Catholic congregations and inciting riots. This might be explained by the lifting of Catholic repression with the accession of Elizabeth I. Source C presents a positive view of Protestant attitudes in London because it is written by the Protestant minister of an underground congregation there to a contact in Switzerland, so is unlikely to express a negative view of Protestant attitudes. He does, however, suggest that London Protestants hate and resist the Catholic authorities. On the other hand, Source D is written by one Catholic to another, at the court of Philip II, at that time recently widowed after Mary’s death, and champion of the Catholic Reformation in Europe, so the view of Protestant attitudes is likely to be negative. The English were xenophobic and Philip II was unpopular in England. The implication is that repressed hatred of Mary and her Catholic advisers has led to a boiling over of feelings on Elizabeth’s accession. Catholic attitudes expressed in Source C are seen as cruel and tyrannical, shown by the proclamation about free speech and the secretive, underhand way in which the Bishop of London deals with the later wave of heretics. The Bishop mentioned in Source D, however, is open in his views, thinking Elizabeth may retain Catholicism, and given sympathy by its Catholic author. 2(b) Study all the Sources.

Use your own knowledge to assess how far the Sources support the interpretation that the rulers of England followed similar religious policies between 1552 and 1559. [70]

Focus: Judgement in context, based on the set of Sources and own knowledge. Religious policies may cover a range of approaches from doctrine to the questions of approach, enforcement, observance and encouragement. The emphases of candidates may vary and examiners will need to be flexible. Successful answers will need to make use of all five Sources, testing them against contextual knowledge and evaluating their strengths and weaknesses, any limitations as evidence. A range of issues may be addressed in focusing upon the terms of the question but no set conclusion is expected.

52

Page 156: History A - The Bicester Schoolthebicesterschool.org.uk/wp-content/uploads/2015/09/Mark... · 2015-09-28 · History A Mark Schemes for the Units January 2009 H106/H506/MS/R/09J

F963 Mark Scheme June 2009

At first glance it seems obvious that Edward and Elizabeth followed a Protestant religious policy, whereas Mary's was Catholic, and there is plenty of evidence in the Sources to aid evaluation of this view. However, their religious policies have similarities - for example, establishing obedience to the official, uniform religion, healing religious divisions and controlling religious opposition. The personal part played by the monarch in religious policy is consistently evident. There are examples of a policy of leniency and persuasion; of education, regulation and inspection; and of controlling opposition by compulsion, punishment and persecution. Sources A, B and E show monarchs proposing a policy of doctrinal uniformity, whether Protestant or Catholic. Uniformity and obedience may be cross-referenced in all three Sources. The provenance of B and E, female monarchs, differs from Source A as Northumberland rules making the minor Edward's policies merely suggestions. An evaluation of the extent to which Edward influenced policy may be supplied by own knowledge. Source E mentions both similarity and difference between Edward's and Elizabeth's Prayer Books, which may be developed using own knowledge. Sources A and B suggest a policy of persuasion, followed by a repressive policy towards non-conformity, whereas Source D suggests there is some confusion about Elizabeth’s early attitudes about a uniform doctrine, but with some leniency in Source E. Own knowledge might be used to compare the broad 1559 Settlement with the narrowness of Mary's and Edward’s later religious policy. The encouragement of preaching and education are royal policies mentioned in Sources A, B, D and E. There may be a discussion of the extent to which policy prioritised Scriptures, preachers and universities in Protestant reigns compared to the Catholic reign of Mary. The use of bishops and injunctions to enforce policy might be mentioned as a similarity in Sources C and D, despite shifts in denomination, though their means of enforcement differed. The provenance of C and D reflect the subjectivity of a Protestant in hiding during a Catholic reign and a Catholic observer at the start of a Protestant reign. The audience of these Sources may also affect their reliability. A policy based on religious peace is found throughout the Sources. The banning of unorthodox preaching in Source D links with the licences mentioned in Source B. The repressive proclamation in Source C might be linked with the punishments for non-conformity given in Source E, and compared for their severity. Whereas Edward’s suggestions for religious policy in Source A are positive, evaluation using own knowledge might mention the speed of Protestant reform under Northumberland and attempts to force Mary to give up the Mass. Mary’s apparently lenient policy at the start of her reign seems similar to Elizabeth's in D and E. Own knowledge of the return to Rome might be used here. More negative policies are threatened in Source B and adopted in C, the burnings marking a change of royal policy to repression. Own knowledge is likely to focus on the changes in the official form of religion. Candidates are likely to consider a range of policies: enforcement of uniformity, religious peace and control of the religious opposition. Policy had some consistency: leniency and persuasion; education, regulation and inspection, but also inconsistency in the balance of compulsion, punishment and persecution. They are likely to set their argument within a context of changes to the official religion. It is up to candidates to assess the similarity of royal policies, there being no set conclusion.

53

Page 157: History A - The Bicester Schoolthebicesterschool.org.uk/wp-content/uploads/2015/09/Mark... · 2015-09-28 · History A Mark Schemes for the Units January 2009 H106/H506/MS/R/09J

F963 Mark Scheme June 2009

3 The English Civil War and Interregnum 1637-60

(a) Study Sources D and E Compare these sources as evidence for criticism of Cromwell’s rule as Lord Protector. [30]

No set answer is expected, but candidates need to compare the contents, evaluating such matters as authorship, dating, utility and reliability, so using the sources ‘as evidence for…’. The headings and attributions should aid evaluation and reference to both is expected in a good answer.

Both sources are retrospective accounts, written after the death of Cromwell and the Restoration of the monarchy, so the authors can speak their minds. Both are hostile to Cromwell and condemn Cromwell’s reliance on deception, and the illegality of his rule (source D talks of the Major-Generals ‘ruling according to their wills and not according to law’ while source E sees the protectorate as ‘full of oppression and injustice’). Both sources see his rule as oppressive and his religious commitment as skin deep. But the sources also emphasize different facets of Cromwell’s arbitrary rule: D sees the army as Cromwell’s partner, though evidently subordinate to him since he purges them; source E addresses his over-arching aim (self-interest), his ingratitude (his treatment of Vane), and his opportunism (abandoning the Major-Generals).

(b) Study all the Sources Use your own knowledge to assess how far the sources support the interpretation that Cromwell’s main aim as Lord Protector was to build a godly society. [70]

Focus: Judgment in context, based on the set of sources and own knowledge Successful answers will need to make use of all five sources, testing them against contextual knowledge and evaluating their strengths and weaknesses, and any limitations as evidence. A range of issues may be addressed in focussing upon the terms of the question but no set conclusion is expected.

Source A certainly supports the proposition, and candidates may note that this was an early move (1654) in Cromwell’s time as Lord Protector, suggesting that reform of the ministry was a priority, and the theme of religious reform reappears in source C, in 1656. Baxter however, at a distance that saw Puritan hopes diminish, may well look back too approvingly. Part of the Major-Generals’ brief was security, but also ’godly reformation’ and Cromwell takes pride in their (to him) considerable achievements in advancing this agenda. Source B suggests another priority – that of stability and conciliation after years of civil war, and perhaps also division amongst supporters of the regicide, demonstrated spectacularly in April 1653 when, goaded by the army, Cromwell had dissolved the Rump with military force. Yet both B and C are Cromwell justifying himself to parliament. Sources D and E offer a very different reading as to be expected from critics: of Cromwell cynically using religion to advance himself and allowing ‘true religion’ to atrophy (source D). Instead, as source E states and source D implies, Cromwell was really interested in the accumulation and then exercise of power; and the implication here is that Cromwell’s professed commitment to ‘godly reformation’ was simply a smokescreen to gather support which would then underpin his power. So there is much to debate here, and candidates may suggest that ‘healing and settling’ was a priority in 1654 but was thrown off course by Royalist uprisings such as Penruddock’s (source C) while a more reformed society was never far from Cromwell’s mind. But as sources D and E indicate, his motivation has been debated from that day to this.

54

Page 158: History A - The Bicester Schoolthebicesterschool.org.uk/wp-content/uploads/2015/09/Mark... · 2015-09-28 · History A Mark Schemes for the Units January 2009 H106/H506/MS/R/09J

F963 Mark Scheme June 2009

55

F963/02 British History Enquiries Option B: British History Enquiries 1815-1945

1 The Condition of England 1815-1853

(a) Study Sources C and D Compare these sources as evidence for attitudes towards the events at St Peter’s Fields (Peterloo) in August 1819 [30]

Focus: Comparison of two Sources No set answer is expected, but candidates need to compare the contents, evaluating such matters as authorship, dating, utility and reliability, so using the Source ‘as evidence for…’ The headings and attributions should aid evaluation and reference to both is expected in a good answer. Both stress the importance of the Manchester Yeomanry and both agree that casualties were caused by the panic in the crowd, people being trapped and trampled in the confusion (‘they cannot get away in ‘C’ and ‘panic’ and some were killed ‘by the crush to get out of the way’ in D). At that point the similarities end and the attitudes shown are in marked contrast to each other. They differ over who is responsible for Peterloo. ‘C’ stresses that the cavalry were to blame; pointing out that they were first welcomed but that their indiscipline led to ‘confusion’ and a deliberate hacking through defenceless people, targeting women and children. In contrast ‘D’ blames the radicals and those of that persuasion in the crowd who wanted to be martyrs. ‘D’ argues they instigated it by holding an illegal meeting with ‘revolutionary’ banners (‘starts’ hostilities). The troops merely responded, although D is not entirely consistent. His comment that the ‘attack’ gave added impulse to the troops could be read to mean that they started it, especially as they had already penetrated to the centre wagons to arrest Hunt. On provenance neither source is especially reliable given their respective slants. Although ‘C’ is from an eyewitness he could only see a small part of Peterloo (‘stood on tiptoe’), although he acknowledges this (‘as I understood it’). As a Radical, writing some time after an event that became enshrined in radical mythology, his tone is very slanted. Troops were ‘welcomed’ but then they ‘hewed’ through the ‘naked hands and defenceless heads’ of the people. ‘D’ was not there but was from a member of the ruling aristocracy, concerned to stress sedition and to minimise casualties. The blame is thrown on to the Radicals who are seen as cowardly (Martyrs tried to make good their escape). It too was written long after the event, although based on ‘family documents’ (how reliable / useful?), although we are told the family is prominent). Both are useful pieces of evidence on the contrasting political attitudes of the time.

(b) Use your own knowledge to assess how far the sources support the

interpretation that the Radicals of 1815-20 failed because of government repression. [70]

Focus: Judgement in context, based on the set of Sources and own knowledge. Successful answers will need to make use of all five Sources, testing them against contextual knowledge and evaluating their strengths and weaknesses, including any limitations as evidence. A range of issues may be addressed in focusing upon the terms of the question but no set answer is expected. Candidates will need to focus on the relative importance of the role of government repression, placing it in relation to other possible explanations such as economic recovery and the nature of that economy (declining artisans), a divided leadership, division between radical and moderates over methods, a failure of coordination and the lack of any unifying

Page 159: History A - The Bicester Schoolthebicesterschool.org.uk/wp-content/uploads/2015/09/Mark... · 2015-09-28 · History A Mark Schemes for the Units January 2009 H106/H506/MS/R/09J

F963 Mark Scheme June 2009

force beyond Parliamentary Reform. The sources provide evidence for a variety of views, although only one (Buckingham in D) is from a government perspective. The others are from a variety of Radicals, mainly moderates. All the sources stress government repression, to be expected given that it was one of the radicals’ rallying cries and so it figures heavily in their description and analyses of some of the major events of the period. Those who would agree with the view of repression could cite the use of spies referred to in both A and B. In the former Bamford warns the Blanketeers that they could be infiltrated whilst Shelley in ‘B’, keen to exonerate some of the Pentrich leaders from responsibility, argues that Oliver acted as ‘agent provocateur’. Own knowledge would suggest their comments to be reliable, despite the sympathetic ‘tone’ of Shelley in ‘B’. Troops are conspicuous in the sources. B comments on their use at the Pentrich trial (at Brandreth’s execution ‘cavalry hemmed in the multitudes’) and C and D refer to Peterloo where their actions became notorious. Government also liked to use the law to restrict radical activity. Buckingham in D mentions the Manchester authorities banning the Peterloo meeting in 1819, hence the arrest of Hunt, whilst Cruickshank in E graphically demonstrates the impact of the Six Acts which can be amplified by own knowledge (the speeding up of trials, greater press control through stamps, the banning of drilling and training, the limitation of meetings over 50 unless JPs had given their permission and the power to search for arms). Cruickshank’s view is that the government succeeded but is over-reacting and far too repressive, although own knowledge might comment on their temporary nature, how few Radicals were held and how they were designed more to reassure an anxious Commons than to repress Radicals. Government also arrested key ring -leaders like Hunt (and Bamford, the author of A and C). Sources D and E refer explicitly to arrests. There is plenty of other evidence of the government resorting to Acts (in 1817 for example) following in Pitt’s steps.

Yet the sources also mention other factors. Source A implies the impracticality of the Blanketeers proposed methods of protest. They failed to listen to his advice and appeared pathetic. As a balanced account, friendly yet critical and able to judge from hindsight, its evidence is sound. However the stress on numbers in B and C could be counter productive, entrenching propertied conviction that the status quo must be preserved at any cost. Peterloo saw the height of all this. Source C also stresses how moderate the aims were in contrast with the assertions of D as to revolutionary intent. E demonstrates how dependent radicals were on the spoken and printed word and how easily it could be cut-off. Nonetheless this aspect of repression could misfire – it was often impractical, radicals found ways around it and cartoonists like Cruickshank undermined government by such barbed and clever satire. Own knowledge can extend the view that other factors were more important. Radicals like Bamford and Thistlewood opposed each other; the recovery of the economy post 1820 took the wind out of Radical sails, whilst methods (Blanketeers, Pentrich and Cato St.) varied from plot to mass meeting. Geography, distance and climate also played a part. With the exception of Cato St. and Spa Fields radical demonstrations took place in the North, not in London, where the government was more vulnerable.

The sources reveal much, although D and E are clear exaggerations (Castlereagh is portrayed as a devil in E and modern historians have questioned the radical image of repression). Bamford in C is more reliable, although at Peterloo his ability to detect what was happening was limited. Some candidates may question failure – the radical numbers were huge at Peterloo, whilst the crowd at the execution in B were not sympathetic to the government. Most candidates are likely to argue either that failure was due to Government repression or to the mishandling of protest by the radicals themselves. The sources present evidence, of mixed value, either way.

56

Page 160: History A - The Bicester Schoolthebicesterschool.org.uk/wp-content/uploads/2015/09/Mark... · 2015-09-28 · History A Mark Schemes for the Units January 2009 H106/H506/MS/R/09J

F963 Mark Scheme June 2009

2 The Age of Gladstone and Disraeli 1865 – 1886

(a) Study Sources B and D Compare these sources as evidence for the ideas behind Disraeli’s social reforms in the 1870s. [30]

Focus: Comparison of two Sources No set answer is expected, but candidates need to compare the contents, evaluating such matters as authorship, dating, utility and reliability, so using the Source ‘as evidence for…’. The headings and attributions should aid evaluation and reference to both is expected in a good answer.

Both Sources agree on the idea that the state needs to interfere when sanitary issues are at stake. For both this is the issue where there is a justification for breaking with laisser-faire and allowing a measure of state interference. They differ in the relative breadth (B) and the narrowness (D) of their ideas. Disraeli in B takes a very broad ideological view of Social Reform, to include pollution (air), clean water, inspection and food standards, whereas Cross in D has a much more precise and limited view, confined by the principles of laisser-faire. He is simply concerned to raise the existing standards of the housing stock to a reasonable condition. Despite Disraeli’s claim in ‘B’ to be practical in approach, Cross in D is closer to it. Disraeli has a wider mindset – that ideas on health should be a government’s main priority, that the health of the people should be considered as a whole. He makes the historic connection between health and greatness which Cross in D does not. Cross is more concerned to set limits to government action; neither it nor local government should provide cheap housing. Private and individual initiative should not be discouraged.

In part such differences are explained by the provenance. Disraeli in ‘B’ is in opposition and under pressure to rally the Conservative party. The Manchester speech is a piece of public rhetoric to a large audience and is widely publicised. As such its ideas are sweeping and, although it is unusual to discuss health, this is post the Second Reform Act where the artisan vote was important. Candidates might comment that this was only a very small part of a very long speech that stressed other issues. In contrast Cross in D is addressing, as the Home Secretary, a very different audience, not the electorate but MPs. He is likely to emphasise the cautious and liberal ideas underpinning his proposals, those of individualism and a limited role for the state. His emphasis is on what is not done, stressing just the sanitary aspect which had had much publicity in previous decades (cholera, typhoid) the causes of which were now agreed upon. Candidates might consider D to be the better evidence for reform ideas, given that it was based on a particular Bill and the arguments that would sway MPs.

(b) Study all the Sources

Use your own knowledge to assess how far the sources support the interpretation that Disraeli carefully planned a programme of social reform in the 1870s. [70]

Focus: Judgement in context, based on the set of Sources and own knowledge. Successful answers will need to make use of all five Sources, testing them against contextual knowledge and evaluating their strengths and weaknesses, including any limitations as evidence. A range of issues may be addressed in focusing upon the terms of the question but no set condition is expected.

The sources may be used in a variety of ways to assess the interpretation that Disraeli’s aim was a carefully planned programme of social reform. It is likely that candidates will see Sources A and B, both Disraeli, as being more supportive of this view whilst C, D and E, Cross and the modern historian, as being critical of it, pointing to lack of substance, the

57

Page 161: History A - The Bicester Schoolthebicesterschool.org.uk/wp-content/uploads/2015/09/Mark... · 2015-09-28 · History A Mark Schemes for the Units January 2009 H106/H506/MS/R/09J

F963 Mark Scheme June 2009

role of others and the historical context, which may be developed by own knowledge. Evaluatively they may also consider A and B as being of less credibility given the context of opposition (1871 and 1872) and opportunism (debates on Trade Union legislation and public political speeches) than C, D and E. In C Cross is confiding in his autobiography the reality of Disraeli’s inactivity, whilst in D he is concerned to minimise the extent of what is proposed. Vincent in E may well have based some of his critical findings on Cross’ evidence.

The case for a carefully planned programme is to be found in A and B. In A Disraeli outlines his understanding of the Trade Unions as working class bodies and is clearly convinced of their wealth and potential power. Own knowledge could be used to demonstrate a sustained interest in this in his own later legislation, giving the Trade Unions an undefined and largely unlimited right of picketing with legal equality between employer and employee (a careful plan to woo working men’s votes?). In Disraeli’s Manchester Speech (B) Disraeli appears to have a plan and candidates could clearly support this with reference to the legislation of 1874-6. It would appear to be a ‘practical’ and carefully planned programme with the focus clearly on sanitary reform, as the evidence of Cross in Source D would corroborate. Cross is clear in C that Disraeli has talked much about such a programme in ‘all his speeches’. Certainly his ‘colleagues’, especially Cross, did have some modest proposals based, as D acknowledges, on firmly liberal and laisser-faire principles. Vincent in E makes reference to the ‘miraculous year’ of 1875 and candidates can point to Disraeli presiding over Public Health, Dwellings Acts, and Food and Drugs (and possibly speculate on the role of Disraeli’s literary social vision the Young England novels). Some candidates might comment on a possible political motive with a plan to develop working class conservatism, securing their votes via social reform (Tory democracy). There is a hint of this in ‘B’, the Manchester Speech. Vincent’s account, which questions Disraeli’s commitment, is rather unbalanced in its comments and this could be used to question his comments. The case for a more limited and unplanned programme, in which Disraeli played a more casual role, is largely in C, D and E, with candidates challenging the value of A and B. Clearly in ‘A’ Disraeli is not well informed as to working class problems, confusing the working class with skilled Trade Unions and assuming both to be ‘wealthy’. No careful programme is suggested and candidates could argue that his later Union legislation was more a reaction to Gladstone’s mistakes than a conscious part of a careful working class programme. Similarly in B, there are some very general and ill-thought through claims as to the health of the people, as is to be expected in a politician’s speech. It was presumably this sort of speech that Cross had in mind when he comments in C that there was no programme of large legislative schemes, points developed by Vincent in E when he comments that Disraeli had to rely on the suggestions of others. Cross is a largely reliable source, although one might expect a tendency to focus on his own contribution, ignoring Disraeli’s ‘chairing’ role in Cabinet and his support in Parliament. His speech in D makes no mention of Disraeli. Vincent questions both Disraeli’s personal interest, in comparison with his colleagues, and the practical effect of what was achieved, given the lack of money and the permissive nature of the legislation. There was no hint of a welfare state and indeed Conservatives were content with liberal structures, eschewing reform programmes. Own knowledge might point to Disraeli’s electoral promise in 1874 to leave well alone (‘to cease from harassing ‘liberal’ legislation). Vincent also has the advantage of being able to put Disraeli’s social reform into long term perspective, which sees it as a continuation of Liberal ideas. Other points to support a limited view of what was done are that Disraeli was later distracted by foreign policy and illness. His interest in the working class lacked real knowledge or understanding. Candidates are likely to conclude that the sources point to different interpretations of whether the social legislation of 1875 was carefully planned but can equally point more to one than the other.

58

Page 162: History A - The Bicester Schoolthebicesterschool.org.uk/wp-content/uploads/2015/09/Mark... · 2015-09-28 · History A Mark Schemes for the Units January 2009 H106/H506/MS/R/09J

F963 Mark Scheme June 2009

3 England and a New Century

(a) Study Sources B and C Compare these two Sources as evidence for attitudes towards the Irish Home Rule Bill. [30]

Focus: Comparison of two Sources. Content. Source B (Carson) is strongly opposed to the Third Home Rule Bill (introduced by Asquith in April 1912), seeing it as a “conspiracy” against Ulster. However, Source C (Redmond) supports the Bill, and is opposed to any attempt to weaken it, particularly through any plans to exclude Ulster. Carson presents Ulster as a special case. Richer and more industrialised than the south, but doomed to be permanently out voted in a United Ireland if the Bill is passed. He stresses the loyalty of Ulster to the Crown, and the need to oppose the Bill in the interests of Ireland, Britain and the Empire. In Redmond’s speech, the emphasis is upon moving forward towards greater Irish self-government, and of avoiding Irish division. Ireland should be one nation: Catholics and Protestants together. ‘Partition is unthinkable.’ This represents the patriotic appeal to Irish history as viewed by moderate Irish Nationalism. Unlike Carson his threat is a parliamentary one rather than that of civil disobedience and possibly armed resistance. Provenance. During this period, Carson emerged as the leader of the Ulster Unionists. Later, he was to organise the Ulster Volunteer Force. His Belfast speech (September 1912), launching the Covenant, is an appeal to popular protest. Nearly half a million people signed the Covenant, some in their own blood. Not surprisingly, the tone of the speech is extreme. It is clear that the Ulster Unionists would rather be separated from the rest of Ireland than be subject to the Home Rule Bill. This is a speech that launched a campaign of resistance and is drenched in the rhetoric of an appeal to king and empire. Interestingly it avoids any mention of Protestantism and religion, unlike Redmond in C. Redmond, the leader of the Irish Nationalists, is speaking in the House of Commons attempting to pressurize Asquith in the context of parliament. By January 1913, the suggestion that Ulster might be excluded from the provisions of the Bill has come to dominate political debate. At this stage, Redmond, confident in his political position (offering necessary parliamentary support to Asquith’s Liberal Government) is unwilling to compromise over the Bill. Later this was to change due to various political pressures. Both sources are effective evidence of the stances taken on Home Rule. (b) Study all the Sources

Use your own knowledge to assess how far the Sources support the interpretation that a crisis arose in Irish affairs in the period 1912 to 1914 mainly as a result of Asquith’s mishandling of events. [70]

Successful answers will need to make use of all five Sources, testing them against contextual knowledge, and evaluating their strengths and weaknesses and any limitations as evidence. A range of issues may be addressed in focussing upon the terms of the question, but no set conclusion is expected. The context is the period following the 1910 Elections (which left the Liberal Government dependent on the support of the Irish Nationalists) and the Parliament Act of 1911 (which meant that the Conservatives could no longer use the House of Lords to block Home Rule). Once Redmond had persuaded Asquith to introduce the Third Home Rule Bill (a moderate measure providing for an Irish Parliament in Dublin with limited powers), the years 1912-14 saw a very intense political crisis, centred on what should be done about Ulster and its Protestant majority.

59

Page 163: History A - The Bicester Schoolthebicesterschool.org.uk/wp-content/uploads/2015/09/Mark... · 2015-09-28 · History A Mark Schemes for the Units January 2009 H106/H506/MS/R/09J

F963 Mark Scheme June 2009

The only Source which directly blames Asquith for the crisis is Source E. The modern historian describes Asquith’s ‘Wait and See’ approach, giving examples of what later historians have seen as the Prime Minister’s failings. Obviously, not all historians have taken this line. However, candidates may wish to add further examples of Asquith’s prevarication over events like gun running, and the Curragh Mutiny. The outbreak of the Great War, which saved the Government in some respects, but also destroyed its Home Rule policy, is mentioned in the Source. By then, despite a conference at Buckingham Palace, chances of progress were minimal. Elsewhere, Asquith and his Government are blamed by implication (at least). The dangers in the situation were clear from the outset. The Home Rule Bill is attacked strongly by Bonar Law (Source A) and Carson (Source B). The Irish Nationalists (Source C and Source D) welcome Home Rule as a first step, but soon despair of Asquith’s dithering, which leads to a suggested policy of exclusion for Ulster, the cause of further divisions. Source D is particularly useful here. The nationalist Healy is speaking in March 1914, at a time when feelings are running high, and the Government is rapidly losing control. The tone in Source D is one of anger and disappointment. It is clear that Healy is critical of both Asquith and Redmond. He is particularly hostile to his leader, Redmond, who (according to Healy) has compromised on nationalistic principles, by giving in to Asquith. By now, exclusion had become a main area of dispute. That it was a thorny problem can be illustrated by the fact that the initial compromise involved four counties being excluded, later increased by the House of Lords to nine counties as a blocking move, and much later on finalised as six. But the crisis could also be blamed upon the irresponsible attitude of Bonar Law (Source A) and the determination of Ulster Protestants led by Carson (Source B). Source C and Source D taken together indicate both Redmond’s inconsistency over exclusion, and the pressures on Redmond from more extreme nationalism. Candidates might show awareness that all the main actors were under pressure from both sides e.g. James Craig; the Ulster Volunteer Force; the Irish Volunteers in response to the UVF; Sinn Fein etc. And, to an extent, the problem was insoluble. Ulster men like Carson were certain that the industries of the North would be destroyed if governed from the less-developed South. While on the other side, the Nationalists were committed to getting a United Ireland.

60

Page 164: History A - The Bicester Schoolthebicesterschool.org.uk/wp-content/uploads/2015/09/Mark... · 2015-09-28 · History A Mark Schemes for the Units January 2009 H106/H506/MS/R/09J

F963 Mark Scheme June 2009

4 Churchill 1920-45

(a) Study Sources C and D Compare these Sources as evidence for Churchill’s character. [30]

Focus: Comparison of two Sources No set answer is expected but candidates need to compare the contents, evaluating such matters as authorship, dating, utility and reliability, so using the Source ‘as evidence for…’. The headings and attributions should aid evaluation and reference to both is expected in a good answer.

Content: C and D offer different views. In C we find Churchill unable to see the wider picture and being opportunistic. We find him on bad terms with Britain’s top soldier, unable to resolve a disagreement, petulant and childish. D finds him not living for the moment but able to distinguish main factors in a situation! We find him an inspiring leader, not a furious and immature person, unable to take advice. D suggests an understanding and energetic leadership absent from C, where the energy is misplaced and directed in fits of temper. Provenance: both are from military men who worked closely with him although D is Assistant Secretary to the War Cabinet and may over the wartime period have fallen over Churchill’s spell more than Brooke, but the big difference lies in the date. Brooke was writing under great pressure of the colossal responsibility of the day to day running of the war in 1943. Jacob was writing years after in a volume honouring Churchill and looking back on much more positive aspects. He is writing about a different side of Churchill. Yet there is also agreement. In C Brookes refers to ‘toys’ and Jacob in D corroborates this with reference to Churchill’s obsession with anything new, radar, warships etc. For Brooke that was not the point – by 1943 he did not need heroic speeches or ‘toys’ but sound decisions about a war that involved a great deal of planning and coordination now that the US had entered, not wasting time as D acknowledges – ‘matters which attracted his passing attention’. The nature of the sources differs – C is written directly after a confrontation in a diary which though later published was at the time for private use. Jacob is writing very publicly at a time when Churchill’s reputation stood very high and may not choose to recall the day to day frustrations. Some may know that Brook wrote very warmly about Churchill in other parts of his diaries and memoirs – but that is not looked for here. Brooke may be unreliable as an overall judgement as it expresses Churchill’s limitations as war leader – his childishness and tendency to interfere; Jacob may be painting too rosy a picture, given the limitations of some aspects of Churchill’s role in making wartime decisions, but there is also balance given his point that some of Churchill’s ideas were ‘old-fashioned’. (b) Study all the Sources

Use your own knowledge to assess how far the Sources support the interpretation that Churchill was a great war leader. [70]

Focus: Judgement in context, based on the set of Sources and own knowledge Successful answers will need to make use of all five Sources, testing them against contextual knowledge and evaluating their strengths and weaknesses, including any limitations as evidence. A range of issues will be addressed in focusing upon the terms of the question but no set answer is expected.

The debate centres on Churchill’s wartime leadership – did he take on too much, insist on personal involvement in leading decisions, interfere with the appointment of generals and have too much say in strategies which have come in for considerable criticism ? For example the involvement in Greece and Crete, the Mediterranean strategy in general, the trust he placed in Montgomery and his treatment of both Wavell and Auchinleck. On the other hand is his understanding of the importance of good relations with the USA; his towering moral force in insisting on victory and the rightness of the allied cause and his

61

Page 165: History A - The Bicester Schoolthebicesterschool.org.uk/wp-content/uploads/2015/09/Mark... · 2015-09-28 · History A Mark Schemes for the Units January 2009 H106/H506/MS/R/09J

F963 Mark Scheme June 2009

inspiration. Not every aspect of this topic need to be tackled given the possible range of subject matter. Nor is a detailed knowledge of the decisions in the Desert War looked for. Grouping. In considering the Sources, Source E and B are critical in a considered way; C is critical but is expressing the emotions of the moment; D offers praise though accepting that Churchill was ‘somewhat old fashioned’ and A could certainly be used to support Churchill as an inspirational leader. The case against in E considers that Churchill interfered with the army too much, a point which also appears in B, a contemporary criticism. B is spoken after the major defeats in the Far East – which were Churchill’s fault to some extent. The nature of B might lead to some caution in accepting it at face value. To censure a leader in time of war is unusual and Laidlaw Milne would be looking for evidence to support his attack on the PM. C is very critical and Churchill was deeply upset by the publication of Brooke’s diaries. However we are not looking here at a considered judgement but a heat of the moment response by a man under considerable pressure and facing a huge burden of responsibility. Nevertheless it does reflect the day to day strains of working with Churchill. Like D Thompson in E finds him old fashioned and he uses the example of the sacking of Sir Claude Auchinleck in 1942 as an example of Churchill’s lack of judgement and vanity – finding in Montgomery a similarly showy and egotistical figure. Some may know that El Alamein in October 1942 was largely the result of Auchinleck’s preparations and Monty’s plan was not well-judged. Some may find other examples of Churchill’s interference; or may reject the view because of eventual victory and the more evident leadership qualities referred to in D and A. The tone of E is somewhat polemical and strident. A could be evaluated in the context of Britain’s position in 1940 when it stood alone and really needed confidence at the top. For all its rhetoric many found it it inspiring – it was intended to be a rallying cry and also a message to the USA. D could be evaluated in terms of Churchill being able to see many aspects of the war very clearly – the need for strong relations with FDR and Stalin; the need to ensure that D Day was well planned and the realism to see that the US had to lead, for example. The source is from someone close to Churchill, but it was part of an anthology written more than 20 years after the war when Churchill had become a legend. There could be a wide range of relevant own knowledge to discuss the question, so it is important to consider how candidates are using what they know rather than expecting specific information. In terms of judgements, some may stress the major contribution made in 1940 to morale and the courage of continuing the war against Germany. Others may be more critical of Churchill’s wartime style of government. It is important not to expect a particular view but to see how well supported opinions are from both Sources and contextual knowledge.

62

Page 166: History A - The Bicester Schoolthebicesterschool.org.uk/wp-content/uploads/2015/09/Mark... · 2015-09-28 · History A Mark Schemes for the Units January 2009 H106/H506/MS/R/09J

F964 Mark Scheme June 2009

63

F964/01 European and World History Enquiries Option A: Medieval and Early Modern 1073-1555

1 The First Crusade and the Crusader States 1073-1130

(a) Study Sources A and C Compare these Sources as evidence for Pope Urban II’s attitude to Islam. [30]

Focus: Comparison of two Sources. No set answer is expected, but candidates need to compare the contents, evaluating such matters as authorship, dating, utility and reliability, so using the Source ‘as evidence for ...’. The headings and attributions should aid evaluation and reference to both is expected in a good answer. The Sources both can be compared in their tone and language content. Both take a tough line towards the Islamic faith. A is a clear and powerful appeal, using much emotive language towards Islam and promising remission of sins as a key outcome; the audience is a wide one, given that the appeal was reported and disseminated through Europe from small groups whereas C has wider feudal appeal; there is urgency in tone and appeal, a strong emphasis upon war against the Muslims who are identified as pagans. Both suggest the hostility towards the Muslim invaders and this is reflected in the language used. In A the focus is on the extermination of a ‘vile race’ and on aiding fellow Christians against pagans; in C, the latter are ‘barbarians’, ‘frenzy’, ‘invaded and ravaged’ figure strongly; liberation of the Holy Land is the goal. Both mention vows to fight Muslims. Differences lie in part in the target audiences and in part in the language used. Typicality lies in the nature of how the Muslim threat is expressed. In both the pope is the figurehead, the key mover, but he operates in somewhat different ways according to his target audience, the language being adjusted in some parts to fit the audience. The language is, in turn, focused and applied, powerful (A), more measured but strong still (C). There is a contrast between the feudal language in C and spiritual appeal in A, e.g. ‘Christ commands’.

Comment on the provenances may include the reliability of the author of A, using a measured but intense tone in reporting the Pope’s words, and the Pope himself in C, intense, personalised, subjective in tone in his letter, pouring out concern and zeal. Both are very typical of Christian views at the time.

(b) Study all the Sources

Use your knowledge to assess how far the sources support the interpretation that Urban II, in preaching the First Crusade, appealed largely to social and material motives. [70]

Focus: Judgement in context based on the set of Sources and own knowledge. Successful answers will need to make use of all five Sources, testing them against contextual knowledge and evaluating their strengths and weaknesses, including any limitations as evidence. A range of issues may be addressed in focusing upon the terms of the question but no set answer is expected. The question focuses on the nature of Urban’s appeal for the Crusade, setting up argument and counter-argument. The Sources can be grouped according to both areas. Traditionally, religious appeal has have been seen as paramount but it is clear that other factors were there. The Sources offer a range of useful points, explicit and implicit. Sources A, C and D mention religious zeal, in different ways and there is reference also

Page 167: History A - The Bicester Schoolthebicesterschool.org.uk/wp-content/uploads/2015/09/Mark... · 2015-09-28 · History A Mark Schemes for the Units January 2009 H106/H506/MS/R/09J

F964 Mark Scheme June 2009

in Source E. Source B and a good part of E feature more material, non-religious factors. A focuses strongly on religious zeal and zest, with powerful language devised to reach out to a wide audience. Like A, C dwells upon the power and role of remission of sins; again, the language is powerful; a religious appeal is paramount. E reinforces this before moving to other issues. D mentions religious appeal (salvation, liberation) and calls for a close liaison of church and lay leaders. Own knowledge of Urban’s appeal can be adduced: plenary indulgences; his concept of the armed pilgrimage; the wide range of crusader types he was appealing to. Source B focuses upon lands, resources, overpopulation, the removal of the reasons for dissent and unrest and can be linked to a good part of E, where Urban’s wider motives are addressed. Again, own knowledge can supply support in the form of evidence for economic and social tensions, political unrest, disorder as well as evidence for these being the issues that appealed. Therefore, though the Sources do have an obvious religious appeal and direction, but there are enough references to other factors to suggest that religious zeal was not the sole issue behind the preaching of the crusade (reviewed in Source E). Contextual knowledge can be used, as above and with reference to features such as: Urban’s activist preaching of the crusade; contemporary religious devotions and enthusiasms, including pilgrimages; Urban’s awareness of pressures on lords and knights; knightly codes of service and feudal ties; socio-economic pressures; civil unrest levels; popular enthusiasm and energy levels (The People’s Crusade being but one major example). These areas can be linked to all the Sources. Better answers will see the linkage between Urban’s strands of appeal but be able to offer a good counter-argument to the interpretation.

64

Page 168: History A - The Bicester Schoolthebicesterschool.org.uk/wp-content/uploads/2015/09/Mark... · 2015-09-28 · History A Mark Schemes for the Units January 2009 H106/H506/MS/R/09J

F964 Mark Scheme June 2009

2 The German Reformation 1517-1555: The Impact of Lutheranism 1524-1530

(a) Study Sources A and E Compare these sources as evidence for the religious attitudes and beliefs of German townspeople. [30]

Focus: Comparison of two Sources. No set answer is expected, but candidates need to compare the contents, evaluating such matters as authorship, dating, utility and reliability, so using the Sources ‘as evidence for …’. The headings and attributions should aid evaluation and reference to both is expected in a good answer. The Sources both suggest that many German townspeople embraced the reformation in the 1520s, and that this encouraged a desire for liberty, unrest and disobedience to authority. In both cases, the context of the Sources is the calling of an imperial Diet with the aim of reconciling Catholics and Protestants, so feelings might be untypical, heightened due to the atmosphere and the presence of princes attending the Diet. However, there are differences. Whereas Catholic practices have been ‘utterly destroyed’ according to the Catholic friar in Source A, perhaps exaggerating the heretical nature of his opponents in an emotive style, the Venetian observer in Source E takes a more positive line, recording that ‘the papists still have their churches, images and Masses’, though they are few in number. This suggests a considerate attitude among the townspeople of Augsburg, in contrast to the ridicule of pope, Catholic hierarchy, saints and practices suggested in Source A. Source A also suggests that Luther is the central inspiration for reform, whereas Source E shows majority support for Zwingli’s ideas. This follows the Marburg Colloquy and the dispute between Luther and Zwingli over Communion. The tone of Source E is more objective and suggests that doctrine is important in shaping attitudes, whereas propaganda and disobedience to authority are the underlying attitudes to religious reform noted in Source A. Division of the city into opposing religious factions is the key issue in Source E, unlike Source A, where Lutherans are the only group mentioned. Source A generalises to include conversions to reform in other cities, whereas Source E refers only to those in Augsburg. Both Sources are written by Catholic observers about a city where an imperial Diet is being held, but they are five years apart and attitudes have radicalised. Religious change has become more entrenched and diverse.

(b) Study all the Sources

Use your own knowledge to assess how far the Sources support the interpretation that the impact of the German Reformation between 1524 and 1530 was under the control of the authorities. [70]

Focus: Judgement in context, based on the set of Sources and own knowledge. Successful answers will need to make use of all five Sources, testing them against contextual knowledge and evaluating their strengths and weaknesses, any limitations as evidence. A range of issues may be addressed in focusing upon the terms of the question but no set conclusion is expected. Different types of authority are mentioned: the Emperor in Source D; the princes in Sources A, B, C and D; and town authorities in Sources A and E. As far as the Emperor and princes are concerned: Source C suggests Philip of Hesse was in control of the reformation in his territories, as he is directing the dissolution of the monasteries there. Own knowledge might be used to discuss whether other princes also controlled the spread of the reformation, perhaps using own knowledge of the princes who signed the Protest in 1529. In Source D the Emperor seems to have had some control: some Catholic princes have succeeded in enforcing the Edict of Worms, so are preventing the spread of the Reformation. However, he has lost control of other princes: in Source D the

65

Page 169: History A - The Bicester Schoolthebicesterschool.org.uk/wp-content/uploads/2015/09/Mark... · 2015-09-28 · History A Mark Schemes for the Units January 2009 H106/H506/MS/R/09J

F964 Mark Scheme June 2009

spread of sects has occurred due to princely support for reform against his will. His request to ban further reform lacks conviction, suggesting that only some princes may be able to achieve this, and he concedes that the ban may not be ’humanly possible’. His tone shows resignation that his instructions may be unenforceable, based on past experience of his failures. Own knowledge might include the emperor’s distractions, past Diets and Edicts, the unhelpful attitude of the papacy, and the unrest amongst imperial knights and peasants, hinted at in Source B, which had increased princely control within the Empire.

Source B suggests that reformation should be controlled by the princes or else the lower orders will take up arms and gain reformation by force, a spectre of civil war also raised in Source A by the Catholic friar. Müntzer, the prophetic author of Source B, played a major part in the subsequent Peasants’ War showing a shift of control from Luther, in Source A, to the radicals. The Reformation subsequently spun out of control of the authorities until military victory allowed the princes greater control, so showing the irony of Müntzer’s words. Own knowledge of this context might be used in evaluation.

However, Sources A and E suggests that the princes have no control of the imperial free cities or have lost control in their towns, where the reformation is proceeding according to the wishes of the nobles, merchants and townspeople. Own knowledge might include the cities which signed the Protest in 1529. In Source E, the authorities of Augsburg seem to have control, but they have conceded to the wishes of the majority of Zwinglians in order to keep law and order. Public order also seems under threat in Nuremburg in Source A, and the general view of the Catholic observer is that the authorities have no control of towns and cities. Own knowledge might be used to add that Luther has lost control of the movement he began, and cannot appear at the Diet due to the imperial ban.

Candidates are likely to consider alternative views on the extent of control over the impact of the reformation, by the Emperor, the princes, the town and city authorities. They are likely to see the reformation in a variety of ways: under the control of princes, towns and peasants, but outside the control of Emperor and papacy. It is up to candidates to assess and decide upon relative importance here, there being no set conclusion.

66

Page 170: History A - The Bicester Schoolthebicesterschool.org.uk/wp-content/uploads/2015/09/Mark... · 2015-09-28 · History A Mark Schemes for the Units January 2009 H106/H506/MS/R/09J

F964 Mark Scheme June 2009

67

F964/02 European and World History Enquiries Option B: Modern 1774-1975

1 The origins and course of the French Revolution 1774-1795

(a) Study Sources B and D Compare these Sources as evidence for contemporary views of the Revolution in the period from 1793 to 1794. [30]

Focus: Comparison of Two Sources No set answer is expected, but candidates need to compare the contents, evaluating such matters as authorship, dating, utility and reliability, so using the Sources ‘as evidence for..’. The headings and attributions should aid evaluation and reference to both is expected in a good answer. In terms of content they offer a very different view of the Revolution. Robespierre in B sees peace and enjoyment of liberty as the key – D sees not peace but crimes and blood. B sees France aiming to become a model for other nations; D merely a country dominated by madness and ambition. This is a long way from the ideal of Virtue in B. This public virtue is very different from the people losing morale and having a blow dealt to social institutions. In Robespierre's vision in B law is in men’s hearts; in D it is imposed only by the iron rod of a vile scoundrel. In terms of provenance, Robespierre in B is making a public speech at a time of war and terror to provide a vision to encourage the development of a revolution under pressure; D is writing after the terror and the fall of Robespierre, in a state of shock about the extent of the violence and repression and able now to speak freely. Robespierre is speaking of aims; Des Essarts is speaking about practical consequences. Both adopt highly emotional tone and both are writing in public statements. B is far more political and from a figure at the very heart of the revolution; D is written from the perspective of an observer of the victims of this public figure and is not himself making a political statement, or at least not one to advance particular political policies as is the case with B. Both may exaggerate – Robespierre in B offers a very idealistic picture of laws which were highly repressive and a regime which adopted extreme measures; D sees nothing but a scoundrel and ignores the visionary and idealistic aspects of the revolution and its leader. In terms of utility, B offers the ideology which justified the Terror, while D shows how the Terror came to be seen out of context of the emergency conditions which contributed to its development – both represent a degree of mythology

(b) Study all the Sources

Use your own knowledge to assess how far the Sources support the interpretation that harsh measures were introduced during the Terror mainly in order for France to protect itself from its enemies. [70]

Focus: Judgement in context, based on the set of Sources and own knowledge Successful answers will need to make use of all four Sources, testing them against contextual knowledge and evaluating their strengths and weaknesses, including any limitations as evidence. A range of issues may be addressed in focusing upon the terms of the question but no set answer is expected. The discussion here is whether the Terror and the repressive laws were simply measures necessary to defend France from threatening foreign armies, émigré forces and counter revolution at home, or had a wider social and political agenda. Candidates could see the

Page 171: History A - The Bicester Schoolthebicesterschool.org.uk/wp-content/uploads/2015/09/Mark... · 2015-09-28 · History A Mark Schemes for the Units January 2009 H106/H506/MS/R/09J

F964 Mark Scheme June 2009

threat coming from within – from profiteers, non-juring priests, aristocrats, federalists. Émigré came to be a term of abuse which included internal enemies. In terms of grouping the Sources, B seems the clearest defence of measures being based on the need to fight the tyrants of Europe. A offers a justification based on foreign threats. C and A both suggest other motives and D rejects the necessity for harsh laws, seeing merely the exercise of tyranny. Knowledge of the context in which the revolution became radicalized will help to assess the significance of A – there is reference to the ‘enemies of Liberty’ and the relatives and agents of émigrés which might indicate that the harsh measure was defensive. . However, the law was wide – who was to interpret the ‘customs, connections, remarks or writings ' of those who were suspected? The law itself could be seen either as defensive measure in time of war or an extension of revolutionary extremism with the purpose being not only to attack domestic opposition, but to wage war on political and class opponents. This is certainly the view in D which see ‘our best citizens; being killed by ‘madness and ambition’ not a desire to defend France from enemies. Here is not legitimate defence against enemies, but abominable tyranny, a return to ancient regime oppression, undermining public morale. Evaluation: this is written in 1797 after the fall of Robespierre and when immediate dangers from foreign counter-revolution had subsided. Unlike B which is written in the heart of the revolutionary wars, with France having raised the largest conscripted army in history and Carnot being in charge of the nearest thing to total war before the twentieth century. Robespierre rejects the ideas of laws being subverted and stresses the need for consent, but this view is open to criticism and may be justification for very severe wartime controls. Or perhaps Robespierre did see his own vision of Virtue – a sort of hard edged Classical vision of citizens showing their moral strength by defending the Republic – as an ideal in citizens’ hearts. C has a vision of imprisonment which goes beyond functional necessity in time of war – powerful laws are to be linked with the very nature of a Republic. The object of imprisonment and confiscation seems to be to defeat the foreigner, but they are also a way of celebrating liberty. The confiscation of goods for the Republic seems to go beyond the simple needs of war or defeating internal enemies to a sort of redistribution of property. Harsh measures are not seen as extraordinary but a part and parcel of the new state. Evaluation might refer to St.Just’s position on the Committee of Public Safety and the mixture of practical response to enemies and the class warfare that can be seen in A (‘Spare the aristocracy and you will bring fifty years of trouble’) St.Just’s political radicalism has to be taken into account when assessing how genuine his stated motives for Terror were. Additional knowledge about the ideals and aims of the revolutionaries like St.Just and Robespierre would be relevant; as would the reorganization of the state for war and the nature of the war in 1793-4. Also the way that the revolutionaries destroyed their political rivals would support the view that there were motives beyond the purely defensive.

68

Page 172: History A - The Bicester Schoolthebicesterschool.org.uk/wp-content/uploads/2015/09/Mark... · 2015-09-28 · History A Mark Schemes for the Units January 2009 H106/H506/MS/R/09J

F964 Mark Scheme June 2009

2 The Unification of Italy 1815-70

(a) Study Sources A and E Compare these Sources as evidence for Cavour’s views on constitutional government. [30]

Focus: Comparison of two Sources. No set answer is expected, but candidates need to compare the contents, evaluating such matters as authorship, dating, utility and reliability, so using the Sources ‘as evidence for …’. The headings and attributions should aid evaluation and reference to both is expected in a good answer. Both sources indicate that Cavour favoured a form of liberal and constitutional government. This is spelled out in the first sentence of Source A and the last line of Source E. An associated point of similarity, expressed in both Source A and Source E, is that Cavour believed government depended on popular support. Source A stresses Cavour’s belief in strong leadership and his suspicion of political parties which dovetails with the comment in Source E highlighting Cavour’s opposition to faction. A major issue developed in both is Cavour’s hostility to street or mob action. This is articulated in the final sentence of Source A and in the reference to Cavour’s fear of ‘revolutionary passions’ expressed in Source E. They differ in one respect at least. In Source A the implication of the second sentence is that Cavour was prepared to act in a dictatorial manner, a point developed later with reference to ‘the constitution being reduced to a mere machine’. However, in Source E Cavour subtly portrays himself as merely part of a government and that, if anything, the King was the dominant political force.

The typicality of the sources and their dates should be evaluated. The author of Source A is favourable in his assessment of Cavour because he was a friend and he was writing just after unification and the death of Cavour when there was a tendency to exaggerate Cavour’s role to the point of glorification. As such the author is prepared to accept Cavour’s tendency towards authoritarianism as a price worth paying for the achievement of unity. Cavour himself was hardly likely to admit to being authoritarian which explains why in Source E he emphasises the role of the King and the government of Piedmont in taking the initiative in the Papal States. Cavour is speaking at a time of national triumph following the unification of all of Italy except Rome and Venetia which might explain the patriotic tone of this piece. In addition, Cavour was likely to trumpet the cause of liberalism to check revolution in addressing an assembly who owed their position to the constitutional system in place in Piedmont: some candidates may provide knowledge of the Statuto.

(b) Study all the Sources

Use your own knowledge to assess how far the Sources support the interpretation that Cavour’s priority was to promote the interests of Piedmont rather than the unification of Italy. [70]

Focus: Judgement in context, based on the set of Sources and own knowledge. Successful answers will need to make use of all four Sources, testing them against contextual knowledge and evaluating their strengths and weaknesses, any limitations as evidence. A range of issues may be addressed in focusing upon the terms of the question but no set conclusion is expected.

Most candidates are likely to suggest that Source B and Source C lend most support to the view under consideration whilst Source A appears to refute the statement. Most are likely to regard Source D and Source E as ambivalent. The second and third sentences of Source B state bluntly that Cavour’s priority was to advance the interests of Piedmont. Given the reference to French support for Piedmont in a war with Austria candidates might refer to the terms of Plombieres which aimed to establish a northern state headed by the

69

Page 173: History A - The Bicester Schoolthebicesterschool.org.uk/wp-content/uploads/2015/09/Mark... · 2015-09-28 · History A Mark Schemes for the Units January 2009 H106/H506/MS/R/09J

F964 Mark Scheme June 2009

king of Piedmont. The comments made about Cavour might reflect disappointment on the part of the author that Cavour was not more ambitious to unite the whole of Italy or, more likely, that he had reservations about French involvement in Italian affairs. Either way it does not alter the impression that Cavour’s aim was the promotion of Piedmont rather than the unity of Italy.

Source C confirms Cavour’s aims were limited to the creation of a Kingdom of Italy in the north only. Some candidates will explain the reference to ‘promises ought to be kept’ mentioning the deal struck at Plombieres highlighting the territorial arrangements agreed and the understanding that war would only be ended by mutual agreement. This may be linked to the comments arising from the analysis of Source B as indicated above. The ‘years he had worked hard’ allows scope to elaborate on the foreign and domestic policies of Cavour in anticipation of liberating Italy from Austrian influence for the benefit of Piedmont. Cavour’s apparent willingness to continue the war without French support might be regarded as proof of his determination to achieve his aims of establishing an independent northern state under Piedmont. His commitment could be emphasised by explanation of the reference to 1848 and the caution of the military to carry on the war alone given the defeat at Custozza in 1848 and the fierceness of the battles of Magento and Solferino in 1859.

The imperative of Piedmontese interests is confirmed in Source E. This emphasises Cavour’s clear concerns to ensure stable political conditions and protect the king and government of Piedmont from infection by revolutionary ideas. Knowledge about the Mazzinian views of Garibaldi and the rivalry between the latter and Cavour could be applied allowing some to debate Garibaldi’s attitude to the monarchy in light of the support he received from Victor Emmanuel during his campaign and the transfer of his conquests to the King at Teano. Some might suggest that Cavour’s action in intervening in the Papal States was motivated by his anxiety about French troops stationed in Rome. Others may interpret Cavour’s caution as practical in the circumstances rather than as proof of his reluctance to support the full unity of Italy as he indicates his preference for such an outcome in describing ‘our wonderful Italian movement’ and, later in the source, his desire not to derail ‘our national movement’. Many will appreciate that Cavour was trying to gain the support of the Piedmontese Senate which obliged him to pay lip service, at least, to the national cause.

Source A presents Cavour as an Italian nationalist backing ‘the great enterprise’ and that his policy was ‘exclusively Italian’. The fact that Cavour enjoyed the support of the nation, which put their faith in his leadership, reinforces the nationalist credentials of Cavour. Yet, some might suggest that the nation referred to is Piedmont and its destinies are not defined, explicitly, in nationalist terms. The fact that ‘Cavour’s policy became more exclusively Italian’ allows for the interpretation that if Cavour did eventually support the unity of Italy he may not always have done so. With knowledge the degree to which Cavour’s aims shifted from the early 1850s, with policies aimed to modernise and strengthen Piedmont, to the events of 1859-60 when the state of Italy was created, might be discussed. Source D suggests Cavour aimed to unite Italy as he wanted to support Garibaldi (‘We could not have stopped Garibaldi’) and actually did so (in allowing ‘the despatch of arms and ammunition’) but that Cavour ‘cannot support him openly’ (for fear of France and Naples). Yet, the passage shows Cavour wanted to stop Garibaldi and by implication the unity of Italy but to do so ‘he would have become a real domestic problem’. The popularity of Garibaldi in Piedmont was a factor Cavour considered with parliamentary elections imminent. Was Cavour’s simply opposed to Garibaldi’s campaign rather than concerned about the international response? The final sentence of this source underlines its ambiguous nature. Cavour is expressing his opinions in a private letter to a man who had demonstrated his commitment to Piedmont and the completion of the unification of Italy: candidates could refer to the events in the Central Duchies in 1859-60 and Ricasoli’s role in them.

70

Page 174: History A - The Bicester Schoolthebicesterschool.org.uk/wp-content/uploads/2015/09/Mark... · 2015-09-28 · History A Mark Schemes for the Units January 2009 H106/H506/MS/R/09J

F964 Mark Scheme June 2009

3 The origins of the American Civil War 1820-61

(a) Study Sources A and B Compare these Sources as evidence for views about the 1850 Compromise. [30]

Focus: Comparison of two Sources. No set answer is expected, but candidates need to compare the contents, evaluating such matters as authorship, dating, utility and reliability, so using the Sources ‘as evidence for …’. The headings and attributions should aid evaluation and reference to both is expected in a good answer. The main difference is that Seward, in Source A, argues the Compromise breached the Constitution of 1787 which is denied by Clay in Source B. Similarly, in Source A Seward considers that the Compromise threatens the Union claiming that the denial of freedom implicit in the proposals will mean that ‘the United States will cease to exist’. By contrast, Source B regards the proposals as a ‘healing system of measures to reunite this Union’, a theme developed further in Clay’s speech. Source A emphasises the ‘sovereignty of the people’ rather than the states whereas Clay in Source B is more concerned to protect the rights of the states and interest groups. In Source B Clay declares ‘Shall any man oppose the onward march of a great nation?’ accepting the inevitability of compromise whereas Seward’s comments in Source A are laced with a moral tone of responsibility to God not the desires of men. Both agree that the stakes were high as Seward in Source A implies unhappiness will result from the Compromise and in Source B Clay argues that the ‘consequences … will be disastrous’ unless the Compromise is accepted. In both speakers are patriotic and aware of the gravity of the debate: Source A wants to uphold the principles that underpinned the Union and Clay, in Source B, talks of his ‘beloved country’. Evaluation that explains Seward’s and Clay’s views on the Compromise as consistent with Northern and Southern views respectively can be expected. Details of the proposals would help demonstrate the perspectives of the two speakers. As a prominent member of the administration Seward supported President Taylor’s stubborn opposition to Compromise. When Seward spoke in the early stage of the debate positions on both sides of the divide were stridently opposed to Compromise if for different reasons. The prospect of secession and war was very real, reflected in the stubborn tone of Seward’s speech. As the author of the Compromise it is not surprising that Clay spoke passionately for it. However, Clay had a reputation as a conciliator based on many years of public service and the Compromise was typical of his views and approach. Also, Clay wanted to win the leadership of the Whig Party, led by Taylor, so he adopted a contrary position. The death of Taylor on 9 July 1850 was a further incentive for Clay to promote the Compromise with more chance of success: the new President, Fillmore, was less hostile to it. (b) Study all the Sources

Use your own knowledge to assess how far the sources support the interpretation that southern interests were unrestricted between 1850 and 1860? [70]

Focus: Judgement in context, based on the set of Sources and own knowledge. Successful answers will need to make use of all four Sources, testing them against contextual knowledge and evaluating their strengths and weaknesses, any limitations as evidence. A range of issues may be addressed in focusing upon the terms of the question but no set conclusion is expected. The case in favour of the interpretation is strongly supported by Source E which summarises the conspiracy known by many Northerners as Slave Power claiming this was evident in the events of the period including the Compromise allowing candidates the

71

Page 175: History A - The Bicester Schoolthebicesterschool.org.uk/wp-content/uploads/2015/09/Mark... · 2015-09-28 · History A Mark Schemes for the Units January 2009 H106/H506/MS/R/09J

F964 Mark Scheme June 2009

chance to comment on different aspects of it, perhaps emphasising the option of slavery in New Mexico and Utah or the failure to ban slavery in Washington DC or the Slave Fugitive Law. In addition, the charge that slave-masters ‘made Presidents judges’, etc could be explained with reference to the election of Taylor, Pierce and Buchanan and the Dred Scott judgement. The reference to the election of Lincoln allows candidates to talk of his position on slavery. The more discerning will question the validity of this evidence. As a servant of Grant – one of the war heroes of the North – or simply as a Northern republican politician the author’s perspective is clearly partial, even, partisan. Further, some of the charges made are not entirely convincing. Taylor may have been a slave owner but during the debate of 1850 he was opposed to Compromise. Neither was the decision in the Dred Scott judgement a simple case of Southern judges out-voting Northern ones.

Seward, in Source A, represented the view of many at the time that they were at a turning point when the union might sunder unless slave power was restrained. A link with Source E can be made and the background to the crisis in Texas could be referenced. Lincoln, in Source C, expresses reservations about the Kansas-Nebraska Act not least because of the repeal of the Missouri Compromise it entailed and because he saw it as a reverse for Northern principles of freedom. Rather as Seward implies, the outcome was likely to threaten the Union the protection of which was their main concern. The problem in Kansas-Nebraska and the subsequent resolution of the issue in 1854 as well as the events which engulfed the area in 1856 (‘bleeding Kansas’) could be explained and their importance in confirming the slave power theory in the minds of Northerners. The authors of Sources A and C both had political ambitions which they hoped to advance by adopting the positions they articulated. Seward’s loyalty to Taylor was important and is outlined in the previous answer. Lincoln and Douglas were rivals for political power in Illinois. Victory for either in the debate on the issue of Kansas-Nebraska would help each gain support within the state. Most candidates will know that the rivalry continued to the Senate elections of 1858 and the famous series of debates between them.

A counter argument can be constructed on three of the sources. Source D clearly refutes the notion that southern interests were unrestricted. The article accuses the North of exploiting the South which it defends against the charge of slave power by emphasising her commitment to compromise. The reference to ‘invaded our states’ might be linked to John Brown’s activities in Kansas in 1856 and Virginia in 1859. Their fears of Lincoln after his election could be explained and cross reference here to Source E would confirm the reality of these fears. Southern concerns about the ‘complete ruin of her social, political and industrial institutions’ might be explained by highlighting the trends in population and wealth in favour of the North. Southern claims to have compromised could be considered with reference to the 1850 Compromise or the Lecompton Constitution. The partial viewpoint of the article only a few weeks before South Carolina was to secede from the Union is worth comment.

Source B also refutes the interpretation. It explicitly states that the Compromise ‘deals unjustly by no part of the Republic’ and ‘saves the interests of all quarters of the country’; rejecting the idea that Southern interests were given preferential concern. Details about the Compromise which restricted Southern ambitions might be cited including the ban on slave trading in Washington DC and the fact that California was declared a free state. Candidates might emphasise the author’s Kentucky roots to explain his position.

72

Page 176: History A - The Bicester Schoolthebicesterschool.org.uk/wp-content/uploads/2015/09/Mark... · 2015-09-28 · History A Mark Schemes for the Units January 2009 H106/H506/MS/R/09J

F964 Mark Scheme June 2009

4 Dictatorship and Democracy in Germany 1933-63

(a) Study Sources A and C Compare these Sources as evidence for the aims of the Western powers towards the German Democratic Republic. [30]

Focus: Comparison of two Sources. No set answer is expected, but candidates need to compare the contents, evaluating such matters as authorship, dating, utility and reliability, so using the Sources ‘as evidence for …’. The headings and attributions should aid evaluation and reference to both is expected in a good answer. Both sources are very similar in the view of Western aims in 1961. Both date from the immediate aftermath of the building of the Wall and see a Western attempt to undermine the DDR through an aggressive policy of external and internal subversion. They differ in their emphasis, Ulbricht in A as the Communist leader of the DDR, is more concerned with the specific and allegedly serious impact on his country and in particular on East Berlin of Western policy. He claims that the West is sending a wide variety of agents and subversives to undermine the DDR politically, economically and socially. In contrast, the Soviet source Isvestia, in C, is more focused on the wider Cold War agenda of subversion and espionage. This can be borne out by comments on tone. Ulbricht (A) is more intemperate, exaggerating the Capitalist ‘low life’ sent by the West to infect good communists (teenage hooligans, prostitutes, traffickers and profiteers). Better answers might link his images of blood, life force, seeds etc with Nazi racial imagery. Isvestia (C) is more conventionally Cold War – agents, spies and saboteurs, the latter possibly a reference to the June 1953 uprising. Ulbricht (A) focuses on Western penetration of East Berlin but candidates might read this as implying that the West is using its attractions to ‘suck the life force from the people’, a veiled reference to the loss of skilled workers to the West and portrayed here as a deliberate Western aim. The provenance of both is questionable in relation to Western aims as both are official communist sources, heavily controlled and censored, and it is likely that there was collaboration between East Berlin and Moscow, Ulbricht and Kruschev, on the general ‘line’ to be taken re Western aggression. One would expect to hear that their purpose is to justify the building of the Wall as a response to Western aggression (A’s ‘anti-fascist protection wall’), and, as official newspapers, A the main DDR organ, and C one of the two key Soviet papers, this is what one gets. Both are remarkably similar. The more exaggerated tone of Ulbricht (A) plays on the fear of Western ‘negatives’ as its audience, its own citizens, are portrayed as directly at threat from crime and sleaze. He is naturally more immediately concerned with the exodus of East Germans and is concerned to stress that the bright lights of the West are deceptively corrupt. The USSR’s Isvestia has a more Cold War emphasis on spin and the achievement of protection and security for all ‘working people’. In terms of judgement neither is especially helpful evidence of Western aims towards the USSR as they take a standard propaganda line but they are useful for documenting Cold War rhetoric. Espionage was a reality. Some may consider Ulbricht as more useful given the influence and more specifically DDR audience.

73

Page 177: History A - The Bicester Schoolthebicesterschool.org.uk/wp-content/uploads/2015/09/Mark... · 2015-09-28 · History A Mark Schemes for the Units January 2009 H106/H506/MS/R/09J

F964 Mark Scheme June 2009

(b) Study all the sources Use your own knowledge to assess how far the Sources support the interpretation that in building the Berlin Wall the German Democratic Republic was mainly concerned to prevent the fleeing of East Germans to the West

[70] Focus: Judgement in context, based on the set of Sources and own knowledge. Successful answers will need to make use of all four Sources, testing them against contextual knowledge and evaluating their strengths and weaknesses, any limitations as evidence. A range of issues may be addressed in focusing upon the terms of the question but no set conclusion is expected. The question requires an assessment of the relative importance of the DDR’s motives in building the Berlin Wall as suggested in the sources. There are 3 main possibilities, one internal to the DDR and two external. The internal motive is suggested in the question as the main reason - to stop the East German skill drain to the West that was threatening the economic survival of the DDR. The external factors are either a defensive reaction to Western attempts at subversion or as part of an offensive, continuing and gradual by both the USSR and the DDR to absorb Western Berlin into the East German state. The sources can bear a variety of interpretations and the following groupings would suggest themselves. The need to arrest the exodus to the West is best stated in source E, the historian, and, through inference, in source A, Ulbricht. Berghan states quite clearly the threat posed by the loss of skilled labour to the most advanced of the eastern Communist states, one based on industrial modernisation and a skilled and educated workforce. Its statistics are reliable for the 1949-1961 periods. Rather obliquely Ulbricht confirms this with his references to ‘sucking on our workers’ and peasants’ republic’. As the Communist leader of DDR such inferences carry great weight. Berghan, although a Western historian writing during the Cold War (1982), does give a balanced view, referring to both the official line and then the reality beneath. His evidence on the skilled nature of the drain is especially telling. Knowledge can be used to substantiate this view – the open frontiers between sectors in Berlin that allowed movement – the push/pull factors of poorer DDR conditions and shortages that had already led to a rising in June 1953 on the building sites. Source D could be used here to highlight the lack of basics (food etc) in contrast to the subsidised entertainment and cheap classic books and records. This was heightened in 1961. The numbers leaving increased because of a renewed drive to agricultural collectivisation. The wall would close off their last escape route to the West. The figures of those leaving in the first half of 1961 was 200,000, threatening to top the previous record of 300,000 during the disturbed year of 1953 which also saw renewed oppression in the aftermath of the June uprising. The view that the Wall was built as a reaction to Western subversion can be found in sources A and C (Ulbricht and Isvestia) and is also commented upon as the official line in source E (Berghan). There is also some indirect support for this in the reported comments of Ulbricht in B and in Trenkner‘s memoirs in D. However, as source E points out this in the official line. Both A and C are propaganda, the view of the Eastern leadership, and much is obviously made of the corrupt attraction of the West. Candidates might point out that Ulbricht and the old guard of the SGD saw American youth culture as a threat and the language of A could be usefully discussed here. Trenkner in D, a reasonably reliable personal account from the West, corroborates Ulbricht’s comment about ‘sucking the life blood of the DDR’ as he buys up DDR goods and regularly visits the East to do so. However, candidates could question his typicality and he hardly seems the subversive implied by A. Nonetheless knowledge could be used here to point out that the West and the Federal Republic poured large sums into West Berlin to make it a magnet and a showcase of Western capitalism. West German television was broadcast into the East. This would suggest, if not subversion, at least an attempt to undermine Communist ideals.

74

Page 178: History A - The Bicester Schoolthebicesterschool.org.uk/wp-content/uploads/2015/09/Mark... · 2015-09-28 · History A Mark Schemes for the Units January 2009 H106/H506/MS/R/09J

F964 Mark Scheme June 2009

The view that the Wall was built as part of a strategy gradually to absorb Western Berlin into East Germany is found in B (Ulbricht’s comments to the Conservative MPs, who, as was no doubt expected, quickly reported to MacMillan the British PM) and by implication in C (Isvestia), Ulbricht, no doubt deliberately, is using the language of creeping absorption of demanding some controls at Tempelhof (one of the main Western airports in Berlin) and on limiting airspace and over-flights, a clear reference to events during the airlift in 1949, and to the use of visas. Isvestia in C, talks of tightening the borders around West Berlin. However, how much weight is to be put on such evidence? The context of B, the Leipzig fair, and the off-the-cuff remarks to British MPs, would suggest the usual war of words rather than specific intent. Was he bullying and intimidating or reflecting a very real ‘ratcheting up’? The wall is not mentioned but the tightening of the secure border around West Berlin is. Knowledge would suggest that in 1953 Krushchev had presented an ultimatum to the West demanding re-integration of West Berlin with the East over the next six months. It was ignored and nothing happened, as in the case here. He tried again with the new President Kennedy in 1961, this time insisting that US troops be withdrawn from Berlin. Kennedy refused. It could thus be argued that the Wall was the reaction. Something had to be done that would save face and to which the US could react only by useless protest. Sources B and C can be seen as just part of the sabre-rattling and their influence contexts (the Leipzig bookfair and Britain as a US ally) would suggest this. If it was a diversionary tactic then this suggests that the Wall was mainly a means of preventing the fleeing of East Germans. Given that the figures in E are largely accepted this would appear to be the main trigger for the Wall.

75

Page 179: History A - The Bicester Schoolthebicesterschool.org.uk/wp-content/uploads/2015/09/Mark... · 2015-09-28 · History A Mark Schemes for the Units January 2009 H106/H506/MS/R/09J

F964 Mark Scheme June 2009

5 The USA and the Cold War in Asia 1945-75

(a) Study sources B and D Compare these Sources as evidence for the impact of the Tet Offensive on American domestic opinion. [30] 

No set answer is expected but candidates need to compare the contents, evaluating such matters as authorship, dating, utility and reliability, so using the Sources ‘as evidence for...’. The headings and attributions should aid evaluation and reference and reference to both is expected in a good answer.

The Sources agree that American TV influenced US domestic opinion at the time of the Tet offensive. Source B suggests that the evidence shows that there will be no US victory, as suggested by the optimists in the past and that Communists hope ‘any success in the offensive will improve their position in eventual negotiations’. Source D suggests that the media focussed on the ‘initial success of the Viet Cong’.

But the Sources also disagree. Source B describes the situation as one of ‘stalemate’, as ‘for every means we have to escalate, the enemy can match us’. But it does say that the Tet Offensive may have been a desperate attempt to end the war of attrition which the North could not win. Source D on the other hand, states that the media are presenting ‘everything as a big American defeat, like that of the French at Dien Bien Phu.’ Source D thus suggests a much more extreme reaction amongst the American public, and focuses on particular anti-war protest groups such as ‘powerful’ liberals and ‘the young’ who had already expressed opposition to the war.

Contextual knowledge might be used to show understanding of the significance of the particular incidents mentioned in Source D, the events in Saigon, such as the footage of an ARVN officer shooting a VC suspect in the head in full view of the TV cameras, which caused such shock in the USA. Johnson’s advisers had turned against the war and he had declared in March that he would not stand in the forthcoming election.

The provenance of the Sources is revealing here. Source B is a spontaneous CBS broadcast at the time of the Tet Offensive, when shocking TV footage showed US ‘superpower’ forces coming under attack by Third World forces within their power-bases in the South Vietnamese capital. It does not tell us the impact on the US public, only the experienced reaction of an influential US broadcaster. Source D however is the considered opinion of the CIA in a secret intelligence briefing during the summer. The suggestion in Source D is that this media coverage distorted the truth of the military situation and failed to report US successes, so boosting support for the anti-war movement.

A supported judgement should be reached on which Source provides better evidence. Source B is useful in showing the immediate reaction broadcast to the US public. Source D may represent the military situation more accurately than Source B, and is much more useful for the impact on the US public, due to its later date. No set conclusion is expected.

(b) Study all the Sources.

Use your own knowledge to assess how far the Sources support the interpretation that the Tet Offensive of 1968 was a turning-point in the Vietnam War [70]

Successful answers will need to make use of all five Sources, testing them against contextual knowledge and evaluating their strengths and weaknesses, any limitations as evidence. A range of issues may be addressed in focussing upon the terms of the question but no set conclusion is expected.

76

Page 180: History A - The Bicester Schoolthebicesterschool.org.uk/wp-content/uploads/2015/09/Mark... · 2015-09-28 · History A Mark Schemes for the Units January 2009 H106/H506/MS/R/09J

F964 Mark Scheme June 2009

The Sources contain references to both sides of the argument, so they may be grouped by interpretation. Sources A, B and E support the view of Tet being of major significance in changing the US direction of the War, but Sources C and D raise doubts and suggest the opposite, that it need not have caused a reassessment in Washington. The context is a change from NLF guerrilla tactics to a broad offensive on the cities, co-ordinated with a political uprising in the South. Source A sets up the objectives of the Tet Offensive and Source B suggests its impact, above all on the mindset of many Americans, while Source E confirms this and offers further contextualisation as to the domestic impact, not least linking it to the growing anti-War movement there. The realisation that ‘crossover point’ would never be reached, in Source B, when more VC died than could be replaced, made observers suggest that the War was unwinnable.

Own knowledge might be used to confirm this shift in US public opinion and its impact on domestic American politics, including the decision not to grant Westmoreland 200 000 more troops and Johnson’s decision not to stand in the 1968 presidential election, leading to Nixon’s promise to seek an honourable peace and the beginnings of military de-escalation. Source E adds the perception of South Vietnamese collapse and lack of co-operation with US objectives, leading to the beginnings of an attempt to ‘Vietnamese’ the War after 1968 and make Saigon shoulder more responsibility for its conduct.

In contrast, Source C questions the significance of the Tet Offensive, suggesting it was, in fact, a defeat for the North, written itself from a Northern perspective and with hindsight after the North had succeeded in winning the War. This might bring its purpose and reliability into question. The impact of Tet in undermining the morale of the Communists in Source C – ‘we were surprised the Offensive had any success at all’ – is confirmed in Source D – ‘the Vietcong has suffered badly’. There seems to be an element of surprise in Source C’s admission that ‘we were told the Americans felt as is they had lost’. There is surprise too in Cronkite’s spontaneous and influential report, in Source B, suggesting that it was the media coverage, rather than the Tet Offensive itself, which marked a turning-point – the end of optimism, featured in Source B, and a growing anti-War movement in Sources D and E. Own knowledge might be used to extend the discussion of the extent of public outrage during 1968 and the impact of media reports in the light of international as well as domestic opinion.

Candidates are likely to consider whether or not Tet itself caused the change of US policy, as it revealed the nature of the war – a stalemate, or conversely, that a misinterpretation of the situation by the media unleashed a crisis of confidence in public opinion, which began the move towards US military de-escalation. They are likely to set the Sources within the changed context – from NLF guerrilla tactics in rural areas to co-ordinated attacks on a wide range of targets, including the cities, and US military de-escalation. It is up to candidates to assess and decide upon relative importance here, there being no set conclusion.

77

Page 181: History A - The Bicester Schoolthebicesterschool.org.uk/wp-content/uploads/2015/09/Mark... · 2015-09-28 · History A Mark Schemes for the Units January 2009 H106/H506/MS/R/09J

78

Grade Thresholds

Advanced GCE History (H506) Advanced Subsidiary GCE History (H106) June 2009 Examination Series Unit Threshold Marks

Unit Maximum Mark

A B C D E U

Raw 100 69 60 52 44 36 0 F961/01 UMS 100 80 70 60 50 40 0 Raw 100 70 61 52 44 36 0 F961/02 UMS 100 80 70 60 50 40 0 Raw 100 66 58 50 42 35 0 F962/01 UMS 100 80 70 60 50 40 0 Raw 100 69 61 53 46 39 0 F962/02 UMS 100 80 70 60 50 40 0 Raw 100 65 58 51 44 37 0 F963/01 UMS 100 80 70 60 50 40 0 Raw 100 70 62 54 46 39 0 F963/02 UMS 100 80 70 60 50 40 0 Raw 100 66 59 52 45 38 0 F964/01 UMS 100 80 70 60 50 40 0 Raw 100 70 63 56 49 42 0 F964/02 UMS 100 80 70 60 50 40 0

Specification Aggregation Results Overall threshold marks in UMS (ie after conversion of raw marks to uniform marks) Maximum

Mark A B C D E U

H106 200 160 140 120 100 80 0

The cumulative percentage of candidates awarded each grade was as follows:

A B C D E U Total Number of Candidates

H106 20.1 42.8 66.6 83.6 93.5 100.0 10478

For a description of how UMS marks are calculated see: http://www.ocr.org.uk/learners/ums_results.html Statistics are correct at the time of publication.

Page 182: History A - The Bicester Schoolthebicesterschool.org.uk/wp-content/uploads/2015/09/Mark... · 2015-09-28 · History A Mark Schemes for the Units January 2009 H106/H506/MS/R/09J

OCR (Oxford Cambridge and RSA Examinations) 1 Hills Road Cambridge CB1 2EU OCR Customer Contact Centre 14 – 19 Qualifications (General) Telephone: 01223 553998 Facsimile: 01223 552627 Email: [email protected] www.ocr.org.uk For staff training purposes and as part of our quality assurance programme your call may be recorded or monitored

Oxford Cambridge and RSA Examinations is a Company Limited by Guarantee Registered in England Registered Office; 1 Hills Road, Cambridge, CB1 2EU Registered Company Number: 3484466 OCR is an exempt Charity OCR (Oxford Cambridge and RSA Examinations) Head office Telephone: 01223 552552 Facsimile: 01223 552553 © OCR 2009

Page 183: History A - The Bicester Schoolthebicesterschool.org.uk/wp-content/uploads/2015/09/Mark... · 2015-09-28 · History A Mark Schemes for the Units January 2009 H106/H506/MS/R/09J

Oxford Cambridge and RSA Examinations

GCE

History A Advanced GCE A2 H506

Advanced Subsidiary GCE AS H106

Mark Schemes for the Units January 2010

HX06/MS/R/10J

Page 184: History A - The Bicester Schoolthebicesterschool.org.uk/wp-content/uploads/2015/09/Mark... · 2015-09-28 · History A Mark Schemes for the Units January 2009 H106/H506/MS/R/09J

OCR (Oxford Cambridge and RSA) is a leading UK awarding body, providing a wide range of qualifications to meet the needs of pupils of all ages and abilities. OCR qualifications include AS/A Levels, Diplomas, GCSEs, OCR Nationals, Functional Skills, Key Skills, Entry Level qualifications, NVQs and vocational qualifications in areas such as IT, business, languages, teaching/training, administration and secretarial skills. It is also responsible for developing new specifications to meet national requirements and the needs of students and teachers. OCR is a not-for-profit organisation; any surplus made is invested back into the establishment to help towards the development of qualifications and support which keep pace with the changing needs of today’s society. This mark scheme is published as an aid to teachers and students, to indicate the requirements of the examination. It shows the basis on which marks were awarded by Examiners. It does not indicate the details of the discussions which took place at an Examiners’ meeting before marking commenced. All Examiners are instructed that alternative correct answers and unexpected approaches in candidates’ scripts must be given marks that fairly reflect the relevant knowledge and skills demonstrated. Mark schemes should be read in conjunction with the published question papers and the Report on the Examination. OCR will not enter into any discussion or correspondence in connection with this mark scheme. © OCR 2010 Any enquiries about publications should be addressed to: OCR Publications PO Box 5050 Annesley NOTTINGHAM NG15 0DL Telephone: 0870 770 6622 Facsimile: 01223 552610 E-mail: [email protected]

Page 185: History A - The Bicester Schoolthebicesterschool.org.uk/wp-content/uploads/2015/09/Mark... · 2015-09-28 · History A Mark Schemes for the Units January 2009 H106/H506/MS/R/09J

CONTENTS

Advanced GCE History (H506)

Advanced Subsidiary GCE History (H106)

MARK SCHEMES FOR THE UNITS

Unit/Content Page

AS/A2 HISTORY SYLLABUS-SPECIFIC MARKING INSTRUCTIONS 1

F961 British History Period studies 14

F962 European and World History Period Studies 29

F963 British History Enquiries 57

Option A: Medieval and Early Modern 1066-1660 57

F964 European and World History Enquiries 72

F966 Historical Themes 88

Grade Thresholds 110

Page 186: History A - The Bicester Schoolthebicesterschool.org.uk/wp-content/uploads/2015/09/Mark... · 2015-09-28 · History A Mark Schemes for the Units January 2009 H106/H506/MS/R/09J

Marking Instructions

AS/A2 HISTORY SYLLABUS-SPECIFIC MARKING INSTRUCTIONS

AS UNIT F961 & UNIT F962 – PERIOD STUDIES Distribution of marks for each level that reflects the Unit’s AOs and corresponds to the UMS 2 answers: each maximum mark 50.

A01a A01b IA 21-24 24-26

IB 18-20 22-23

II 16-17 19-21

III 14-15 16-18

IV 12-13 13-15

V 9-11 11-12

VI 4-8 6-10

VII 0-3 0-5

Notes:

(i) Allocate marks to the most appropriate level for each AO.

(ii) If several marks are available in a box, work from the top mark down until the best fit has been found.

(iii) Many answers will not fall at the same level for each AO.

(iv) Analysis refers to developed explanations; evaluation refers to the argued weighing up/assessment of factors in relation to their significance in explaining an issue or in explaining linkages between different factors.

1

Page 187: History A - The Bicester Schoolthebicesterschool.org.uk/wp-content/uploads/2015/09/Mark... · 2015-09-28 · History A Mark Schemes for the Units January 2009 H106/H506/MS/R/09J

Marking Instructions

AOs AO1a AO1b Total mark for each question = 50

Recall, select and deploy historical knowledge appropriately, and communicate knowledge and understanding of history in a clear and effective manner.

Demonstrate understanding of the past through explanation, analysis and arriving at substantiated judgements of: - key concepts such as causation, consequence, continuity, change and significance within an historical context; - the relationships between key features and characteristics of the periods studied

Level IA

Uses a wide range of accurate, detailed and relevant evidence

Accurate and confident use of appropriate historical terminology

Answer is clearly structured and coherent; communicates accurately and legibly

21-24

Clear and accurate understanding of key concepts relevant to analysis and to the topic

Clear and accurate understanding of the significance of issues in their historical context

Answer is consistently and relevantly analytical with developed and substantiated explanations, some of which may be unexpected

The argument evaluates a range of relevant factors and reaches clearly substantiated judgements about relative importance and/or links.

24-26

Level IB

Uses accurate, detailed and relevant evidence Accurate use of a range of

appropriate historical terminology Answer is clearly structured and

mostly coherent; writes accurately and legibly

18-20

Clear and accurate understanding of most key concepts relevant to analysis and to the topic

Answer is mostly consistently and relevantly analytical with mostly developed and substantiated explanations

Clear understanding of the significance of issues in their historical context.

Substantiated judgements about relative importance of and/or links between factors will be made but quality of explanation in support may not be consistently high.

22-23

Level II

Uses mostly accurate, detailed and relevant evidence which demonstrates a competent command of the topic Generally accurate use of historical terminology Answer is structured and mostly

coherent; writing is legible and communication is generally clear

16-17

Mostly clear and accurate understanding of many key concepts relevant to analysis and to the topic

Clear understanding of the significance of most relevant issues in their historical context

Much of the answer is relevantly analytical and substantiated with detailed evidence but there may be some description

The analysis of factors and/ or issues provides some judgements about relative importance and/or linkages.

19-21

Level III

Uses accurate and relevant evidence which demonstrates some command of the topic but there may be some inaccuracy

Answer includes relevant historical terminology but this may not be extensive or always accurately used

Most of the answer is organised and structured; the answer is mostly legible and clearly communicated

14-15

Some/uneven understanding of key concepts relevant to analysis and of concepts relevant to their historical context

Answers may be a mixture of analysis and explanation but also simple description of relevant material and narrative of relevant events OR answers may provide more consistent analysis but the quality will be uneven and its support often general or thin.

Answer considers a number of factors but with very little evaluation of importance or linkages between factors/issues

Points made about importance or about developments in the context of the period will often be little more than assertions and descriptions

16-18

2

Page 188: History A - The Bicester Schoolthebicesterschool.org.uk/wp-content/uploads/2015/09/Mark... · 2015-09-28 · History A Mark Schemes for the Units January 2009 H106/H506/MS/R/09J

Marking Instructions

AOs AO1a AO1b Level IV

There is deployment of relevant knowledge but level/accuracy of detail will vary; there may be some evidence that is tangential or irrelevant.

Some unclear and/or under-developed and/or disorganised sections; mostly satisfactory level of communication.

12-13

Understanding of key concepts relevant to analysis and the topic is variable but in general is satisfactory.

Limited and patchy understanding of a few relevant issues in their historical context.

Answer may be largely descriptive/ narratives of events and links between this and analytical comments will typically be weak or unexplained OR answers will mix passages of descriptive material with occasional explained analysis.

Limited points made about importance/links or about developments in the context of the period will be little more than assertions and descriptions

13-15

Level V

There is some relevant accurate historical knowledge deployed: this may be generalised and patchy. There may be inaccuracies and irrelevant material also

Some accurate use of relevant historical terminology but often inaccurate/ inappropriate use

Often unclear and disorganised sections; writing will often be clear if basic but there may be some illegibility and weak prose where the sense is not clear or obvious

9-11

General and sometimes inaccurate understanding of key concepts relevant to analysis and of concepts relevant to the topic

General or weak understanding of the significance of most relevant issues in their historical context

Attempts at analysis will be weak or generalised, based on plausible but unsubstantiated points or points with very general or inappropriate substantiation OR there may be a relevant but patchy description of events/developments coupled with judgements that are no more than assertions

There will be some understanding of the question but answers may focus on the topic not address the focus of the question

11-12

Level VI Use of relevant evidence will be limited; there will be much irrelevance and inaccuracy Answer may have little

organisation or structure; weak use of English and poor organisation

4-8

Very little understanding of key concepts Very limited understanding of the topic or of the

question’s requirements Limited explanation will be very brief/ fragmentary The answer will be characterised by generalised

assertion and/or description/ narratives, often brief

6-10 Level VII No understanding of the topic or of

the question’s requirements; little relevant and accurate knowledge Very fragmentary and disorganised

response; very poor use of English and some incoherence

0-3

No understanding of key concepts or historical developments. No valid explanations Typically very brief and very descriptive answer

0-5

3

Page 189: History A - The Bicester Schoolthebicesterschool.org.uk/wp-content/uploads/2015/09/Mark... · 2015-09-28 · History A Mark Schemes for the Units January 2009 H106/H506/MS/R/09J

Marking Instructions

AS UNIT F963 & UNIT F964 – Historical Enquiries Maximum mark 100. 1 answer: 2 parts. Question (a) Maximum mark 30 A01a A01b AO2a

IA 6 8 16

IB 6 7 13-15

II 5 6 11-12

III 4 5 9-10

IV 3 4 7-8

V 2 3 5-6

VI 1 2 3-4

VII 0 0-1 0-2

Notes related to Question (a) (i) Allocate marks to the most appropriate level for each AO (ii) If several marks are available in a box, work from the top mark down until the best fit has

been found (iii) Many answers will not fall at the same level for each AO Question (b) Maximum mark 70 A01a A01b AO2a AO2b

IA 9-10 11-12 26-28 20

IB 8 9-10 23-25 17-19

II 7 8 20-22 14-16

III 6 6-7 17-19 11-13

IV 4-5 4-5 14-16 8-10

V 3 3 11-13 6-7

VI 2 2 5-10 3-5

VII 0-1 0-1 0-4 0-2

Notes related to Part B: (i) Allocate marks to the most appropriate level for each AO (ii) If several marks are available in a box, work from the top mark down until the best fit has

been found (iii) Many answers will not be at the same level for each AO

4

Page 190: History A - The Bicester Schoolthebicesterschool.org.uk/wp-content/uploads/2015/09/Mark... · 2015-09-28 · History A Mark Schemes for the Units January 2009 H106/H506/MS/R/09J

Marking Instructions

Marking Grid for Question (a)

AOs AO1a AO1b AO2a Total for each question = 30

Recall, select and deploy historical knowledge appropriately, and communicate knowledge and understanding of history in a clear and effective manner.

Demonstrate understanding of the past through explanation, analysis and arriving at substantiated judgements of: - key concepts such as causation, consequence, continuity, change and significance within an historical context; - the relationships between key features and characteristics of the periods studied.

As part of an historical enquiry, analyse and evaluate a range of appropriate source material with discrimination.

Level IA

Accurate use of a range of appropriate historical terminology

Answer is clearly structured and coherent; communicates accurately and legibly

6

Answer is consistently and relevantly analytical with developed comparison and judgement

Clear and accurate understanding of key concepts relevant to analysis and to the topic

Clear and accurate understanding of the significance of issues in their historical context

8

Response provides a focused comparison and/or contrast of both content and provenance

Evaluates qualities such as reliability, completeness, consistency, typicality, and especially utility, in relation to the question

16 Level IB

Accurate use of a range of appropriate historical terminology

Answer is clearly structured and coherent; communicates accurately and legibly

6

Judgements are supported by appropriate references to both content and provenance

Very good level of understanding of key concepts

Clear and accurate understanding of the significance of issues in their historical context

7

Response provides an effective comparison and/or contrast of both content and provenance

Evaluates a range of qualities of authenticity, completeness, consistency, typicality and usefulness in relation to the question

13-15 Level II

Generally accurate use of historical terminology Answer is structured and mostly coherent; writing is legible and communication is generally clear

5

Good attempt at explanation/ analysis but uneven overall judgements

Mostly clear and accurate understanding of key concepts

Clear understanding of the significance of most relevant issues in their historical context

6

Provides a relevant comparison and/ or contrast of both content and provenance

Answer lacks completeness in evaluating most of the range of available criteria (eg. limited use of the introductions and/ or attributions)

11-12

5

Page 191: History A - The Bicester Schoolthebicesterschool.org.uk/wp-content/uploads/2015/09/Mark... · 2015-09-28 · History A Mark Schemes for the Units January 2009 H106/H506/MS/R/09J

Marking Instructions

AOs AO1a AO1b AO2a

Level III

Answer includes relevant historical terminology but this may not be extensive or always accurately used

Most of the answer is organised and structured; the answer is mostly legible and clearly communicated

4

A mixture of internal analysis and discussion of similarities and/or differences. A judgement is unlikely Some/uneven understanding

of many key concepts relevant to analysis and of many concepts relevant to the topic

Uneven understanding of the significance of most relevant issues in their historical context

5

Provides a comparison and/ or contrast

Makes limited links with the sources by focusing too much on content or on provenance

The organisation is uneven, confining the comparison to the second half of the answer or simply to a concluding paragraph

9-10

Level IV

There may be some evidence that is tangential or irrelevant

Some unclear and/or under-developed and/or disorganised sections; mostly satisfactory level of communication

3

Mostly satisfactory understanding of key concepts

Mostly satisfactory explanation but some unlinked though relevant assertions, description / narrative

There is no judgement

4

Response attempts a comparison and/or contrast but the comment is largely sequential

Few points of comparative provenance or discussion of similarity/difference of content

7-8 Level V

There may be inaccuracies and irrelevant material.

Some accurate use of relevant historical terminology but often inaccurate/ inappropriate use

Often unclear and disorganised sections; writing will often be clear if basic but there may be some illegibility and weak prose where the sense is not clear or obvious

2

General and sometimes inaccurate understanding of key concepts relevant to analysis and of concepts relevant to the topic

General or weak understanding of the significance of most relevant issues in their historical context

3

Identifies some points of agreement and/or disagreement

The comparison and/or contrast is implicit

There is no judgement

5-6 Level VI

There will be much irrelevance and inaccuracy Answer may have little organisation or structure; weak use of English and poor organisation

1

Limited explanation but mainly description / narrative

Very little understanding of key concepts

2

Very weak commentary on one point of agreement/ disagreement

Sources may be paraphrased with no real attempt to compare and/or contrast

3-4

Level VII

No understanding of the topic or of the question’s requirements Totally irrelevant answer Very poor use of English

0

Weak explanation, and descriptive / narrative commentary on the sources

No understanding of key concepts

0-1

No attempt to provide a comparison and/or contrast

Sources are paraphrased or copied out

0-2

6

Page 192: History A - The Bicester Schoolthebicesterschool.org.uk/wp-content/uploads/2015/09/Mark... · 2015-09-28 · History A Mark Schemes for the Units January 2009 H106/H506/MS/R/09J

Marking Instructions

Marking Grid for Question (b)

AOs AO1a AO1b AO2a AO2b Total mark for the question = 70

Recall, select and deploy historical knowledge appropriately, and communicate knowledge and understanding of history in a clear and effective manner.

Demonstrate understanding of the past through explanation, analysis and arriving at substantiated judgements of: - key concepts such as causation, consequence, continuity, change and significance within an historical context; - the relationships between key features and characteristics of the periods studied.

As part of an historical enquiry, analyse and evaluate a range of appropriate source material with discrimination.

Analyse and evaluate, in relation to the historical context, how aspects of the past have been interpreted and represented in different ways.

Level IA

Uses a wide range of accurate, detailed and relevant evidence

Accurate and confident use of appropriate historical terminology

Answer is clearly structured and coherent; communicates accurately and legibly

9-10

Clear and accurate understanding of key concepts relevant to analysis and to the topic

Clear and accurate understanding of the significance of issues in their historical context

Answer is consistently and relevantly analytical with developed explanations leading to careful judgements

11-12

Excellent analysis and evaluation of all sources with high levels of discrimination

Analyses and evaluates the limitations of the sources and what is required to add to their completeness as a set

26-28

Excellent analysis and evaluation of the historical interpretation using all sources and own knowledge to reach a clear conclusion

Fully understands that the sources may either support or refute the interpretation

20 Level IB

Uses accurate, detailed and relevant evidence Accurate use of a

range of appropriate historical terminology

Answer is clearly structured and mostly coherent; writes accurately and legibly

8

Clear and accurate understanding of most key concepts relevant to analysis and to the topic

Clear understanding of the significance of issues in their historical context

Judgements are supported by appropriate references to both content and provenance

9-10

Focussed analysis and evaluation of all sources with high levels of discrimination

Analyses and evaluates the limitations of the sources and what is required to add to their completeness as a set

23-25

Focussed analysis and evaluation of the historical interpretation using all sources and own knowledge to reach a clear conclusion

Understands that the sources may either support or refute the interpretation

17-19

7

Page 193: History A - The Bicester Schoolthebicesterschool.org.uk/wp-content/uploads/2015/09/Mark... · 2015-09-28 · History A Mark Schemes for the Units January 2009 H106/H506/MS/R/09J

Marking Instructions

AOs AO1a AO1b AO2a AO2b

Level II

Uses mostly accurate, detailed and relevant evidence which demonstrates a competent command of the topic Generally accurate use of historical terminology Answer is structured and mostly coherent; writing is legible and communication is generally clear

7

Mostly clear and accurate understanding of key concepts

Clear understanding of the significance of most relevant issues in their historical context.

Good attempt at explanation/ analysis but uneven overall judgements

8

Focussed analysis and evaluation of most of the sources with good levels of discrimination

Analyses and evaluates some of the limitations of the sources and what is required to add to their completeness as a set

20-22

Focussed analysis and evaluation of the historical interpretation using most of the sources and appropriate own knowledge to reach a clear conclusion

There may be some imbalance between discussion of the sources and use of external knowledge in evaluating the interpretation

14-16 Level III

Uses accurate and relevant evidence which demonstrates some command of the topic but there may be some inaccuracy

Answer includes relevant historical terminology but this may not be extensive or always accurately used

Most of the answer is organised and structured; the answer is mostly legible and clearly communicated

6

Shows a sound understanding of key concepts. Sound awareness of

the significance of issues in their historical context

Attempts an explanation/ analysis but overall judgement may be incomplete

6-7

Refers to most of the sources to illustrate an argument rather than analysing and evaluating their evidence

Aware of some of the sources’ limitations either individually or as a set

17-19

Sound analysis and evaluation of the historical interpretation.

There may be some description and unevenness between use of own knowledge and use of sources

Answers which use the sources but no own knowledge in assessing the interpretation have a Level III ceiling

11-13 Level IV

There is deployment of relevant knowledge but level/ accuracy of detail will vary; there may be some evidence that is tangential or irrelevant

Some unclear and/or under-developed and/or disorganised sections; mostly satisfactory level of communication

4-5

Mostly satisfactory understanding of key concepts

Some explanation but not always linked to the question

Assertions, description / narrative will characterise part of the answer

4-5

Sources are discussed sequentially

Considers some of the limitations of the sources; but may not establish a sense of different views

14-16

Some analysis and evaluation of the historical interpretation with increasing amounts of description

Response is more imbalanced than Level III in using sources and own knowledge

Answers that use own knowledge but make no use of the sources in assessing the interpretation have a Level IV ceiling

8-10

8

Page 194: History A - The Bicester Schoolthebicesterschool.org.uk/wp-content/uploads/2015/09/Mark... · 2015-09-28 · History A Mark Schemes for the Units January 2009 H106/H506/MS/R/09J

Marking Instructions

AOs AO1a AO1b AO2a AO2b

Level V There is some relevant historical knowledge deployed: this may be generalised and patchy. There may be inaccuracies and irrelevant material Some accurate use of relevant historical terminology but often inaccurate/ inappropriate use Often unclear and disorganized sections; writing will often be basic and there may be some illegibility and weak prose where the sense is not clear or obvious

3

General and sometimes inaccurate understanding of key concepts relevant to analysis and of concepts relevant to the topic General or weak understanding of the significance of most relevant issues in their historical context

3

Limited attempt to use the sources or discriminate between them; they are discussed sequentially Sources will be used for reference and illustration of an argument

11-13

Mainly description with limited comment on the context of the question Little effective analysis of how far the sources support the interpretation

6-7 Level VI

Use of relevant evidence will be limited; there will be much irrelevance and inaccuracy Answer may have

little organisation or structure

Weak use of English and poor organisation

2

Very little understanding of key concepts.

No explanation. Assertion, description

/ narrative predominate

2

Weak application of the sources to the question

Weak attempt at analysis

5-10

Weak contextual knowledge

Mainly description with weak evaluation of the historical interpretation

3-5 Level VII

No understanding of the topic or of the question’s requirements; little relevant and accurate knowledge Very fragmentary and disorganised response; very poor use of English and some incoherence

0-1

No understanding of key concepts Weak explanation,

assertion, description / narrative

0-1

Very weak application of the sources to the question

No attempt at analysis

0-4

Very weak attempt at evaluating the historical interpretation

Heavily descriptive No contextual

knowledge

0-2

9

Page 195: History A - The Bicester Schoolthebicesterschool.org.uk/wp-content/uploads/2015/09/Mark... · 2015-09-28 · History A Mark Schemes for the Units January 2009 H106/H506/MS/R/09J

Marking Instructions

F966 Maximum mark 120 for this unit. 2 answers: Each maximum mark 60

A01a A01b IA 18-20 36-40

IB 16-17 32-35

II 14-15 28-31

III 12-13 24-27

IV 10-11 20-23

V 8-9 16-19

VI 4-7 8-15

VII 0-3 0-7

Notes:

(i) Allocate marks to the most appropriate level for each AO.

(ii) If several marks are available in a box, work from the top mark down until the best fit has been found.

(iii) Many answers will not fall at the same level for each AO.

(iv) Candidates will demonstrate synoptic skills by drawing together appropriate techniques, knowledge and understanding to evaluate developments over the whole of the period

10

Page 196: History A - The Bicester Schoolthebicesterschool.org.uk/wp-content/uploads/2015/09/Mark... · 2015-09-28 · History A Mark Schemes for the Units January 2009 H106/H506/MS/R/09J

Marking Instructions

AOs AO1a AO1b

Total mark for each question = 60

Recall, select and deploy historical knowledge appropriately, and communicate knowledge and understanding of history in a clear and effective manner.

Demonstrate understanding of the past through explanation, analysis and arriving at substantiated judgements of: - key concepts such as causation, consequence, continuity, change and significance within an historical context; - the relationships between key features and characteristics of the periods studied

Level IA

• Uses a wide range of accurate and relevant evidence • Accurate and confident use of appropriate historical terminology • Answer is clearly structured and coherent; communicates accurately and legibly. 18-20

• Excellent understanding of key concepts (eg. continuity and change) relevant to analysis in their historical context • Excellent synthesis and synoptic assessment • Answer is consistently and relevantly analytical with developed explanations and supported judgements • May make unexpected but substantiated connections over the whole period 36-40

Level IB

• Uses accurate and relevant evidence • Accurate use of a range of appropriate historical terminology • Answer is clearly structured and mostly coherent; communicates accurately and legibly

16-17

• Very good level of understanding of key concepts (eg. continuity and change) in their historical context. • Answer is consistently focused on the question set • Very good level of explanation/ analysis, and provides supported judgements. • Very good synthesis and synoptic assessment of the whole period 32-35

Level II

• Uses mostly accurate and relevant evidence • Generally accurate use of historical terminology • Answer is structured and mostly coherent; writing is legible and communication is generally clear 14-15

• Good level of understanding of key concepts (eg. continuity and change) in their historical context • Good explanation/ analysis but overall judgements may be uneven • Answer is focused on the issues in the question set • Good synthesis and assessment of developments over most of the period 28-31

11

Page 197: History A - The Bicester Schoolthebicesterschool.org.uk/wp-content/uploads/2015/09/Mark... · 2015-09-28 · History A Mark Schemes for the Units January 2009 H106/H506/MS/R/09J

Marking Instructions

AOs

AO1a AO1b

Level III

• Uses relevant evidence but there may be some inaccuracy • Answer includes relevant historical terminology but this may not be extensive or always accurately used • Most of the answer is structured and coherent; writing is legible and communication is generally clear 12-13

• Shows a sound understanding of key concepts, especially continuity and change, in their historical context • Most of the answer is focused on the question set • Answers may be a mixture of analysis and explanation but also description and narrative, but there may also be some uneven overall judgements; OR answers may provide more consistent analysis but the quality will be uneven and its support often general or thin • Answer assesses relevant factors but provides only a limited synthesis of developments over most of the period 24-27

Level IV

• There is deployment of relevant knowledge but level/ accuracy will vary. • Some unclear and/or underdeveloped and/or disorganised sections • Mostly satisfactory level of communication 10-11

• Satisfactory understanding of key concepts (eg. continuity and change) in their historical context • Satisfactory focus on the question set • Answer may be largely descriptive/ narrative of events, and links between this and analytical comments will typically be weak or unexplained • Makes limited synoptic judgements about developments over only part of the period 20-23

Level V

• General and basic historical knowledge but also some irrelevant and inaccurate material • Often unclear and disorganised sections • Adequate level of communication but some weak prose passages 8-9

• General understanding of key concepts (eg. continuity and change) in their historical context • Some understanding of the question but answers may focus on the topic and not address the question set OR provides an answer based on generalisation • Attempts an explanation but often general coupled with assertion, description / narrative • Very little synthesis or analysis and only part(s) of the period will be covered 16-19

12

Page 198: History A - The Bicester Schoolthebicesterschool.org.uk/wp-content/uploads/2015/09/Mark... · 2015-09-28 · History A Mark Schemes for the Units January 2009 H106/H506/MS/R/09J

Marking Instructions

13

AOs

AO1a AO1b

Level VI • Use of relevant evidence will be limited; there will be much irrelevance and inaccuracy • Answers may have little organisation or structure • Weak use of English

4-7

• Very little understanding of key concepts (eg. continuity and change) in their historical context • Limited perhaps brief explanation • Mainly assertion, description / narrative • Some understanding of the topic but not the question’s requirements 8-15

Level VII • Little relevant or accurate Knowledge • Very fragmentary and disorganised response • Very poor use of English and some incoherence 0-3

• Weak understanding of key concepts (eg. continuity and change) in their historical context • No explanation • Assertion, description / narrative predominate • Weak understanding of the topic or of the question’s requirements 0-7

Page 199: History A - The Bicester Schoolthebicesterschool.org.uk/wp-content/uploads/2015/09/Mark... · 2015-09-28 · History A Mark Schemes for the Units January 2009 H106/H506/MS/R/09J

F961 Mark Scheme January 2010

F961 British History Period studies

Option A: Medieval and Early Modern England 1035-1642 From Anglo-Saxon England to Norman England 1035-1087

1 How far was Edward the Confessor’s personality the most important cause of the

problems he faced as king of England?

No set answer is looked for but candidates will need to address the question. Candidates should consider a range of reasons for the problems and at the top levels evaluate their relative importance in causing the problems. It might be said that Edward lacked the strong qualities to make an effective king. His piety and artistic interests were admired but for his ideals rather than their practical relevance to kingship. Better answers may start by identifying the problems that Edward faced and this may include issues such as his lack of knowledge of the country, his upbringing, the power of the Godwin family, the problems created by his marriage to Edith, his support base and the problem of the lack of an heir. It is likely that candidates may suggest that the power of the Godwin family played a large role in causing many of the problems as Edward was heavily dependent on them, some may use their exile to show the power they had. The power of the Godwins may be linked to many of the problems and this may be an approach taken by those reaching the higher levels-for example his upbringing meant that he was even more dependent upon the support of the Godwin’s and therefore it might be argued that his marriage to Edith was almost inevitable and that this created further problems and may even have led to the succession crisis at the end of his reign. There may be some consideration of the problems that followed from his continuing patronage of Normans. There might be mention of foreign dangers, especially from Scandinavia. A king’s powers were limited and he needed to be able to implement whatever authority he possessed.

2 ‘Military factors were the most important reason for William of Normandy’s success

at the Battle of Hastings.’ How far do you agree?

No set answer is looked for but candidates will need to address the question. Military factors is a wide ranging term and might include issues such as tactics, forces and weaponry available, military leadership and the previous invasion by Harald. If candidates use the term to encompass all of these they may find it difficult to consider other issues, however issues such as luck for William or misfortune for Harold may be considered as candidates might point to the timing of Harald’s invasion and the impact it had on Harold, particularly following the changing direction of the wind, which allowed William to invade. Some may consider the mistakes made by Harold as more important, suggesting that if he had not rushed back from the north and waited until he had rested and had a full force he might have won, given how close Hastings was, even with such a depleted force. Some answers might also consider religious factors and argue that it was only with papal blessing that William was able to gather a large enough force to be able to make the challenge.

14

Page 200: History A - The Bicester Schoolthebicesterschool.org.uk/wp-content/uploads/2015/09/Mark... · 2015-09-28 · History A Mark Schemes for the Units January 2009 H106/H506/MS/R/09J

F961 Mark Scheme January 2010

3 To what extent did William I change the government and administration of England?

No set answer is looked for but candidates will need to address the question. There is a wide range of material available for candidates to consider. Some may consider the issue of personnel and the fate of the Anglo-Saxon earls and their replacement by Normans. However, some answers may focus on the nature and methods of government and this may result in consideration of the use of the feudal system, but it must be linked to methods of government. There may be some consideration of the nature of the monarchy and candidates might consider the use made of crown wearing sessions. The personal rule of the monarchy became more important. Writs were used, a legacy of Anglo-Saxon government, although they were not usually in English and they were used more frequently to enforce William’s orders. Sheriffs and shire courts were continued but sheriffs were evidently more important as royal officials.

15

Page 201: History A - The Bicester Schoolthebicesterschool.org.uk/wp-content/uploads/2015/09/Mark... · 2015-09-28 · History A Mark Schemes for the Units January 2009 H106/H506/MS/R/09J

F961 Mark Scheme January 2010

Lancastrians, Yorkists and Tudors 1450-1509 4 How successful was Richard III’s government of England?

No set answer is looked for but candidates will need to address the question. Some may argue that Richard was not successful in his government of England because he lost at Bosworth, however this point needs to be fully explained if it is to be relevant. Candidates could point to his failure to win the support of the nobility and that his government was over-reliant on northern nobility at the expense of the southerners. Answers may also consider his relationship with parliament and the issue of finances, this may lead to a discussion of benevolences and candidates might discuss whether his approach was successful. Some may argue that his government was not successful as he did not possess a wide enough basis of power because of the nature of his accession and therefore lacked sufficient patronage. There may be some consideration of the nature of his accession, but this needs to be linked to the question.

5 How effectively did Henry VII deal with England’s domestic problems?

No set answer is looked for but candidates will need to address the question. It is likely that better answers will identify the domestic problems that Henry faced and then consider how well he was able to deal with them. Many answers are likely to focus on the problem of the Yorkist challenge, particularly Simnel and Warbeck. Candidates may argue that these were dealt with successfully as both were defeated and some may also argue that the threat was reduced by his marriage to Elizabeth of York. Candidates may also consider the problem of the nobility, some may conclude that Henry’s policy was successful as he prevented the emergence of over mighty subjects and through his policy of bonds and recognisances was able to reduce their power, but at the same time win loyalty through such methods as the Order of the Garter. However, others might argue that his last years were so oppressive that the country was close to civil war. The question of the succession might also be discussed and some may argue that initially this was successful with Arthur and Henry, but that Arthur’s death left the succession hanging by a thread. There might be some consideration of the financial problems and how well they were solved.

6 ‘Marriage agreements were the most important achievement of Henry VII’s foreign

policy.’ How far do you agree?

No set answer is looked for but candidates will need to address the question. Candidates should consider a range of achievements and evaluate their relative importance in order to access the higher levels. Some may argue that the marriages were important as they brought him European recognition, which was important because of his weak claim, and also gave him an ally with the most powerful nation-Spain. It might also be argued that the marriage of Margaret to James brought at least short term peace with Scotland and also removed the potential threat of Warbeck, which was important to Henry. However, others may consider that his most important achievement was achieving security from the Yorkist threat, although it might be argued that this was only achieved at the end of the period. There might be some consideration of how successful he was in dealing with the threat presented by Margaret of Burgundy. Some might argue that financial gain was the most important achievement given the nature of his finances and use the French pension to support this and also the development and protection of trade, although the latter issue can be debated.

16

Page 202: History A - The Bicester Schoolthebicesterschool.org.uk/wp-content/uploads/2015/09/Mark... · 2015-09-28 · History A Mark Schemes for the Units January 2009 H106/H506/MS/R/09J

F961 Mark Scheme January 2010

Henry VIII to Mary I 1509-1558 7 How successful were Wolsey’s domestic policies?

No set answer is looked for but candidates will need to address the question. There is a wide range of domestic policies that candidates might consider and it is not expected that candidates should cover all; it is the quality of analysis that matters. However, candidates should cover a range of areas and this might include legal, financial, social and economic aspects and the church. Some candidates might establish criteria against which to judge success and this could include pleasing Henry so as to remain in power, gaining personal wealth and prestige or improving the government of the country. It is possible that candidates will argue that his legal reforms were the most successful and point to the increase in cases and the availability of justice for all. However, it is possible to argue that his financial reforms were successful, particularly in the early years and candidates may use the example of the subsidy to support this and the funding of Henry’s foreign policy, which won Wolsey support. However, if this line is taken it can be balanced by consideration of the Amicable Grant. In discussing social and economic policies candidates might focus on the issue of enclosure and argue that in the short term it appeared to be successful, but had to be abandoned because of financial needs. The problem of the church may figure in some essays and although some might point to his success in dissolving some monasteries others might argue that, given the power he had, this was a missed opportunity and that he even brought the church into disrepute. There might be some consideration of the divorce and his failure and the consequences, but this should not dominate the answer.

8 How far did Tudor government and administration change in the 1530s?

No set answer is looked for but candidates will need to address the question. There is a wide range of issues that candidates might consider and it should not be expected that all will be addressed, what matters is the quality of analysis, although examiners should expect to see a range. There may be consideration of the changing role and regularity of parliament and its increased competence as it became involved in religious issues and some might raise the issue of the importance of statute law or point to Henry’s comment about power in the time of parliament. There might be some consideration of the financial courts that were established, although it should be noted that most were short-lived. Candidates might consider the issue of Wales and the Act of Union of 1536, with the establishment of the county system etc. Some answers might raise the Elton ‘Tudor Revolution’ debate, but this is not to be expected as historiography is not a requirement at AS and examiners should also be aware of answers that simply describe the Elton thesis and do not use it to answer the question.

9 How effectively did the governments of Edward VI and Mary I deal with unrest?

No set answer is looked for but candidates will need to address the question. Candidates understanding of the idea of unrest may be a determining factor in the quality of the answer. It is likely that many answers will focus solely on the rebellions of the period: Western, Kett and Wyatt, although the Lady Jane Grey affair may also receive mention. Candidates may argue that Mary was more successful as Wyatt was defeated without battle, whereas the unrest of 1549 was at least a contributory factor in the downfall of Somerset. However, they may also argue that ultimately both the Western and Kett were crushed. Some answers may take a broader approach and consider economic and social issues, such as vagrancy or the problems created by the collapse of the cloth trade and this is acceptable.

17

Page 203: History A - The Bicester Schoolthebicesterschool.org.uk/wp-content/uploads/2015/09/Mark... · 2015-09-28 · History A Mark Schemes for the Units January 2009 H106/H506/MS/R/09J

F961 Mark Scheme January 2010

Church and State 1529-1589 10 How important was Thomas Cromwell in influencing religious policy in the 1530s?

No set answer is looked for but candidates will need to address the question. There are a large number of factors that candidates may consider, but it is not expected that they will consider all issues, even at the highest level. There should be some consideration of Cromwell’s role and some may argue that his more radical beliefs were important in influencing the king to dissolve the monasteries and introduce some more protestant views. However, this might be balanced against Cromwell’s desire and need to please the king, arguing that he dissolved the monasteries to make Henry’ the richest man in Christendom’. It may also be argued that Cromwell’s views were not important as once he displeased Henry and became too radical he was removed. Candidates might suggest there were other more important factors and issues such as power, money, the foreign situation and threat of a Catholic crusade and Henry’s own religious beliefs might be considered. There might also be some who argue that the condition of the church and the need to reform in response to popular pressure was important.

11 How much support was there for Protestantism in England in 1558 and 1559? No set answer is looked for but candidates will need to address the question. The focus of 1558 and 1559 is important and candidates could therefore discuss the problems Elizabeth faced in the passing of the Elizabethan Settlement in Parliament. Candidates are also likely to look at the level of support for Catholicism and/or Protestantism at the end of Mary’s reign. Some may argue that the country was largely Catholic on Mary’s death, whilst others may argue that the burnings and Marian persecution had turned England protestant, depending on their argument so they will determine the strength of Protestantism in England on Elizabeth’s accession. Candidates whose answers range back into Edward’s reign and either suggest that it was difficult for Edward to turn England protestant or that he succeeded and therefore Elizabeth had a harder/easier task can receive credit, but the focus of the essay must be on the situation in 1559.

12 How successfully did Elizabeth I deal with the Catholic challenge from 1559 to 1589? No set answer is looked for but candidates will need to address the question. Candidates may identify the nature of the Catholic challenge and consider how it changes over the period. Answers may look at the threat from home and abroad and suggest that at the start of the period it was the threat from home that was the strongest, given the strength of Catholicism. However, they may argue that Elizabeth handled this well; there was no serious unrest, the moderate nature of the settlement and her avoidance of creating martyrs. Some answers may also consider how well she handled the Catholic challenge of the Northern Earls. There may also be consideration of her policy towards seminary priests and again the avoidance of creating martyrs, but executing for treason. The handling of the foreign threat might include her ambivalent policy in the early years, although some may argue that Philip needed her support just as much. There may be some who argue that her policy towards the end of the period was less successful as she provoked war with Spain and this could have led to Catholics at home rising.

18

Page 204: History A - The Bicester Schoolthebicesterschool.org.uk/wp-content/uploads/2015/09/Mark... · 2015-09-28 · History A Mark Schemes for the Units January 2009 H106/H506/MS/R/09J

F961 Mark Scheme January 2010

England under Elizabeth I 1558-1603 13 How important was the Privy Council in the government of England during the reign

of Elizabeth I?

No set answer is looked for but candidates will need to address the question. Candidates will need to consider the role of the Privy Council in the government of England, but this should be weighed up against other elements of government in order to reach a judgement about its relative importance. It is likely that many will write in greater depth about the role of parliament and some may be sidetracked in to the debate about parliament. Better answers might also consider local government and the role of JPs etc.

14 ‘Inflation was the most serious financial problem facing Elizabeth I and her

government’. How far do you agree?

No set answer is looked for but candidates will need to address the question. There is a range of financial problems that candidates can consider, but they must give due attention to the named factor, even if they argue that it was not the most important. Inflation had a major impact on crown revenue and impacted on the cost of warfare, which would be a major item of expenditure at the end of the period. Inflation also had an impact on taxation returns, although some may suggest that it was Elizabeth’s failure to update assessments that was the bigger problem. Some may consider the problem of crown expenditure and selling of crown lands, others may look at customs or monopolies as issues.

15 How successful was Elizabeth I in dealing with the issue of the succession during

her reign?

No set answer is looked for but candidates will need to address the question. The issue of the succession concerned many, but Elizabeth did not want the matter discussed by parliament and did not want to name a successor. Candidates might argue that parliament did try to discuss the issue, but were largely unsuccessful in getting any answer from Elizabeth. Some answers might argue that Elizabeth handled the situation very well, given the fact she was seen as illegitimate by some. They may point to her handling of the issue of Mary Queen of Scots, who was the potential heir, but by not naming her it discouraged attempts to hasten her accession. Elizabeth was also masterful in exploiting her position as the ‘Virgin Queen’ and candidates might consider the various marriage proposals and how well they were handled and exploited by her. There might be some consideration of the last years and the position of James VI.

19

Page 205: History A - The Bicester Schoolthebicesterschool.org.uk/wp-content/uploads/2015/09/Mark... · 2015-09-28 · History A Mark Schemes for the Units January 2009 H106/H506/MS/R/09J

F961 Mark Scheme January 2010

The Early Stuarts and the Origins of the Civil War 1603-1642 16 Assess the reasons why financial issues caused conflict between James I and his

parliaments.

No set answer is looked for but candidates will need to address the question. There is a range of issues that candidates can consider and it is not expected that all areas will be covered, what is important is the quality of analysis. Some answers may identify the financial problems that James faced, such as the inherited debt and the inadequacy of royal finances and suggest that it was the scale of the problem that was the major issue. However, others may suggest that it was James’ extravagance, particularly money spent at court or on royal favourites that caused conflict. There may be some consideration of foreign policy and the differing views of James and parliament and this can be linked to financial clashes. Some answers might argue that parliament used the issue of supply to try to obtain redress of grievances, whilst others may suggest that there was a lack of trust between the two, shown in the failure of the Great Contract. Issues such as monopolies and impositions may also receive consideration.

17 ‘Charles I desire for financial independence from parliament was the most important

reason for the establishment of personal rule in 1629’. How far do you agree?

No set answer is looked for but candidates will need to address the question. The focus of the question should be on the establishment of Personal rule and candidates who write about the nature of rule should not receive high credit. Candidates will need to focus on Charles’ aims and problems in the period from 1625 to 1629 to be able to address fully the demands of the question. Answers may consider the problematic relationship between Charles and his parliaments in this period and suggest that he wanted political independence and link this to his belief in Divine Right or even suggest that parliament was not a permanent part of the constitution. Some answers will look at other areas of conflict, such as foreign affairs or his relationship with Buckingham and again may argue that Charles wanted to avoid criticism and prevent parliament from linking supply to redress of grievance. Charles’ attitude towards parliament and his view of their role is also an area that might be considered.

18 To what extent was Charles I personal rule the most important cause of the outbreak

of Civil War in England in 1642?

No set answer is looked for but candidates will need to address the question. The question invites candidates to weigh up a range of factors causing the civil war. Those who focus on personal rule may consider the impact of the financial and religious policies of Charles and the growing opposition generated by polices such as Thorough and the fear it created. However, this may be balanced against the lack of a united opposition or the lack of a royalist party, suggesting that at this stage there could not be a war. Candidates who argue that the causes were short term will focus on developments during the period 1640-2. Some may argue that the war was unlikely in the summer of 1641 as Charles had compromised, others may suggest that the Grand Remonstrance was the turning point, others may suggest it was attempted arrest of the Five MP’s, whilst others may suggest it was either parliament taking control of the army or the Nineteen Propositions. There is a great deal that candidates could consider and it is not expected that all issues will be looked at, what matters is the quality of analysis.

20

Page 206: History A - The Bicester Schoolthebicesterschool.org.uk/wp-content/uploads/2015/09/Mark... · 2015-09-28 · History A Mark Schemes for the Units January 2009 H106/H506/MS/R/09J

F961 Mark Scheme January 2010

Option B: Modern 1783-1994 From Pitt to Peel 1783-1846

1 To what extent was the support of the crown the most important reason for Pitt’s domination of politics from 1783 to 1793?

No set answer is looked for but candidates will need to address the question. Candidates will need to write a good paragraph on the named factor if they are to access the higher levels, even if they conclude that it was not the most important reason. Candidates will need to assess the role of George III in ensuring the survival of Pitt’s ministry, particularly in the 1784 election and the Regency Crisis of 1788 and then attempt to evaluate this against other factors if they are to achieve the higher levels. The king played an important role through patronage, control of the frequency of elections and his own distaste for the Whigs, particularly Fox and North. The partnership between Pitt and George should not be understated. It might also be noted that once the king withdrew support Pitt soon fell. Other factors for survival could include Pitt’s successful domestic policy in the 1780s, which saw economic and financial recovery and his use of repression in the 1790s, the division and weakness of the Whig opposition, made worse by splits over the French Revolution and Pitt’s mastery of parliamentary business and debate.

2 How far would you agree that the Conservative party was more liberal from 1822 to 1830 than from 1812 to 1822?

No set answer is looked for but candidates will need to address the question. The question of how liberal the Tories were in the period after 1822 remains an open one, but most are likely to argue that they were more liberal after 1822 than before. To support the view candidates could use: the appointment of younger politicians such as Canning, Peel and Huskinsson after 1822-3, the economic legislation passed by both Robinson and Huskinsson and the reforms of Peel at the Home Office. However candidates might argue that there were also some illiberal measures such as the refusal to accept Roman Catholic Emancipation, the fact that the repeal of the Test and Corporation Acts were forced upon them, and the refusal to entertain the issue of parliamentary reform. These issues should be contrasted with the measure of the earlier period in order to reach a conclusion. Candidates are likely to argue that even if the later period was not that liberal, the earlier period was repressive and point to the Corn Laws, the Suspension of Habeas Corpus, the Six Acts and Seditious Meetings. However, this might be balanced against the need to tackle unrest and that the acts were no more repressive than Pitts. There might also be some consideration of the more liberal reforms of the earlier period.

3 How successful was Peel’s leadership of the Conservative party to 1846?

No set answer is looked for but candidates will need to address the question. Answers must focus on Peel as the party leader and not simply examine his reforms in the ministry of 1841-6, although some of these might be used to show he was not a great party leader. Some may argue that in the early years he was a good party leader as he reorganised the party after the disasters of the Great Reform Act and with his Tamworth Manifesto adapted the party to a changed set of electoral conditions. There might also be consideration of the reforms at the centre with reorganisation, the creation of the Carlton Club and Registration issues. Peel’s attempts to broaden the appeal of the party might also be discussed and candidates might be aware that the 1841 election results suggest he failed in this aspect and was returned to power on traditional Tory votes. There might be some discussion about the significance of the 100 days and also his support for some Whig measures to argue that he had shown the party was responsible and fit to govern. It is likely that many will consider his treatment of backbenchers once he was in power and his belief that it was his duty to serve the nation and monarch not the party. This might result in some discussion of his abandonment of key Tory ideas over protection and issues in Ireland. Many are likely to suggest that his action over the Corn Laws suggests he was a poor party leader as he split the party and the result was years in the political wilderness. Some might argue that it was not Peel’s successes that brought the Conservatives back into power but the mistakes and failings of the Whigs

21

Page 207: History A - The Bicester Schoolthebicesterschool.org.uk/wp-content/uploads/2015/09/Mark... · 2015-09-28 · History A Mark Schemes for the Units January 2009 H106/H506/MS/R/09J

F961 Mark Scheme January 2010

Liberals and Conservatives 1846-1895 4 How important was the influence of Gladstone in the emergence of the Liberal party

by 1868?

No set answer is looked for but candidates will need to address the question. Candidates are likely to focus on how Gladstone contributed to the key issues in Liberal development such as party development, winning elections, party cohesion and winning key electoral groups. Gladstone was able to identify the party with an attractive financial and economic package of low taxation and free trade in his budgets. Some may argue that it was Palmerston who founded the Liberal party in 1859 over the Italian principle and also note that Gladstone was absent from the Willis Rooms meeting. However, some might argue that Gladstone was important as he created a liberal press by his repeal of the Paper Duties; he made contacts with the Trade Unions and forged contacts with the radicals. These issues will be balanced against other factors, such as the split in the Conservative party on the role of the Peelites. There might be consideration of the roles of radicals such as Bright and Cobden, unity over Italy, common support for free trade, trust in the party over finance and the abandonment of the aristocratic Whiggish image associated with Grey and Melbourne.

5 ‘The loss of working class support was the most important reason for the defeat of

the Liberal party in the 1874 election.’ How far do you agree?

No set answer is looked for but candidates will need to address the question. Candidates will need to write a good paragraph on the named factor if they are to access the higher levels, even if they conclude that it was not the most important reason. Candidates will need to explain how, despite a generally good press for Gladstone’s legislative achievements, he lost the 1874 election. They must consider the loss of working class support, even if they conclude it was the not the most important reason. In looking at this factor they might include artisan and working class disapproval of Trade Union legislation that was repressive on peaceful picketing and acts in restraint of trade as well as resentment over the Licensing Act. They may assess the fall out from many of his reforms – The Whig upper class was unhappy over the Irish reform, particularly land, they also disliked the Abolition of the Purchase of Commissions in the Army and exams in the Civil Service. They might consider the importance of the non-conformists, unhappy with Forster’s Education Act which implicitly sided with the Anglicans. The administrative reforms and increased efficiency, which the government achieved, would hardly bring votes. Candidates might also consider the impact of an ‘apparently weak foreign policy’, Gladstone losing steam and a faltering leadership in 1873/4 and the failure to find a rallying cry beyond income tax.

22

Page 208: History A - The Bicester Schoolthebicesterschool.org.uk/wp-content/uploads/2015/09/Mark... · 2015-09-28 · History A Mark Schemes for the Units January 2009 H106/H506/MS/R/09J

F961 Mark Scheme January 2010

6 To what extent did Disraeli’s ministries of 1867 and 1874 - 1880 follow the ideas of Tory democracy?

No set answer is looked for but candidates will need to address the question. Candidates will need to show an understanding of the concept of Tory democracy – knitting up the social divisions between rich and poor by paternalistic social reform, cementing an upper and working class bond at the expense of the middle classes. In discussing this issue candidates might consider the Second Reform Act. There might be some mention of the origins of Tory democracy in his novels and Young England to suggest that he was likely to follow such a policy, although some may argue these were simply idealistic. In order to support the argument candidates might make reference to his speeches at Crystal Palace and Manchester, from the reforms themselves, Artisan Dwellings Act, Trade Union Act, Public Health Act, Food and Drink, Pollution and Merchant Shipping. In considering these acts candidates might discuss their intentions and practice to show whether they did uphold the ideals. Against the argument candidates might suggest that such reforms were already in the pipeline, that they built on existing practice, they simply followed a liberal framework, that developments in technology made such developments possible, that their impact was limited and that Tory Democracy was a later phrase which made little sense electorally.

23

Page 209: History A - The Bicester Schoolthebicesterschool.org.uk/wp-content/uploads/2015/09/Mark... · 2015-09-28 · History A Mark Schemes for the Units January 2009 H106/H506/MS/R/09J

F961 Mark Scheme January 2010

Foreign and Imperial Policies 1856-1914 7 To what extent was the maintenance of the balance of power the most important

factor influencing British foreign policy from 1856 to 1902?

No set answer is looked for but candidates will need to address the question. Candidates will need to write a good paragraph on the named factor if they are to access the higher levels, even if they conclude that it was not the most important reason. There are a number of factors that candidates might consider when addressing the question and examiners should not expect them all to be discussed, what is important is the quality of analysis. Some might argue that although the balance of power was an important issue the countries that threatened it changed from Russia to Germany. This issue might also be linked to trade and the need to preserve trade routes, particularly to India and how this impacted on relations with Russia. This might also be linked to imperial concerns over India and therefore the issue of the Mediterranean and the Suez Canal might feature. Issues of naval supremacy and ‘blue water’ might also be considered and the question of the two power principle. There might be some consideration of the importance of support for nationalist movements and Britain’s relationship with Italy, Germany and Poland in this period might be considered.

8 How far did support for imperialism decline from 1880 to 1902?

No set answer is looked for but candidates will need to address the question. Examiners need to ensure that candidates focus on ‘how far’ and do not simply assume that it did decline and assess ‘why’. Some answers might challenge the assumption that it did decline and point to the positive images of the ultimate victory or the result of the Khaki election. It is possible to argue that victory in the Boer War in 1902 only reinforced an image of invincibility and there was no decline and this can be seen in the popular jingoism of the music hall and added to the ideal that the ‘sun never sets ‘ on the British Empire. They might also point to the celebrations of Queen Victoria. However, this might be balanced against the negative reaction to the Boer War and some may suggest that it was a turning point as a small force had caused such difficulty for the might of the Empire; mention might also be made of the brutal use of concentration camps that damaged the reputation of imperialism. This can be supported by reference to the questioning of the wisdom of Chamberlain’s imperial vision and the social and medical problems highlighted by the Boer War caused some to argue that Britain should concentrate on domestic reform.

9 Assess the reasons why Britain’s attitude to major European powers changed from

1902.

No set answer is looked for but candidates will need to address the question. Candidates might argue that this was due to the resolution of areas of dispute with France, particularly in colonial issues following the Fashoda incident. They may argue that this led to the Entente Cordiale and some might develop this and suggest that relations with Germany changed following the Entente as Germany feared what might have been agreed. However, some candidates might suggest that it was the growing power and fear of Germany that caused Britain’s attitude to change; there might be reference to the development of the German navy, her economic growth or support for the Boers. Some candidates might explain how these developments, particularly after the Anglo Japanese alliance of 1902 encouraged an improvement in relations with Russia. There might be some candidates who argue that Britain’s attitude to European countries did not change and argue that the Entente did not commit Britain to war, that the agreements made followed on from the earlier Mediterranean agreements or that Britain still followed a policy of splendid isolation: this approach is valid and should be credited accordingly. Some might argue that attitudes did not change and that Britain was just concerned to avoid war and that it was the means that changed.

24

Page 210: History A - The Bicester Schoolthebicesterschool.org.uk/wp-content/uploads/2015/09/Mark... · 2015-09-28 · History A Mark Schemes for the Units January 2009 H106/H506/MS/R/09J

F961 Mark Scheme January 2010

Domestic Issues 1918-1951 10 How successful was the Conservative party from 1918 to 1929?

No set answer is looked for but candidates will need to address the question. Some candidates might argue that the party was very successful and point to the electoral recovery after 1906. It can be argued that they dominated the period electorally: the Coupon election and resulted in Conservative dominance, they won in 1922, were still the largest party in 1924 and won the subsequent election. However, some might balance this against the importance of Lloyd George in their victory of 1918, the party ‘split’ in 1924 and the subsequent Second Eleven Cabinet and the success of Labour in 1929. Some might debate how well the Conservatives handled the major issues such as the General Strike and social issues or they might consider whether Baldwin was successful in creating a new Conservatism, which resulted in electoral dominance throughout this period and up to 1945. It might be argued that Baldwin was able to heal the divides in society that had been created by the General Strike.

11 ‘Poor Trade Union leadership was the most important reason for the failure of the General Strike.’ How far do you agree? No set answer is looked for but candidates will need to address the question. There are a number of reasons that candidates might assess in order to decide the most important reason for failure. Candidates will need to write a good paragraph on the named factor if they are to access the higher levels, even if they conclude that it was not the most important reason. They might argue that the Unions were reluctant to embark on a General Strike and did so in light of their apparent weak actions on Black Friday. They might link this to the shortness of the General Strike and why the miners were abandoned so quickly. This might lead them to argue that they were pushed into it by the Coal Unions. On the other hand some might argue that the government was well prepared for the strike, having stockpiled coal and were aided by the timing. They might also point to the government’s ability to win the propaganda war and the role of Churchill and the British Gazette in this. Some might consider the reaction of a section of the public who were willing to help and enjoyed the opportunities the strike presented.

12 How successful were the Labour governments’ reforms of 1945-51 in improving social and economic conditions? No set answer is looked for but candidates will need to address the question. The approach to the question might depend on how success is judged; is it seen against the desires of many committed socialists or against the problems and difficulties they faced. It might be argued that Attlee’s government accomplished a modest redistribution of wealth through fiscal policy and therefore improved social conditions, there was also full employment and improved living standards for the working class this could be contrasted with the feeling of relative deprivation among the middle class. It might be argued that they suffered more austerity with food shortages and rationing than during the war. In considering social conditions it is likely that many will focus on issues such as the NHS and education opportunities. In considering social conditions candidates might refer to the National Insurance Act, the industrial Injuries Act and National Assistance Act and Family Allowance Act. Some historians have argued that these measures were so significant that they should be seen as achieving a social revolution, suggesting they were a success. There might also be a discussion as to whether the nationalisation programme was successful in improving economic conditions. However, this might be balanced against the economic problems they faced such as debts and argue that even though there was some recovery it was not enough to meet expectations and link this to the dollar gap and the defence spending associated with the Cold War. Some may argue that the government lacked any carefully thought-out plans of social reconstruction.

25

Page 211: History A - The Bicester Schoolthebicesterschool.org.uk/wp-content/uploads/2015/09/Mark... · 2015-09-28 · History A Mark Schemes for the Units January 2009 H106/H506/MS/R/09J

F961 Mark Scheme January 2010

Foreign and Imperial Policies 1945-1990 13 How far did British foreign policy change from 1945 to 1964?

No set answer is looked for but candidates will need to address the question. There are a large number of areas and issues that candidates might consider, but it is not expected that they will deal with all, what matters is the quality of the analysis. Candidates might choose to approach this by looking at either themes or their relationship with individual countries or the EEC. If they take the latter approach it is likely that they will focus on Britain’s relationship with the USA and USSR and this might be linked to the issues of the Cold War. Some might argue that there was a significant change after the war because of Britain’s financial position and point to the change seen in Greece. The development of a close relationship with the USA might be stressed, although some might argue that after Suez this did see a shift. The desire to remain a great power remained a constant and answers might mention Britain’s place on the Security Council and the desire for an independent nuclear deterrent.

14 How far was the decline of the British Empire the most important reason for the change in British attitudes towards Europe from 1945 to 1973? No set answer is looked for but candidates will need to address the question. Candidates are likely to consider a number of reasons, but in order to access the higher levels they must write at least a good paragraph on the given factor, even if they conclude it was not the most important. Candidates may consider the perception that Britain had to make a choice between the Empire and Europe and with ‘the wind of change’ shift to de-colonisation altered Conservative views. There might also be consideration of the perception that Britain had to make a choice between USA and Europe; this might also be linked to the new direction under Eden and the application to join the EEC in 1963. There might be some consideration of Heath’s attitudes. Some answers might consider the economic success of the EEC and compare this with the failure of EFTA. This might be compared with Britain’s go it alone attitude up to 1960. Many in Britain did not take European integration seriously until 1960, pointing to Britain’s world status and her desire to shape Europe rather than be shaped by it. This might be linked to distrust of the Schumann Plan and EEC.

15 To what extent did Thatcher achieve her aims in foreign policy? No set answer is looked for but candidates will need to address the question. It is likely that candidates will identify Thatcher’s aims before assessing how far she achieved them. There might be some consideration of her desire to reassert Britain’s position as a major power and this might be linked to the Falklands War, although some might balance this with the Grenada incident. There is a case that Thatcher wanted to improve relations with the US and that this was achieved through events such as the Libya bombings. Answers may discuss her aims regarding the EU, particularly the question of the budget and are likely to conclude that she achieved her aims, even if some have argued that her stance damaged Britain’s position in Europe. Thatcher wanted to see the defeat of communism and it is likely that some will argue that this was achieved and that her role was of some significance. There might be some consideration of the Rhodesian question where it might be argued that Thatcher achieved her aim of improving Britain’s position in the Commonwealth.

26

Page 212: History A - The Bicester Schoolthebicesterschool.org.uk/wp-content/uploads/2015/09/Mark... · 2015-09-28 · History A Mark Schemes for the Units January 2009 H106/H506/MS/R/09J

F961 Mark Scheme January 2010

Post-War Britain 1951-1994 16 ‘Labour weakness was the most important reason for Conservative dominance from

1951 to 1964’. How far do you agree?

No set answer is looked for but candidates will need to address the question. There are a large number of reasons that candidates might consider, but it is not expected that they will cover all areas, what matters is the quality of analysis. However, in order to achieve the higher levels they must consider the named factor even if they conclude it was not the most important. Candidates will need to identify the weakness of Labour opposition during the period. The weakness of Labour opposition in this period was focused on the left versus right wing debate over the future of the party. Modernisers or revisionists under Gaitskell wished to increase the private sector involvement and remove Clause 4. They were opposed by traditionalists under Bevan who wished to expand public sector involvement and oppose nuclear weapons. The divisions did not go down well with the electorate. However, it is likely that this will be contrasted with the positive elements of Conservative rule. It might be argued that prosperity made Labour disputes appear petty. Economic recovery was underway and the Conservatives were able to dismantle the apparatus of austerity and gain the credit. Politics were devised to manage this by Butler, Maudling, Powell and MacLeod. A property owning democracy had more appeal than Bevan’s expanded public sector or Gaitskell’s social democracy. The Conservatives were able to reduce taxes yet maintain and even increase social expenditure, completing the promised ‘homes’ programme. Full employment spread the gain more widely and affluence became more marked. The Conservatives timed elections well to coincide with ‘boom’ and avoided moments of disaster such as Suez or Profumo.

17 How successful was Heath as leader of the Conservative party? No set answer is looked for but candidates will need to address the question. Candidates may argue that Heath was a failure as leader and point to his defeat in 1974, when he called an election despite a large majority, and subsequent loss of the leadership of the party to Thatcher. He failed in nearly of all his aims: reducing the price rise, increasing productivity and reducing unemployment. In particular candidates might discuss his failure to deal with industrial relations, which led to his eventual downfall. It resulted in conflict with the miners and his misjudgement in calling an early election resulted in defeat. Some might argue that he failed to carry through his tough programme of economic and industrial reform on which the party had won the election and therefore should be judged to have failed and these were the grounds of criticism from the Thatcherite wing of the party, even their emergence might be used to argue he failed. There might be consideration of the number of U turns he made and the weak image that this created among the electorate. However, some might argue that he should not be judged as failing in his leadership, but these problems need to be seen in the context of the extraordinary and unstable domestic and international problems with which he was faced.

27

Page 213: History A - The Bicester Schoolthebicesterschool.org.uk/wp-content/uploads/2015/09/Mark... · 2015-09-28 · History A Mark Schemes for the Units January 2009 H106/H506/MS/R/09J

F961 Mark Scheme January 2010

28

18 Assess the reasons why governments were unable to solve the Irish problem in the period to 1994. No set answer is looked for but candidates will need to address the question. Answers may start by identifying the problems faced by the governments and this might include issues such as the divisions, the growth of terrorism, economic discrimination, the use of violence, the development of paramilitary groups and perceptions of the police and army. Although the topic starts in 1951 some answers might place the problems in context of earlier developments and this can be credited provided the main focus is on the period from 1951. Candidates might consider the problem of the division between Nationalists and Loyalists and the emergence of more extreme forms within the period which resorted to more violent approaches. The sizeable proportion of the population, about 1/3, who felt resentment against the government made the problems more difficult to resolve. Candidates might examine the importance of the events of 1968-9 in exacerbating the problems and they might also argue that the issues had largely been ignored until then and that this had matters worse. There might be an examination of the issues of discrimination in policing, social and economic areas and the anti-Catholic nature of the Unionist majority that added to the difficulties and this might be linked to the problem of a Unionist dominated government. Answers are likely to consider the role of the IRA and the emergence of the Provisional IRA following the split in 1969. Attitudes towards the British army among many Catholics did not make the situation any easier. Policies such as internment may also have exacerbated the problem and this was added to by events such as Bloody Sunday. There might be an exploration as to why the Sunningdale Agreement and power sharing failed and this might include the change in government and a lack of decisive leadership. There might also be some consideration of international support for terrorism and reference made to the hunger strikes of the 1980s. Some candidates might also examine the problem of relationship between Britain and the Irish government, particularly when Fianna Fail was in power, particularly during the Falklands crisis. Candidates might also make reference to the political success of Sinn Fein, the divisions within the Unionist movement and the limited support among both communities for a lasting peace.

Page 214: History A - The Bicester Schoolthebicesterschool.org.uk/wp-content/uploads/2015/09/Mark... · 2015-09-28 · History A Mark Schemes for the Units January 2009 H106/H506/MS/R/09J

F962 Mark Scheme January 2010

F962 European and World History Period Studies

Option A: Medieval and Early Modern 1095-1609 The Crusades and Crusader States 1095-1192 1 Assess the reasons why people joined the First Crusade.

No set answer is looked for but candidates will need to address the question. This question seeks to elicit responses which assess the relative significance of the different motives that people had for joining the First Crusade. There needs to be real assessment for the top bands. Candidates may discuss religious motivation in some detail and focus in on the crusade as an ‘armed pilgrimage’ and the plenary indulgence that Pope Urban II promised crusaders. Such discussion may be balanced against other motives such as those that can be inferred from the reports of Urban’s sermon at Clermont: revenge for the atrocities committed against eastern Christians by the Turks; aid to Christians in the East; the chance of ‘righteous’ warfare; the recovery of the Holy Land (and the focus on Jerusalem that emerged as a key factor as the crusade recruitment campaign got underway). Candidates may also suggest more worldly motives: the prospect of a land of milk and honey, an escape from the hardships of life in western Christendom, the chance to carve out reputations and lands as a result of victory and conquest. Candidates may discuss motivation in relation to general groups as well as particular individuals.

2 To what extent was shortage of manpower the main problem facing the Crusader

States in the twelfth century?

No set answer is looked for but candidates will need to address the question. Candidates must deal with the question of shortage of manpower even if they wish to argue other problems were more significant. Candidates may discuss the chronic nature of the manpower problem from which the erratic and sporadic arrival of ‘crusaders’ from Western Europe at best provided temporary alleviation. Even when there were major crusades, most crusaders saw their sojourn in the Holy Land as temporary. The appeals for western aid can be used as evidence of the manpower shortage. The problem of controlling the fragile crusader states with limited manpower was one which faced every ruler in every state. Candidates may discuss the role of the military orders, the strategy of establishing defensible strongpoints manned by relatively small numbers of knights and men, the relatively small size of the total forces available to the rulers of the states, even when they came together and so on. Such discussion needs to be balanced against other problems and candidates may discuss some of the following: the divided natures of the Christian states (even accepting the nominal sovereignty of the King of Jerusalem); the long and indefensible frontiers; the hostility (and growing unity) of neighbouring states; the rivalries between states; the problems associated with governance; the rivalries between different rulers and within states between different factions; the problems associated with succession crises and so on.

29

Page 215: History A - The Bicester Schoolthebicesterschool.org.uk/wp-content/uploads/2015/09/Mark... · 2015-09-28 · History A Mark Schemes for the Units January 2009 H106/H506/MS/R/09J

F962 Mark Scheme January 2010

3 To what extent was the rivalry between Richard I of England and Philip II of France the main reason for the limited success of the Third Crusade?

No set answer is looked for but candidates will need to address the question. Candidates must deal with the given factor even if they wish to argue other factors were more significant. In relation to the given factor, candidates may refer to the background to the tension between the two monarchs, the journey to Acre, the differences that emerged there, the departure of Philip after the siege and the difficulties with the French contingent thereafter. Certainly it would be hard to argue that the divisions between the two had no impact; indeed, candidates may also argue that fears about what Philip might do in Europe whilst Richard remained in the Holy Land affected the conduct and outcome of the Crusade. Against this candidates may argue that success at Acre would not have been possible without some cooperation between the two. In discussing other reasons for the limited success of the Crusade candidates may refer to the disaster that befell Frederick Barbarossa and the disintegration of the German contingent, the parlous position in the Holy Land, the rivalries within the Christian camp at Acre over who should be King of Jerusalem, and the strengths of the opposition and Saladin in particular.

30

Page 216: History A - The Bicester Schoolthebicesterschool.org.uk/wp-content/uploads/2015/09/Mark... · 2015-09-28 · History A Mark Schemes for the Units January 2009 H106/H506/MS/R/09J

F962 Mark Scheme January 2010

The Renaissance from c. 1400-c. 1550 4 How important was the fall of Constantinople (1453) in the development of the

Renaissance in Italy?

No set answer is looked for but candidates will need to address the question. This is a question about the Renaissance generally and not about art specifically. Candidates must deal with the given factor even if they wish to argue other factors were more significant. In relation to the given factor, candidates may discuss the argument that after the fall of Constantinople there was a flood of Greek scholars into Europe who brought with them manuscripts of the Greek classics that were to inform much of the development of Renaissance ideas. This may be countered by the suggestion that the Renaissance pre-dated 1453 and that western scholars had had access to classical learning before this date. They may argue therefore that at best the effects of the Fall of Constantinople accelerated a process that had already begun. Candidates need to address ‘How important?’ and are likely to do this not just by the above but also by setting the fall of Constantinople in the context of other factors that contributed to the development of the Renaissance, such as the cultural, economic and political conditions in 15th century Italy and in particular the significance of wealth, trade, city states (like Florence), patronage and other influences.

5 ‘Renaissance artists and architects did no more than copy classical art and

architecture.’ How far do you agree?

No set answer is looked for but candidates will need to address the question. Candidates are likely to argue against the contention in the question along the lines that, although Renaissance artists and architects did draw inspiration from the works of Rome and Greece, they developed something innovative in their various fields. They may draw on their knowledge of individual artists and architects to illustrate their argument. They may point, for example, to Brunelleschi’s marriage of classical features such as Corinthian columns and a concern with proportion with Romanesque arches and Byzantine inspired domes. In art they may point to the classical themes and the use of light and atmospheric colour that inspired much Renaissance art and the revival (by, for example, Donatello) of free-standing sculpture, but stress the development of perspective, realism and the close observation of nature that is apparent in the works of artists from Masaccio onwards. They may also discuss the differences of subject matter in Renaissance art. No specific answer is looked for but the quality of exemplar material is likely to be a key discriminator.

31

Page 217: History A - The Bicester Schoolthebicesterschool.org.uk/wp-content/uploads/2015/09/Mark... · 2015-09-28 · History A Mark Schemes for the Units January 2009 H106/H506/MS/R/09J

F962 Mark Scheme January 2010

6 To what extent did the Italian Renaissance influence cultural developments in northern Europe?

No set answer is looked for but candidates will need to address the question. Candidates may well argue that whilst the Italian Renaissance did influence developments in art, architecture and ideas north of the Alps, developments here were essentially distinct. In developing their ideas candidates may refer to the importance of Rome and Italy more generally as a place of pilgrimage, art and learning (with its universities) to which scholars and artists from across Europe came. They may also point to the spreading of Renaissance ideas and influences via merchants and diplomats. Candidates may also refer to the Italian influences apparent in the works of painters like Holbein and Dürer. They may also point to the influence of Renaissance humanism. To balance this, candidates may discuss the distinctiveness of developments north of the Alps such as the protestant prejudice against religious art and the development of Christian humanism that was less inspired by the examination of Greek and Roman classics and more by a concern to apply humanist ideas in a Christian context. In relation to this they may refer to the Devotio Moderna and the importance placed on the reading of scripture and the reality of religion.

32

Page 218: History A - The Bicester Schoolthebicesterschool.org.uk/wp-content/uploads/2015/09/Mark... · 2015-09-28 · History A Mark Schemes for the Units January 2009 H106/H506/MS/R/09J

F962 Mark Scheme January 2010

Exploration and Discovery c. 1445-c. 1545 7 How important was royal patronage in Portuguese overseas exploration in this

period?

No set answer is looked for but candidates will need to address the question. Candidates must deal with the role of royal patronage even if they wish to argue that other factors were more or as significant in explaining Portuguese overseas exploration. In relation to the given factor, candidates are likely to focus on Henry the Navigator and John II. In relation to the former, candidates may refer to: his sponsorship of voyages to explore the African coast, leading to the discovery of the Azores, the Cape Verde Islands and the coast of West Africa (a slave trading base at Lagos); and, his attracting some leading cartographers to help map the coast. In relation to the latter, John II sponsored and planned expeditions to find Prester John and a route to the Asian sources of spices. In this context, they may refer to the voyages of Bartholomew Diaz and the expedition of Covilha (overland) to India. Such discussion may be set in the context of other factors that promoted or helped develop Portuguese exploration, such as Portugal’s geographic position, its established sea-faring tradition, its relative political stability, the interest of nobles (not least in a desire to serve their rulers), the role of individuals, such as Diaz, Cabral and Da Gama, and the incentives to find gold, slaves, and spices and to find Prester John and spread Christianity.

8 ‘No more than a series of trading posts.’ How far do you agree with this view of the

Portuguese Empire?

No set answer is looked for but candidates will need to address the question. Candidates are likely to argue that to a degree the Portuguese Empire was a series of trading posts, and that trade was central to all of her empire. However, they are likely to argue also that to conclude it was ‘nothing more’ is wrong. In relation to the notion that the Empire was a series of trading posts, candidates are likely to point to the strategy of Henry the Navigator and those that followed of establishing secure bases for trade and security of trade routes along the coast of Africa and the estimated 50 forts and trading posts established between southern Africa and Japan during the 16th century. No attempt was made to colonise Africa but bases were used to exchange European goods for slaves and the raison d’être of posts from Goa to Macao was the lucrative spice trade. Against this, candidates may argue that this is not the whole picture, as in the islands of the coast of West Africa (Madeira and the Cape Verde Islands) active settlement took place and a thriving agriculture based on sugar was established. Similarly in Brazil, the east coast was settled for plantation agriculture supported by the export of slaves from Africa. What is more the capture of important trading posts like Goa, Malacca and Macao led in time to the development of colonies as from these the Portuguese could monopolise not only trade but could also plunder and tax.

33

Page 219: History A - The Bicester Schoolthebicesterschool.org.uk/wp-content/uploads/2015/09/Mark... · 2015-09-28 · History A Mark Schemes for the Units January 2009 H106/H506/MS/R/09J

F962 Mark Scheme January 2010

9 To what extent was the spread of Christianity Spain’s main aim in the development of its Empire?

No set answer is looked for but candidates will need to address the question. Candidates must deal with the given factor even if they wish to argue other factors were more significant. In relation to the given factor, candidates may discuss the religious motivation of royal patrons such as Isabella, the injunctions of the Pope (as in his bull of 1493 to Isabella), and the accompaniment of priests with Spanish expeditions to the New World. It would be hard to argue that religion was not an important motive as Christianity was spread with conquest. However, candidates are likely to argue that it was not the main motive and certainly not the only one. Candidates are likely to argue that the main motive was profit (gold, silver or spices) and discuss the expeditions of Cortes and Pizarro in Latin America as evidence of this. Other motives that candidates may discuss include the desire for settlement and farming, the desire for fame and reputation (not least for Cortes). Candidates should explore a range of aims and draw a reasoned conclusion as to the relative importance of the spread of Christianity.

34

Page 220: History A - The Bicester Schoolthebicesterschool.org.uk/wp-content/uploads/2015/09/Mark... · 2015-09-28 · History A Mark Schemes for the Units January 2009 H106/H506/MS/R/09J

F962 Mark Scheme January 2010

Spain 1469-1556 10 How successful were Ferdinand and Isabella’s religious policies?

No set answer is looked for but candidates will need to address the question. Success may be assessed in a number of ways: against aims; against results; and in the light of historical context. Candidates will need to identify and analyse Ferdinand and Isabella’s religious policies. The areas considered may include: the conquest of Granada, policies towards Muslims in Granada including the expulsion of 1502; policies towards the Catholic Church (to achieve royal domination of the Church, to access the Church’s wealth, and to reform the abuses within the Church); and the role of the Inquisition and policies towards the Jews. Candidates may argue that whilst the conquest of Granada was successful the policies towards the Muslims there had mixed success (the Morisco ‘problem’ was to remain until the 17th century, for example). They may argue also that Ferdinand and Isabella enjoyed considerable success in their policy towards the Church, effectively controlling appointments and winning considerable concessions from the Papacy in Granada and the New World. However, they may argue that Cisneros’ attempts to reform the clergy were less successful. Judgement on the work of the Inquisition, policies towards conversos and the expulsion of the Jews is likely to be one of success.

11 Assess the reasons why Charles I faced so many problems in Spain from 1516 to

1524.

No set answer is looked for but candidates will need to address the question. Candidates will need to analyse reasons and evaluate their relative significance and/or links between them. Candidates may well suggest that some problems were inherited from Ferdinand and Isabella and that others were of his own making or that his actions/inaction exacerbated the situation. Candidates are likely to discuss some or all of the following: the Communeros and Germania revolts (the latter not fully resolved until the pardon issued in 1524); the tensions between towns and grandees; the problem of raising money via the Cortes of Castile, Aragon and other provinces; the privileges of the same; the appointment of ministers; Charles’ delay in arriving and his subsequent absence; his other ambitions and commitments (and the use of Spanish resources to pursue them) and so on.

12 Assess the strengths and weaknesses of Spain in 1556.

No set answer is looked for but candidates will need to address the question. Candidates need to assess both strengths and weaknesses. In discussing strengths and weaknesses, candidates may address some of the following areas: the Church and religion; government and administration; Castile and the other provinces; relations with the nobility; finance; the impact of the New World; the impact of foreign policy; the economy. They may argue that the pattern of strengths and weaknesses is a mixed one. The failure of the Reformation to make any headway in Spain may be considered a strength – religious unity remained strong – although the problem of the moriscos remained; royal government at a local level was perhaps dependent on the nobility, but worked reasonably effectively at the centre through its councils and candidates may pay tribute to the work of Gattinara, los Cobos and the bureaucracy of letrados – the conciliar system, despite corruption worked well enough in normal circumstances, but proved less effective at times of crisis. Many may argue that finance was an area of weakness, partly because of noble exemption from taxation, but mainly because of the demands made on Castile especially by Charles’ expensive commitments outside Spain. New World bullion became more significant later in the reign, but debt was a constant feature. Many will argue that the economy was Charles’ greatest failure: heavy taxation and the failure to use New World revenues effectively distorted the economy.

35

Page 221: History A - The Bicester Schoolthebicesterschool.org.uk/wp-content/uploads/2015/09/Mark... · 2015-09-28 · History A Mark Schemes for the Units January 2009 H106/H506/MS/R/09J

F962 Mark Scheme January 2010

Charles V: International Relations and the Holy Roman Empire 1519-59 13 ‘Without the protection of princes, Lutheranism would not have survived.’ How far

do you agree?

No set answer is looked for but candidates will need to address the question. Candidates must deal adequately with the issue raised in the quotation even if they wish to argue that other factors were as or more significant. However, many are likely to argue a strong case in favour of the quotation, pointing to the role from early on of Frederick of Saxony, the difficulties facing Charles V in imposing his will without the support of the princes, the formation of the Schmalkaldic League and the eventual acceptance of Lutheranism in the Peace of Augsburg (cuius regio, eius religio). They may also point to the reluctance of Catholic princes to take up arms against Protestant princes. Such discussion needs to be balanced against other considerations such as: the power of Luther’s ideas; their spread (including the role of the printing press and the context of anti-papal feeling); the role of the towns, peasants, Imperial Knights; the intermittent attention Charles V was able to give to the issue given the distractions of Habsburg-Valois rivalry, his absences, his desire for religious division to be settled by a Church Council, the Ottoman threat and so on.

14 To what extent was personal rivalry the main reason for the Habsburg-Valois wars?

No set answer is looked for but candidates will need to address the question. Candidates must deal adequately with the issue of personal rivalry even if they wish to argue that other factors were as or more significant. In relation to the given factor, candidates may argue that in an age of personal monarchy, personal rivalry was bound to play a part. Certainly there was rivalry: Charles defeated Francis I in the bid to become Holy Roman Emperor and Francis would never accept that Charles was the pre-eminent ruler in Christendom. Their contrasting characters also helped to shape events. However, candidates may well argue that conflict between these rivals was more than personal. The election of Charles as Holy Roman Emperor and his interests in Italy and the Netherlands meant that strategically France felt surrounded, whilst Charles also wanted to recover his ancestral Burgundian lands and Francis Navarre. Candidates should support their arguments by reference to the developments in the struggle (including the reign of Henry II) and may refer to events in Italy, the Holy Roman Empire, relations with the Turks and England and so on to illustrate and develop their argument.

15 Assess the reasons why Charles V was unable to remove the Ottoman threat.

No set answer is looked for but candidates will need to address the question. Candidates must identify and analyse a range of reasons and evaluate their relative significance and/or linkages. Candidates are likely to discuss the Ottoman threat in relation both to the Holy Roman Empire and Habsburg lands in Austria, Bohemia and Hungary, and to the Mediterranean. They may point to the aggressive and expansionist nature of the Ottoman Empire (and refer to the actions of Barbarossa and Dragut in the Mediterranean as well as the opening up the Balkans by the Ottoman army’s victory at Belgrade in 1521). They may also discuss the impact of the different commitments that Charles V’s vast territories imposed upon him, that meant he could never focus on one issue for long. They may also argue in relation to that that France was willing to use the Ottoman threat as a weapon in its wars with Charles. They may also argue that the resources available to Charles were limited in a number of ways (costs; no effective navy to counter the naval forces available to the Turks; the refusal of the German army to cross the frontier into Hungary) and that (as with the Turks) distance also limited what could be achieved.

36

Page 222: History A - The Bicester Schoolthebicesterschool.org.uk/wp-content/uploads/2015/09/Mark... · 2015-09-28 · History A Mark Schemes for the Units January 2009 H106/H506/MS/R/09J

F962 Mark Scheme January 2010

37

Philip II, Spain and the Netherlands, 1556-1609 16 ‘The most serious problems Philip II faced in ruling Spain were economic and

financial.’ How far do you agree?

No set answer is looked for but candidates will need to address the question. Candidates must deal with the claim made in the quotation even if they wish to argue that other problems were as or more serious for Philip. However, many are likely to agree with the quotation arguing that insufficient finance (itself dependent on the economy) underpinned and limited his ability to deal with many other serious problems. Candidates may discuss the general inadequacy of funds and how New World bullion shipments provided at best short term relief. They may argue that there was a vicious circle of rising debt as future income was mortgaged and interest rates rose. Increases in taxation impacted on the Castilian economy and by the 1590s the strain told. Candidates may argue that financial problems were also a symptom as well as a cause of other problems. It was the strain of constant warfare that demanded increased taxation and inefficient administration meant corruption. Candidates may also discuss other problems, such as faction (Perez affair), relations with the nobility, the problem of the moriscos and conversos.

17 ‘Philip II enjoyed more success in his dealings with the Turks than with England or

France.’ How far do you agree?

No set answer is looked for but candidates will need to address the question. In assessing success, candidates may take note of aims, results and the historical context. Candidates may suggest that at best Philip had not dealt with the Turkish menace, but had contained it and secured a measure of peace by the 1580s. In reaching a judgement they may discuss the extent of the Ottoman/Corsair threat, the defeat at Djerba in 1560, the subsequent aggression of the Corsairs (even raiding Granada), the relief of Malta, the victory at Lepanto, and the armistice of 1580. Candidates may argue that Philip’s chances of success were limited by the diversion of resources to other problems (such as the Netherlands) and the different interests of the Papacy and Venice that made cooperation difficult. In relation to England, candidates may refer to marriage to Mary, attempts to woo Elizabeth, growing differences, conflict over the New World and the Netherlands and the failures of his armadas. Candidates may well judge his policy here as a failure (although England was excluded from the New World). In relation to France, candidates may discuss early fears of a Guise empire, but are likely to focus on his support for the Catholic League against Henry of Navarre. They, again, may judge his policy a failure, although in the end France remained Catholic.

18 How far was Maurice of Nassau’s leadership the main reason for the success of the

northern provinces by 1609?

No set answer is looked for but candidates will need to address the question. Candidates must deal with the role of Maurice of Nassau even if they wish to argue that other factors were as or more significant in the success of the northern provinces. Candidates may argue that Maurice’s key contribution was as a military leader who was able to deliver a series of victories to the northern provinces that made it clear that the Spanish, whilst they might retain a hold on the south could not re-take the north. Candidates may balance such discussion with consideration of other factors such as: the role of William of Orange and the failures of Spain prior to Maurice’s prominence; the role of England and France; the diversion of Spanish forces from the Netherlands against England and France in the 1580s and 90s; the financial difficulties facing Spain and the mutinies that affected their forces; the skills of Oldenbarnevelt; the divisions within the government of the southern provinces and so on.

Page 223: History A - The Bicester Schoolthebicesterschool.org.uk/wp-content/uploads/2015/09/Mark... · 2015-09-28 · History A Mark Schemes for the Units January 2009 H106/H506/MS/R/09J

F962 Mark Scheme January 2010

Option B: Modern 1795-2003 Napoleon, France and Europe 1795-1815 1 ‘The weaknesses of the Directory were the main reason for Napoleon’s rise to

power.’ How far do you agree? No set answer is looked for but candidates will need to address the question. Candidates must deal with the given factor even if they wish to argue other factors were more significant. In relation to the given factor, candidates may discuss: the nature of the constitution of the Directory which made it weak and increasingly dependent on the military; the internal politics of the Five Directors and their rivalries; unrest at home and the growing desire for change; and the impact of defeat against the Second Coalition. Candidates may well argue that the weakness of the Directory provided Napoleon with the opportunity to seize power, but that, on its own, it does not explain his rise. Candidates may refer to other factors, such as: Napoleon’s rise in the military and the reputation he gained from Toulon to Egypt; the significant role played by politicians like Barras; aspects of the Coup of Brumaire such as the role of Napoleon’s brother and the miscalculation of Sieyes and others who had hoped for a tame general (and here the reluctance of generals like Moreau to play the role is significant); and, of course, Napoleon’s own ambitions.

2 To what extent was Napoleon responsible for his own downfall? No set answer is looked for but candidates will need to address the question. This question focuses on the reasons for Napoleon’s downfall. Candidates must deal with the given factor even if they wish to argue other factors were more significant. The arguments for Napoleon’s own responsibility include: whilst he was a great warrior, he was not a great statesman and failed to seek a permanent settlement for Europe; the Spanish and Russian campaigns revealed the limitations of Napoleon as a grand strategist; the view that Napoleon’s abilities declined in his later years; Napoleon became predictable. Candidates may question some of these points and certainly they should discuss some of the alternative explanations, such as: the relative decline of the French army; the determined opposition of Britain, supreme at sea, critical in the Iberian peninsula and providing finance for those willing to take up arms against Napoleon; the reorganization of enemy armies in the light of French victories; the impact of Napoleon’s defeat in Russia in 1812; Napoleon’s loss of support in France; the impact of the Continental System; the drawing up of the Fourth Coalition; the significance of Leipzig and Waterloo, and so on.

38

Page 224: History A - The Bicester Schoolthebicesterschool.org.uk/wp-content/uploads/2015/09/Mark... · 2015-09-28 · History A Mark Schemes for the Units January 2009 H106/H506/MS/R/09J

F962 Mark Scheme January 2010

3 To what extent did Napoleon export the ideas of the French Revolution to the areas he conquered? No set answer is looked for but candidates will need to address the question. The question focuses on the impact of Napoleon’s hegemony over much of Europe for at least part of his reign. Candidates are likely to argue that the impact of French ideas varied from place to place and from time to time. They may argue that much of Napoleonic rule was pragmatic, and that although French ideas of government, principles of the Revolution and law were introduced in many areas it is difficult to discern a consistent pattern. Candidates may argue that much depended on nearness to France, the length of French influence and control, the attitudes of the local population, the differing status of conquered or occupied areas (absorbed within ‘France’ or satellite states or allies), and the exigencies of the particular time. There may be discussion of the impact of the Code Napoleon, relations with the Church, nobility and middle classes, the requirements of the French military machine and so forth.

39

Page 225: History A - The Bicester Schoolthebicesterschool.org.uk/wp-content/uploads/2015/09/Mark... · 2015-09-28 · History A Mark Schemes for the Units January 2009 H106/H506/MS/R/09J

F962 Mark Scheme January 2010

Monarchy, Republic and Empire: France 1814-1870

4 To what extent was Charles X responsible for his own downfall?

No set answer is looked for but candidates will need to address the question.

Candidates must deal with the issue raised in the question even if they wish to argue that other factors were as or more important in Charles X’ overthrow in 1830. In discussing Charles X’ degree of responsibility, candidates may argue that as he was king he was responsible for the ministers he appointed and the policies adopted, his association with the Ultras and the circumstances of his coronation suggested a reactionary policy from start. The policies pursued by Villele seemed to confirm this: compensation for émigrés, return of Jesuits and clerical control of education etc. Candidates may also argue that crucially Charles appointed the ultra Polignac in 1829 and then in 1830 issued the Ordinances of St Cloud at a time when the Crown’s best troops were in Algeria. On the other hand, candidates may point to Charles’ liberalization of the press on his accession, the existence of liberal opposition and press, the tradition of revolution and the return of economic crisis as key factors.

5 Assess the reasons why Louis Napoleon came to dominate the Second Republic.

No set answer is looked for but candidates will need to address the question.

Candidates will need to identify and analyse a range of reasons and evaluate their relative significance and/or linkages. Candidates may argue that there are longer term and shorter term reasons why Napoleon emerged as the dominant figure between 1848 and 1852. They may point to the longer term development of Bonapartism, Louis Napoleon’s failed expeditions of 1836 and 1840, the publication of the Extinction of Pauperism (1844), the association with Napoleon and the revival of the Napoleonic legend (return of body, and completion of the Arc de Triomphe) under Louis Philippe. In the shorter term they may point to the developments after the February Revolution in 1848: the growth of reaction, Louis Napoleon’s appeal to all classes (peasants, workers, businessmen, monarchists and ultramontane Catholics), Louis Napoleon’s alliance with conservative forces, problems in Paris, the election of November 1848, the Bonapartists’ efficient organization. Candidates may stress that Louis Napoleon’s greatest asset was his name. Some may argue that Louis Napoleon did not dominate the Second republic and that is why he abolished it.

6 Assess the reasons why Napoleon III’s foreign policies in the 1860s were unsuccessful.

No set answer is looked for but candidates will need to address the question.

Candidates will need to identify and analyse a range of reasons and evaluate their relative significance and/or linkages. In developing their answers, candidates are likely to discuss some of the following aspects of foreign policy: the attempts to secure territory along the Rhine (aired at Biarritz in 1865); attitude to the Polish Revolt in 1863; the Austro-Prussian war (1866); the Luxembourg crisis (1867); the Mexican adventure; colonial policy; the dispute over the Spanish succession and the background to the Franco-Prussian War. In discussing reasons for failure candidates may refer to: misjudgement, lack of clear aims, unrealistic ambitions and expectations, increasing isolation, pursuit of foreign aims as a means of appeasing domestic opinion (eg by keeping troops in Rome); policies which led to French isolation/loss of potential friends; the arousing of suspicions in Prussia, German states and Britain over attempts to expand French influence and territory on the Rhine; misconceived desire to offer support to Polish nationalism led to break-up of his understanding with the Tsar; the blow to French prestige when attempts to mediate in the Austro-Prussian war failed; unrealistic unilateral action against Mexico motivated by a mistaken desire for glory, economic opportunity and a desire to appease catholic opinion; being out-manoeuvred by Bismarck from 1865 onwards and so on.

40

Page 226: History A - The Bicester Schoolthebicesterschool.org.uk/wp-content/uploads/2015/09/Mark... · 2015-09-28 · History A Mark Schemes for the Units January 2009 H106/H506/MS/R/09J

F962 Mark Scheme January 2010

The USA in the 19th Century: Westward Expansion and Civil War 1803-c. 1890 7 How important were the policies of Federal governments in opening up the West to

settlement? No set answer is looked for but candidates will need to address the question. Candidates will need to deal with the given factor even if they wish to argue that other factors were as or more important. In assessing the importance of the policies of Federal governments, candidates may discuss some of the following areas: Federal sponsorship of exploration and surveying; Federal acquisition of territory; the organization of acquired lands into territories and states; the role of the Federal army in policing the frontier, the trails west and dealing with Native Americans; Federal sponsorship of communications (especially the trans-continental railway); Federal encouragement to settlement through legislation such as the Homestead Act. To balance such discussion candidates may argue that Federal policy often followed rather than preceded settlement and the real stimulus came from the needs of fur traders, cattlemen, farmers and miners as well as those seeking refuge, like the Mormons, from persecution, and the development of communications.

8 Assess the reasons why southern states decided to secede from the Union in 1860 and 1861. No set answer is looked for but candidates will need to address the question. Candidates need to identify and analyse reasons and evaluate their relative significance and/or linkages. Candidates may argue that the immediate cause of the first wave of secession was the election of a sectional president in Lincoln and that the stimulus to the secession of some Upper South states was the decision for war in the spring of 1861. Candidates may then go on to explore the short and longer term issues that led to secession. Central to their arguments is likely to be the issue of slavery and they may discuss some of the crises of the 1840s and 50s that made the issue of slavery and the issue of the possible westward expansion of slavery so contentious. In this context, candidates may refer to the Mexican War, Wilmot Proviso, Calhoun doctrine, the ‘Compromise’ of 1850, fugitive slaves, Dred Scott, ‘Bleeding Kansas’, the development of the Republican Party, Harper’s Ferry and so on. Candidates may also discuss the issue of States’ rights, the apparent social, cultural and economic divisions between North and South and the suspicions of ‘Slave Power conspiracy’ and ‘northern aggression’.

41

Page 227: History A - The Bicester Schoolthebicesterschool.org.uk/wp-content/uploads/2015/09/Mark... · 2015-09-28 · History A Mark Schemes for the Units January 2009 H106/H506/MS/R/09J

F962 Mark Scheme January 2010

9 ‘Grant was a better general than Lee.’ How far do you agree?

No set answer is looked for but candidates will need to address the question. This question requires candidates to compare the strengths and weaknesses of Grant and Lee as generals. Candidates may consider a range of issues in drawing their comparison: personal qualities, reputation, leadership, strategic thinking (eg at different levels – Grand Strategy, campaign strategy, battlefield command), relationship with political masters, use of resources, impact on morale, quality of opposition, quality of subordinates and so on. In making their analysis candidates may draw on their knowledge of specific campaigns and battles such as: in relation to Lee, Seven Days Campaign, Second Manassas, Antietam, Fredericksburg, Chancellorsville, Gettysburg, Wilderness Campaign, Petersburg; and in relation to Grant, Fort Donelson, Shiloh, Vicksburg, Lookout Mountain and Missionary Ridge, Wilderness Campaign, Petersburg, Atlanta and March through Georgia (Sherman in command). No specific answer is looked for and candidates can legitimately argue that both were great generals in their different ways.

42

Page 228: History A - The Bicester Schoolthebicesterschool.org.uk/wp-content/uploads/2015/09/Mark... · 2015-09-28 · History A Mark Schemes for the Units January 2009 H106/H506/MS/R/09J

F962 Mark Scheme January 2010

Peace and War: International Relations c. 1890-1941 10 To what extent was the alliance system the main cause of the First World War?

No set answer is looked for but candidates will need to address the question. Candidates will need to deal with the given factor even if they wish to argue that other factors were as or more important. In relation to the given factor, candidates may discuss the intentions and nature of the alliances that preceded the First World War. Certainly the division of Europe into two potentially hostile alliances (Ententes and Dual/Triple Alliances) could be said to make war a possibility and in the event the war between the Dual Alliance and the Entente. Candidates may, however, argue that other factors were more important and discuss the relative merits of other factors such as military and naval arms races, aggressive German foreign policy, Russia’s hopes and fears in relation to the Balkans, British and French policy, domestic problems and pressures and so on. They may discuss the significance of particular crises in contributing to making war more likely, such as the Bosnian Crisis, the Moroccan Crises and the July Crisis of 1914.

11 Assess the reasons why the First World War was not ‘over by Christmas’ 1914.

No set answer is looked for but candidates will need to address the question. Candidates will need to identify and analyse a range of reasons and evaluate their relative significance and/or links between them. Candidates may discuss the relative strengths and weaknesses of the Schlieffen Plan both conception and as it operated under Moltke. They may point to the role of the Belgians, the BEF, the ‘Miracle on the Marne’ and the subsequent ‘race to the sea’. They may also point to the relatively rapid mobilisation of the Russians and the impact of their offensives in the East. Candidates may also point to the issues of technology that affected the initial course of the war, pointing to the exposure of cavalry and infantry in attacking strong positions, the impact of disciplined rifle, machine gun and artillery fire, the difficulties of supply for a rapidly advancing army, the ‘digging in’ of defending forces and so forth. Candidates may also argue that no side had a decisive advantage in numbers, tactics or strategy.

43

Page 229: History A - The Bicester Schoolthebicesterschool.org.uk/wp-content/uploads/2015/09/Mark... · 2015-09-28 · History A Mark Schemes for the Units January 2009 H106/H506/MS/R/09J

F962 Mark Scheme January 2010

12 Assess the reasons why Japan followed an increasingly aggressive foreign policy in the period from 1931 to 1941.

No set answer is looked for but candidates will need to address the question. Candidates should discuss a number of reasons and assess their relative significance and/or the linkages between them. Candidates may well focus on the ambitions of Japan in China and South East Asia more generally. They may point to the aggressive foreign policy pursued as a consequence of the Depression, the growth of nationalism and historic claims on the Chinese mainland. There may be discussion of effects of the successful invasion of Manchuria, the outbreak of the Sino-Japanese war, the alliance with Germany and Italy, the Co-prosperity Sphere and the decision to attack Pearl Harbour. Discussion of Japanese motives needs to be set in the context of the attitudes of the USA, Britain and France, the weakness and failure of the League of Nations, the distractions of events in Europe, and the relative weakness of and internal divisions in China. Candidates may argue therefore that whilst there were compelling internal reasons why Japan wanted to pursue an expansive foreign policy, she was encouraged to do so by the weaknesses of her immediate opponents (notably China) and the failure of the major powers (Britain, France and the USA) to stop Japan because of their own internal problems and the developing events in Europe.

44

Page 230: History A - The Bicester Schoolthebicesterschool.org.uk/wp-content/uploads/2015/09/Mark... · 2015-09-28 · History A Mark Schemes for the Units January 2009 H106/H506/MS/R/09J

F962 Mark Scheme January 2010

From Autocracy to Communism: Russia 1894-1941 13 Assess the reasons for opposition and unrest in Russia from 1894 to 1905.

No set answer is looked for but candidates will need to address the question. Candidates will need to identify and analyse a range of reasons and evaluate their relative significance and/or linkages. Candidates may discuss the longer term context of opposition to the Tsars, but are likely to focus on the development of opposition and unrest after 1894. They may point to the economic causes of unrest and point to the economic downturn that provoked strikes and unrest in the period after 1900 alongside the appalling conditions for workers in Russia’s growing industries. They may also discuss the problem of land shortage for the peasants. Such discussion may be linked to the developments in political opposition through the Zemstva and in the development of Russian social Democracy, the Socialist Revolutionaries and the Union of Liberation Party. They may also point to the humiliation of defeat in the Russo-Japanese War, disillusion with Tsarism, the desire for liberal reforms, the problem of nationalities and the outbreak of the 1905 Revolution.

14 How far was Russia politically stable from 1905 to 1914?

No set answer is looked for but candidates will need to address the question. Candidates need to assess ‘How far?’. They may contrast the revolutionary crisis of 1905 with the relative quiet of the period of Stolypin’s influence and the development of renewed unrest in the few years before the First World War. Candidates may discuss the political repression that followed the October Manifesto and the Fundamental Laws and the nullifying of the potential of the Dumas as a check on Tsarism. They may point to the decline in agitation and the collapse in the membership of the RSDLP as evidence of increased stability. They may also point to Stolypin’s ‘wager on the strong’ as evidence of a different approach and the celebrations of the Romanov dynasty in 1913 of the Tsar’s popularity. They may also suggest that whilst there was comparative quiet in the period after 1906 stability was more apparent than real and that the pressures that brought about the revolutionary crisis of 1905 were still unresolved.

45

Page 231: History A - The Bicester Schoolthebicesterschool.org.uk/wp-content/uploads/2015/09/Mark... · 2015-09-28 · History A Mark Schemes for the Units January 2009 H106/H506/MS/R/09J

F962 Mark Scheme January 2010

15 Assess Lenin’s strengths and weaknesses as leader of Russia from 1917 to 1924. No set answer is looked for but candidates will need to address the question. Candidates must address both strengths and weaknesses. In discussing Lenin’s strengths and weaknesses candidates may refer to key decisions and developments in this period: the determination to ensure Bolshevik rule to the exclusion of other socialist parties (despite the brief flirtation with the left SRs); the ending of the war with Germany; the Civil War, reign of terror and war communism; the NEP and political repression. Candidates may discuss Lenin’s determination, his ruthlessness, his ability to force through unpopular decisions, his hard work, his ability to change policy when necessary, his commitment to the Bolshevik Party, his vision of a Communist Russia. What may be seen as strengths may also been seen as weaknesses – his unwillingness to compromise or work with other socialists, his pursuit of power (or Communist ideas depending on interpretation) whatever the cost, his decline after his strokes, and so on.

46

Page 232: History A - The Bicester Schoolthebicesterschool.org.uk/wp-content/uploads/2015/09/Mark... · 2015-09-28 · History A Mark Schemes for the Units January 2009 H106/H506/MS/R/09J

F962 Mark Scheme January 2010

Democracy and Dictatorship: Italy 1896-1943 16 Assess the reasons for unrest in Italy from 1896 to 1915.

No set answer is looked for but candidates will need to address the question. Candidates need to identify and analyse a number of reasons and evaluate their relative significance and/or linkages. Candidates may discuss some of the following areas: national humiliation after Adowa, famine and economic hardship; the desire for economic improvements (to working conditions); police violence; the North:South divide; the limited franchise; the growth of socialism; nationalist agitation and so on. Candidates may argue that real hardship was certainly an issue in the late 1890s when famine led to food riots. They may suggest that after 1900 much agitation was still economically based and strikes and protests aimed at better working conditions. They may suggest that whilst Giolitti’s economic policies led to economic expansion this only accentuated the divide between the industrial north and the poverty-stricken agricultural south. Candidates are likely to spend some time discussing the growth of socialism and the rise in political and economic unrest in the pre-war years, such as ‘red week’. They may also point to nationalist agitation that led to the expensive conquest of Libya in 1911-12.

17 To what extent was fear of socialism the main reason for Mussolini’s rise to power in 1922? No set answer is looked for but candidates will need to address the question. Candidates must deal with the given factor adequately even if they wish to argue that other factors were more important. In relation to the significance of fear of socialism, candidates may discuss the growth of socialism, its electoral profile and the red week and the fears aroused amongst the middle and upper classes, the Church and the establishment by the ‘red menace’. They may link such discussion to the impact of unemployment, inflation, post-war economic restructuring, problems in the countryside and the north-south divide to illustrate the potential scale of the threat. Mussolini and the Fascists were able to play on these fears and pose as the men of action - the direct action Mussolini was willing to take against strikers and communists (albeit after the main crisis had passed). Such a pose contrasted with the apparent failure of the liberal governments of Nitti and Giolitti to deal with the problems effectively. Candidates may also discuss the other weaknesses of the liberal governments (for example, the failure to gain a creditable peace settlement, the failure of transformismo), the legacy of nationalism, the ability and opportunism of Mussolini and the fascists, the attitude of the King and the establishment and the fateful decisions of 1922.

47

Page 233: History A - The Bicester Schoolthebicesterschool.org.uk/wp-content/uploads/2015/09/Mark... · 2015-09-28 · History A Mark Schemes for the Units January 2009 H106/H506/MS/R/09J

F962 Mark Scheme January 2010

18 ‘A dictator in name only.’ How far do you agree with this view of the extent of Mussolini’s power in Italy after 1922?

No set answer is looked for but candidates will need to address the question. Candidates should discuss the extent and nature of Mussolini’s dictatorship. Candidates may refer to some of the following in developing their argument: Mussolini’s consolidation of power after 1922 (Acerbo Law, Aventine Secession, abolition of the party system, the restrictions on the power of the monarchy, rule by decree, the fusion of state and party under the Duce). They may also refer to censorship, propaganda and other aspects of a police state (such as OVRA). To balance this they may also discuss the extent of Mussolini’s control of the party, the continued existence of the monarchy, the need to come to agreement with the Church (Concordat), the inefficiency of the Fascist state. Candidates could point to the circumstances of his overthrow in 1943 as evidence of his limitations to his power.

48

Page 234: History A - The Bicester Schoolthebicesterschool.org.uk/wp-content/uploads/2015/09/Mark... · 2015-09-28 · History A Mark Schemes for the Units January 2009 H106/H506/MS/R/09J

F962 Mark Scheme January 2010

The Rise of China 1911-1990 19 Assess the reasons why it took so long to establish effective government after the

1911 revolution.

No set answer is looked for but candidates will need to address the question. Candidates will need to discuss and evaluate a range of reasons. In assessing reasons, candidates may discuss some of the following: the state of China in 1911; ‘sudden’ nature of the revolution in 1911 and resulting power vacuum; ambitions of Yuan Shikai; the limited authority of government and local power/rivalries of warlords (the significance of the warlords may be stressed); the extent and nature of support for Sun Zhongshan (Sun Yat-sen) and the Nationalists (party formed only in 1912); the significance of the 4 May Movement; Sun Zhongshan (Sun Yat-sen) and the reorganization of the Guomindang; Foundation of CCP. Candidates may argue that whilst the overthrow of the Manchu dynasty met little resistance, there was no consensus about what next and that there was no one source of power able to assert its authority in the short term. Not until the 1920s were the nationalists in a position to establish their authority and this remained patchy. Some may argue that no really effective government was established until after 1949. No specific answer is being looked for.

20 Assess the reasons why the Nationalists were unable to crush the Communists in the period to 1945.

No set answer is looked for but candidates will need to address the question. Candidates need to identify and analyse a number of reasons and evaluate their relative significance and/or linkages. In discussing reasons for Nationalist failure, candidates may refer to the united front with Sun Yat-sen’s nationalists after the 4 May Movement during which time the party began to grow, the White Terror of 1927, the retreat to the countryside and the Chungkang Mountains, the development of the Red Army, the Nationalist campaigns and the resulting Long March of 1934-5, Yenan, the united front against the Japanese, and the Civil War. Candidates may discuss reasons to do with the communists: leaders like Mao, the active aid to and support from peasants, the skills of the Red army and its development of guerrilla tactics, the significance of the Long March and the honing of ideas in Yenan, the distinguished role in the war against Japan and so on. They may also discuss the failings of the Nationalists under Jiang: distraction of the warlords, corruption, failure to win over the workers and peasants (lack of support in the countryside), the loss of middle class support, poor performance in the war against the Japanese and so on. They may also discuss the fact that the Nationalists were also forced to accept communist help in the fight against the Japanese.

49

Page 235: History A - The Bicester Schoolthebicesterschool.org.uk/wp-content/uploads/2015/09/Mark... · 2015-09-28 · History A Mark Schemes for the Units January 2009 H106/H506/MS/R/09J

F962 Mark Scheme January 2010

21 To what extent were Mao’s domestic policies successful in the 1950s?

No set answer is looked for but candidates will need to address the question. Candidates need to identify and analyse a range of policies and assess their success; this can be done against aims, results and/or historical context. Candidates may consider some of the following policy areas: the establishment of communist rule including military rule and reunification campaigns, the use of terror, propaganda and the imposition of one party rule; the ‘three’ and ‘five’ ‘anti-movements’; attacks on the middle classes and landlords; the first Five Year Plan; the Hundred Flowers Campaign; collectivisation; the Great Leap Forward. The last may not be considered in terms of success as it lasted into the 1960s. Candidates may well argue that the CCP managed to establish its authority effectively and achieved considerable successes but at a cost. The results of the first five year plan, for example, were impressive, but heavily dependent on Soviet aid and support; the middle classes were attacked and denounced and maybe a million landlords in the countryside were killed. They may argue that the Hundred Flowers Campaign backfired and had to be abandoned.

50

Page 236: History A - The Bicester Schoolthebicesterschool.org.uk/wp-content/uploads/2015/09/Mark... · 2015-09-28 · History A Mark Schemes for the Units January 2009 H106/H506/MS/R/09J

F962 Mark Scheme January 2010

Democracy and Dictatorship in Germany 1919-1963 22 To what extent did the Weimar Republic in the 1920s overcome the problems it

faced? No set answer is looked for but candidates will need to address the question. Candidates need to identify a number of problems facing the Weimar Republic and assess how far these were overcome during the 1920s. Candidates may discuss some of the following problems: the economic and social problems arising out of the war; the Treaty of Versailles and its impact; the problems of political extremism; the problems arising from the nature of the new Weimar constitution; the difficulties of Germany’s international position and so on. In relation to economic and social problems candidates may refer to the unemployment, inflation and economic dislocation after the war and the impact of reparations, hyperinflation and the invasion of the Ruhr. They may discuss the work of Stresemann, the Dawes and Young Plans and the rise in foreign investment in relation to this. They may discuss the threats posed by left and right to the regime and the difficulties associated with a democratic constitution based on proportional representation and how (far) these were dealt with. They may discuss German attempts to revise the Treaty of Versailles and the impact of Stresemann’s foreign policy (Locarno, League of Nations). No specific answer is looked for.

23 ‘Their use of terror was the main reason that the Nazis retained control in Germany after 1933.’ How far do you agree? No set answer is looked for but candidates will need to address the question. Candidates must give adequate treatment to the use of terror even if they wish to argue that other factors were as or more significant. In relation to terror, candidates are likely to discuss various aspects of the police state such as the roles of censorship and propaganda, the nazification of the judicial system, the use of arbitrary imprisonment and the roles of the SS, Gestapo and concentration camps. They may also point to the stifling of political opposition with the arrest of communists and social democrats and the ‘law’ banning other political parties. However, candidates may balance their discussion of these areas with the role of indoctrination, the attempts to control all aspects of people’s lives and the impact of war (with reference to the German Labour Front, Strength through Joy, the Hitler Youth etc.). Candidates may also refer to the apparent benefits of Nazi rule: the end of the communist threat, the restoration of ‘order’, employment and economic recovery, and foreign policy successes.

51

Page 237: History A - The Bicester Schoolthebicesterschool.org.uk/wp-content/uploads/2015/09/Mark... · 2015-09-28 · History A Mark Schemes for the Units January 2009 H106/H506/MS/R/09J

F962 Mark Scheme January 2010

24 Assess the reasons why Adenauer kept power for so long after 1949.

No set answer is looked for but candidates will need to address the question. Candidates need to identify and analyse a number of reasons and evaluate their relative significance and/or linkages. Answers may discuss the personal role of Adenauer in maintaining power and weigh this up against other factors This may include factors such as the political situation in West Germany, economic issues and issues of foreign relations in developing their argument. In relation to politics, candidates may refer to the strength and stability of the CDU/CSU coalition and the reliable support this received from the Liberals until the early 60s; they may also point to the weaknesses of the main opposition party – the SPD – (internal divisions, unable to adapt to the new prosperous West Germany). They may also suggest that the strategy of emphasising reconstruction (rather than recrimination) was a powerful political argument. In relation to foreign policy, candidates may point to: acceptance of West Germany in Europe; the recognition given to the FRG after 1955 and the end of the ‘occupation’; Britain’s support for Germany’s entry to NATO and hence the creation of her own army; West Germany membership of the OEEC, the ECSC and then the EEC. In relation to the economy candidates are likely to point to the work of Erhard, the social market economy, the Marshall Plan, cheap labour, good industrial relations, the survival of much of Germany’s industrial base after the war. They may point to the fall in unemployment and the average growth rate of 8%, and improving living standards. Candidates may well point to economic factors as being most important in explaining Adenauer’s success but no specific answer is looked for.

52

Page 238: History A - The Bicester Schoolthebicesterschool.org.uk/wp-content/uploads/2015/09/Mark... · 2015-09-28 · History A Mark Schemes for the Units January 2009 H106/H506/MS/R/09J

F962 Mark Scheme January 2010

The Cold War in Europe from 1945 to the 1990s 25 Assess the reasons for the growth of tension between the Allies at the Yalta and

Potsdam Conferences. No set answer is looked for but candidates will need to address the question. Candidates need to identify and analyse a range of reasons and to evaluate their relative significance and/or linkages. Candidates may discuss the longer term context of ideological differences and tensions that had arisen in the wartime alliance that provided grounds for mutual suspicion. In relation to Yalta, candidates are likely to focus on discussion of the Polish issue and the differences this aroused. There may be discussion of Stalin’s desire for security. In relation to Potsdam, candidates may refer to the change in personnel and it significance (particularly in relation to Truman), the context of the successful testing of the atom bomb, the continuing issue of Poland, the question of governance in liberated states and the issue of reparations in relation to Germany. Candidates may suggest that underpinning apparent agreements lay real difficulties as mutual fear and suspicion grew.

26 How serious were the problems facing the Soviet Union in controlling Eastern

Europe in the 1950s and 1960s?

No set answer is looked for but candidates will need to address the question. Candidates will need to identify and analyse a range of problems and evaluate their relative seriousness. They may discuss particular developments such as the problems in East Germany, Poland, Yugoslavia, Albania and Hungary in the 1950s and the Prague Spring of 1968. However, they may place such discussion in the broader context of the issue for Soviet leaders in balancing control against local situations and reform, the impact of the Cold War and the impact of de-Stalinisation. Candidates may discuss individual crises and the threat they posed to the stability of Soviet controlled Eastern Europe more generally and here there may be developed treatment of Hungary in 1956, the problem of refugees and the building of the Berlin Wall and the Dubcek regime in Czechoslovakia. The seriousness of the problems may be assessed in relation to the reaction of the Soviet government and the actions it took.

53

Page 239: History A - The Bicester Schoolthebicesterschool.org.uk/wp-content/uploads/2015/09/Mark... · 2015-09-28 · History A Mark Schemes for the Units January 2009 H106/H506/MS/R/09J

F962 Mark Scheme January 2010

27 Assess the consequences of the end of Communist rule in the Soviet Union.

No set answer is looked for but candidates will need to address the question. Candidates need to identify and analyses a range of consequences and evaluate their relative significance and/or linkages. Candidates may place their discussion of the context of the loosening of Communist Party control within the Soviet Union under Gorbachev’s policies of glasnost and perestroika and date their treatment of the question from the formal surrender of Communist power in February 1990. Candidates may point to the disintegration of the Soviet Union as Baltic States, Georgia and others bid for independence; the attempted coup against Gorbachev and the rise of Yeltsin (elected president of the Russian republic); the continued collapse of the economy and food rationing; the independence of the Ukraine; the creation of the Commonwealth of Independent States and Gorbachev’s resignation which signalled the formal end of the Soviet Union; the freedom of the media and political life; Yelstin’s economic ‘shock programme’ and the economic and social problems of the 1990s. Candidates may distinguish between political, social and economic consequences and between the immediate and longer term consequences.

54

Page 240: History A - The Bicester Schoolthebicesterschool.org.uk/wp-content/uploads/2015/09/Mark... · 2015-09-28 · History A Mark Schemes for the Units January 2009 H106/H506/MS/R/09J

F962 Mark Scheme January 2010

Crisis in the Middle East 1948-2003 28 Assess the consequences of the first Arab-Israeli War (1948-49).

No set answer is looked for but candidates will need to address the question. Candidates need to identify and analyse a range of consequences and evaluate their relative significance and/or linkages. Candidates may set their discussion in the context of the fighting that had already begun between Zionists and the Arab League even before the British left and the state of Israel was declared. They may refer to immediate consequences of the fighting such as the casualties, the razing of Arab towns and villages within Israel, the defeats inflicted on the Arab League forces, the agreement of cease-fires with Egypt and Syria (but not Iraq). Candidates may point out that there was no formal peace and that therefore the wars of 1956 and 1967 can be seen as continuations of the conflict. Candidates may point out that Israel at the end of the war was larger than that envisaged by the UN partition proposal and that the remnants of the Palestinian state were effectively taken over by Egypt and Transjordan. There was also the plight of the approximately 1 million Palestinian refugees that remained a destabilizing element in the Middle East. Candidates may also point to the consequences for the Arab states that faced internal instability. Candidates may make links between the war and political developments in these states.

29 To what extent have the violent actions of some Palestinians been the main obstacle

to the establishment of a Palestinian state?

No set answer is looked for but candidates will need to address the question. Candidates must deal adequately with the given factor even if they wish to argue that other factors were as or more significant. In relation to Palestinian violence candidates may discuss some of the following: the sporadic attacks of fedayeen in the 1950s, the actions of Yasser Arafat and Fatah, raids in the 1960s, hijacks, Black September, the Munich Olympics, the PLO in Lebanon, the actions of Hezbollah, the First and Second Intifadas, the attitude and actions of Hamas, and so on. Candidates may argue that violence was the Palestinians only effective weapon, that it brought the issue of the Palestinians to world attention and made possible an eventual two-state solution (proposed by Arafat). However, candidates may also argue that the impact of violence made settlement with the Palestinians impossible. Others may argue that there were other obstacles to the establishment of a Palestinian state, not least the attitude and actions of Israel and the unwillingness of the USA to force a solution. Candidates may also discuss the policies and attitude of the PLO, the significance of the Six Day War and Resolution 242. The issue of the Palestinians was, of course, also bound up with the other causes of distrust and friction within the Middle East.

55

Page 241: History A - The Bicester Schoolthebicesterschool.org.uk/wp-content/uploads/2015/09/Mark... · 2015-09-28 · History A Mark Schemes for the Units January 2009 H106/H506/MS/R/09J

F962 Mark Scheme January 2010

56

30 Assess the reasons for United Nations military action against Iraq in 1991.

No set answer is looked for but candidates will need to address the question. Candidates need to identify and analyse a range of reasons and evaluate their relative significance and/or linkages. Candidates may point to growing concerns, especially in Israel and the US, over the development of Scud missiles with the potential to carry nuclear warheads, Saddam’s announcement of a new rocket with a range of over 1200 miles and the misspending of agricultural credits. Meanwhile the Soviet Union was reducing its involvement in Iraq under Gorbachev and relations with Britain deteriorated (eg over the so-called super-gun and the execution of a British journalist by the ‘butcher of Baghdad’). However, the main issue was Iraq’s actions over Kuwait. Despite diplomatic pressure from both the US and the Arab League Saddam did not back off from his threats to Kuwait and in August 1990 he invaded, annexed the state (and its oil) and held foreigners as hostages. Candidates may explain how the coalition against Iraq was built up with the West, most Arab states and Gorbachev (unwilling and unable to intervene). Candidates may suggest that it was the end of the Cold War that enabled the UN Security Council to vote for an ultimatum. Candidates may argue that underpinning all the minor and more obvious reasons to justify UN action was the question of oil supply security.

Page 242: History A - The Bicester Schoolthebicesterschool.org.uk/wp-content/uploads/2015/09/Mark... · 2015-09-28 · History A Mark Schemes for the Units January 2009 H106/H506/MS/R/09J

F963 Mark Scheme January 2010

F963 British History Enquiries

Option A: Medieval and Early Modern 1066-1660

1 The Normans in England 1066-1100

(a) Study Sources A and C Compare these Sources as evidence for military campaigns fought by the Norman Kings of England. [30]

Both sources show that the kings relied on taking important castles. In both sources the king of France is appealed to by the enemies of the English and in both peace is made. In both members of the royal family are pitted against each other. The sources differ in that in A the Bretons fought hard and defeated William I, a rare event for the victor of Hastings, whereas in C William Rufus was successful, albeit through bribery. In A William was even worsted by his son Robert. Similarly in A the intervention of the French king was successful but in C he was bought off by Rufus. C has more to say about the attitudes of the nobles some of whom wanted a peace made. Source A mentions the corollary of war in France, an invasion by the Scots and the results of this. Both authors are chroniclers who list and describe. They have similar attitudes to the fighting, but A is more focused on the impact the events in France had on England, whereas C shows the bad effects of the war in Normandy and the impact of land ownership on both sides of the Channel. In terms of provenance A is Anglo-Saxon and C is Anglo-Norman but both approach the campaigns from an English perspective. As an Englishman, Henry of Huntingdon stresses the danger to England of the threat from the Scots while the king is Normandy. William of Malmesbury is only referring to the Norman perspective. Source A might be seen to provide a broader perspective.

(b) Study all the Sources

Use your own knowledge to assess how far the Sources support the interpretation that the problems arising from the linking of Normandy and England were caused mainly by members of William I’s own family. [70]

Sources A, C and D make it clear that Robert of Normandy was a troublemaker, leading a rebellion and being cursed by his father in A, appealing to the French king for help against his brother in C and being castigated as a rebel in D. In D he is also condemned for his failure to keep order. Source B outlines the effects in England of William’s absence when his regents, one of whom was his half brother, exploited their position. This evidence agrees with the statement. But in C Robert lacked courage to resist so his threat may not have been that serious. Source C suggests there were other reasons, describing a civil war that lasted a long time and showing that the nobles took advantage of the situation. But Source C also shows that the nobles saw their interests were served by peace, In Source A there is intervention from the French king, taking advantage of the situation and Source C supports this point. Also in C the Scots were causing problems. Source D shows the problems inherent in the linking of England and Normandy and the dual loyalties of the barons. There is a hint that Rufus himself was less effective than his father. Candidates could argue that despite the assertions in B, William I was a strong ruler in both England and Normandy as D makes clear. Odo was later overthrown and Lanfranc was a better regent. Candidates could refer to the role of Ranulf Flambard under Rufus. The fear of the barons that they might have different overlords in England and Normandy is backed up in the sources and candidates might argue that the greatest problems arose in the period 1087-95 when the kingdom and the duchy had different rulers.

57

Page 243: History A - The Bicester Schoolthebicesterschool.org.uk/wp-content/uploads/2015/09/Mark... · 2015-09-28 · History A Mark Schemes for the Units January 2009 H106/H506/MS/R/09J

F963 Mark Scheme January 2010

2 Mid-Tudor Crises 1536-1569 Royal Advisers 1540-1569

(a) Study Sources A and E

Compare these Sources as evidence for relations between monarchs and their advisers. [30]

Focus: Comparison of two Sources. No set answer is expected, but candidates need to compare the contents, evaluating such matters as authorship, dating, utility and reliability, so using the Sources ‘as evidence for …’. The headings and attributions should aid evaluation and reference to both is expected in a good answer.

The Sources agree that the advisers of both monarchs were ambitious, ‘seeking their own profit’ and with ‘projects’ the king wished to stop succeeding in Source A, and ‘if you think to rule’ in Source E. Both Sources agree that monarchs had to deal firmly with advisers – ‘reproaching’ them in Source A and humbling Dudley in Source E. They also agree that advisers did serve the monarch well: in Source A Cromwell is referred to as ‘the most faithful servant’ Henry had had and in Source E Elizabeth has ‘many servants whom she favours’. In Source A Henry is ‘gloomy’ and mistrusts his advisers, as in Source E Elizabeth mistrusts Dudley. The Sources also agree that monarchs had the power to make advisers afraid – Dudley was ‘so alarmed’ in E, perhaps because of examples like the execution of Cromwell in Source A.

But the Sources disagree concerning the extent of control monarchs exercised over their advisers. Source A suggests that Henry had been persuaded to execute Cromwell by the lies of Cromwell’s enemies, whereas in Source E Elizabeth is said to have deliberately created parties and factions to ‘divide and rule’. In Source E the public reprimand is to Elizabeth’s favourite, while in Source A, Henry’s reprimand is to the advisers who have made him remove his minister. Contextual knowledge may be used to show understanding of the comparison. Perhaps Henry is wishing to salve his conscience. His annoyance at the Cleves marriage undermined Cromwell’s favour with the king, and the Bill of Attainder against him was introduced under the king’s prerogative. Dudley was Elizabeth’s favourite, recently made Earl of Leicester and a rival of Cecil on the Privy Council, but she had no intention of marrying him, as he hoped, because he was a ‘mere’ subject. Both Sources therefore suggest that factional rivalry played a part in the relationship between monarchs and their advisers.

The provenance of the Sources may be used to determine which is more useful or reliable for explaining relations between monarchs and their advisers. Neither Source can be fully trusted. In Source A this is because of the nationality of the author and the context of the Habsburg-Valois wars. As ambassador at a time when the Anglo-German alliance had collapsed, he might be reassuring leading advisers, such as Montmorency, that Henry was weak. Late in Henry VIII’s reign the conservative and reform factions were influential due to his poor health and increasing age. Source E is written by a Stuart politician who was a child at the time of the incident he describes. Source E might seem to be less useful evidence as the assertion of Elizabeth’s strong control of her advisers has little factual support except this one incident. In Source A there is more balance between an infirm king who knows the good servants from the flatterers, and can act forcefully, as with Cromwell, but can also be manipulated. No set conclusion is expected, but substantiated judgement should be reached for the top levels of the Mark Scheme.

58

Page 244: History A - The Bicester Schoolthebicesterschool.org.uk/wp-content/uploads/2015/09/Mark... · 2015-09-28 · History A Mark Schemes for the Units January 2009 H106/H506/MS/R/09J

F963 Mark Scheme January 2010

(b) Study all the Sources. Use your own knowledge to assess how far the Sources support the interpretation that advisers could be trusted to serve royal interests between 1540 and 1569. [70]

Focus: Judgement in context, based on the set of Sources and own knowledge. Successful answers will need to make use of all five Sources, testing them against contextual knowledge and evaluating their strengths and weaknesses, any limitations as evidence. A range of issues may be addressed in focusing upon the terms of the question but no set conclusion is expected.

The Sources contain references to both sides of the argument, so they may be grouped by interpretation. The supporting view is shown predominantly in Source D and to varying extents in Sources A, B and E, whereas the opposing view predominates in Sources A, B, and E.

The supporting view is that Tudor royal servants helped establish strong system and served the monarchy well. Source D suggests that at the start of her reign, as a young female ruler, Elizabeth I trusts Cecil as hardworking, honest and discreet. Own knowledge might add that he was an experienced politician and a committed Protestant to help her implement a Church Settlement. However, at the time of the Source England was religiously divided, and he had not yet been tested, so these are only Elizabeth’s hopes. Henry VIII, in Source A, is reflecting on the loyalty of Cromwell, but his execution suggests that Henry had previously felt he had failed him. In Source C, Renard himself is acting as a loyal adviser of Mary, keeping Charles V informed should problems require his support for her and his son Philip. Here the context is less typical, as the Spanish marriage has caused factions at court to develop a xenophobic flavour unlike those in Sources A, B and E. Some candidates might point out that factions were part of the normal running of all governments. Dudley, in Source E, is humbled to be a loyal servant because of the Queen’s forcefulness. But this may be Naunton’s attempt to make a contrast with the weaker Stuart kings. In Source B, Somerset claims to be serving Edward VI by encouraging the common people to rise in support of the Lord Protector, who is taking the side of the people oppressed by the greedy gentry class, though the authors of the Source see his action as undermining the monarchy and nation.

So the Sources also support the opposing view. Somerset’s claims are unconvincing as described in the charges against him in 1549, made by his enemies on the Privy Council, the authors of Source B. They blame him for political and social instability, though knowledge of rebellions might be added to suggest other causes. In Source B he is said to have acted against the King’s interests and for his personal ambition during the royal minority. Knowledge of provenance might reveal the Privy Council’s own ambition, to oust Somerset in favour of Northumberland, who might be added, from knowledge, to best exemplify an adviser seeking power against the interests of the Tudor dynasty by placing Lady Jane Grey on the throne in 1553. In Source C, Paget and Arundel are revealed to be scheming against Gardiner, the chancellor. Paget is also out of favour with Mary and cultivating Philip’s support, leading to factions and disunity in the Council, hence political instability. The author, Renard, however, is Mary’s close adviser, so may not be objective. Source A suggests factions had caused instability due to their schemes and personal ambition, whereas in Source E, individual favourites are suggested as failing to prosper under Elizabeth, who is fully in control of her advisers using the patronage system. A supported overall judgement should be reached on the extent to which the Sources accept the interpretation that advisers could be trusted to serve royal interests. No specific judgement is expected.

59

Page 245: History A - The Bicester Schoolthebicesterschool.org.uk/wp-content/uploads/2015/09/Mark... · 2015-09-28 · History A Mark Schemes for the Units January 2009 H106/H506/MS/R/09J

F963 Mark Scheme January 2010

Candidates are likely to consider a range of themes within the Sources: factional rivalry, disunity, personal ambition, manipulation and loyal service. They are likely to set the Sources within the context of strong or weak monarchies, perhaps due to age or gender. It is up to candidates to assess and decide upon relative importance here, there being no set conclusion.

3 The English Civil War and Interregnum 1637-1660

The Second Civil War and the Trial of King Charles I

(a) Study Sources A and C Compare these Sources as evidence for issues involved in the King’s negotiations during 1647-8. [30]

Focus: Comparison of two Sources. No set answer is expected, but candidates need to compare the contents, evaluating such matters as authorship, dating, utility and reliability, so using the Sources ‘as evidence for …’. The headings and attributions should aid evaluation and reference to both is expected in a good answer.

Both Sources are examples of the terms negotiated by the King. The provenance of the Sources should be integrated into the comparison. There are both similarities and differences in the terms discussed. In Source A, the King negotiates with the Scots to gain their military support which he had already had earlier with the Solemn League and Covenant, whereas in Source C he negotiates with commissioners sent by Parliament, who are hostile and in a position of strength after his defeat in the Second Civil War. Source A is signed by Charles I himself so is more reliable than Source C, which is a later record of hearsay information perhaps justifying a husband’s involvement in the later execution of the King. The purpose of the King in both sets of negotiations is to restore his power. In Source A the Scots ‘will restore him to his government, royal rights and full revenues’ and Source C refers to ‘terms they agreed for his restoration’.

Source A includes terms concerning religion: the King wishes ‘to preserve and establish religion’ and Source C agrees ‘He would not give up the bishops, but only lease out their revenues’. However, in Source A ‘Separatists and Independents will be suppressed’ and the King will set up ‘a Presbyterian system for three years’, whereas Source C takes an opposite view on bishops. Hutchinson is giving her husband’s view, that such terms ‘betrayed their whole cause’. Context might be used to explain the divisions which Hutchinson shows, between those Presbyterians who wished to continue negotiations and the Army officers who now took a hard line attitude towards the King as ‘that man of blood’, complicating the process of negotiation in 1648. Source A records the treaty with the Scots which began the Second Civil War, the basis of the charge that the King had caused bloodshed by waging war against his people, and this charge seems to have been put to him in Source C, which states that the King ‘acknowledged himself guilty of the blood spilt in the late war’. His untrustworthiness is shown in Source C, by his request that this concession ‘should not be used against him’. In Source A, he accepts Presbyterianism, though himself a crypto-Catholic, and concedes valuable fortresses to the Scots, showing his untrustworthiness.

Both Sources suggest that the King tried to use bribery to buy his restoration to power. In Source A, he offers the Scots fortresses, ‘arrears of £200,000 and all the expenses of their army in this future war’. In Source C he offers ‘great honours and offices’. But whereas in Source C, Colonel Hutchinson’s view is that acceptance of the terms is ‘inconsistent with the liberty of the people’, in Source A, Charles is claiming that his Engagement with the Scots is to ‘defend the privileges of Parliament and the liberties of the subject’. Context might be used to discuss this claim in the light of national and religious hostility between

60

Page 246: History A - The Bicester Schoolthebicesterschool.org.uk/wp-content/uploads/2015/09/Mark... · 2015-09-28 · History A Mark Schemes for the Units January 2009 H106/H506/MS/R/09J

F963 Mark Scheme January 2010

the Scottish Presbyterians and the Independents in the Army and Parliament, who would lose their liberty by the terms of the Engagement.

So the Sources are very different in provenance: authorship, date and context, but many of the issues involved remain the same. In relation to the attempt to gain a settlement with the King, Source C is better evidence, despite its unreliability. No set conclusion is expected, but substantiated judgement should be reached for the top levels of the Mark Scheme.

(b) Study all the Sources

Use your own knowledge to assess how far the Sources support the interpretation that the King was brought to trial because after 1647 he could no longer be trusted. [70]

Focus: Judgement in context, based on the set of Sources and own knowledge. Successful answers will need to make use of all five Sources, testing them against contextual knowledge and evaluating their strengths and weaknesses, any limitations as evidence. A range of issues may be addressed in focusing upon the terms of the question but no set conclusion is expected.

The untrustworthiness of the King was also a reason for his trial. Negotiations failed partly because Parliament and the Army did not trust him to keep his promises. He also is said, in Source C, to have tried to bribe the commissioners to get the treaty signed by Parliament. This certainly supports his view in Source E that he was leaving behind a ‘corruptible crown’. His claim, in Source E, that he did not rule by the power of the sword might be evaluated by cross-reference with Source A. By laying the foundations of the ‘cult of King Charles the Martyr’ with his propaganda in Source E, he is trying to overcome the view in the title.

There are two main charges against the King in his trial in Source D: firstly that ‘He traitorously and maliciously levied war against Parliament’; and was ‘the author and continuer of the unnatural, cruel and bloody wars’, ‘guilty of the treasons, murders, burnings and damages to this nation caused by these wars’. Secondly, that he tried ‘to erect an unlimited and tyrannical power’ and ‘overthrow the people’s liberties’. In Source E the King protests his innocence saying that he did not ‘rule in an arbitrary way, to have all laws changed according to the power of the sword’. Thus he refutes the second charge of trying to create an unlimited and tyrannous power. He claims to have tried to protect ‘the people’s liberty’ and ‘their life and goods’.

He denies that he acted in an arbitrary way. Cross reference might be made with Source A, where he invites a Scottish army into England to restore him, but here too he claims to be protecting the ‘liberties of the subject’. This claim might be evaluated in the light of English views of the Scots and his promise to establish Presbyterianism for three years. There is evidence in Source A that the King caused the Second Civil War by signing the Engagement with the Scots, so the charge, which is repeated in Sources B and C, may have some foundation.

But, on the other hand, the introduction to Source D shows that the High Court set up to try Charles was created by a Rump of the Commons after Pride’s Purge in December 1648. John Bradshaw represents this, and the King did not recognise the court. Thus the power of the Army lay behind the trial of the King.

The king had escaped from Army control when he called in the Scots with the Engagement, so Army power might be seen to lie behind this desperate move. However, there were still those parliamentarians who wished to continue to negotiate with the King, as revealed in Source C, and there was considerable debate in Parliament about the

61

Page 247: History A - The Bicester Schoolthebicesterschool.org.uk/wp-content/uploads/2015/09/Mark... · 2015-09-28 · History A Mark Schemes for the Units January 2009 H106/H506/MS/R/09J

F963 Mark Scheme January 2010

62

acceptance of the terms negotiated by the Parliamentary commissioners in the autumn of 1648. The provenance of this Source is unreliable, as there may be a purpose of justifying Lucy Hutchinson’s husband, and showing that the parliamentary cause would be lost if a treaty with the King were signed. However, if it is taken at face value, pressure from the Army played an important part in the decision to end negotiations.

As for the decision to bring the king to trial, Source B suggests that some parts of the Army had prejudged his guilt. Source B reveals that some elements of the Army viewed the King as ‘that man of blood’ as early as the Windsor prayer meeting in April 1648. The ‘reopened’ negotiations mentioned in Source C was unpopular with them, as it came after the defeat of the King in the Second Civil War ‘after his defeat and capture’, and the repeal of the ‘Vote of No Addresses’ which had originally been passed under pressure of the Army in January 1648. Therefore the power of the Army had played a part in the ending of negotiations, trying the king and finding him guilty. However, the author of Source B is an agitator, writing long after the event, emphasising the danger of the Second Civil War. The context of Source B is widespread revolts against the Army throughout 1648, so that may suggest the Army was not so powerful at that time. Certainly this context hardened their views of the king’s role in events.

A supported overall judgement should be reached on the extent to which the Sources accept the interpretation that the King was brought to trial because after 1647 he could no longer be trusted. No specific judgement is expected.

Candidates are likely to consider a range of themes within the Sources, such as the King’s part in the outbreak of the Second Civil War, his untrustworthiness, the extent of his defence of the law, the nature of his rule and the part played by the Army and the Scots. They are likely to set the Sources within the context of events, such as the Vote of No Addresses and Pride’s Purge. It is up to candidates to assess and decide upon relative importance here, there being no set conclusion.

Page 248: History A - The Bicester Schoolthebicesterschool.org.uk/wp-content/uploads/2015/09/Mark... · 2015-09-28 · History A Mark Schemes for the Units January 2009 H106/H506/MS/R/09J

F963 Mark Scheme January 2010

Option B: Modern 1915-1945 1 The Condition of England 1815-1853

(a) Study Sources A and E Compare these sources as evidence for attitudes towards improving the condition of factory workers. [30]

No set answer is expected, but candidates need to compare the contents, evaluating such matters as authorship, dating, utility and reliability, so using the sources ‘as evidence for…’. The headings and attributions should aid evaluation and reference to both is expected in a good answer. Both sources agree on the need for improvement, emphasising the need for some sort of education, although E is vaguer (the 3 ‘R’s in A, an ‘increase of intelligence’ in E). They also agree that the way forward is for factory workers to become ‘industrious’ (A) or to work in E. However they differ on how best to do it. There is disagreement on what holds workers back. The stress in A is on poor living and working conditions whilst in E it is government restrictions on both trade and labour. For Bright trade barriers, monopolies and factory reforms all deny workers the opportunity to find work. Similarly they disagree on labour. Owen in A has ended child labour under 8 and although Bright in E doesn’t refer to this he is adamant that all be given the opportunity or ‘liberty’ to work. The way forward for Owen (A) is for employers to take responsibility, providing education from 5 to 10, better streets and housing, restricting alcohol and ending the Truck System by effective bulk buying and then selling at low rates. For Bright in E it is the free market which allows both employers and workers to make individual choices (workers voluntarily deciding to limit their work). Owen stresses the employer as the key, Bright both the individual worker and employer operating in a free market. These differences are explained by the provenance. Both are employers and mill owners and perhaps significantly neither mentions better wages as the way forward, although Owen does refer to the need to lower the price of necessities. Both are radicals and not necessarily typical of their class, although Bright will speak for most of his type. There the similarities end. Owen was a radical paternalist and early socialist who believed in cooperation rather than competition as the basis for the new industrial society. In contrast Bright was an MP and spokesman of the northern millowners who campaigned for Free Trade and a society based on the principles of laissez faire, hence his stress on achieving a free market as the means of improvement (‘markets of the world’, ‘liberty to work’). Owen’s comments are based on social experimentation at New Lanark and he is keen to use the experience to ‘prove’ his case – that cooperation will lead to workers who are ‘industrious, faithful and kind’. This may have led to some exaggeration of the beneficence of his changes given that he is arguing a case in his book ‘New View of Society’. In contrast Bright is talking of the economy in general. He assumes that improvement will come not from employer paternalism but from individual effort, if only government would ‘give them the power’. Both are optimistic. There is also a difference in the dates. Owen is talking of an earlier experiment, before the first proper Factory Act in 1833, whilst Bright is opposing Graham’s Factory Act, the 3rd main instalment of workplace change, in a parliamentary speech that is strong on rhetoric. Both are useful pieces of evidence on how best to improve the industrial working class. Although there were other examples of paternal millowners Owen may be less typical than Bright, although many employers may not have taken Bright’s more theoretical view, preferring to stress profit.

63

Page 249: History A - The Bicester Schoolthebicesterschool.org.uk/wp-content/uploads/2015/09/Mark... · 2015-09-28 · History A Mark Schemes for the Units January 2009 H106/H506/MS/R/09J

F963 Mark Scheme January 2010

(b) Study all the Sources Use your own knowledge to assess how far the Sources support the interpretation that factories in the period 1831 to 1844 were places of exploitation for all workers. [70]

Successful answers will need to make use of all five Sources, testing them against contextual knowledge and evaluating their strengths and weaknesses, including any limitations as evidence. A range of issues may be addressed in focusing upon the terms of the question but no set conclusion is expected.

The sources may be used in a variety of ways to assess the interpretation that factories were places of exploitation for all. It is likely that candidates will see Sources B and D as more supportive of this view whilst A, C and E are critical of it, pointing to Factories as more positive places, the ‘best schools’ in the words of McCulloch in C. Evaluatively C and E may be considered less credible given their abstract and theoretical nature (both are referring to the economy). McCulloch in C is an economist who may well have less experience than Bright and Owen (A and E) in the running of factories. However they too have interests that lie elsewhere (Bright’s middle class radical campaign against aristocratic government and the Corn Laws and his position as an MP; Owen’s trade union and written work). B and D may be considered more useful given Horner’s hands-on evidence as the key Factory Inspector a decade into his work, and the report of the Leeds newspaper with a slightly better off readership in Yorkshire, yet whose tone appears sympathetic to the cause of reform (‘aptly named little victims’, an acceptance of the case that exploitation was prevalent). The case against factories is mainly to be found in B and D. The Manchester demonstration in B, organised by the Short Time Committees, was designed to stress the cruelty of the system for children and thus implicitly for men as well. It deftly demonstrates the experience of radicalism, the phrasing linking to an Anti Slavery movement that was about to triumph in 1833 (candidates may refer to Oastler’s Yorkshire Slavery article) – ‘Am I not a brother and a man’. Children sing of the 12 hours they work and emotively carry whips and straps. However this is obviously propaganda, milking a Manchester audience for support and clearly succeeding. It is the sort of ‘exaggeration’ that McCulloch in C complains of. Nonetheless the response and scale might suggest deeply felt grievances with workers yet to learn to love the factory. Horner in D may be considered better evidence given the provenance. It is based on much visiting and inspection and although the true picture may have been hidden on occasions his comments command authority and he was listened to by governments. However Horner does not comment on children, confining his comments to adult labour. He does not necessarily accept the argument that conditions and hours for men needed interference, but he is concerned that the rise of female labour is based on abuse (their alleged physical incapacity and the ‘deterioration of their health’) and that this has an impact on adult men (neglected domestic duties). He believes that women, as unfree agents, have been exploited as cheap labour. Whilst not a supporter of 10 hours, he does consider 12 exploitative. Candidates could also use Owen in A. Given that he refers only to his own experiment at New Lanark, one can infer exploitation was common elsewhere (‘the practice of employing children…’; a truck system that needed challenging, via his Cooperative movement, to prevent employers controlling their workforce as consumers). Own knowledge could extend the discussion here, pointing to findings in the reports of 1831 and 1832 and to the legislative struggle before and beyond 1833. Candidates could also note that McCulloch admits there is some foundation to these shocking reports and that ‘abuse’ is ‘certain’ in some factories (and own knowledge may point to the older, smaller and more water powered ones by the 1830s and 1840s). Bright in E could be interpreted as a theoretical justification of worker exploitation, given that no mention is made of wages and that labour shortages were increasingly a thing of the past.

64

Page 250: History A - The Bicester Schoolthebicesterschool.org.uk/wp-content/uploads/2015/09/Mark... · 2015-09-28 · History A Mark Schemes for the Units January 2009 H106/H506/MS/R/09J

F963 Mark Scheme January 2010

The case that hours and conditions were not necessarily exploitative for all can mainly be found in A, C and E. Yet Owen in A is not referring to a typical factory and candidates might discuss whether there were other model factories. He is certainly keen to include children and adults in his improving measures. C and E carry the main weight of the pro factory argument, as to be expected from wealthy men and employers who invested their time and careers in advancing the cause of a factory based society governed by the free market and self made men, offering the blessings and opportunities of hard work. Clearly they are concerned to resist any attempt by government to interfere via factory acts. However McCulloch was right to challenge some of the stories of exploitation emanating from the factory reformers – professional cripples and the coaching of witnesses by reformers was a tactic used in the early 1830s. Non factory labour (workshops and agriculture) may well have been just as exploitative, and with worse conditions for all, than the factories, the owners of which often felt unduly focused on. McCulloch refers to ‘other classes’ and comments on the alternatives – beggary in the streets and crime. He is convinced that the Factories especially have disciplined, ordered and protected children. Bright in E shares this view although candidates could question the reality behind his rhetoric. He is short on evidence, long on claims (‘speedily make them independent’?). Could workers look forward to the sort of means which would enable them to choose more ‘recreation and enjoyment’? Horner in D will not accept less hours for adult men whilst B could be dismissed as unreliable anti factory propaganda. The Sources provide mixed messages and much will depend on an evaluation of their respective worth and the relative conditions prevailing in the variety of workplaces at the time. There is much special pleading in all the sources.

65

Page 251: History A - The Bicester Schoolthebicesterschool.org.uk/wp-content/uploads/2015/09/Mark... · 2015-09-28 · History A Mark Schemes for the Units January 2009 H106/H506/MS/R/09J

F963 Mark Scheme January 2010

2 The Age of Gladstone and Disraeli 1865 -1886

(a) Study Sources B and D Compare these Sources as evidence for the attitudes towards Forster’s 1870 Education Act. [30]

No set answer is expected, but candidates need to compare the contents, evaluating such matters as authorship, dating, utility and reliability, so using the sources ‘as evidence for…’. The headings and attributions should aid evaluation and reference to both is expected in a good answer. The sources agree on the importance both of education and the 1870 Act, as to be expected from two such advocates, but both have reservations. These are religious. Bright in B is convinced that the Act was designed to enhance the role of the Church of England and to continue a system of religious voluntary schools that favoured Anglicans. As such the Nonconformists had long given up a demand for equity and instead put their trust in a nondenominational system that would strip the Church of its pre-eminence in elementary education, hence Bright’s hope that Board schools would be established everywhere and voluntary schools induced to come under their control. This explains Bright’s frustration that in many areas, especially rural ones, nondenominational Board Schools would not be established and thus a choice not forthcoming for nonconformists. Barry in D partly agrees but on different grounds, stressing those more respectable workers will want to send their children to largely Anglican voluntary schools, possibly on grounds of snobbery and class. He thus agrees with Bright that the Act in part advantages denominational Anglican education but he considers this to be the fault of Nonconformists like Bright. His argument is that by strongly objecting to fee payments for the poor to attend Voluntary schools they confine them in effect to what is perceived to be a second class Board School education. Where Barry in D disagrees is his view that the Act has dealt a huge blow to the religious voluntary system, financially (no building grants and a failure to plug into the local rates as an assured means of securing income) and religiously (the Cowper Temple conscience clause allowing withdrawal and an enforced nondenominational religion to be taught in Board schools). These differences are explained by provenance. Bright, as a key nonconformist MP, is denouncing the Act in a speech, possibly to a nonconformist audience. He represents militant nonconformity, anxious to achieve religious equity with established Anglicanism and his tone is evidence of this. In contrast Barry is more the educational expert and Headmaster and, although he approaches it from an Anglican viewpoint, he is concerned to make more thoughtful points about its impact. In part a little distance in date helps him. In 1873 Bright, who resigned as President of the Board of Trade, is still the outraged leader of thwarted Nonconformity whilst in 1874 Barry is more impressed with its secular, nondenominational drift. He can see, as Bright cannot, that the Anglican Voluntary schools will benefit from class distinctions (whether he approves is another matter) as better off workers seek to segregate their children from the urban poor, destined for the Board schools. On these grounds it may be the better evidence for more informed and balanced attitudes 4 years into its operation and on the eve of an election that saw Bright’s Nonconformists contribute to Gladstone’s downfall.

(b) Study all the Sources

Use your own knowledge to assess how far the Sources support the interpretation that the reforms of 1868-74 were designed to satisfy interest groups which supported the Liberal Party. [70]

Successful answers will need to make use of all five Sources, testing them against contextual knowledge and evaluating their strengths and weaknesses, including any limitations as evidence. A range of issues may be addressed in focusing upon the terms of the question but no set conclusion is expected.

66

Page 252: History A - The Bicester Schoolthebicesterschool.org.uk/wp-content/uploads/2015/09/Mark... · 2015-09-28 · History A Mark Schemes for the Units January 2009 H106/H506/MS/R/09J

F963 Mark Scheme January 2010

Sources A and in part B and E suggest that the reforms were intended to satisfy liberal interest groups and that those groups played an important part both in their occurrence and their drafting. These sources may be considered effective in their contribution, coming as they do from a variety of liberal or moderate and reformist views (Arnold from the vantage point of reason and justice, Bright from militant middle class nonconformity and Matthews from a modern historian’s view with a close understanding of Gladstone’s thinking). In contrast Sources C, D and in part B and E would suggest otherwise – that the reforms were designed to attack vested interest wherever it might be. They too come from a variety of liberal angles and provide a balanced view, Matthews in E being a case in point. He provides a balanced account of the Trade Union reforms. The satisfying of Liberal interests is seen in Arnold (A) who strongly suggests that the key interest group to be appeased were the nonconformists, citing their influence on Irish Disestablishment in 1869. Arnold would clearly have preferred a redistribution of property rather than disendowment and cites liberal concepts of equity and justice. Knowledge might support this. Gladstone, it has been argued, used Irish Church reform to unite a divided Liberal party in the wake of the debacle over the 2nd Reform Act and to win the 1868 election. Not just nonconformists were pleased by Disestablishment. Radicals and Whigs also united around it. However whether Arnold’s ideas were realistic is another matter. Clearly Forster in 1870 tried to satisfy the nonconformists and the Birmingham based National Education League over education but Bright in B and the League would settle for nothing less than an ending of grants to the voluntary schools and a compulsory, free, state, non denominational elementary educational system. They looked to his Board Schools to do it and there is evidence that he thought he had done enough to satisfy them. In E Bruce is clearly acting to address Lib/Lab concerns over Union legal status and funds, thinking it achieved by the two acts of 1871. Matthews makes it clear that the Liberal reforms intended to allow moderate peaceful picketing and negotiation, only to be frustrated by conservative and restrictive rulings in the Courts. Knowledge might also be used to demonstrate Whig interests (government posts and a more moderate Irish Land Act) being satisfied. Other Liberal and nonconformist concerns were addressed over alcohol, the aristocratic and patronage ridden army and over the principle of merit in Universities, Civil Service and the Army. The alternative view is provided in C, D and in parts of B and E – that far from pandering to interest groups, Liberal or otherwise, the reforms took on vested interest and indeed alienated key Liberal groups, notably the nonconformists, the Drink Trade and the Lib/Labs of the respectable artisan upper working class with their newly formed TUC. Bright in B is outraged by the apparent Anglican victory and candidates could point to clause 25 and other loopholes that allowed rate subsidy for Voluntary schools via poor pupils and became the later focus for nonconformist hostility. Far from satisfying those the reforms seemed to ride roughshod over this particular ‘interest’. The Economist in C is staggered at the Licensing Act’s treatment of both the brewers and the working class. It stresses how bold it was to take on such a vital electoral interest. Gladstone blamed his own second place in the Greenwich election in 1874 on this act – the torrent of gin and beer! Barry in D can be interpreted either way – that the Liberals achieved a balance between Anglican and nonconformist interests or that it was too concerned at economising by not building Board schools throughout and therefore did advantage Anglicans whilst simultaneously undermining their catechism. Matthews in E points to the lack of interest Gladstone and the Cabinet (or was it lack of agreement) had over addressing worker concerns over the Criminal Law Amendment Act. It would appear that the reforms were not that interested in pandering to specific Liberal interest groups. They were prepared to live with Union anger, something Disraeli was easily able to capitalise upon in 1875. Sources A to D are all varieties of Liberal thinking and provide a mixed message as to the focus of the reforms. Only A suggests out and out pandering to sectional interest and Arnold stood somewhat aloof from mainstream Liberalism. The rest, including the historian of Gladstone, Matthews, suggest a more mixed set of motives and, indeed, the antagonism of some liberal groups, whether Nonconformist, TUC, Brewer or Whig Irish landowner, to achieve retrenchment, reform, civic equality and justice.

67

Page 253: History A - The Bicester Schoolthebicesterschool.org.uk/wp-content/uploads/2015/09/Mark... · 2015-09-28 · History A Mark Schemes for the Units January 2009 H106/H506/MS/R/09J

F963 Mark Scheme January 2010

3 The Fortunes of the Conservative Party 1900-1914

(a) Study Sources C and D Compare these Sources as evidence for attitudes towards Tariff Reform. [30]

Focus: Comparison of two Sources No set answer is expected, but candidates need to compare the contents, evaluating such matters as authorship, dating, utility and reliability, so using the Sources ‘as evidence for...’. The headings and attributions should aid evaluation, and reference to both is expected in a good answer.

Content: Both Sources refer to Conservative opinion. But while Source C supports Tariff Reform and the necessity of Food Taxes, Source D is opposed to both. Source C is confident that Tariff Reform is the right policy, and claims that it is most successful when discussed openly and vigorously. The emphasis is on Tariff Reform protecting British jobs. But this argument does not play well with the Lancashire county association (Source D). The Source claims that ‘Lancashire hates Tariff Reform’, and candidates might pick up on the point that generally the cotton industry supported both free trade and low food prices. According to Lord Derby, Tariff Reform has been a divisive issue in the party, especially since the crushing electoral defeat of 1906, and now further failure in the two elections of 1910.

Provenance: Source C. The background is the General Election of January 1910, when the Conservatives narrowly failed to defeat the Liberal Government. Austen Chamberlain had taken over leadership of the Tariff Reform movement from his father Joseph Chamberlain. Here, he encourages Balfour to continue with the controversial policy. As this election was fought against the background of the crisis over the People’s Budget and the House of Lords, one might wonder if Chamberlain is exaggerating the importance of Tariff Reform in the minds of the electorate. However, at this stage, it remains true that the supporters of Tariff Reform (‘whole-hoggers’) are still an influential group in the Conservative Party. Source D. Nearly three years later, Balfour has gone, having initially promised a referendum on Tariff Reform. Bonar Law has now withdrawn this offer. Hence the anger in Lancashire. The source reference is local, but the dangers of a split in the party are wider. Walter Long was a moderate supporter of Tariff Reform. But Derby hopes he will respond to the danger. Soon after this, Bonar Law (always a ‘whole-hogger’) is forced to abandon Tariff Reform. A good answer may not require all of this information. The essential point is the potential split in Conservative leadership (and support) arising from these differing attitudes to Tariff Reform. The dates are important in bringing this out.

(b) Study all the Sources

Use your own knowledge to assess how far the Sources support the interpretation that the issue of cheap food was the main reason for fluctuating working class support for the Conservative Party between 1900 and 1914. [70]

Successful answers will need to make use of all five Sources, testing them against contextual knowledge, and evaluating their strengths and weaknesses and any limitations as evidence. A range of issues may be addressed in focusing upon the terms of the question, but no set conclusion is expected.

Fluctuating working class support for the Conservative Party is much in evidence during this period. After victory in the Khaki Election of 1900, the Conservatives faced many difficulties in an era of increasing working class political influence. These problems included: The unpopular policies of the Balfour Government 1900-1905. Electoral defeat in 1906. The domination of the Liberal Party 1906-1914. The rise of the Labour Party. The House of Lords Crisis. Continuing splits over Tariff Reform (the issue of cheap food). The

68

Page 254: History A - The Bicester Schoolthebicesterschool.org.uk/wp-content/uploads/2015/09/Mark... · 2015-09-28 · History A Mark Schemes for the Units January 2009 H106/H506/MS/R/09J

F963 Mark Scheme January 2010

problems of Ulster and Home Rule. And failure to oust the Liberals in the two elections of 1910. And yet, as the modern historian suggests in Source E, the Conservatives continued to enjoy an underlying electoral strength as the natural party of government. Despite the Liberal reforms, and Labour’s rise, there was still strong support for the Conservatives, even among the working classes. Certainly, the issue of cheap food weakened the Conservatives because their policy of Tariff Reform was associated with dearer food, and because not all Conservatives supported the policy. Nevertheless, for sound reasons, it remained a Conservative policy for almost a decade. Besides, there were many other factors for fluctuations in working class support. Most of the Sources can be used on either side of the debate, although a likely grouping is A, B and E questioning the centrality of ‘cheap food’ as the key factor in working class political allegiance, C and D asserting its primacy. Source A clearly explains electoral defeat in 1906 by reference to unpopular policies which alienated the working class (and others). It points to the Conservatives being seen generally as the party of the rich and selfish. However, ‘taxing the food of the poor’ is only one of the reasons given for loss of working class support. Chinese Slavery (selfish imperialism) and Taff Vale (attacks on trade unionism) are seen as equally important. In addition, candidates might suggest that the Quarterly Review (a Liberal magazine) is presenting a one-sided case. It could be argued that Joseph Chamberlain’s Tariff Reform campaign was a serious effort to deal with the problems of Empire, trade and national efficiency. Furthermore, own knowledge could provide examples of useful Conservative legislation appealing to working class support. For example: the Education Act; Wyndham’s Land Act; and the Unemployed Workmen Act. Balfour’s Government also introduced a commission to investigate the poor law. However, there is no doubt that the Liberals won a landslide in 1906. In Source C, Austen Chamberlain, who has taken over his father’s campaign, urges the uncertain Balfour to stick with Tariff Reform, despite recent defeat in the January Election of 1910. Its main thrust is to praise Tariff Reform, and to suggest that electoral defeats have occurred when the party members were insufficiently bold and open about the policy. Chamberlain’s optimistic view is that the working class could be converted to Tariff Reform by an emphasis on the threat of unemployment rather than on the price of food. However, he admits that Food Taxes may well have contributed to the party’s unpopularity with workers: ‘dear food, black bread and horse flesh’. Candidates might point out that the main issue in January 1910 was the House of Lords rejecting the People’s Budget rather than the food issue. In Source D, it is clear that ‘Lancashire hates Tariff Reform’ because of ‘Food Taxes’. Derby, as leader of the county association, would be well aware of working class discontent in Lancashire. However, as a member of the ‘shadow cabinet’, he has wider concerns for his party. The Source concentrates on divisions in the party resulting from the policy of Food Taxes, which are the main problem of continuing to support Tariff Reform. The Source also implies problems of Conservative leadership following Bonar Law replacing Balfour. Bonar Law stuck with Tariff Reform as long as possible. Balfour had been more diffident, and had offered a referendum on the question. Source B provides the clearest evidence against the assertion in the question. In the Source, Sir Edward Stanley plays down the impact of Tariff Reform as an explanation for Conservative electoral defeat in 1906. In particular, Stanley sees the rise of the Labour Party (Lib-Lab Pact implied?), the strengthening trade unions, and the growth of working class independence as the main factors in his own defeat in Lancashire. However candidates might point out that Stanley (the Lord Derby of Source D) will change his opinion by 1912, accepting the damage done to the party by the issue of cheap food. Credit answers that make an effective provenance point here – B, as a private conversation, may more accurately reflect Derby’s position than the letter to Long in D. In Source E, the modern historian explains the broadening popular appeal of the party despite all the difficulties of the period. The Source ignores the damaging issue of cheap food. Instead, it explains the revival of working class support by references to Irish Home Rule, Empire and Employment. The Conservatives were seen as patriotic; and this created

69

Page 255: History A - The Bicester Schoolthebicesterschool.org.uk/wp-content/uploads/2015/09/Mark... · 2015-09-28 · History A Mark Schemes for the Units January 2009 H106/H506/MS/R/09J

F963 Mark Scheme January 2010

jobs, an important contrast in some areas to Lancashire’s cotton industry. They supported the Empire against Little Englanders and Socialists. They backed Ulster. Candidates might also point out that, in the second election of 1910, the Conservatives received more votes than the Liberal Party, which now had to rely on Labour and Irish support to stay in power.

4 (a) Study Sources A and B.

Compare these Sources as evidence for views about self-government for India as expressed in 1931. [30]

Focus: Comparison of two Sources No set answer is expected, but candidates need to compare the contents, evaluating such matters as authorship, dating, utility and reliability, so using the Sources ‘as evidence’ for….The headings and attributions should aid evaluation and reference to both is expected in a good answer.

Content: A offers a view of a moderate and popular Indian policy in contrast to the view in B. Churchill in B speaks of surrender, while A speaks of all parties cooperating to bring a new constitution. There is no mention in A of the loss of trade which Churchill fears. In B India is being abandoned to upper caste Hindus, but in A Muslims and Hindus are in agreement and there is no reference to dominant caste politics. A sees the government acting to meet the political hopes of India, but B sees ignominious surrender to Gandhi and the ‘Brahmins’. Political liberties for all Indians are guaranteed in A, but the Untouchables are being consigned to tyranny in B. A sees peaceful conditions, but by implication that is far from the case in B’s view. B talks in terms of high emotion – ‘shame’ and ‘guilt’ but A is the language of agreement and consensus. The tone is very different. Provenance: A is a statement made in the calm of the House of Commons, pondered and governmental. B is a rousing rhetorical speech in a large hall before committed Imperialist opponents of self government for India. It appeals to emotion and self-interest in a way that B deliberated avoids. A is from a government minister wanting to show progress for moderation; B is from a political maverick, whose stance has alienated him from the Conservative establishment and who rejects moderation and cross-party agreement. The aim of A is to build agreement; the aim of B is to use extra parliamentary pressure to wreck agreement. A is typical of moderate opinion on India, hoping to end the Congress agitation and build on previous acts to share power. B is not very typical of Conservative opinion, but entirely typical of Churchill’s willingness to take on establishments and pursue anti-appeasement policies. A is useful for showing the calm tone of Macdonald’s approach; B is useful for seeing how India led Churchill to the rhetorical excesses which kept him in the political wilderness. Some may judge A to be more useful for establishing the cross-party view of India because B is so unrepresentative of all but a minority. Others may see B as more useful for understanding the disagreements and for seeing that the tone of the opposition to reform in India was not likely to be generally acceptable.

(b) Study all the Sources

Use your own knowledge to assess how far the Sources support the interpretation that Churchill’s policies towards India showed serious misjudgement on his part. [70]

Focus: Judgement in context, based on the set of Sources and own knowledge. Successful answers will need to make use of all five Sources, testing them against contextual knowledge and evaluating their strengths and weaknesses, any limitations as evidence. A range of issues may be addressed in focusing upon the terms of the question but no set conclusion is expected. The debate here is whether Churchill was blinkered by his early experiences in India and his determination to hold India at all costs and whether this led him to serious

70

Page 256: History A - The Bicester Schoolthebicesterschool.org.uk/wp-content/uploads/2015/09/Mark... · 2015-09-28 · History A Mark Schemes for the Units January 2009 H106/H506/MS/R/09J

F963 Mark Scheme January 2010

71

miscalculation. There have also been suggestions that Churchill was blinded by racial prejudice about India. So he has been seen as an out-of-touch imperialist who ruined his career in campaigning against India reform and turned moderate Conservative opinion against him, a misjudgement which made his criticisms of appeasement less effective when applied to Germany. There is a view that Churchill was wiser in practice than in making speeches, but he came in for a great deal of criticism for the governing of India during the war – by Amery, the Colonial secretary and Wavell, the Viceroy and Source D condemns him for the policy adopted over the Bengal famine. Source E attempts some justification and Source C shows that as Prime Minister he softened the approach he took to India. The most critical Sources are A, C and D. Source A by implication shows that the press and public opinion, and both Hindus and Muslims within India were prepared to come together for moderate reform. ‘Some quarters’ – presumably Churchill and his die hard allies – the extremists referred to in C cannot accept the consensus. However, a minister in Benn’s position would want to stress the degree of unity about this issue. The tone of B rather tells against Churchill here and the Imperial expert, Wrench, even at a time when Churchill’s reputation stood very high in the early 50s finds the 1931 position that Churchill took hard to understand. However this is a speech in a large hall to an audience likely to support the reactionary views of the organizers and Churchill’s public and private views were not always expressed in the same terms. Churchill after all had wide ministerial experience and had been sympathetic to national causes in South Africa, Ireland and Belgium, but he aligned himself with imperial extremists. The most damning criticism comes from an Indian source (D) looking back on the Bengal Famine. However, this is based on one family and the evidence is not given for British neglect. 1942 was a very difficult year for Britain and there were some hard choices. This is a journalistic rather than a historical account. It is also bound to be limited as it is essentially based on family experiences. However, other sources do confirm widespread resentment about British inaction. C has eye witness accounts of meetings with Gandhi and the Cripps mission of 1942; but whether Gandhi would actually in the end have been satisfied with dominion status may be questioned – certainly on the basis of the limited evidence given here. India and the Empire were causes of dissent between Churchill and the Americans and there is some doubt about whether a post-war Conservative government would have given up India. Candidates may be aware of Churchill’s continuing commitment to an imperial role right up to the end of the war. However Wrench (C) does offer some modification to the harsher view. Churchill himself defends his policy in 1931 on humanitarian grounds, though the sincerity and factual basis of this may be questioned. He was convinced that Congress would oppress lower castes and, indeed, Muslims. This was actually the case in Congress administrations after 1935 – at least in terms of Muslims. However, Churchill’s concerns were probably more about preserving the Victorian empire and a belief in the importance of trade links. Source E offers a justification in terms of eventual outcome, but this may be questioned. By the 1930s there was a consensus for change as Source A shows, and a gradual constitutional reform was seen as inevitable and more likely to prevent extremism and communalism. In 1931 Churchill was something of a failed politician and may well have played the Indian card with some irresponsibility. If subsequent events proved some of his concerns to have some justification, then that does not entirely justify his position in 1931. In terms of both assessing the needs of India and of his reputation within his own party and with the influential political leaders of his day, it could well be argued that Churchill had misjudged and candidates are free to assess how serious this was.

Page 257: History A - The Bicester Schoolthebicesterschool.org.uk/wp-content/uploads/2015/09/Mark... · 2015-09-28 · History A Mark Schemes for the Units January 2009 H106/H506/MS/R/09J

F964 Mark Scheme January 2010

F964 European and World History Enquiries

Option A: Medieval and Early Modern 1073-1555 The First Crusade and the Crusader States 1073-1130 1 The People’s Crusade

(a) Study Sources A and B Compare these Sources as evidence for the popular response raised by the preaching of the Crusade. [30]

Focus: Comparison of two Sources No set answer is expected, but candidates need to compare the contents, evaluating such matters as authorship, dating, utility and reliability, so using the Sources ‘as evidence for...’. The headings and attributions should aid evaluation and reference to both is expected in a good answer. A makes much of the divine nature of Peter’s appeal (‘little short of divine’), in the context of his audience and of the enthusiasm generated; B echoes elements of this but also points up the sense of a lack of control, an over-confidence, a belief in success, all features, arguably, of excessive popular enthusiasm. Both focus on the role of Peter; both imply receptive audiences, audiences waiting for a message. B refers to soldiers as well as unarmed men, women and children, adding to the sense of the range of appeal. The reference to palms and wearing crosses is of note. The tone and language of both Sources can be engaged here. So, too, the provenances, and, given the apparent hostility of the Byzantine rulers towards the People’s Crusade, the authorship of B may be significant. Source B mentions Peter ‘inspired people’, the wide range of those affected. The Source makes much of ‘palms and wearing crosses’ (links here to Palm Sunday) and of the mass response. There is a strong sense of popular involvement. Source A mentions ‘gifts’, ‘holiness’, ‘godlike’. Both point to a sense of the charismatic popular preacher as well as to a mass longing for such a leadership with purpose. A mentions ‘the common people’, ‘crowds of people’, while B mentions ‘unruly, difficult, restless’ supporters. Comments on the provenances will aid evaluation. Authorship is important. Both assess from hindsight, Anna from a highly privileged Byzantine perspective which expresses horror at the disorderly aspects of the Peoples’ Crusade and may well exaggerate this. The gap in dates may be viewed as important. A comes from a reliable source and offers insight while B, though later, comes from a source usually seen as critical of the crusaders yet here having some positive comments to make. Sympathy and empathy in A, a more cautious, possibly semi-critical viewpoint in B, can be assessed as well and the provenances linked to the contents and their nature.

(b) Study all the Sources

Use your knowledge to assess how far the Sources support the interpretation that Peter the Hermit was an ineffective leader. [70]

Focus: Judgement in context, based on the set of Sources and own knowledge Successful answers will need to make use of all five Sources, testing them against contextual knowledge and evaluating their strengths and weaknesses, any limitations as evidence. A range of issues may be addressed in focusing upon the terms of the question but no set conclusion is expected. The interpretation finds support in Sources C, some of D and some of E, while Sources A, B, some of D and E point to other possible factors; D says that the Emperor did send help in the end having neglected the crusade beforehand. Source E presents an overview and sees some merit in Peter’s leadership and mentions

72

Page 258: History A - The Bicester Schoolthebicesterschool.org.uk/wp-content/uploads/2015/09/Mark... · 2015-09-28 · History A Mark Schemes for the Units January 2009 H106/H506/MS/R/09J

F964 Mark Scheme January 2010

the nature, size and scale of the enterprise (linked to A and B) as well as sense of the loss of control by Peter (as in C). Peter is seen as having an impact as a preacher and a leader (A, B, E) but also as lacking the necessary qualities as a military and political leader (especially in C and E). His role was important but there were other factors involved (E is useful here), some related to his leadership, others less so (for example, the sheer military power and ferocity of the Turks his force encountered). Apart from the issues of the attitude of Alexius I, the Byzantine Emperor, and the nature of Peter’s leadership, military factors are raised in C and D, and suggested in E. A and B point to the intensity of responses to the call for a crusade. Source B does include the warning from the Byzantine Emperor. Source C mentions Peter’s visit to Constantinople, and the Turks ‘full of glee’, ready to attack. Source D also mentions Constantinople and conveys a sense of followers abandoned to the Turks. The popular element of this Crusade was large but the force was ill-equipped and ill-trained. Contextual knowledge can supply some details of the Crusade and its fate as well as of the role of the Byzantine Emperor; this Crusade had flaws in leadership, organisation, weaponry (Source A points to the advantages that princes and nobles had in preparations, resources); there is a wider issue of the Byzantine response to the presence of this Crusade, indeed of the Crusade as a whole. Source E gives a succinct survey of some of the key ‘requirements for success’ and these can be exemplified by reference to the problems encountered by the People’s Crusade. Peter was a charismatic preacher, but he was no military leader. The peasant-based force he gathered was transported into Asia Minor by Alexius I and, it could be argued, abandoned there. The lack of control by Peter, evident early on, may be viewed as significant; so, too, the very nature of this Crusade, as evidenced by references to its make-up in B and E. Both B and D point to the attitude of the Emperor; the reference to ‘permission’ in D is of note. The provenances of the Sources can add to analysis here; so, too, the tone, not least of C (the Emperor rejoiced, survivors were disarmed). Candidates who make valid comments on provenance should be rewarded. References to the Emperor in C and D could be used to qualify the reliability of Anna in B who writes from a generally pro-Byzantine stance. The author of C, as an ex-Crusader, might be deemed to have greater knowledge of conditions those of A and B, whose eye-witness accounts may not have been fully reliable. The time lag in B and D is also significant in assessing reliability. Topic knowledge will add in points about (for example) the general attitude of the Emperor to the Crusaders and about the military problems facing all crusaders as well as the Turkish advantages in battle against unprepared, undisciplined opponents.

73

Page 259: History A - The Bicester Schoolthebicesterschool.org.uk/wp-content/uploads/2015/09/Mark... · 2015-09-28 · History A Mark Schemes for the Units January 2009 H106/H506/MS/R/09J

F964 Mark Scheme January 2010

The German Reformation 1517-1555 2 Reactions to Luther and his Ideas 1519-21

(a) Study Sources A and C Compare these Sources as evidence for Catholic reactions to Luther’s teachings. [30]

Focus: Comparison of two Sources. No set answer is expected, but candidates need to compare the contents, evaluating such matters as authorship, dating, utility and reliability, so using the Sources ‘as evidence for…’. The headings and attributions should aid evaluation and reference to both is expected in a good answer.

The provenance of the Sources is a good starting point for a comparative analysis here. The author of Source A is an internationally respected and influential humanist, thought by some to have ‘helped him write his books’, though he seems to be trying to distance himself from the association. On the other hand, the author of Source C is Pope Leo X, the target of much of Luther’s criticism. Their authority is very different. Erasmus has intellectual and theological authority but no official religious authority within Germany. The audience of Source A is Albrecht of Mainz, who had aided the Pope in the sale of indulgences, and a corrupt churchman himself whereas the audience of Source C is Frederick the Wise of Saxony, Luther’s prince, whom the Pope hopes may be able to suppress Luther’s heresy even though he lies in saying that Frederick has never favoured Luther, opening the way for Frederick to distance himself from supporting Luther. This would avoid further confrontation before the Bull ‘Exsurge Domine’ becomes final and perhaps limit the damage to the Church: hence his purpose in writing to Frederick. Erasmus, on the other hand, has the purpose of trying to save his own reputation from being dragged down by Luther’s heresy from which he wishes to distance himself.

The Sources are similar, in that Source A says ‘All they can say is ‘heresies’, and Source C shows the Pope, having declared Luther a heretic, complaining that he ‘ignores the punishment of heretics, papal decrees and church councils’. Source A states that ‘churchmen publicly ridicule him with their crazed howling’, and the Pope, in Source C, calls Luther ‘that son of iniquity’, ‘mad’ and ‘a scabby sheep who infects the flock’. Source A suggests that his enemies wish only to catch and crush Luther, and similarly, in Source C, the Pope suggests Frederick takes him captive to suppress his views. Erasmus, in Source A mentions ‘the vicious venomous lies’ with which his enemies tear Luther apart, and in Source C the Pope is claiming Luther believes only his own opinion, whereas in Source A Erasmus states that Luther’s ideas are based on St Augustine, so the Pope would seem to be unreliable.

The Sources therefore also differ. In Source A, Erasmus is suggesting Luther should not be crushed, whereas in Source C the Pope is trying to suppress his views. Whereas Erasmus sees Luther’s teachings as showing ‘brilliant sparks of Gospel learning’, the Pope, in Source C, says Luther ‘perverts the faith’, a claim which Source A refutes as ‘they are considered orthodox, even godly’ in St Augustine. Source A suggests Luther’s critics are unlearned, and ‘have never read a word Luther has written’ whereas Source C is written by the Pope, the fount of canon law and Roman Catholic doctrine, who has selected particular errors by which Luther has ‘seduced the simple’. But whereas Erasmus, in Source A, says that Luther’s enemies would prefer him to be a ‘dead man rather than a good man’, Pope Leo, in Source C, is offering Luther clemency if he ‘returns to his sanity’.

74

Page 260: History A - The Bicester Schoolthebicesterschool.org.uk/wp-content/uploads/2015/09/Mark... · 2015-09-28 · History A Mark Schemes for the Units January 2009 H106/H506/MS/R/09J

F964 Mark Scheme January 2010

Brief comments on context must be credited only in so far as they aid the comparison. The recent publication of the Bull ‘Exsurge Domine’ urging Luther to recant within sixty days has made it more urgent to make one final attempt to put pressure on Luther by targeting his princely patron.

A supported judgement should be reached on their relative value as evidence. Source C is a last attempt at persuasion by the highest authority in the Roman Catholic Church, whereas Source A is a request for support from a leading German churchman in holding back the tide of the attack on Luther and justifying him, to prevent Erasmus getting tarnished by his previous association with Luther. They therefore represent the divided reactions within Germany at this time. Both are useful as evidence for ‘behind the scenes’ views, but Source C shows more of the official face of the Roman Catholic Church, asserting papal authority but also trying one last attempt at persuasion. No set conclusion is expected, but substantiated judgement should be reached for the top levels of the Mark Scheme.

(b) Study all the Sources

Use your own knowledge to assess how far the Sources support the interpretation that the main reason the authorities failed to suppress Luther’s heresy was because he had influential supporters. [70]

Focus: Judgement in context, based on the set of Sources and own knowledge. Successful answers will need to make use of all five Sources, testing them against contextual knowledge and evaluating their strengths and weaknesses, any limitations as evidence. A range of issues may be addressed in focusing upon the terms of the question but no set conclusion is expected.

The Sources contain references to different interpretations, so they may be grouped according to their view. The supporting view is shown predominantly in Sources A, C, D and implicitly E, whereas the opposing view features in Source B and explicitly in C, D and E, though the reference in C is false and misleading in E.

The supporting view of influential religious patronage is in Source A, while political patronage features in Sources C, D and implicitly E. Source C is an appeal to Elector Frederick of Saxony who had not pressured Luther to recant and supported him. Knowledge might be used to explain that the Pope’s attempt to bring Luther to Rome to face trial had failed due to Frederick’s support for Luther’s trial within Germany, where he had popular support. Frederick’s military influence is hinted at In Source D, by the reference to defending Luther with a ‘military bodyguard’. ‘Support with their resources’ also implies Frederick, but in Source C, the Pope denies that Frederick has supported him rather than accuse Frederick openly, due to his power in the Empire. The hint lies in the ‘lofty and dazzling dignity’ mentioned. The Pope had sent Frederick the coveted ‘Golden Rose’ to try and gain his support for the papal candidate in the Imperial election in 1519. Other princes are also said to have left Worms, in the introduction to Source E, before Charles V feels able to issue the Edict of Worms, suggesting Luther had influential princely support.

The Sources also support the opposing view. Source E shows that the most influential patron in Germany, Charles V, did not support Luther and was attempting to suppress his heresy. Knowledge might be used to explain the Emperor’s role as the political arm of the papacy, but also to assess the extent of his influence over the princes of the Holy Roman Empire. But though the Emperor is asserting that Luther is hated by all God-fearing persons, he contradicts this by implying that Luther has been protected and supported, not only by influential patrons but by the wider German public, who have published, bought, read and sold his books. Knowledge of the part played by the printing press might aid evaluation. However, Charles’ audience is the more compliant group of princes remaining

75

Page 261: History A - The Bicester Schoolthebicesterschool.org.uk/wp-content/uploads/2015/09/Mark... · 2015-09-28 · History A Mark Schemes for the Units January 2009 H106/H506/MS/R/09J

F964 Mark Scheme January 2010

at Worms after Frederick and Luther’s other princely supporters have left. Charles’ hope to ban or capture Luther remains in vain as long as his patrons and the German public continue to support him, as perhaps shown by using knowledge of Frederick’s secret ‘kidnap’ of Luther for his own protection, his lodging in the Wartburg and safe return to Wittenberg.

Further limitations of his support are suggested in Sources B and D. The author of Source B is not a patron, but a Luther supporter, with pupils he may influence. He shows this by his comment about ’wise, learned men’, suggesting that Luther may have some theologians on his side, such as Erasmus in Source A, but he has not gained majority support among the audience of his disputation with Eck. Erasmus at face value is attempting to gain Luther the patronage of Albrecht of Mainz, but knowledge of his part in the sale of Indulgences and his purchase of his office would suggest this is unlikely to happen. Rather Erasmus may be trying to gain himself an influential patron to protect him from association with Luther. Source C reveals that the simple folk, who are among Luther’s main supporters, may be influential, though not patrons, and are a large group who have been ‘seduced by him’. These are perhaps among the ‘god-fearing persons’, the ‘faithful subjects’ to whom Charles is appealing in Source E. Fear of civil war is always in the background.

The provenance of the Sources should be integrated into the discussion. The authorship, tone, audience and purpose of the Sources are particularly revealing, as shown above.

Supported overall judgement should be reached on the extent to which the Sources accept the interpretation that influential patrons were the main reason why the authorities failed to suppress Luther’s heresy. No specific judgement is expected.

Candidates are likely to consider a range of reasons within the Sources: the influence of patrons, support from theologians, hesitation by the authorities to condemn Luther, outbursts and lies which attracted support for Luther, the printing press, the weakness of the Emperor and public adulation. They are likely to set the Sources within the context of Luther’s condemnation for heresy. It is up to candidates to assess and decide upon relative importance here, there being no set conclusion.

76

Page 262: History A - The Bicester Schoolthebicesterschool.org.uk/wp-content/uploads/2015/09/Mark... · 2015-09-28 · History A Mark Schemes for the Units January 2009 H106/H506/MS/R/09J

F964 Mark Scheme January 2010

Option B: Modern 1774-1975 1 (a) Study Sources A and D.

Compare these Sources as evidence for opinions about Robespierre. [30]

Focus: Comparison of two Sources No set answer is expected, but candidates need to compare the contents, evaluating such matters as authorship, dating, utility and reliability, so using the Source ‘ as evidence for…..’ The Headings and attributions should aid evaluation and reference to both is expected in a good answer.

Content: Source A sees Robespierre as eloquent and a famous man whose speeches will be read in the future. Source D sees him as puffed up with pride and not revered but the butt of sarcastic comments by his fellows and of contempt by at least one member of ‘the people’. Robespierre is not being ‘elevated to the sky’ by mercy in D, but rather by eccentric delight in a pseudo-religious cult. The writer of A has affection for Robespierre as an old friend, D writes in a hostile and ironic way (With what joyful pride! Which contrasts with A’s ‘My dear Robespierre’). There is no suggestion in A that Robespierre has theocratic ambition and D does not suggest that he has been excessively bloodthirsty and severe in the way that A does. Robespierre’s popularity is directly mentioned in D and rather more indirectly in A in which his writings are thought to be likely to be read by posterity. A is a warning; D is a judgement. They both reflect criticisms of Robespierre among the political elite – Source A by its nature of a warning and Source D by the reporting of Robespierre’s mocking colleagues.

Provenance: Source A is a public article intended to be a clear warning against violence and immoderation, a warning that cost Desmoulins his life, despite his friendship with Robespierre. Source D is a public document, but not one that was so bravely written, as Robespierre had fallen and was dead by the time it was disseminated. It reflects on his fall while, in contrast, A is trying to prevent it. Both men were deeply involved in the politics of their day– therefore these are political ‘insiders’. Note the dates – A is written before the massive increase in terror which it failed to prevent; D after the terror which helped (see C) to bring about Robespierre’s fall. A is still full of the revolutionary idealism typical of this author, whereas D seems much more cynical in tone – with Robespierre’s idealism being mocked and his popularity creating envy – however, note the rapturous crowds. In terms of making a judgement about their utility, A could be seen as more useful from someone who knew Robespierre or B could be seen as more useful as revealing more about Robespierre’s actual rule. Neither is an objective source, but it could be argued that A is more balanced. No set answer is expected.

(b) Study all the Sources.

Use your own knowledge to assess how far the Sources support the interpretation that the main reason for the fall of Robespierre in 1794 was because his rule was dominated by the policy of Terror. [70]

Focus: Judgement in context, based on a set of Sources and own knowledge. Successful answers will need to make use of all five Sources, testing them against contextual evidence and evaluating their strengths and weaknesses, any limitations as evidence. A range of issues may be addressed in focusing upon the terms of the question but no set conclusion is expected. The debate here is whether the Terror brought the end of Robespierre, or whether it was Robespierre’s potential to be a dictator based on his popularity with the populace together with his increasing eccentricities and religious views. Did the easing of the external threat make his extremism less necessary or was the key the split in the convention and the

77

Page 263: History A - The Bicester Schoolthebicesterschool.org.uk/wp-content/uploads/2015/09/Mark... · 2015-09-28 · History A Mark Schemes for the Units January 2009 H106/H506/MS/R/09J

F964 Mark Scheme January 2010

committee about the irrational desire for Reason and the alienation of the propertied classes?

A and C stress concerns about terror – one before it reached its height by a personal friend and associate; C a private letter which does recognize the positive achievements – so neither by a necessarily unsympathetic or counter-revolutionary source, but both expressing concerns about violence. Neither may be typical – both are Paris-based and both supporters of revolution. Provincial opinion may have been more strongly against Robespierre and many may have found the Terror obliterated any good opinions about the Committee of Public Safety. Candidates may use own knowledge of the terror to good effect here. D and, by implication, B do not refer to violence but are evidence of unease about the eccentricity and pseudo religious views of Robespierre. B seems quite ridiculous with all manner of elements being celebrated on holy days. As a Decree its authenticity cannot be questioned, but the extent to which it reflected opinion in both government and country might be considered and how far it reflected aspiration rather than reality. This might be linked to other manifestations of extreme change and perhaps too to the association of Revolution with impiety and anticlericalism that fuelled provincial unrest. The scepticism, therefore, shown in D may be typical and candidates might know the relief in the Thermidor period when the fatuous processions, emblems and rhetoric stopped – and of course the violence. What D stresses is the splits in the revolutionary elite by 1794 and candidates may know the background of the receding threat from invasion and the reaction against the political murder of Robespierre’s previous opponents – Roland, Danton etc. ‘Liberty what crimes are committed in thy name’ etc. E tries to balance but the negativity of Robespierre’s political vision comes out strongly and this can be assessed in the light of knowledge of the terror and the political in-fighting. The fanatical self-belief is supported by D and the repression by C. The reliance on Robespierre on sans culotte support in C may be picked up and linked to own knowledge of his rise and appeal and also cross referenced to the envy shown in D.

2 (a) Study Sources B and D Compare these Sources as evidence for Cavour’s influence in Piedmontese politics in the period 1851-57. [30]

Focus: Comparison of two Sources No set answer is expected, but candidates need to compare the contents, evaluating such matters as authorship, dating, utility and reliability, so using the Sources ‘as evidence for …’. The headings and attributions should aid evaluation and reference to both is expected in a good answer. Candidates are likely to highlight some of the following factors which may be described as similarities or differences according to the emphases adopted by candidates. Both sources refer to the limited popularity of Cavour and the impact this had on his political actions. In Source B the suggestion is that Cavour adopted ‘fine liberal sentiments’ as a way of reinforcing ‘public faith in his liberalism’ and in Source D his reluctance to reform the press laws is explained by his ‘fear of lowering his own popularity’. The challenge Cavour faced from the Right is stressed in both sources with Source B implying it was increasingly confident to act independently of the government and Source D refers to a swing to the Right in the elections of November 1856. The connubio is identified as a key factor in Piedmontese politics with Source B explaining how Cavour created it and Source D confirming Cavour’s continued reliance on it. The way French affairs impacted on Cavour is referred to in Source B explaining how Napoleon III’s coup weakened the Left in Piedmont, to Cavour’s advantage, and Source D explicitly demonstrates the influence the French tried to exert and the support they enjoyed from the King which, by implication, weakened Cavour’s position. Most are likely to agree that an obvious difference is the position of Cavour for in Source B he is described as ‘in complete control’ but in Source D his position is ‘seriously weakened’.

78

Page 264: History A - The Bicester Schoolthebicesterschool.org.uk/wp-content/uploads/2015/09/Mark... · 2015-09-28 · History A Mark Schemes for the Units January 2009 H106/H506/MS/R/09J

F964 Mark Scheme January 2010

Candidates may claim the sources are reliable as they are both reports intended to be a matter of record. On the other hand each comment contains an element of subjective opinion. Candidates will consider the author of Source B to be either supportive or critical of Cavour depending on their interpretation of the references to Cavour’s speeches and the views expressed about the conduct of cabinet business. Either way the author was a member of the Cabinet with direct knowledge of the politics of the time. The accuracy of the comments of the author of Source D may be considered questionable as the views of one observer only and those of an outsider. The utility of the sources may be considered. Both provide an insight into the political manoeuvrings of the time and some may be able to substantiate with reference to the division between d’Azeglio and Cavour in 1852 and the rise of the Right in 1856 based on the anti-clerical policies of the previous years. Some may identify the comments in Source D about attacks on the Emperor with disappointment in Piedmont that France appeared reluctant to promote the Italian cause following the expectations of the peace of Paris in 1856.

(b) Study all the Sources

Use your own knowledge to assess how far the Sources support the interpretation that Piedmont developed into a liberal state in the 1850s. [70]

Focus: Judgement in context, based on the set of Sources and own knowledge. Successful answers will need to make use of all four Sources, testing them against contextual knowledge and evaluating their strengths and weaknesses, any limitations as evidence. A range of issues may be addressed in focusing upon the terms of the question but no set conclusion is expected.

How candidates define ‘a liberal state’ will determine the shape and direction of their response. Many are likely to group the sources into those that suggest Piedmont developed into a liberal state (Sources A, B and D) and others that suggest the opposite (Sources C and E) although there is room for debate within most of these sources. Some might adopt a thematic approach and consider the economic, political and religious constituents of the liberal state. If so they may treat Source A as indicative of a liberal trade policy. Sources B, C, D and E reflect the nature of political issues and Sources C and E religious matters. There is scope for differences of emphasis in the interpretation of these sources and the evaluation of their provenance Tariff reform in pursuit of free trade, championed in Source A, was a basic element of the liberal state of the period. Candidates should add details about the trade treaties agreed with foreign states and the increased competition opened up in Piedmont. They could also expand on the reference to ‘economic progress’ and explain how industry, agriculture and transport were modernised in the 1850s. However, the fawning tone of Cavour’s remarks, intended to win Cobden’s support for the engineer despatched to England, exaggerates the strength of the free trade lobby in Piedmont especially as the argument for free trade was not as secure as Cavour implies. Candidates might argue that Sources B, C and D confirm parliamentary government – a key feature of a liberal state – functioned in Piedmont. Further, the dominant politician of the period, Cavour, is portrayed as committed to liberalism (Sources B and D) and Source D makes it clear that a free press existed. Knowledge of the powers of Parliament, the political manoeuvrings described in Sources B and D and the type of criticism directed against the French Emperor referred to in Source D could be considered in support of these points. On the other hand, some will detect traces of authoritarianism in all three sources. Source B implies that cabinet government was undermined by Cavour’s actions and dominance. In Source D the freedom of the press is considered of little worth in terms of its influence. In Sources C and D the King appears to be conservative in his attitude to Church reform and press freedom. Candidates may comment on the opportunism of Cavour as a politician or the limitations of the Statuto such as the considerable powers of

79

Page 265: History A - The Bicester Schoolthebicesterschool.org.uk/wp-content/uploads/2015/09/Mark... · 2015-09-28 · History A Mark Schemes for the Units January 2009 H106/H506/MS/R/09J

F964 Mark Scheme January 2010

the King which was why the proposed marriage laws referred to in Source B were dropped. In December 1851 d’Azeglio was prepared to restrict the powers of the press to appease the political right. In Source E Acton argues that the pursuit of ‘the greatness of the State’ was more important to Cavour than ‘the liberty of the people’: candidates could refer to the events of 1859-61. The charge that freedom was alien to Piedmont could be explored by comparisons with England, even America. The specific interests of the authors of Sources B, C and D might be considered. Sources C and E illustrate the move to a more liberal secular State. ‘The law on religious orders’ mentioned in Source C refers to the closure of some monasteries. The King’s opposition to the law is implicit in the source which was consistent with his religious convictions, demonstrated in his hostility to the earlier marriage bill referred to in Source C. Some may stress the Pope’s threat to excommunicate anyone supporting the reform to explain the fawning tone of the King’s letter. Candidates might highlight the public anger against the Pope’s intervention as an indication of the strength of liberal opinion in Piedmont although loyalty to the Church, emphasised in the final lines, highlights the depth of conservatism. At face value Source E views Piedmont as an illiberal state and there is evidence to support certain accusations. D’Azeglio did introduce ecclesiastical reforms without consulting the Church, most notably the Siccardi Laws of 1850. That ‘her governments were profoundly hostile to the Church’ was true in so far as both d’Azeglio and Cavour were resolutely anti-clerical. However, the author was a Catholic whose loyalty to the Church could explain his opposition to the religious reforms of the period. Further, Acton ignores Cavour’s pledge to allow a ‘free Church in a free State’. Despite this Lord Acton was committed to liberty in the broadest sense so his views are not entirely prejudiced.

80

Page 266: History A - The Bicester Schoolthebicesterschool.org.uk/wp-content/uploads/2015/09/Mark... · 2015-09-28 · History A Mark Schemes for the Units January 2009 H106/H506/MS/R/09J

F964 Mark Scheme January 2010

3 (a) Study Sources A and B Compare these Sources as evidence for the right of South Carolina to nullify the Tariff Law. [30]

Focus: Comparison of two Sources. No set answer is expected, but candidates need to compare the contents, evaluating such matters as authorship, dating, utility and reliability, so using the Sources ‘as evidence for …’. The headings and attributions should aid evaluation and reference to both is expected in a good answer.

The sources disagree in every respect. Calhoun (A) considers nullification to be legal, describing the exercise of the veto as ‘legitimate’ whereas Webster (B) states clearly that nullification is ‘unlawful’. The justification offered for these positions differ too. Calhoun argues that the Constitution was formed by the will of the States which is denied by Webster who claims ‘the people erected it’. In similar vein Calhoun asserts ‘that no authority is higher than theirs’ (the States) in contrast to Webster who states ‘the laws of the United States are supreme’. To emphasise this further it is stated that intervention by the General Government would mean ‘violating the Constitution’. In contrast, Webster argues that resistance to ‘a law is treason which the US could not ignore’ implying that interference in the internal affairs of a State is justified. Calhoun implies nullification is vindicated to ensure liberty (first line) whereas Webster argues liberty is dependent on the Union, hence his reference to ‘Liberty and Union’.

The evaluation of the differences should consider the debate about States Rights and the impact of the Tariff. Calhoun was invoking some of the notions raised earlier by Jefferson and Madison in the Kentucky and Virginia Resolutions respectively which were interpreted by some as justifying nullification. The State of Massachusetts had consistently argued against any concession to States Rights so Webster’s views are unsurprising. The Tariff was resented in South Carolina because it was perceived to damage agricultural interests and, by implication, was a threat to the peculiar institution of slavery. Candidates should pick up on the vehemence of Calhoun’s hostility to the Tariff as ‘obnoxious’ and something that ‘impoverishes us’: it was called ‘The Tariff of Abominations’ in South Carolina. By contrast Webster considers the Tariff as ‘beneficent’ which can be explained because of the protection it offered northern manufacturers. Calhoun appears to favour accommodation with the General Government whereas Webster does not betray any doubt about his position. This was in part because Calhoun was the newly elected Vice-President with an obligation to uphold the integrity of the Union (which explains his desire for anonymity). Furthermore, Source A was written two years earlier when positions were less entrenched than when Webster was debating. Calhoun was trying to explain a theory whilst Webster was defending the status quo to an audience that was largely conservative on matters of the Constitution.

(b) Study all the Sources

Use your own knowledge to assess how far the Sources support the interpretation that the Nullification Crisis threatened the existence of the United States. [70]

Focus: Judgement in context, based on the set of Sources and own knowledge. Successful answers will need to make use of all four Sources, testing them against contextual knowledge and evaluating their strengths and weaknesses, any limitations as evidence. A range of issues may be addressed in focusing upon the terms of the question but no set conclusion is expected.

81

Page 267: History A - The Bicester Schoolthebicesterschool.org.uk/wp-content/uploads/2015/09/Mark... · 2015-09-28 · History A Mark Schemes for the Units January 2009 H106/H506/MS/R/09J

F964 Mark Scheme January 2010

Most candidates are likely to group Sources B, C and D as supportive of the interpretation with Sources A and E providing the counter-argument. Sources C and D consider nullification will result in the separation of South Carolina from the Union. Source C is effectively an ultimatum and provides a deadline after which the State will form its own government. Source D claims South Carolina is intent on secession. The reliability of both sources might be queried as they are public statements intended to stake positions unequivocally and to indicate the commitment of both parties to fulfil their duty; in the case of the Convention to articulate popular sentiment in the State and in the case of the President to show his determination to discharge his responsibility to uphold the Union. To some extent both may be seen as bluffs as both challenge the other side to make the first move: Jackson also appeals to the people of South Carolina by questioning the wisdom of those leading them. The fact that Jackson was a slave-owning Southerner, with sympathies for the South, might be assessed. However, candidates might argue that the threat to use force, made in both sources, is convincing. Following the revision of the Tariff in 1832 anger in South Carolina was widespread and radicals like McDuffie and Rhett in the Convention were sincere in the position expressed in Source C. Similarly, Jackson had just secured re-election for a second term so he was confident of a mandate to uphold the Union and, as an army general with a reputation for firmness, there is little doubt that he was sincere in his views. South Carolina was raising an army, as Jackson claims, if only in anticipation of having to defend itself. Candidates may cross refer to Source B which expresses similar concerns about the future of the Union even if they are less explicit. This may be because in 1830 there was still room for manoeuvre on nullification. At least Webster’s anticipation of a ‘broken union’ leading to ‘civil feuds’ suggests that he regarded the nullification issue to be a threat to the existence of the United States. Some may know that Webster retained an uncompromising stance on the issue till the very end arguing strongly against Clay in the debates of February 1833 so allowing some cross reference to Source E.

Sources A and E appear to refute the interpretation that nullification threatened the United States if for very different reasons. It is clear that Source A does not see any contradiction between nullification and the continuance of the Union. Calhoun clearly hopes for a peaceful solution to the problem of the Tariff and only as a last resort does he envisage applying a veto. Indeed, he is optimistic that the new President will address the concerns of South Carolina if only, perhaps, because as the new Vice-President, he believed he could represent the interests of the State directly in the White House. He does not refer to the separation of the State from the Union. At this stage the notion of nullification was only being defined as the introductory comments make clear so positions were more fluid and open-ended than was to be the case later. Source E explicitly rejects the idea that South Carolina ‘ever desired to become a separate State’ and as such refutes the interpretation that the nullification crisis threatened the existence of the United States. Clay bases his view on the impracticality of the State surviving out of the Union. Candidates might consider the financial, economic, political and military difficulties independence would create. In addition, Clay identifies a key weakness of South Carolina in that she lacked support ‘across the continent’. Candidates should know that no other State, even those in the South, supported her: indeed, many like Louisiana and Alabama publicly disowned her. Furthermore, the hostility of public opinion, alluded to by Clay, might be substantiated by the formation of so-called ‘Washington Societies’ of volunteers prepared to take action against South Carolina. Clay’s pitch may be seen for what it was: an attempt to win support for a compromise Tariff from both South Carolinians and nationalists, of which he was one, by his appeal to history. As such, some may argue he was glossing over the seriousness of the situation at a time when the Force Bill was being debated too.

82

Page 268: History A - The Bicester Schoolthebicesterschool.org.uk/wp-content/uploads/2015/09/Mark... · 2015-09-28 · History A Mark Schemes for the Units January 2009 H106/H506/MS/R/09J

F964 Mark Scheme January 2010

4 (a) Study Sources D and E Compare these Sources as evidence for the effectiveness of the Nazi regime’s attack on the Churches. [30]

Focus: Comparison of two Sources. No set answer is expected, but candidates need to compare the contents, evaluating such matters as authorship, dating, utility and reliability, so using the Sources ‘as evidence for …’. The headings and attributions should aid evaluation and reference to both is expected in a good answer.

Content: Similarities: Both sources are complaining of lack of success. Both refer to church attendance and both refer to restrictions put in the way of religious education. Both refer to the general absences from church services among young people. Both refer, albeit obliquely in D, to the influence of the Church on opinion. Differences: The obvious difference is that D is lamenting the continuing decline in church attendances among the population as a whole, especially among the young whereas E is lamenting the fact that, during war time, attendances are rising, again, apart from among the young. D implies that the Protestant church is effectively stifled or paralysed by rigorous Nazi activity whereas E is stating effectively the opposite, that the Church has more manpower at its disposal and is able to exert its influence over opinion, which it is unable to do in D. However, perceptive candidates might point out that in E the report suggests that the Catholic rather than the Protestant Church is exerting more influence. D refers exclusively to the Protestant Church whereas E refers to both Catholic and Protestant churches.

Provenance: There are obvious differences in the provenance and dates. D comes from a confidential report from Protestants in peacetime after five years of the consolidation of Nazi power. It comes from Bavaria where the Protestant Church was historically weaker, the Catholic stronger. Bavaria was also more supportive of the Nazis. E is a summary of reports from Nazi gauleiters in wartime where the effectiveness is questioned, especially amongst adults. Both are confidential summaries of reports, not aimed at the public and therefore inclined to be reliable, especially as both report negatively upon themselves. Both are of equal value.

In terms of ‘evidence for’, candidates should highlight the differences between peacetime and war time. The date of D is 1938, a time when the regime’s attacks on the Protestant Church have had mixed success; The ‘German Christian’ movement has largely failed, but, as pointed out in D, the regime has largely managed to woo youth away from Church influence and hinder religious teaching. The whole tone and content of D implies effectiveness.

In E, clearly the war time situation is fundamental. The date, mid 1943 is important. By this time, the tide has turned and the war is going badly. As more and more Germans read between the lines and realise this, naturally they turn more to the churches for spiritual comfort in the wake of huge losses at Stalingrad. Clearly then, by this time, the regime’s attacks on the Churches has largely ceased to be effective, particularly given the strain on manpower that the Churches would not be subject to.

83

Page 269: History A - The Bicester Schoolthebicesterschool.org.uk/wp-content/uploads/2015/09/Mark... · 2015-09-28 · History A Mark Schemes for the Units January 2009 H106/H506/MS/R/09J

F964 Mark Scheme January 2010

(b) Study all the Sources. Use your own knowledge to assess how far the Sources support the interpretation that the Churches were willing collaborators with the Nazi regime. [70]

Focus: Judgement in context, based on the set of Sources and own knowledge. Successful answers will need to make use of all four Sources, testing them against contextual knowledge and evaluating their strengths and weaknesses, any limitations as evidence. A range of issues may be addressed in focusing upon the terms of the question but no set conclusion is expected. In terms of grouping: A clearly suggests willing collaboration B suggests collaboration but for different reasons; necessity rather than willingness. The content of C is rather ambiguous, but collaboration of a kind particularly over the need to stamp out Communists is implied. In opposition to this view D strongly suggests weakness in the face of the regime, but not collaboration. E suggests clearly that not only have the churches survived, but are not collaborating. The supporting argument therefore is supplied by A and C, and the counter argument supplied by D and E Source B can be used to demonstrate that there was collaboration, but not ‘willing’ in all areas.

There is much in the five sources to construct an argument around. Candidates should use their contextual knowledge to evaluate the sources. The key element is to establish the existence of ‘collaboration’ and the extent to which this was ‘willing’. A shows clear collaboration, not in order for the Catholic Church to survive but because the Nazis were welcomed as a violently anti-bolshevik force. However, this was right at the beginning of the regime when the horrors were not yet apparent. It also comes from Catholic schoolteachers rather then the Church per se. Moreover, the churches were not alone in welcoming Hitler and fearing communists. Candidates can evaluate using contextual knowledge. There was much common ground between the Catholic Church and Nazi ideology over the issues of communism, anti-modernism in culture and the arts, anti-feminism, and, to an extent, anti-semitism. Much can be made of B. The context of course is the Catholic centre Party’s collaboration with Hitler in voting for the Enabling Law, thereby effectively destroying democracy. Candidates may pick up on the word ‘collaboration’ in the source which the author suggests (ironically) would not be possible in the future without the Party siding with Hitler in the vote. A clear sense of helplessness is evident here. However, good candidates will question whether this ‘collaboration’ was ‘willing’, or whether it was done out of necessity. C Implies a form of collaboration. There is continuing support for Hitler with protestations of loyalty, a misguided belief that he means to keep his promise made in 1933 not to interfere with the Church and is being ignored by the ‘little Hitlers’ There is the reference to ‘communists’ and ‘Marxists’ which can be cross-referenced with A. The source can be taken as a counter-argument against ‘willing collaboration’. Contextual knowledge can be used to point out that the Catholic Church did stand up and protest [Archbishop Galen, removal of crucifixes from schools etc] but only did so when its own interests were threatened. D Strongly implies helplessness, and can be cross-referenced with B, but not collaboration. Candidates should use own knowledge to evaluate here .The traditional mainstream of the Lutheran Church succeeded in breaking away from the German Christian Movement, and therefore did not collaborate.. However, this was at great cost as the content and tone of the source indicates. Though there is no implication of collaboration here, let alone ‘willing’, there is evidence of passive acceptance. E Clearly states neither ‘willingness’ nor ‘collaboration’. In fact, quite the opposite. However this is after nearly four years of war when discerning Germans are seeing through the propaganda and beginning to realise that the war might soon be lost. Death and destruction post-Stalingrad would account for religious revival. Candidates should point out the obvious differences between the relations between the regime and the churches during peacetime and wartime.

84

Page 270: History A - The Bicester Schoolthebicesterschool.org.uk/wp-content/uploads/2015/09/Mark... · 2015-09-28 · History A Mark Schemes for the Units January 2009 H106/H506/MS/R/09J

F964 Mark Scheme January 2010

5 The USA and the Cold War in Asia 1945-75 American Policies in Asia 1945-1950

(a) Study Sources C and E

Compare these Sources as evidence for US strategies for the military security of Asia between 1945 and 1950. [30]

Focus: a comparison of Sources No set answer is expected, but candidates need to compare the contents, evaluating such matters as authorship, dating, utility and reliability, so using the Sources ‘as evidence for …’. The headings and attributions should aid evaluation and reference to both is expected in a good answer.

The Sources agree that the main strategy is ‘to rely on the people attacked to resist’ in Source C, supported by the arming of South Korea to prevent border raids in Source E. They agree that a defensive perimeter is in place to hold back enemy advance - the defensive perimeter in Source C, and the thirty-eighth parallel in Source E. In both, the US strategy is to work with the United Nations in a collective security system to prevent or stop war. Both Sources agree that there are global considerations: ‘other pressing problems’ in Source C and threats to independent nations in Source E. Source C says that the USA cannot guarantee ‘other Asian Areas against military attack’, and this has been shown to be the case in Source E, where mainland Korea has been invaded. Even here the US makes no mention of using ground troops, only air and sea cover for the South Koreans.

But the Sources also disagree. Source C prioritises Japan and the Philippines as part of the chain of offshore islands of the ‘defensive perimeter strategy’, whereas Source E refers to the invasion of Korea, on mainland Asia, an area which had received no US military protection and was excluded from the defensive perimeter. This is shown by the South Korean forces being armed for ‘internal security’ only and proving unable to resist attack without US support now being supplied after the event. Source E suggests that the Communists have moved from subversion to open war, whereas in Source C the enemy is not identified, but a general global threat is identified.

Contextual knowledge may be used to show understanding of the comparison. Source C is set in the context of America’s confidence being shaken by the ‘fall’ of China to Mao’s CCP, creating a powerful communist bloc in Asia and public disquiet at home. McCarthyist criticism leads Truman into a more active policy towards Asia and he may wish to prove he has no communists within his administration. Nonetheless Acheson is concerned to put limits on what the US can do in mainland Asia. By the time of Source E, the Cold War has intensified and America has lost its nuclear monopoly after the successful Soviet nuclear test in August 1949. NSC 68 has been secretly proposed in April, suggesting a tripling of military expenditure and Truman is wishing to gain support in Congress in order to adopt a more forceful policy.

The provenance of the Sources may be used to determine which is more useful or reliable for explaining US military strategies. The author of Source C is the Secretary of State, Dean Acheson, who had established the half-hearted ‘defensive perimeter strategy' and failed to contain communism in Asia, a strategy he sticks with in C, whereas that of Source E is the President himself, so Source E carries extra weight. Truman’s purpose is to pave the way for a massive increase in funding from Congress for a more forward policy to rescue his and America's reputation.

A supported judgement should be reached on which Source provides better evidence. Source E is more useful and reliable in that it shows the strategy the President wishes to adopt rather than the ill-advised exposure of US weakness by his Secretary of State in the

85

Page 271: History A - The Bicester Schoolthebicesterschool.org.uk/wp-content/uploads/2015/09/Mark... · 2015-09-28 · History A Mark Schemes for the Units January 2009 H106/H506/MS/R/09J

F964 Mark Scheme January 2010

wake of the loss of China. However, Source E explains why the strategy in source C had to be changed. (b) Study all the Sources

Use your own knowledge to assess how far the Sources support the interpretation that US policy in Asia was weak in the period between 1945 and 1950. [70]

Focus: Judgement in context, based on the set of Sources and own knowledge. Successful answers will need to make use of all five Sources, testing them against contextual knowledge and evaluating their strengths and weaknesses, any limitations as evidence. A range of issues may be addressed in focusing upon the terms of the question but no set conclusion is expected.

The Sources contain references to both sides of the argument, so they may be grouped by interpretation. The view that US policy was weak, in achieving its aim to contain communism, is based on the key argument that the USA became too reliant on its nuclear monopoly, prioritised Europe and failed to realise the strength of Asian national movements. This view is predominantly in Source B and to an extent in Sources C, D and E, whereas the view that US policy was strong is in Sources A, D and to an extent E.

The view that US policy was weak is stressed by Source B, that though US policy was outwardly strong and aggressively imperialist, it was weak because it depended on the atomic bomb to back the spread of western ideologies. It calls the US atom bomb a 'paper tiger', the nuclear weapons underpinning defensive perimeter strategy boasted about in Source A merely alienating local peoples. The view is that the USA would not dare use atomic weapons again after the bombings of Hiroshima and Nagasaki in August 1945, which might be supplied from own knowledge. Source C may be linked to this view, suggesting ‘it is wrong to become obsessed with military considerations in Asia . . there are other pressing problems’. Own knowledge might be used in evaluation: the USA had made military cutbacks, as it had no tradition of taxation for the armed forces in peace-time. It prioritised defence of democracy in Europe. Source A talks of bases from ‘Alaska to the Azores in the South Atlantic’ and other global commitments, and Source B, ‘to dominate the world’. This point might be evaluated as 'US overstretch'. Source C suggests that the USA has prioritised Japan and the Philippines within the Asian defensive perimeter, omitting Korea. Own knowledge might be used of the policy of ‘model’ democratic states, a weak containment policy, as it depended on corrupt puppet rulers. Acheson in Source C expects local peoples in Asia to defend themselves, 'we rely on the people attacked to resist'. Sources B and D agree that 'Asian affairs will be settled by the Asian peoples themselves'. Knowledge of the fall of China to communism by the time of Sources C and D reveals the weakness of Truman's administration, especially Acheson's policy. Source E supports the view in C that the USA felt they could not defend other areas of Asia, and Source D is an example of American supplies to the Nationalists failing to be sufficient to prevent the fall of China. Own knowledge might be used to develop this idea, such as the misuse of aid to the Chinese Nationalists. This links to Source E where US 'air and sea forces' are to cover and support South Korean troops but there is no mention of US ground troops being sent except as part of UN forces, showing a weak policy. Knowledge might be used of the involvement of the UN to shield the US from the threat of a global nuclear war. Discussion of subjective provenance might mention the anti-American standpoint of B and D, both members of the Chinese Communist leadership, while the authors of C and E are Truman and his Secretary of State, much criticised by Republicans and McCarthyists. Own knowledge of this and of strong Soviet support for Communist China might be used to develop this view.

86

Page 272: History A - The Bicester Schoolthebicesterschool.org.uk/wp-content/uploads/2015/09/Mark... · 2015-09-28 · History A Mark Schemes for the Units January 2009 H106/H506/MS/R/09J

F964 Mark Scheme January 2010

87

US weakness can be contrasted with the confidence and strength of the view in Source A, which supports the view that US policy was strong. Source A takes a confident line based on the US nuclear monopoly a few months after they had defeated Japan with the dropping of atomic bombs on Hiroshima and Nagasaki. It suggests that off-shore defensive bases are all the strength needed for defence of Asia and the wider world, although it assumes a willingness to use them (‘guts’). Own knowledge and provenance might be used to evaluate this view - the USA had recently emerged as the stronger of the two superpowers, but the subjective, outspoken tone of Source A reveals this as unreliable, as the introduction mentions. This was a misplaced confidence based on the delusion it could be had on the cheap. Source D might also be used for this argument, that US policy was outwardly strong and aggressively imperialist, killing millions and removing freedoms by supplying arms and aid to Nationalists in the Chinese civil war, although Chou En-lai is clearly exaggerating US intent (annexation of China). Knowledge might be used to develop this point - at the time of Sources C, D and E the USSR had its own nuclear capability, ending the US monopoly. By 1950 also the USSR had formed an alliance with Communist China, explaining the more aggressive policy forced on the USA in Korea in Source E, though under a UN umbrella.

Candidates are likely to consider a range of themes within the Sources: strengths and weaknesses of defensive perimeter strategy, reliance on the atom bomb and weakness in cutting back US military forces, prioritising Europe over mainland Asia. They are likely to set the Sources within the shifting context of the Cold War. It is up to candidates to assess and decide upon relative importance here, there being no set conclusion.

Page 273: History A - The Bicester Schoolthebicesterschool.org.uk/wp-content/uploads/2015/09/Mark... · 2015-09-28 · History A Mark Schemes for the Units January 2009 H106/H506/MS/R/09J

F966 Mark Scheme January 2010

F966 Historical Themes

Option A: Medieval and Early Modern 1066-1715 English Government and the Church 1066 – 1216 1 Assess the view that reasons for rebellion remained the same in the period from

1066 to 1216. [60]

Rebellions happened at times across the period. They include those from 1066 to 1070, 1075-6, 1088, 1095, the rebellion against Stephen which ended in civil war, the Great Rebellion of 1173-4 and rebellion against John from 1215-6. Examiners should not expect equal treatment of all of these. Some reasons for rebellion are seen throughout the period and, although they are not all present in all rebellions, rebellions were multi-causal. Reasons include support for an alternative ruler. This is seen in 1088 when Odo of Bayeux supported Robert of Normandy instead of William II, in 1095 when the rebels planned to replace William with his nephew Stephen, in the civil war in Stephen’s reign when some of the barons supported Matilda’s claim, and in the 1173-4 rebellion where rebels supported Young Henry. The strength of royal government was also a cause of rebellion: in 1075-6 Roger of Hereford balked at William’s sending sheriffs to hear pleas on his marcher lands; in 1095 Robert of Mowbray resented royal interference in his fief; one of the reasons for the 1173 rebellion was the tension caused by 20 years of strong government and 40 years later, reaction to the strength of Angevin rule was an important factor in the rebellion against John. Some barons rebelled because of the problems caused by the continental possessions, especially the difficulty of serving two different lords. This is seen in 1088 when barons were faced with the prospect of serving Robert of Normandy for their lands in Normandy and William II for their English lands, and from 1144 in Stephen’s reign. Continental possessions caused other problems too, especially the heavy taxation needed to retain them, a possible factor in the rebellion against Henry II, or to try to win them back, certainly a factor in the rebellion against John. Behind much of this, throughout the period, lay baronial self interest, the belief that barons’ rights were being undermined in some way, as can be seen from some of the clauses of Magna Carta, and/or that they could win better personal rewards from another ruler. However, there are also reasons which apply only to specific rebellions. Those from 1066 to 1070 were reactions to the invader. The 1088 rebellion and that against Stephen were partly the result of disputed succession. The rebellion against John was prompted partly by military failure abroad. The rebels of 1215 also arguably took up arms not only against Angevin government but because they wished to enforce some regulation of the king’s rights. The best responses are likely to be analytical examinations of a range of reasons, looking at both continuity and change, and evaluating how consistent they were across the period. They will probably point to the multi-causal nature of all rebellions but might also examine how far there is a change in emphasis on particular reasons in different rebellions. Most candidates will deal with a number of rebellions, analysing reasons and reaching a conclusion. Weaker responses will probably do this sequentially, typically describing rebellions and then deducing reasons. Least effective responses are likely to deal with only continuity or change and a very limited range of rebellions. Examiners must be open to alternative approaches. If in doubt, they should consult their Team Leader.

88

Page 274: History A - The Bicester Schoolthebicesterschool.org.uk/wp-content/uploads/2015/09/Mark... · 2015-09-28 · History A Mark Schemes for the Units January 2009 H106/H506/MS/R/09J

F966 Mark Scheme January 2010

2 'They were never fully in control of the English Church.' How far do you agree with this view of the archbishops of Canterbury in the period from 1066 to 1216? [60]

Most candidates will probably limit their answers to Lanfranc, Anselm , Becket and Langton and this is acceptable as these are the only archbishops mentioned in the specification. However, credit should be given for relevant reference to others such as Theobald or Hubert Walter. Candidates are likely to argue that there is plenty of evidence for lack of control over the Church by archbishops of Canterbury. Absence through exile as in the case of Anselm or Becket, or inability to enter England in the case of Langton, reduced the amount of control they could exercise. There was also no unequivocal resolution of the primacy issue. There was recognition of Lanfranc's personal primacy but as Thurstan of York's refusal to profess obedience to Canterbury in 1115 demonstrated, this remained a problem. This was usually resolved by the pope granting legatine authority to Canterbury but this was not automatic, as when Henry of Blois was made papal legate instead of the archbishop. By the later twelfth century, Canterbury's primacy did not confer much real power over the Church, and at times both Canterbury and York were made papal legates in their own provinces. Archbishops' authority was deliberately undermined by popes who wished to eradicate primatial control in order to maximise their own authority over the Church and who took advantage of the prevailing English situation to do so as occurred under Becket and Langton. Popes also encouraged appeals to Rome (eg in Stephen's reign) which further reduced archbishops' control. Archbishops sometimes suffered lack of support from their own bishops. For example, Becket's quarrel with Henry II lost him the support of some of his bishops such as Foliot. In Stephen's reign the divided government of the English Church caused problems between Archbishop Theobald and Henry, Bishop of Winchester. Henry II later took the opportunity to weaken Becket's authority by having York instead of Canterbury crown Young Henry. Monasteries also created problems for archbishops as they tried to free themselves from archiepiscopal control eg the Canterbury monks. Good responses should examine a range of evidence, recognising that Canterbury's lack of control was not consistent. Most candidates are likely to discuss the degree of control exercised by Lanfranc, Anselm, Becket and Langton in turn before reaching a judgement. Weaker answers may be characterised by partial treatment of just a few archbishops. Examiners must be open to alternative approaches. If in doubt, they should consult their Team Leader.

89

Page 275: History A - The Bicester Schoolthebicesterschool.org.uk/wp-content/uploads/2015/09/Mark... · 2015-09-28 · History A Mark Schemes for the Units January 2009 H106/H506/MS/R/09J

F966 Mark Scheme January 2010

3 How far was the impact of the papal reform movement in England from 1066 to 1216 characterised more by change than by continuity? [60]

The papal reform movement led to increased papal power and it is this which had an impact in England. There was considerable change in the extent of the impact of the papal reform movement. It had very little impact under William I and Lanfranc when William refused to become a papal vassal and Lanfranc refused Gregory’s summons to Rome. By the thirteenth century, the power of the pope had increased so much as a result of the papal reform movement that Innocent III was able to impose an archbishop of Canterbury unacceptable to the king, excommunicate John, place England under an interdict and go on to suspend Langton, so demonstrating papal power over both the English monarch and the church. There were also changes in the nature of the impact. In the reigns of William II and Henry I its main manifestation was through the Investiture Contest which resulted in a compromise in 1107 in which the king lost investiture with ring and staff. Under Stephen the growing power of the papacy led to more appeals to Rome. At times the main impact could be seen as attempts by popes to further their power by undermining primatial authority, eg by making Henry of Blois papal legate or by supporting York against Becket. At other times worsening relations between kings and their archbishops could be regarded as the main impact: for example, relations between Anselm and Henry I deteriorated because of the Investiture Contest and the quarrel between Becket and Henry II was prompted partly by the papal reform movement’s encouragement of ideas of separate ecclesiastical justice. It is likely that most candidates will examine these changes and possibly conclude that change was more noticeable than continuity. Weaker answers might tend to deal with the changes chronologically.

However, candidates might also argue that the impact of the papal reform remained essentially the same throughout most of the period. It was characterised by increased freedom of the church from royal control starting with Investiture Contest, increased papal authority over the king, the archbishop and the English church as a whole, and, although not consistently, at times throughout the period after William I and Lanfranc when it led to poor relations between kings and their archbishops. Stronger responses will be aware of both change and continuity and will evaluate the relative strength of each, reaching a substantiated conclusion. Some might well point out that although there was much consistency in the nature of the impact the emphasis was on different aspects at different times. Examiners must be open to alternative approaches. If in doubt, they should consult their Team Leader.

4 ‘Tudor rebellions were essentially the responses of local communities to local

grievances.’ How far do you agree with this view on the causes of Tudor rebellions? [60]

It might be useful if candidates define ‘local communities’ and ‘local grievances’ to distinguish between causes that had their origin in a village or town before spreading more widely, and causes that were not particularly linked to local factors but owed more to high politics and personal attitudes. An argument in support of the latter might refer to dynastically motivated rebellions such as Simnel, Warbeck and Northumberland, or to rebellions whose leaders had personal grievances, eg Northern earls, Essex, Kildare and Shane O’Neil. Candidates are likely to suggest that rebellions that were mainly social and economic in origin usually reflected local issues. Reactions to unfair or excessive taxation in Yorkshire (1489), Cornwall (1497), Suffolk among several counties (1525), Lincolnshire (1536) and Devon (1549), complaints about enclosures triggered the Kett and Oxfordshire disturbances, and responses to biased local JPs and the conduct of county gentry led to uprisings in Devon and Norfolk in 1549. It may also be argued that religious grievances manifested themselves locally but were really a response to government policies that

90

Page 276: History A - The Bicester Schoolthebicesterschool.org.uk/wp-content/uploads/2015/09/Mark... · 2015-09-28 · History A Mark Schemes for the Units January 2009 H106/H506/MS/R/09J

F966 Mark Scheme January 2010

affected the whole country. Disturbances in 1536 in Lincolnshire and Yorkshire, for example, resulted from national policies and local circumstances; and similar cases can be made for the Catholic reaction to reforms in Yorkshire and Durham by a reform-minded bishop in 1569. Some rebellions, such as Wyatt and Tyrone, were actuated by a mixture of personal and political motives and had their origin in local responses to national issues. Candidates should demonstrate the interplay between local and national developments and focus on similarities and differences in the causes of rebellion. Examiners must be open to alternative approaches. If in doubt, they should consult their Team Leader.

5 ‘English rebellions were far more successful than those in Ireland.’ How far do you

agree with this view of the period from 1485 to 1603? [60]

Candidates should seek to compare the outcome of rebellions in England and Ireland before reaching a judgement on the extent of success. Good essays will be aware that while most rebellions failed, there were some successes in both countries – Tyrone O’Neil in Ireland and the Yorkshire, Amicable Grant and Mary’s defeat of Northumberland in England. Some answers might include Henry Tudor’s defeat of Richard III as it occurred in 1485. Several English rebellions achieved some of their subsidiary objectives eg taxation was reduced in 1497, illegal enclosures reversed in 1537 and 1597, the Statute of Uses repealed in 1540, religious reforms suspended after 1539 and the Council of the North reconstituted in 1572. Irish rebellions had fewer successes and most failed to achieve their main objectives. Tyrone recovered his dispossessed lands and received a pardon but lost his title of ‘The O’Neil’. The other revolts resulted in the leaders’ deaths, military defeat, the imposition of martial law and confiscated lands. Attempts to oust English settlers in some of the plantations and preserve the Catholic faith were successful but Kildare, Shane, Fitzgerald and Tyrone all failed to expel the English administration and garrisons. It could be argued that the Irish forced English governments to spend a disproportionate amount of time, money and resources in combating rebellions, which enabled them to frustrate English aims of colonisation and conquest. Candidates should be rewarded for discussing areas of similarity and difference and for evaluating successes and failures than for explaining the reasons for successes and failures, which may be a characteristic of weaker essays. Examiners must be open to alternative approaches. If in doubt, they should consult their Team Leader.

6 Assess the role of the nobility in maintaining political stability in Tudor England.

[60]

The nobility was vital to the crown in maintaining political stability in Tudor England. Candidates are likely to argue that many nobles served as royal councillors both in London and at a regional capacity, as sheriffs, JPs, lords lieutenants, and as special commissioners in the counties, and suggest how these officers upheld political stability in the country. As leading landowners, nobles were also expected to arm their tenants and servants to suppress disturbances and, when necessary, fight in royal armies. Nobles such as Surrey, Oxford and Pembroke assisted Henry VII; Norfolk, Suffolk and Shrewsbury suppressed rebellions in Henry VIII’s reign; Russell, Warwick and Grey led armies against the Western, Kett and Oxford rebels in 1549; and Pembroke, Clinton and Norfolk dealt with Wyatt and his rebels. Elizabeth similarly called upon Sussex, Clinton and Hunsdon to combat the Northern Earls, and the Earl of Nottingham was called upon to arrest Essex in 1601. Better candidates should point out that not all nobles supported the crown and in 1485 several families presented a serious challenge to internal stability. However, the Percy, Neville and Howard families declined between 1536 and 1572, especially in the north of England, and only a small minority of nobles remained a problem in Elizabeth’s reign. It may also be pointed out that the crown increasingly relied not just on the nobility but the gentry and lesser landowners for keeping law and order in the provinces and counties. These groups of people dealt with most local grievances and as town officials, JPs and MPs, took a keen interest in maintaining stability. The clergy, judiciary and

91

Page 277: History A - The Bicester Schoolthebicesterschool.org.uk/wp-content/uploads/2015/09/Mark... · 2015-09-28 · History A Mark Schemes for the Units January 2009 H106/H506/MS/R/09J

F966 Mark Scheme January 2010

monarchy might also be examined but it is important that candidates focus their answers principally on the nobility. Examiners must be open to alternative approaches. If in doubt, they should consult their Team Leader.

7 ‘The aims and methods of Tudor foreign policy were characterised more by change

than by continuity.’ How far do you agree with this judgement? [60]

Some candidates may well focus on either aims or methods but better essays should examine both elements in terms of continuity and change. Some candidates may assess Tudor relations with particular countries such as Spain, France, Scotland and Burgundy/ the Netherlands. Some might adopt a chronological approach though this may make sustaining a synoptic analysis more difficult. The main aims behind Tudor foreign policy were national security, trade agreements, continental expansion and dynastic alliances. National security and trade agreements were consistently pursued although, if there was a conflict of interest, trade yielded to political security. Keeping the Netherlands neutral or out of enemy hands was also a consistent objective. To this end, France and Burgundy were seen as England’s main enemies at the start of the period but had been replaced by Spain at the end. Dynastic and marriage alliances were pursued by Henry VII, Henry VIII and Mary, but less so by Edward VI and inconclusively by Elizabeth – although the latter used her unmarried status as a pawn in foreign diplomacy. There were inconsistencies, however, in the Tudors’ attitude towards war. Henry VII and Elizabeth avoided it if possible but engaged in continental alliances; Henry VIII and Edward devoted much time and money to waging war against Scotland and France; Mary was opposed to war but was drawn into it by Philip. Religious reforms also brought changes in allies and foreign commitments eg Anglo-Scottish relations. The best candidates are likely to explain the more prevalent trends and account for changes in aims and methods. Examiners must be open to alternative approaches. If in doubt, they should consult their Team Leader.

8 Which presented the greater threat to England’s security in the period from 1485 to

1603: Scotland or France? Explain your answer. [60]

Candidates should focus on and assess both Scotland and France before reaching a conclusion. Some answers are likely to compare the two countries thematically – perhaps in respect of military and naval threats in peacetime and at war, the support given to pretenders, claimants and English rebels, or the impact of making alliances against England and often together (Auld Alliance). Some essays might assess Scotland and France separately before reaching a judgement though this approach could restrict candidates’ ability to demonstrate synoptic skills. The strong military, naval and financial power of France, its desire to recover land held by England, the long-standing rivalry and its commitment to the Catholic faith, might suggest that it posed a serious threat, and every Tudor ruler went to war against France at least once. Better candidates, however, should point out that there were long periods of peace and stability between the two countries eg 1492-1512, 1527-42, 1564-1603. Scotland though much smaller and less powerful than France also presented a threat to England: it supported Warbeck, invaded northern counties at will and had a long-standing arrangement with France of embarrassing English governments. The deaths of James IV and James V after military conflicts weakened Scotland but left it open to a French presence and capacity to intrigue against England, which was not removed until 1560. Thereafter, Scotland did not present a serious threat. Candidates may well conclude that both countries presented rather different threats, which changed over time, before deciding in favour of one of them. Examiners must be open to alternative approaches. If in doubt, they should consult their Team Leader.

92

Page 278: History A - The Bicester Schoolthebicesterschool.org.uk/wp-content/uploads/2015/09/Mark... · 2015-09-28 · History A Mark Schemes for the Units January 2009 H106/H506/MS/R/09J

F966 Mark Scheme January 2010

9 Assess the ways in which Spain affected English domestic affairs during the period from 1485 to 1603. [60]

Candidates are likely to suggest that Spain had a considerable impact on English domestic affairs but should also be aware that the nature of its influence changed over the period. One approach that candidates might take is to look at each of the Tudors in turn and comment on developments that led to continuity and change. Another would be to assess the impact thematically. The main areas of discussion are likely to be political issues – Henry VII’s alliance of 1489 against pretenders and the betrothal of Arthur (and later Henry) to Catherine of Aragon; the influence of the Aragonese faction at court in the 1520s and 1530s, and Charles V’s pressure not to proceed with a divorce; hostility towards Philip in England in the 1550s at the court, council and parliament surfaced in Wyatt’s rebellion; putative Spanish support for Mary Stuart and Catholic plotters. Religious issues – pressure to remain a Catholic country after the break from Rome. This may have curtailed Henry VIII’s reform programme, compromised Somerset, supported and aided Mary’s restoration of the Church, influenced Elizabeth in determining the Settlement of 1559 and her subsequent treatment of Catholics. Economic issues – trade links since 1489 with Aragon and Castile, and from 1515 with the Low Countries. The collapse of the Spanish Netherlands’ woollen trade had a severe impact on England’s economy in the 1550s. English merchants traded extensively with Iberia until the 1580s and privateers became a source of friction from the 1560s. The outbreak of the Dutch Revolt had a serious effect and Elizabeth faced political pressure to intervene. Military issues – war with Spain against France in 1542 and 1557 and against Spain from 1585 had serious financial, economic and political consequences. Spain also intervened in Ireland in the 1580 and 1601 rebellions. Examiners must be open to alternative approaches. If in doubt, they should consult their Team Leader.

10 ‘The success of the Catholic Reformation in the period from 1492 to 1610 depended

mainly on secular rulers (emperors, kings and princes).’ How far do you agree with this view? [60]

Candidates should evaluate the contribution of a number of secular rulers to the Catholic Reformation and decide how far any success was due to their efforts and how much to other factors. Isabella and Ferdinand, Mary Tudor, Philip II, Ferdinand of Styria, Sigismund of Poland, Rudolph II of Austria, and Albert, William and Maximilian of Bavaria, could be usefully examined as secular rulers who advanced the Catholic Reformation in their own lands. Reasons for their success could be attributed to the use of patronage, military influence and political authority. Candidates might argue that some secular rulers who were in a position to implement improvements did little to advance the Church’s revival. Most German princes, Henry VIII, Francis I and the later Valois kings would fall into this category, and criticism can be made of Charles V and Philip II, who constantly quarrelled with the papacy. Candidates should compare secular rulers with other influences and might well reject the title’s premise. For instance, they could argue that the papacy was the key to the Catholic Reformation’s success, and illustrate the point by the negative and positive contributions of popes during this period. They might point to the role of individuals as members of the new orders or to reform-minded clerics, who were not secular rulers but who had a significant influence upon them eg Canisius in Bavaria, Borromeo in Milan, Pole in England. The work of institutions like the Council of Trent, the Inquisition and Index, might be evaluated but better essays will focus on the term ‘mainly’ and might well conclude that the successful impact of individuals and institutions rested on lay support. Where secular rulers were weak or opposed to reform, little headway was made. Examiners must be open to alternative approaches. If in doubt, they should consult their Team Leader.

93

Page 279: History A - The Bicester Schoolthebicesterschool.org.uk/wp-content/uploads/2015/09/Mark... · 2015-09-28 · History A Mark Schemes for the Units January 2009 H106/H506/MS/R/09J

F966 Mark Scheme January 2010

11 To what extent was the Catholic Reformation more concerned with continuity than change in the period from 1492 to 1610? [60]

Candidates can support or refute the proposition by examining a range of features that contributed to the Catholic Reformation. It may be helpful to define ‘continuity’ eg retaining key features, beliefs and institutions of the Catholic Church and faith. The Inquisition and Index might be assessed to show how they repressed liberal ideas and censored unacceptable views, such as those of Illuminists, Erasmians, Protestants, conversos and moriscos. Attempts to reconcile Catholics and Protestants at Regensburg in 1541 resulted in victory for the reactionaries led by Carafa who was intent on suppressing change. Even the new orders struggled to gain respect and recognition from traditional monastic orders. In education, biblical humanism, and its implicit changes to the way the Church interpreted the Scriptures, was rejected in favour of scholasticism. The management of Trent’s three sessions by Jesuits and Dominicans demonstrated the desire to defeat Augustinianism, Lutheranism and Calvinism in order to preserve orthodox ideas, which was underlined by the Tridentine Decrees. No provision was made for lay administrators and female reformers and reforms to the Curia were slow to take effect and very conservative. There are examples, however, of the Catholic Church showing a willingness to change. The Church learned from Protestantism the value of preaching and the sermon in developing the spiritual condition of ordinary people, and the benefit of advocating social welfare, the use of the consistory in keeping discipline, and the role of seminaries in educating the clergy. Candidates could refer to individual popes to suggest that the papacy was capable of embracing enlightened ideas. On balance, candidates are likely to argue that the Church was more conservative than progressive. Examiners must be open to alternative approaches. If in doubt, they should consult their Team Leader.

12 ‘The Inquisition and Index did little to advance the Catholic Reformation in the

period from 1492 to 1610.’ How far do you agree with this view? [60]

Some candidates will agree and some disagree with the proposition. The focus of the answer should be on what the Inquisition and Index set out to achieve, how far they succeeded and assess their contribution to the Catholic Reformation during these years. Most candidates are likely to assess the Spanish and Roman Inquisitions but the Portuguese, Dutch and Italian city state inquisitions are all valid lines of argument. Candidates should be aware that the role of the Index stayed constant from its inception in the 1540s, when titles of forbidden works were proscribed by universities, to the Roman and Spanish Indices which listed titles and authors. Revisions occurred periodically and candidates could usefully discuss the impact of the Index upon Catholic ideas, education and society in states such as Spain and Italy. Most candidates will probably spend more time assessing the Inquisition. Under Isabella and Ferdinand, the Spanish Inquisition tackled moriscos and conversos; under Charles V, Erasmians, Anabaptists and Lutherans were targeted; under Philip II and Philip III, it focused on immorality, paganism and moriscos in Spain, conversos in Portugal and Calvinists in the Netherlands. An argument may be made that the suppression of heresy and strengthening of the Church owed a great deal to the Inquisition. The Roman Inquisition similarly silenced Protestant movements, dealt with cases of immorality, and maintained papal authority in several Italian city states. A counter-view is that the Inquisition and Index were negative influences and real advances were principally due to other developments, such as the Jesuits, reform-minded popes, bishops and secular rulers who implemented the Tridentine Decrees. For Levels I and II, however, there should be a good evaluation of the Inquisition and Index, especially if the balance of the argument is on other factors. Examiners must be open to alternative approaches. If in doubt, they should consult their Team Leader.

94

Page 280: History A - The Bicester Schoolthebicesterschool.org.uk/wp-content/uploads/2015/09/Mark... · 2015-09-28 · History A Mark Schemes for the Units January 2009 H106/H506/MS/R/09J

F966 Mark Scheme January 2010

13 ‘The nobility strengthened rather than weakened royal power in France in the period from 1498 to 1610.’ How far do you agree with this view? [60]

Candidates are likely to assess the ways in which the nobility contributed to or hindered the development of the nation state, and the best answers will not stray from this path. The nobility held key offices in both church and state and not only served the king but also served themselves in the provinces. They led and raised armies during the Italian wars, which strengthened the state, but the same troops were turned against the well-being of the state and monarchy in the wars of religion. Nobles were royal governors, law enforcers and administrators, and most under a strong ruler such as Francis I served the state very loyally. A minority flouted the law, lined their pockets and disobeyed the king, at times demonstrating how much harm they could inflict at all levels of society. Some candidates may use particular noble families, such as the Montmorencys, Bourbons and Guises, to illustrate their argument throughout the period or to distinguish between noble groups eg nobles who were politically ambitious, princes of the blood, and those with strong religious convictions. Candidates should be aware of the changing relationship between the nobility and crown, most notably during the reigns of Francis I and Henry IV, when the major families were subdued, and in the reigns of Francis II, Charles IX and Henry III, when they exercised considerable political influence over the crown to the detriment of royal authority. Some answers might include the role of noble women eg as a mistress (Diane de Poitiers), a regent (Catherine de Medici) or as participants in the wars of religion (Marguerite d’Angouleme, Louise de Montmorency, Jeanne d’Albret). Examiners must be open to alternative approaches. If in doubt, they should consult their Team Leader.

14 To what extent did the French government overcome its economic problems in the

period from 1498 to 1610? [60]

Most candidates are likely to discuss the state of royal finances during this period but higher responses (Levels I and II) should explain some of the difficulties in industry, trade, commerce and agriculture as well. In finance the main problems were the inefficient and unequal tax system, tax farming and insufficient revenue to meet the state’s requirements; long periods of foreign war and civil wars disrupted administration and increased crown debts; and inflation was exacerbated by court affluence and patronage. Francis I implemented reforms that centralised the system but did not tackle issues of corruption and exemption. Henry IV could only begin to solve the difficulties caused by civil wars by cancelling debts and gradually initiating reforms after 1598. The nobility and officiers had the wealth and potential to invest in trade and industry but throughout the period showed little interest as long as rentes, crown pensions and from 1604 the Paulette were more profitable. Trade was in the hands of merchants who were heavily taxed and disadvantaged when competing with foreigners. There were few improvements in agriculture due to the depressed condition of the peasantry and disinterest among landowning nobility who preferred to hunt over the crops. There was little investment in industry and agriculture until Henry IV and Sully began to encourage state subsidies. Population levels rose to 17 million by 1610 (largest in Europe), which put pressure on urban employment and food supplies, and increased the likelihood of plague, poverty and revolts. Local and regional opposition to a more unitary transport system and an excessive number of tolls impeded the movement of goods. Overall the economy remained a weakness in the nation state though there were some improvements, notably under Francis I and Henry IV. Examiners must be open to alternative approaches. If in doubt, they should consult their Team Leader.

95

Page 281: History A - The Bicester Schoolthebicesterschool.org.uk/wp-content/uploads/2015/09/Mark... · 2015-09-28 · History A Mark Schemes for the Units January 2009 H106/H506/MS/R/09J

F966 Mark Scheme January 2010

15 ‘The Catholic Church held back the development of the French nation state in the years from 1498 to 1610.’ How far do you agree with this view? [60]

Candidates need to assess how far the Catholic Church impeded the development of the nation state. Weaker essays may focus heavily on the Wars of Religion as a prime source of national disunity. Some candidates may discuss the impact of religious groups upon the nation state from Francis I’s reign onwards, particularly in dividing the country socially, politically and religiously, and the response of the Catholic Church to them. Some candidates may see the Day of the Placards (1534) as a turning point in that thereafter humanists, Lutherans and Calvinists were under attack and the goal of national unity (one king, one law, one faith) disappeared. Some candidates will see this question in terms of the Wars of Religion and may compare the attitudes of Catholics towards Huguenots both during and after the civil wars. However, better essays will be aware that the Catholic Church was a source of strength to the crown and to most French people for much of the period. Of course, some candidates may point out factors other than the Church that weakened the nation state. Reference to geo-political borders, language, customary and Roman law, social and political disruption caused by civil wars, developments in administrative centralisation and fluctuations in the authority of the monarchy, would be valid areas of discussion in examining the development of the nation state. Examiners must be open to alternative approaches. If in doubt, they should consult their Team Leader.

16 ‘The development of absolute monarchy in France owed more to Louis XIII than

Louis XIV.’ How far do you agree with this judgement? [60]

The most successful responses are likely to compare the two rulers’ contribution to the development of absolute monarchy in France. Candidates might assess them in respect of their power and authority, their skill as politicians and statesmen, their qualities of leadership in domestic and foreign affairs, their appointment of ministers and the success of their reigns. An argument could be made that the extent to which they were absolute rulers rested heavily upon the competence of their ministers but the question requires more than a comparative evaluation of royal councillors. Some essays might compare the absolutism of the two kings, however implicitly, by examining how well they dealt with France’s problems. Among these, we can expect an assessment of their dealings with the princes of the blood and nobility, their management of royal finances, councils and policy making, how far they enhanced royal authority in the face of parlements and provincialism, their relationship with the Catholic Church and Huguenots, and how well they handled foreign affairs. Better candidates should be aware that the issues that faced French kings changed over time (eg rising population and inflation, the decline of Spain, the problem of the princes of the blood), though some continuity remained (eg financial system, particularism, strong Catholic Church), and, of course, the personality and ambitions of Louis XIII and Louis XIV were in sharp contrast. Moreover, Louis XIV was able to build upon the achievements of Louis XIII and his ministers but just how absolute was the monarchy in 1715 compared with 1610? Examiners must be open to alternative approaches. If in doubt, they should consult their Team Leader.

17 Assess which minister contributed most to France becoming a major European

power in the seventeenth century: Richelieu, Mazarin or Colbert. [60]

A case can be made for each of these ministers and much may depend upon how candidates define ‘a major European power’. Some candidates may compare Richelieu and Mazarin in terms of how they dealt with over-powerful nobles, the Estates and parlements, raised money for war, administered the state and church, strengthened the monarchy and waged war. They may suggest that Richelieu contributed more because he solved the Huguenot problem and worked closely with the Catholic Church. He expanded

96

Page 282: History A - The Bicester Schoolthebicesterschool.org.uk/wp-content/uploads/2015/09/Mark... · 2015-09-28 · History A Mark Schemes for the Units January 2009 H106/H506/MS/R/09J

F966 Mark Scheme January 2010

the army and navy, set up trading companies, reduced the power of the estates, humbled the aristocracy, expanded the intendants and laid the foundations for victory in war against Spain. Mazarin’s main contribution lay in negotiating beneficial terms at Westphalia and the Pyrenees, which gained France lands in Savoy, Alsace, the Netherlands and the Rhineland (1648) and lands in Luxembourg, the Netherlands, the Pyrenees and acquired a claim to the Spanish throne (1659). A negative feature was the Fronde, which was partly a result of his own unpopularity and financial mismanagement. Colbert’s claim lay in his management of the economy, which provided the basis for France’s military achievements under Louis XIV and the creation of Versailles. Revenue increased 400%, taxes rose 40%, corruption was reduced in administration and by 1672, the budget was balanced. Unlike Richelieu, he built up an effective navy of 300 ships and 4 new dockyards; improved road and canal transport and revitalised textile industries. He pursued mercantilist policies aimed at acquiring gold and silver bullion at the expense of the Dutch and English. He regulated industries, founded trading companies, established colonies in Canada and the West Indies, expanded the royal navy, maritime fleet and arsenals and naval stores. However, he failed to reform the fiscal system, his law codes could not be enforced and attempts to establish trading companies failed. A comparative synthesis is likely to characterise the better essays. Examiners must be open to alternative approaches. If in doubt, they should consult their Team Leader.

18 Which social groups in France gained most and which lost most during the period

from 1610 to 1715? Explain your answer. [60]

Candidates should consider a range of social groups, assess in what ways and to what extent they were affected by developments across the whole period, and explain why some benefited more than others. The best responses are likely to organise their arguments thematically according to either different social groups or to particular events, and demonstrate a good awareness of continuity and change. The principal groups that are likely to be examined are the nobility (and better candidates should distinguish between different categories ie princes of the blood, noblesse d’épée, noblesse de robe, noblesse de province), the clergy (Catholic and Protestant), merchants (perhaps those engaged in traditional trades, as well as those in newer companies and industries), town and rural workers (especially the urban poor, artisans and agricultural peasants). Explanations for change and continuity in the condition of these groups may be found in the growth of centralisation and administrative developments, long periods of warfare after 1635, an increase in taxation and size of armies, the growth of Paris and other large cities, the creation of Versailles, economic developments and religious issues, such as the Revocation of the Edict of Nantes. Candidates are not required to assess all groups or every development in the period but they are expected to produce a balanced and sustained comparison of different social groups before arriving at an overall judgement. Examiners must be open to alternative approaches. If in doubt, they should consult their Team Leader.

97

Page 283: History A - The Bicester Schoolthebicesterschool.org.uk/wp-content/uploads/2015/09/Mark... · 2015-09-28 · History A Mark Schemes for the Units January 2009 H106/H506/MS/R/09J

F966 Mark Scheme January 2010

Option B: Modern 1789-1997 The Challenge of German Nationalism 1789 – 1919

1 Assess the view that the German people were more divided than united during the period from 1789 to 1919. [60]

Candidates should focus on the extent to which the German people were more divided than united in this period. Candidates may argue both for and against this proposition. Candidates may view the period from 1789 to 1815 as a time when the German people became more united, geographically and in terms of aims. French domination helped to modernize and consolidate Germany and sparked the first upsurge of German nationalism. A popular uprising helped to drive Napoleon out of Germany in 1813. This common fight of people from different German states against the French enemy gave strong impulses to nationalism. The number of independent and semi-independent German states had been around one thousand in 1790 (with between three and four hundred fully independent units). Twenty-five years later only a little over thirty remained. Candidates may argue that the German nation was very divided from 1815 as a consequence of decisions taken at the Congress of Vienna, but could also argue that the German Confederation from 1815 did loosely bind most Germans into a Confederation with a Diet. Candidates may argue that the growing emergence of the nationalist movement after 1815 to the development of more radical nationalism in the late Nineteenth and early Twentieth Centuries demonstrates an emergence of national unity amongst the German people over this period. Candidates may understand that there were significant divisions even within those who were nationalist. Candidates may understand that the Prussian Empire in 1871 represented Kleindeutschland and an enlarged Prussia. They may argue that it was a Prussian Empire rather than a German Empire; it certainly did not unite all the German people even geographically. The exclusion of Austria from the process of German unification may be dealt with. Divisions within the German nation after 1871 might be illustrated through the Kulturkampf and the rise of socialism, or the domination of the Reich by the elites. Candidates may, however, argue that territorial boundaries rarely exactly match where the people of that nationality live and that divisions within a nation based on class or culture do not necessarily define the unity or otherwise of that nation. All modern nations have exhibited such divisions. They could certainly argue that the German Empire from 1871 physically united the majority of Germans. Candidates may argue that the First World War both united the German nation, at first, but that divisions soon arose and were entrenched by 1918. Similarly, whilst Versailles divided the nation geographically it united the nation in condemnation and bitterness.

Examiners must be open to alternative approaches. If in doubt, they should consult their Team Leader.

2 How far do you agree that the development of German nationalism was mainly dependent upon economic factors from 1789 to 1919? [60]

Candidates might focus on the relative importance of economic factors within the development of German nationalism in this period. Candidates should explain the importance of economic factors on developments, for example the impact of the Zollverein after 1834 in developing Prussia’s economic strength and Prussian leadership of Germany. Candidates should understand how developments in the economy in the 1850s paved the way for the military victories of 1864, 1866 and 1870 / 71. Military strength depended upon economic strength however. ‘Coal & Iron’ rather than ‘Blood & Iron’ could be usefully debated. The development of the railways may be seen as significant. The impact of the extraordinary developments in the German economy after 1871 should be discussed. Candidates must however show that they understand that economic factors were not the sole factors determining the fortunes of German nationalism in this period. For example, the Great War left Germany broken and half-starved despite the German

98

Page 284: History A - The Bicester Schoolthebicesterschool.org.uk/wp-content/uploads/2015/09/Mark... · 2015-09-28 · History A Mark Schemes for the Units January 2009 H106/H506/MS/R/09J

F966 Mark Scheme January 2010

economic domination of continental Europe in 1914. Economic factors undeniably contributed to Prussia’s domination of Germany from 1866, but opportunistic and skilful leadership, especially by Bismarck, should not be overlooked. Candidates may wish to place considerable importance on the diplomatic abilities of Bismarck both in terms of the unification of Germany and his management of German nationalism. The development and impact of ideas on the emergence and development of intellectual nationalism may also be usefully explored. Candidates may argue that initially this provided the impetus or springboard for later developments and that, in the Napoleonic period, it was the common fight of people from different German states against their French enemy that gave strong impulses to nationalism

Examiners must be open to alternative approaches. If in doubt, they should consult their Team Leader.

3 ‘Bismarck was much more effective in managing German nationalism than either Metternich or Kaiser Wilhelm II.’ How far do you agree with this view? [60]

Candidates should focus on how effectively Metternich, Bismarck and Wilhelm II managed German nationalism. Candidates will undoubtedly be more successful if they define ‘effective management’ in their answer. Candidates might define the ways in which the three were (or were not) effective: for example in controlling, harnessing or using nationalism. Clearly all three had different aims and different circumstances, which could enable candidates to make convincing cases for all of them. By 1848/49 no leader of the nationalist movement with mass appeal emerged. From 1815 to 1848 the nationalist movement was too weak to effectively challenge the Metternich System: arguably this demonstrates Metternich’s effective control over German nationalists. Equally Metternich fled Vienna in 1848, though his downfall was hardly dominated by German nationalism. Many candidates may argue in favour of Bismarck because of his critical role in the 1860s in the creation of the Second Reich; candidates may argue that he managed German nationalism by hijacking the nationalist cause for Prussia’s ends. This too could be considered effective management of German nationalism. Wilhelm II’s search for world power was undoubtedly populist, mirroring the development of radical nationalism, but it placed Germany in a vulnerable, dangerous position. The ultimate outcome of his policies was defeat in the Great War and humiliation at Versailles.

Examiners must be open to alternative approaches. If in doubt, they should consult their Team Leader.

99

Page 285: History A - The Bicester Schoolthebicesterschool.org.uk/wp-content/uploads/2015/09/Mark... · 2015-09-28 · History A Mark Schemes for the Units January 2009 H106/H506/MS/R/09J

F966 Mark Scheme January 2010

Theme 2: The Changing Nature of Warfare 1792 – 1945 4 Assess the extent to which conscription enabled armies to succeed in war in the

period from 1792 to 1945. The impact of conscription on warfare is not limited to the massing of larger numbers of soldiers but this will probably form the bulk of the answers met. Conscription is applied to warfare right at the start of the period by the development of ‘a nation in arms’ by France in the early part of the Revolutionary Wars developing into more regulated conscription in the later Revolutionary & Napoleonic period. This might be contrasted with the use of long service professionals and mercenaries by the dynastic armies of France’s enemies. The reaction of France’s enemies to conscription might include the tentative use of Frei Korps and Landwehr by Austria or the traditional use of conscripted serfs by Russia’s long service army. A good topic for discussion would be the development of the Krumper system in Prussia after 1808. Candidates might note that Britain never embraced conscription in this period and yet her army was successful. For the period of unification the different systems used by the combatant powers might be examined. Generally candidates will point to the superior organisation of manpower by Prussia and the resulting large size of her army in proportion to her population. This was illustrated by the defeat of France’s long service army by Prussia’s reservists despite superior French weapons technology in the ‘Imperial’ phase of the Franco-Prussian War. The expansion of the use of reservists in the last part of the 19th century is a profitable area for discussion. The First World War is an obvious example where conscription played a key role in warfare. Note that Britain used a long service professional army supported by Territorials at the start of the conflict, replacing this with Kitchener’s army of volunteers and finally conscription. WWII also saw the use of mass armies of conscripts but with more sophisticated technologies in the hands of these soldiers with resulting problems in training and use on the battlefield. The American Civil War falls into the mainstream of the debate, the north having a preponderance in manpower. Alternative explanations are possible and examiners must be open to alternative approaches. If in doubt, consult your Team Leader.

5 How far did developments in command and control of armies determine the

outcome of battles in the period from 1792 to 1945? No set conclusions are to be expected, but candidates must answer the question and address the theme over the full period. The question prompts a discussion of the organisation of war by the combatant powers and the control of armies during the period. Napoleon was the first to develop a true general staff and this was taken to new levels of effectiveness by the Prussian general staff of the middle and later part of the period. The planning of WWI and WWII is an obvious case for discussion. Candidates might point to ineffective command and control as a basis for argument, a good example of deficiencies in this era was the armies of Napoleon III and Austrians in 1866 versus Prussia. The failure of command and control might be a useful way to argue against the premise of the question. The American Civil War fits easily into the debate. The impact of command and control on the outcome of warfare can be discussed on many levels from grand strategy to the tactics. Better candidates may balance these two factors against others, but a discussion of the command and control of armies must form the core of the essay. Examples of the outcome of battles being determined by command and control could be drawn from the individual skills of a given commander in chief or from the use of command and control systems or both. In the Revolutionary and Napoleonic wars virtually any battle with Napoleon and the French staff system directing one side will apply, but of especial interest might be Austerlitz. The allies eventually developed similar methods but examples will have to come from later in the wars, the 1813 period would be useful but even then

100

Page 286: History A - The Bicester Schoolthebicesterschool.org.uk/wp-content/uploads/2015/09/Mark... · 2015-09-28 · History A Mark Schemes for the Units January 2009 H106/H506/MS/R/09J

F966 Mark Scheme January 2010

Napoleon tended to win all of his battles. A good example of success using more antiquated methods of command would be any of the Duke of Wellington’s battles. Of course the Napoleonic system sometimes was found wanting, for example at Borodino or Waterloo. For the battles of the mid century candidates should be aware that the size of actions increased and had an impact on command and control despite developments in this area, Magenta or Solferino in 1859 were both confused affairs. In 1866 and 1870-1 the Prussian staff system brought their army to the battlefield with some efficiency but once again many of the battles themselves demonstrated the difficulty of controlling armies fighting in long linear formations, Konigsgratz is an example as are many of the battles of the Franco-Prussian War although Gravelotte-St. Privat is of especial interest. The First World War has many obvious examples on the Western Front, so too the Second World War. Alternative explanations are possible and examiners must be open to alternative approaches. If in doubt, consult your Team Leader.

6 ‘Industrialisation was most successfully applied to warfare in the First World War.’

How far would you agree with this view of the period from 1792 to 1945?

No set conclusions are to be expected, but candidates must answer the question and address the theme over the whole period. The First World War is an obvious example of the first conflict where the application of industrialisation to conflict was the key to victory. This was partially due to the sheer scale of the conflict. Candidates might argue that the long duration of the war, the size of the armies involved and the geographic spread of the conflict meant that victory went to the side with the greatest industrial might. Alternatives might be the Napoleonic Wars where one might argue that Britain as the first industrial power played a key role in the conflict or that France was successful for much of the period due to an emerging proto-industrial economy. Another is the Crimean War where the industrial might of the Allies caused them to emerge victorious over an industrially backward Russia – a power that played a key role in the final defeat of Napoleonic France. Another alternative would be the impact of a newly industrialised Prussia on the conflicts of the mid-nineteenth century. For later conflicts the Second World War might fit the ‘successful application of industrialisation’ in the title better than the First, i.e. that industrialisation had a much wider impact on this conflict. The American Civil War can be discussed by candidates, the industrialised North defeating the non-industrialised South but the successful application of command in the question must be addressed in this context given that it took time for the Union’s industrial might to overcome the Confederacy. Candidates need to be aware of how industrialisation had an impact on the successful waging of war in the period. Alternative explanations are possible and examiners must be open to alternative approaches. If in doubt, consult your Team Leader.

101

Page 287: History A - The Bicester Schoolthebicesterschool.org.uk/wp-content/uploads/2015/09/Mark... · 2015-09-28 · History A Mark Schemes for the Units January 2009 H106/H506/MS/R/09J

F966 Mark Scheme January 2010

Theme 3: Britain and Ireland 1798-1921. 7 ‘Revolutionary nationalism in Ireland consistently failed to develop mass support.’

How far would you agree with this view of the period from 1798 to 1921?

No set conclusions are to be expected, but candidates must answer the question and address the theme over the whole period. Candidates may well agree with the question’s proposition, citing the low numbers involved in Emmet’s rising (1803), Young Ireland in 1848, the Fenian Outrages in 1867, the Phoenix Park murders of 1882 and in the Easter Rising of 1916. They could refer to the proscribed nature of Republicanism, their preference for rebellion, assassination and risings, the secretive nature of their methods and their violence, which was met with the same. The revolutionary tradition was one of martyrdom (Tone, Emmet, 1916 etc.). They did not hold mass meetings; their oratory was that of the scaffold, finance was a problem until they tapped into the Irish American diaspora, whilst their policy of an independent republic lacked appeal in peasant Ireland, until the Land League in the late 1870s made the connection with Alien landlords. They also failed to gain the support of the Catholic Church pending the development of a conservative and Catholic social policy in the 1910s. However candidates could also challenge the view of ‘consistent failure’ with reference to the post 1870 period when land issues were exploited by Davitt, especially the rural support gained during the Land war of 1879-82 (where boycott was preferred to violence). There was also more success after the 1890s in widening the basis of support through the harnessing of cultural nationalism (the Gaelic Athletics Association was hijacked by the IRB for propaganda and recruitment purposes). Connolly’s socialism made some headway amongst the Dublin working class but it lacked a rural base, as did Griffith’s Sinn Fein. It could be argued that considerable success came in and after 1917 following government mistakes (Conscription, Black and Tans) and with a popular mandate for Sinn Fein in the 1918 election. Collins’ strategies (the Anglo Irish War) and a comparison with Wolfe Tone may prove useful here, with the careful targeting of the police and local authorities. Alternative explanations are possible and examiners must be open to alternative approaches. If in doubt consult your Team Leader. [60]

8 Assess the view that Gladstone’s first Liberal government (1868-1874) was the most

important turning point in Britain’s relationship with Ireland in the period from 1798 to 1921.

No set conclusions are to be expected, but candidates must answer the question and address the theme over the whole period. Candidates could argue that it did, citing Gladstone’s unusual statement on hearing the news – ‘My mission is to pacify Ireland’, followed by a PM focus on solving the issue of an ‘Alien Church’ (Disestablishment of the Church of Ireland in 1869and its ending of the Protestant Constitution and the religious ascendancy of the Anglo Irish establishment); the attempt in the 1st Land Act in 1870 to address the legal issues (the three ‘Fs’) of Land and the encouragement of a more inclusive ruling class via University reform in 1873. In support of this it could be argued that Gladstone was under little pressure to introduce an Irish programme after 20 years of relative indifference and that it inaugurated a focus on the issue that was to last until 1921, by Gladstone and especially, but not exclusively, by Liberal governments in general. However, candidates might take a different view, arguing that Gladstone’s 1st government was entirely consistent with both previous and successive governments who remained wedded to the Union. Gladstone could be seen as following the tradition of Pitt (who wanted to stabilise the relationship and extend the economic benefits of Union) and Peel. The latter’s government had also sought to bring the Catholic Church on board (Maynooth). Peel’s Land Bill, following the Devon Commission, and the University Bill, prefigured almost exactly Gladstone’s 1870 Land Act and University Bill. Gladstone in 1868 was therefore not a turning point, unless one stresses Disestablishment, which

102

Page 288: History A - The Bicester Schoolthebicesterschool.org.uk/wp-content/uploads/2015/09/Mark... · 2015-09-28 · History A Mark Schemes for the Units January 2009 H106/H506/MS/R/09J

F966 Mark Scheme January 2010

neither Pitt nor Peel had wanted to adopt, although both had introduced Catholic Emancipation, arguably a much greater turning point. It could be argued that Gladstone’s 2nd Land Act in 1881 and the Conservative ones of the 1880s and 1890s were of much greater significance, whilst much could be made of the First Home Rule Bill in 1886 and its successors in 1893 and 1912 as marking a much more important turning point in that it represented a return to the pluralism of Grattan’s parliament in 1798-1800 and had become the policy of all governments after 1912, to be applied both North and South post 1918. Another route for candidates would be to argue that reform within the Union remained British policy throughout and as such the turning points lay at the beginning and end of the periods (1800 Act of Union and the Anglo-Irish Treaty 1921). Alternative explanations are possible and examiners must be open to alternative approaches. If in doubt, consult your Team Leader. [60]

9 How successfully did British governments deal with Irish land issues in the period

from 1798 to 1921?

No set conclusions are to be expected, but candidates must answer the question and address the theme over the whole period. Candidates will need to grasp the significance of Irish land issues in the period. Ireland was overwhelmingly peasant and agrarian outside parts of Ulster and Dublin. One key issue was the legal basis of the land settlement in Ireland. The British had imposed this in the 17th century, basing it on religious confiscation and the concept of landlord supremacy in relation to his tenants. This in turn was opposed by Irish tradition, Ulster tenant right, which effectively gave a form of dual ownership to landlord and tenant. Such grievances fuelled much of the discontent in Ireland in the late 18th and 19th centuries and were increasingly harnessed by both constitutional and revolutionary nationalism from the 1870s (the Land League). No British government sought to deal with this before Peel in the 1840s (the Devon Commission and subsequent Bill), Pitt seeing economic union and an efficient landlord class as the key to agrarian prosperity. The Whigs in the 1830s had de-linked the Tithe from tenant payment to landlord but had increased resentment of the latter that raised rents to compensate. Peel had failed in the face of parliamentary landlordism, which remained entrenched well into the 1870s. Gladstone sought to deal with tenant right in the 1870 Land Act but failed given the loopholes. He had more success when confronted with a Land War in 1881 (the Second Land Act which effectively granted dual ownership and a final recognition that Arrears had to be dealt with in an 1882 Act).Until the 1880s it was Tenant Right and evictions that were the focus. The other key issue was the backwardness of Land and it could be argued that this was not dealt with until post 1880. Pitt and Peel were frustrated by the failure of capitalist agriculture to emerge in anything other than moderate and regional form (East and the Midlands). The West remained backward, overpopulated and potato dependent, as the famine was to show. British governments dealt more successfully with this following the agricultural depression of the late 1870s which rendered Landlordism less powerful and more willing to embrace land sales and purchase schemes offered by Gladstone, and the Constructive Unionism of Ashbourne, Balfour and Wyndham. The latter, by providing loans for purchase at rates that were less than rentals, effectively solved the Land problem in relationship to issues of ownership. Nonetheless backwardness remained a problem. Both Conservative and Liberal governments in the 1890s and early 1900s poured money into marketing schemes and fisheries, with some effect. The pattern would seem to be some success from the 1880s onwards. Alternative explanations are possible and examiners must be open to alternative approaches. If in doubt, consult your Team Leader. [60]

103

Page 289: History A - The Bicester Schoolthebicesterschool.org.uk/wp-content/uploads/2015/09/Mark... · 2015-09-28 · History A Mark Schemes for the Units January 2009 H106/H506/MS/R/09J

F966 Mark Scheme January 2010

Theme 4: Russia and its Rulers 1855 – 1964

10 ‘The nature of Russian government was changed more by Stalin than by any other ruler.’ How far do you agree with this view of the period from 1855 to 1964? [60]

No set conclusions are to be expected, but candidates must answer the question and address the theme over the full period. Candidates should focus on the phrase ‘the nature of Russian government’ in their answers. Candidates may argue either for or against Stalin as having changed the nature of Russian government most, but must do so comparatively in the context of other rulers and leaders. Candidates may argue that the highly dictatorial nature of the Stalinist regime justifies this view and are likely to support this by reference to events such as the terror and the purges. Others may argue that this represented continuity with the nature of much previous Russian government, even if the scale was much greater. Many candidates may show awareness that some historians see great continuity between Lenin and Stalin whereas others view Stalin as significantly different from Lenin. This could be very usefully debated. Candidates may argue in favour of Alexander II because of the emancipation of the serfs and his other reforms such as the zemstva. Candidates may argue in favour of Alexander III because of ‘the Reaction’ though many will see this as a reversion to traditional autocracy. Candidates may argue that the end of over 300 years of Romanov rule in February 1917 was the most significant turning point in the nature of Russian government as it ended the 304 year old Romanov dynasty, but may argue that ultimately this led to the replacement of ‘Romanov Tsars’ by ‘red Tsars’. Many candidates will undoubtedly argue that October 1917 and the triumph of Bolshevism significantly changed the nature of Russian government as it crushed all possibility that a liberal democracy might emerge in Russia. Candidates may argue that Khrushchev’s secret speech of 1956 and his subsequent de-stalinisation marked a significant change in the nature of Russian government.

Examiners must be open to alternative approaches. If in doubt, they should consult their Team Leader.

11 Assess the view that all the rulers of Russia had similar aims in domestic policy in the period from 1855 to 1964. [60]

No set conclusions are to be expected, but candidates must answer the question and address the theme over the full period. Candidates should focus on the phrase ‘similar aims in domestic policy’ in their answers. Candidates are likely to want to argue both for and against this assertion. Candidates may well argue that retention of power, whether autocratic or dictatorial, and the crushing of opposition were priorities for all the rulers even if some were singularly unsuccessful in achieving those goals. Candidates may well argue that the modernisation of Russia was an aim for all the rulers, though candidates are likely to differentiate between rulers such as Alexander III and Stalin in terms of motives and extent. Candidates may however wish to argue that the communist rulers had very different core priorities to the Tsars in terms of political ideology and social priorities; others may contend that this should have been the case but that rulers, especially Stalin (though some will also indict Lenin). Candidates may argue that the Tsars were not uniform in their core aims; they are likely to see Alexander II as having different priorities to his successors, citing emancipation and the other reforms of the 1860s in support. Candidates may also argue that the communist rulers were not uniform in their core aims either; they are likely to argue that Khrushchev had very different priorities to Stalin, citing de-stalinisation as support. Candidates may well understand that whether Lenin and Stalin had similar aims is subject to historical debate.

Examiners must be open to alternative approaches. If in doubt, they should consult their Team Leader.

104

Page 290: History A - The Bicester Schoolthebicesterschool.org.uk/wp-content/uploads/2015/09/Mark... · 2015-09-28 · History A Mark Schemes for the Units January 2009 H106/H506/MS/R/09J

F966 Mark Scheme January 2010

12 Assess the view that the lives of the peasants in Russia did not improve in the period from 1855 to 1964. [60]

No set conclusions are to be expected, but candidates must answer the question and address the theme over the full period. Candidates should focus on the phrases ‘lives of the peasants’ and ’did not improve’ in their answers. Candidates may well consider how valid the phrase ’did not improve’ is. Candidates may argue that some rulers, for example Alexander II and Khrushchev made a sustained attempt to improve the lives of the people. Some candidates may argue that despite the brutality, Lenin and even Stalin did do some things that improved the lives of the people. Arguably the communists did much more to introduce social reform, for example in the sphere of education, than the Tsars. Candidates may also argue that there was little real improvement in the lives of the people. For example, peasants were serfs under the Romanovs until 1861, but candidates may argue that there was little real improvement and / or that collectivization was a ‘second serfdom’. Before and after 1917 there was harsh treatment of the peasantry by both regimes; ‘squeezed dry’ to finance industrialization. Famine hit, e.g. 1891, 1921 & 1932, regardless of regime, although arguably Stalin’s denial of the famine of the 1930s made its impact worse. Control over their lives, whether exercised through the Mir, the Land Captains & the Kolkhoz was a common feature, although distinctions may clearly be made. Candidates may use the systematic Russification of the non-Russian peasants both before and after 1917 as another clear example of there being no significant change. Candidates may also wish to argue that there were times when rulers did improve the lives of the peasants, but that these improvements were most typically temporary rather than embedded. For example, the peasants were given glimpses of reform, e.g. the Peasants Land Bank from the 1880s, the Decree on Land in 1917 and the NEP from 1921. All of these changes led to improvements, albeit temporary, in their living and working conditions. Both regimes had a temporary Kulak policy under Stolypin from 1906 & under the NEP from 1921-28 as peasants were encouraged to ‘enrich themselves’.

Examiners must be open to alternative approaches. If in doubt, they should consult their Team Leader.

105

Page 291: History A - The Bicester Schoolthebicesterschool.org.uk/wp-content/uploads/2015/09/Mark... · 2015-09-28 · History A Mark Schemes for the Units January 2009 H106/H506/MS/R/09J

F966 Mark Scheme January 2010

Theme 5: Civil Rights in the USA 1865 – 1992 13 To what extent did the aims of the campaigners for African American civil rights

remain the same in the period from 1865 to 1992?

No set conclusions are to be expected, but candidates must answer the question and address the theme over the whole period. The weaker candidates will concentrate almost exclusively on the various African American leaders and will probably not go beyond contrasting the non-violent methods of Booker T Washington and Martin Luther King with the more radical goals and methods of leaders such as Du Bois, Garvey, Malcolm X, the Black Power movement and the Black Panthers. Weaker candidates are unlikely to have much to say about civil rights campaigns after 1965 beyond observing that King was less successful in his final years. The majority of candidates should be able to contrast the limited aims of Booker T Washington – offering black co-operation with a white-dominated society in return for economic gains – with the legal campaigns of the NAACP to secure their rights under the 14th and 15th amendments and Martin Luther King’s high profile campaign to abolish the whole Jim Crow system. They should also be able to contrast these assimilationist goals with the separatism advocated by leaders such as Garvey and Malcolm X. The best candidates will explain how changed circumstances, especially in the post-war period enabled campaigners to pursue a more ambitious strategy than previous leaders. They will also refer to the importance of grass-roots activism when writing, for example, about the Sit-In campaigns and the Freedom Rides. The better candidates should also be able to analyse the lack of coherence in the aims of the Black Power movement and the Black Panthers as well as analysing the problems the civil rights movement experienced in the 1970s and 1980s with controversy over bussing and affirmative action and perceptions of continued police discrimination (reference could be made to the Rodney King incident of 1992). High level answers will make effective comparisons over the whole period, perhaps referring to the persistence of de facto discrimination and the existence of a large underclass trapped in a cycle of poverty, unemployment, poor housing in contrast to a prosperous black middle class which had taken advantage of the changed attitude to racism and the abolition of de jure discrimination. Examiners must be open to alternative approaches. If in doubt, they should consult their Team Leader.

14 ‘The internal divisions within the trade union and labour movement in the USA the

most important obstacle to the progress of labour rights in the period from 1865 to 1992’. How far do you agree? No set conclusions are to be expected, but candidates must answer the question and address the theme over the whole period. Weaker candidates will probably refer only in general terms to the divisions over membership between the Knights of Labor and the AFL in the late 19th century, and the AFL and the CIO in the 1930s. They might also refer to ethnic conflict between whites and blacks and between native-born workers and the new immigrants of the late 19th and early 20th centuries. Some weaker candidates may also discount (or even ignore) the internal divisions within the labour movement and provide a list of alternative factors without adequately comparing their importance. Many (perhaps most) candidates, while acknowledging the divisions within the labour movement, will concentrate their analysis on the other obstacles to the progress of labour rights. These include the use force of by government authorities to end strikes (for example, President Cleveland during the Pullman strike in 1894 or Massachusetts Governor Coolidge in Boston in 1919), the willingness of the courts to issue injunctions against strikers under the Sherman Anti-Trust Act, specifically anti-labour legislation such as the Taft-Hartley Act of 1947 and the Landrum-Griffin act of 1959, and the employers’ use of both yellow dog

106

Page 292: History A - The Bicester Schoolthebicesterschool.org.uk/wp-content/uploads/2015/09/Mark... · 2015-09-28 · History A Mark Schemes for the Units January 2009 H106/H506/MS/R/09J

F966 Mark Scheme January 2010

contracts and violent strike-breakers (such as Pinkerton guards in the Homestead strike). Most candidates should attempt some evaluation of the relative importance of these factors in relation to divisions among workers. Better candidates will be able to analyse how far labour divisions were exacerbated by a climate in the USA hostile to the progress of labour movements and point out that this climate was, in part, the result of extremist labour movements (such as the Molly Maguires or the Wobblies) but that it also made it easy for employers and governments to associate trade unionism with socialism and communism, thereby undermining middle class sympathy for workers. High level candidates will be aware that the arguments within the labour movement extended beyond union membership and tactics and were bedevilled by divisions over race, class, religion and gender. They will provide intelligently selected examples to illustrate these divisions. High calibre candidates might, as well as analysing these divisions and the obstacles, point out that the progress of labour rights depended crucially on the support of the federal government (as in the New Deal and the JFK-LBJ years). They will also be able to analyse the importance of wider economic conditions, especially in explaining the decline of trade union power since the 1950s. Examiners must be open to alternative approaches. If in doubt, they should consult their Team Leader.

15 ‘The Indian Reorganization Act in the New Deal was the most important turning

point in the development of Native American civil rights in the USA in the period from 1865 to 1992.’ How far do you agree with this view?

No set conclusions are to be expected, but candidates must answer the question and address the theme over the whole period. Most candidates will agree with this statement and point out that the Indian New Deal replaced forced assimilation with respect for Indian culture and customs which, arguably, has persisted to the present. Furthermore, the Indian Reorganization Act prevented the alienation of more tribal land and granted Indian communities a measure of governmental and judicial autonomy. Weaker candidates will probably see this as a stage of “good” policy towards Indians as opposed to the “bad” periods of forced assimilation from 1887 until 1933 and the termination policy of the 1950s. As such, they will regard the Indian Reorganization Act as a significant turning point, although such candidates are unlikely to be able to compare it with other turning points. Most candidates will offer some alternatives (such as the 1887 Dawes Act at the end of the Indian Wars, the end of the Indian New Deal in 1945, the end of termination in the 1960s or the first militant action of Red Power in 1969) but their attempts to compare them with the Indian Reorganization Act may be unconvincing. The best candidates will be aware of the limitations of Indian Reorganization Act and the degree of opposition to it (for different reasons) in Congress and among Native Americans. They might point out that Collier’s policies were as paternalistic in their way as all other federal Indian policies, and that the majority of white Americans (and a significant number of Indians themselves) continued to believe that Indians should assimilate into mainstream US culture. As such, the Indian Reorganization Act could be seen as an aberration rather than a turning point. The best candidates might also contrast the importance of federal policy shifts with the impact of economic and social change in the USA on Native American lives and attitudes (especially the two world wars, the Cold War and the post-war expansion of the US economy). Alternatively, the best candidates will make effective comparisons between the different turning points they analyse and explain fully why one is more important than the others.

Examiners must be open to alternative approaches. If in doubt, they should consult their Team Leader.

107

Page 293: History A - The Bicester Schoolthebicesterschool.org.uk/wp-content/uploads/2015/09/Mark... · 2015-09-28 · History A Mark Schemes for the Units January 2009 H106/H506/MS/R/09J

F966 Mark Scheme January 2010

Theme 6: The Development of Democracy in Britain 1868 – 1997 16 How far did economic factors influence the results of general elections from 1868 to

1997?

No set conclusions are to be expected, but candidates must answer the question and address the theme over the whole period. Economic change undoubtedly influenced the outcome of a significant number of elections during the period particularly where it was linked with a slow down in economic growth and associated rises in unemployment. For example, the parties that lost elections in 1868, 1880, 1922, 1929 and 1979 were all linked, and to an extent blamed for, high levels of unemployment, falling real wages and a lowering of living standards. Some candidates are likely to show that there were also other economic issues that dominated some election campaigns such as tariff reform (1906, 1923), fiscal policy (1910), changes in the industrial infrastructure (1966), a faltering balance of payments (1970), and rising fuel prices (1974). It would, of course, be highly simplistic to argue that elections were always fought over the issue of economic change. Most candidates should be able to discuss a range of other factors. These might include: external influences ( e.g. the effects of wars on the 1906, 1918, 1945 and 1983 elections); social changes ( e.g. the impact and debate over the NHS which influenced the result of the 1951 election; consumerism which prompted Macmillan’s election winning slogan ‘You’ve never had it so good’ in 1959); internal party divisions ( e.g. the Liberals in the 1880s and1920s, Labour in the mid-1950s) and the coming of class based politics ( e.g. the rise of the Labour party to push the Liberals into third place, the rise of general unionism and industrial unrest which affected the outcome of elections in the 1880s, 1906, 1951 and, most significantly, 1979). A strong case could be made for economic factors mainly (but not solely) influencing the results of elections although better responses will probably offer a more balanced analysis and evaluation. Also, a decent, but not exhaustive, range of elections should be referenced to illustrate change and/or continuity over the period. Examiners must be open to alternative approaches. If in doubt, they should consult their Team Leader.

17 ‘There was more continuity than change in the role that Prime Ministers played in

the development of the democratic system from 1868 to 1997.’ How far do you agree?

No set conclusions are to be expected, but candidates must answer the question and address the theme over the whole period. Over the period, parliaments were managed differently, monarchs varied, control of cabinets increased and there was more responsibility for international affairs (especially wars) and disposal of finance (e.g. the Secret Fund). It might be argued that prime ministers handled such changes more skilfully as time went by, whilst having their power ‘checked’ by political institutions such as political parties, pressure groups, the judiciary and the media. Thus, prime ministers were not allowed to abuse their power. They were expected to play a responsible role in the maintenance and enhancement of democracy. Another line of argument might be that the power of prime ministers was never kept in check enough, especially with respect to the latter part of the period. Developments worth discussing might include ‘Prime Ministers Questions’, the appointment system, collective decision making in the Cabinet, control over Cabinet meetings, the exploitation of new technology (e.g. Baldwin’s use of the radio), influence on the Budget, the power to call elections, the right to remove ministers (1905 onwards) and decisions to go to war (e.g Chamberlain, Thatcher). There should be a focus on assessing the extent of continuity and change in the role of prime ministers; there is little need for candidates to discuss the relative importance of other factors on democracy.

108

Page 294: History A - The Bicester Schoolthebicesterschool.org.uk/wp-content/uploads/2015/09/Mark... · 2015-09-28 · History A Mark Schemes for the Units January 2009 H106/H506/MS/R/09J

F966 Mark Scheme January 2010

109

Examiners must be open to alternative approaches. If in doubt, they should consult their Team Leader.

18 Assess the view that educational reform was the most important influence on the

development of democracy from 1868 to 1997.

It is plausible to argue that there was considerable continuity in the way in which educational reforms influenced the development of democracy. The education acts of 1870 and 1880 seemed to influence a raft of political reforms that were implemented before the end of the nineteenth century. Forster’s Act filled the gaps in elementary school provision especially in the countryside and it was no coincidence that the franchise was extended in 1884 to include £10 householders and lodgers residing in rural areas. The other major political reform of the time, the Redistribution of Seats Act, 1885, was closely linked to the Third Reform Act. This pattern of educational reform influencing changes to the democratic system continued in to the twentieth century. The reforms included the education policies of the Liberals from 1906 to 1910 which had a bearing on the 1910 constitutional crisis; Fisher’s Act of 1918 which occurred at the same time as the Representation of the People’s Act; and the policy initiatives of the inter-war years (Hadow, Spens, Tawney) which tied in with a further extension to the franchise (1928). Some educational reforms, such as that of 1870 and 1988, actually allowed direct participation in politics through the educational system (e.g. election to school boards, governing bodies). Although there were other influences on changes to the democratic system, the expansion in the number of people who were educated undoubtedly caused politicians to implement policies that engendered greater participation in the political process. Also, more generally, educational reforms resulted in a demand for a change to the way in which people were represented, hence the rise of the Labour party and Trades Unions. However, better candidates are likely to point out that some changes to education did not improve the democratic system. The 1902 act appeared to be an attempt to maintain the ruling status quo and the 1944 act created a tripartite system that, in the long run, seemed to reflect a divided society in which many experienced a limited educational and, subsequently, vocational choice. More contentiously, governments throughout the whole of the period continued to allow the existence and growth of the private education sector which could also be viewed as divisive and not characteristic of a truly democratic society. Finally, some candidates might point out that educational change was more likely to follow democratic change. This was partly true of the 1870 act (‘we must educate our masters’) and the creation of the Board of Education in 1899. Examiners must be open to alternative approaches. If in doubt, they should consult their Team Leader.

Page 295: History A - The Bicester Schoolthebicesterschool.org.uk/wp-content/uploads/2015/09/Mark... · 2015-09-28 · History A Mark Schemes for the Units January 2009 H106/H506/MS/R/09J

Grade Thresholds

Advanced GCE History (H506) Advanced Subsidiary GCE History (H106) January 2010 Examination Series Unit Threshold Marks

Unit Maximum Mark

A B C D E U

Raw 100 73 64 56 48 40 0 F961/01 UMS 100 80 70 60 50 40 0 Raw 100 73 64 56 48 40 0 F961/02 UMS 100 80 70 60 50 40 0 Raw 100 73 65 57 50 43 0 F962/01 UMS 100 80 70 60 50 40 0 Raw 100 73 65 57 50 43 0 F962/02 UMS 100 80 70 60 50 40 0 Raw 100 68 60 53 46 39 0 F963/01 UMS 100 80 70 60 50 40 0 Raw 100 68 60 53 46 39 0 F963/02 UMS 100 80 70 60 50 40 0 Raw 100 67 60 53 46 40 0 F964/01 UMS 100 80 70 60 50 40 0 Raw 100 67 60 53 46 40 0 F964/02 UMS 100 80 70 60 50 40 0 Raw 80 64 56 48 40 32 0 F965 UMS 80 64 56 48 40 32 0 Raw 120 85 76 68 60 52 0 F966/01 UMS 120 96 84 72 60 48 0 Raw 120 85 76 68 60 52 0 F966/02 UMS 120 96 84 72 60 48 0

Specification Aggregation Results Overall threshold marks in UMS (ie after conversion of raw marks to uniform marks) Maximum

Mark A B C D E U

H106 200 160 140 120 100 80 0

The cumulative percentage of candidates awarded each grade was as follows:

A B C D E U Total Number of Candidates

H106 18.56 46.74 75.61 92.42 99.02 100.00 1371

For a description of how UMS marks are calculated see: http://www.ocr.org.uk/learners/ums_results.html Statistics are correct at the time of publication.

110

Page 296: History A - The Bicester Schoolthebicesterschool.org.uk/wp-content/uploads/2015/09/Mark... · 2015-09-28 · History A Mark Schemes for the Units January 2009 H106/H506/MS/R/09J

OCR (Oxford Cambridge and RSA Examinations) 1 Hills Road Cambridge CB1 2EU OCR Customer Contact Centre 14 – 19 Qualifications (General) Telephone: 01223 553998 Facsimile: 01223 552627 Email: [email protected] www.ocr.org.uk For staff training purposes and as part of our quality assurance programme your call may be recorded or monitored

Oxford Cambridge and RSA Examinations is a Company Limited by Guarantee Registered in England Registered Office; 1 Hills Road, Cambridge, CB1 2EU Registered Company Number: 3484466 OCR is an exempt Charity OCR (Oxford Cambridge and RSA Examinations) Head office Telephone: 01223 552552 Facsimile: 01223 552553 © OCR 2010

Page 297: History A - The Bicester Schoolthebicesterschool.org.uk/wp-content/uploads/2015/09/Mark... · 2015-09-28 · History A Mark Schemes for the Units January 2009 H106/H506/MS/R/09J

Oxford Cambridge and RSA Examinations

GCE

History A Advanced GCE F966/02

Historical Themes Option B: Modern 1789-1997

Mark Scheme for June 2010

Page 298: History A - The Bicester Schoolthebicesterschool.org.uk/wp-content/uploads/2015/09/Mark... · 2015-09-28 · History A Mark Schemes for the Units January 2009 H106/H506/MS/R/09J

OCR (Oxford Cambridge and RSA) is a leading UK awarding body, providing a wide range of qualifications to meet the needs of pupils of all ages and abilities. OCR qualifications include AS/A Levels, Diplomas, GCSEs, OCR Nationals, Functional Skills, Key Skills, Entry Level qualifications, NVQs and vocational qualifications in areas such as IT, business, languages, teaching/training, administration and secretarial skills. It is also responsible for developing new specifications to meet national requirements and the needs of students and teachers. OCR is a not-for-profit organisation; any surplus made is invested back into the establishment to help towards the development of qualifications and support which keep pace with the changing needs of today’s society. This mark scheme is published as an aid to teachers and students, to indicate the requirements of the examination. It shows the basis on which marks were awarded by Examiners. It does not indicate the details of the discussions which took place at an Examiners’ meeting before marking commenced. All Examiners are instructed that alternative correct answers and unexpected approaches in candidates’ scripts must be given marks that fairly reflect the relevant knowledge and skills demonstrated. Mark schemes should be read in conjunction with the published question papers and the Report on the Examination. OCR will not enter into any discussion or correspondence in connection with this mark scheme. © OCR 2010 Any enquiries about publications should be addressed to: OCR Publications PO Box 5050 Annesley NOTTINGHAM NG15 0DL Telephone: 0870 770 6622 Facsimile: 01223 552610 E-mail: [email protected]

Page 299: History A - The Bicester Schoolthebicesterschool.org.uk/wp-content/uploads/2015/09/Mark... · 2015-09-28 · History A Mark Schemes for the Units January 2009 H106/H506/MS/R/09J

F966/02 Mark Scheme June 2010

1

Maximum mark 120 for this unit. 2 answers: Each maximum mark 60

A01a A01b

IA 18-20 36-40

IB 16-17 32-35

II 14-15 28-31

III 12-13 24-27

IV 10-11 20-23

V 8-9 16-19

VI 4-7 8-15

VII 0-3 0-7

Notes:

(i) Allocate marks to the most appropriate level for each AO.

(ii) If several marks are available in a box, work from the top mark down until the best fit has been found.

(iii) Many answers will not fall at the same level for each AO.

(iv) Candidates will demonstrate synoptic skills by drawing together appropriate techniques, knowledge and understanding to evaluate developments over the whole of the period

Page 300: History A - The Bicester Schoolthebicesterschool.org.uk/wp-content/uploads/2015/09/Mark... · 2015-09-28 · History A Mark Schemes for the Units January 2009 H106/H506/MS/R/09J

F966/02 Mark Scheme June 2010

2

AOs AO1a AO1b

Total mark for each question = 60

Recall, select and deploy historical knowledge appropriately, and communicate knowledge and understanding of history in a clear and effective manner.

Demonstrate understanding of the past through explanation, analysis and arriving at substantiated judgements of: - key concepts such as causation, consequence, continuity, change and significance within an historical context; - the relationships between key features and characteristics of the periods studied

Level IA

• Uses a wide range of accurate and relevant evidence • Accurate and confident use of appropriate historical terminology • Answer is clearly structured and coherent; communicates accurately and legibly. 18-20

• Excellent understanding of key concepts (eg. continuity and change) relevant to analysis in their historical context • Excellent synthesis and synoptic assessment • Answer is consistently and relevantly analytical with developed explanations and supported judgements • May make unexpected but substantiated connections over the whole period 36-40

Level IB

Level IB • Uses accurate and relevant evidence • Accurate use of a range of appropriate historical terminology • Answer is clearly structured and mostly coherent; communicates accurately and legibly

16-17

• Very good level of understanding of key concepts (eg. continuity and change) in their historical context. • Answer is consistently focused on the question set • Very good level of explanation/ analysis, and provides supported judgements. • Very good synthesis and synoptic assessment of the whole period 32-35

Level II

• Uses mostly accurate and relevant evidence • Generally accurate use of historical terminology • Answer is structured and mostly coherent; writing is legible and communication is generally clear 14-15

• Good level of understanding of key concepts (eg. continuity and change) in their historical context • Good explanation/ analysis but overall judgements may be uneven • Answer is focused on the issues in the question set • Good synthesis and assessment of developments over most of the period 28-31

Page 301: History A - The Bicester Schoolthebicesterschool.org.uk/wp-content/uploads/2015/09/Mark... · 2015-09-28 · History A Mark Schemes for the Units January 2009 H106/H506/MS/R/09J

F966/02 Mark Scheme June 2010

3

Level III

• Uses relevant evidence but there may be some inaccuracy • Answer includes relevant historical terminology but this may not be extensive or always accurately used • Most of the answer is structured and coherent; writing is legible and communication is generally clear 12-13

• Shows a sound understanding of key concepts, especially continuity and change, in their historical context • Most of the answer is focused on the question set • Answers may be a mixture of analysis and explanation but also description and narrative, but there may also be some uneven overall judgements; OR answers may provide more consistent analysis but the quality will be uneven and its support often general or thin • Answer assesses relevant factors but provides only a limited synthesis of developments over most of the period 24-27

Level IV

• There is deployment of relevant knowledge but level/ accuracy will vary. • Some unclear and/or underdeveloped and/or disorganised sections • Mostly satisfactory level of communication 10-11

• Satisfactory understanding of key concepts (eg. continuity and change) in their historical context • Satisfactory focus on the question set • Answer may be largely descriptive/ narratives of events, and links between this and analytical comments will typically be weak or unexplained • Makes limited synoptic judgements about developments over only part of the period 20-23

Level V

• General and basic historical knowledge but also some irrelevant and inaccurate material • Often unclear and disorganised sections • Adequate level of communication but some weak prose passages 8-9

• General understanding of key concepts (eg. continuity and change) in their historical context • Some understanding of the question but answers may focus on the topic and not address the question set OR provides an answer based on generalisation • Attempts an explanation but often general coupled with assertion, description / narrative • Very little synthesis or analysis and only part(s) of the period will be covered 16-19

Page 302: History A - The Bicester Schoolthebicesterschool.org.uk/wp-content/uploads/2015/09/Mark... · 2015-09-28 · History A Mark Schemes for the Units January 2009 H106/H506/MS/R/09J

F966/02 Mark Scheme June 2010

4

Level VI • Use of relevant evidence will be limited; there will be much irrelevance and inaccuracy • Answers may have little organisation or structure • Weak use of English and poor organisation

4-7

• Very little understanding of key concepts (eg. continuity and change) in their historical context • Limited perhaps brief explanation • Mainly assertion, description / narrative • Some understanding of the topic but not the question’s requirements 8-15

Level VII • Little relevant or accurate Knowledge • Very fragmentary and disorganised response • Very poor use of English and some incoherence 0-3

• Weak understanding of key concepts (eg. continuity and change) in their historical context • No explanation • Assertion, description / narrative predominate • Weak understanding of the topic or of the question’s requirements 0-7

Page 303: History A - The Bicester Schoolthebicesterschool.org.uk/wp-content/uploads/2015/09/Mark... · 2015-09-28 · History A Mark Schemes for the Units January 2009 H106/H506/MS/R/09J

F966/02 Mark Scheme June 2010

5

Theme 1: The Challenge of German Nationalism 1789 – 1919 1 Assess the view that the ideas and aims of German nationalism changed

significantly during the period from 1789 to 1919. [60]

No set conclusions are to be expected, but candidates must answer the question and address the theme over the full period. Candidates should assess whether the ideas and aims of German nationalism changed significantly in this period or whether they predominantly remained the same. Candidates should focus on the dominant ideas and aims of German nationalists during this period, from the aims of the growing emergent nationalist movement from 1789 to the aims of more radical nationalists in the late Nineteenth and early Twentieth Centuries. The development and impact of ideas on the aims of nationalism should be explored. Candidates may argue that the common fight of people from different German states against the French, especially in 1813, gave strong impulses to nationalism. A few intellectuals consequently demanded the unification of all German-speaking lands, although they represented a minority. Candidates are likely to show knowledge of developments in intellectual nationalism in the first half of the Nineteenth Century. Candidates might well demonstrate understanding of the debate about Grossdeutschland or Kleindeutschland in the period 1815 – 1871 and the reasons for the development of more radical nationalism in the remainder of the period. Candidates might explain the importance of economic factors on the changing aims of German nationalism, for example the impact of the Zollverein after 1834 in developing Prussia’s economic strength and Prussian leadership of Germany. Candidates should understand how developments in the economy in the 1850s paved the way for the military victories of 1864, 1866 and 1870 / 71. Bismarck’s opportunistic and skilful leadership clearly had a significant impact on the development of German nationalism too. The change of heart from 1866 when liberals became national liberals may well be stressed by some candidates. The impact of the foundation of the Second Reich from 1871 clearly had a profound effect on the development of nationalism during the latter part of this period as did the accession to the throne of Wilhelm II. Candidates may focus on the more radical nationalism of the Wilhelmine period and its ultimately disastrous impact on the German nation. The Great War left Germany broken and half-starved with the aims of German nationalists in tatters.

Examiners must be open to alternative approaches. If in doubt, they should consult their Team Leader.

2 Assess the view that the Prussian victory over Austria in 1866 was the most

important turning point in the growth of German nationalism in the period from 1789 to 1919. [60]

No set conclusions are to be expected, but candidates must answer the question and address the theme over the full period. Candidates should focus on the phrase ‘most important turning point’ in their answers. Candidates may argue either for or against the Prussian victory over Austria in 1866 as the most important turning point, but must do so comparatively in the context of other turning points. In assessing the significance of the Prussian victory over Austria in 1866 candidates are likely to stress the decline of Austrian influence in Germany, the North German Confederation and the consequent domination of Germany by Prussia. What follows is not an exclusive list of other potential turning-points, but obvious consideration could be given to 1789, 1813, 1815, 1848-49, 1870/71, 1888 (and / or 1890) and 1914. Clearly answers of the very highest quality can be written without considering all of these potential turning points, but the most able candidates will demonstrate a breadth of vision and a good understanding of the moments that shaped the destiny of German nationalism.

Examiners must be open to alternative approaches. If in doubt, they should consult their Team Leader.

Page 304: History A - The Bicester Schoolthebicesterschool.org.uk/wp-content/uploads/2015/09/Mark... · 2015-09-28 · History A Mark Schemes for the Units January 2009 H106/H506/MS/R/09J

F966/02 Mark Scheme June 2010

6

3 To what extent did German nationalism have mass appeal in the period from 1789 to 1919? [60] No set conclusions are to be expected, but candidates must answer the question and address the theme over the full period. Candidates should focus on ‘to what extent’, ‘German nationalism’ and ‘mass appeal’ in their answers in an attempt to evaluate the mass appeal of nationalism in this period. Candidates should evaluate the extent to which nationalism had mass appeal and demonstrate awareness that such appeal was not uniform but fluctuated. Candidates may demonstrate that concepts of romantic nationalism had a limited intellectual appeal. Candidates could consider the extent to which nationalism appealed or failed to appeal to the people at various points, for example from 1789 – 1815, 1815 – 1848, from 1871 - 1914 and in 1918. Candidates may well demonstrate that they understand that Wilhelmine Germany increasingly looked to exploit nationalist yearnings and the mass appeal of German nationalism, pursuing a populist foreign policy to distract the masses from social discontent. Candidates might choose to demonstrate that the mass appeal of nationalism may be compared to the mass appeal of other philosophies. For example the growing industrialization of Prussia and the German Empire was mirrored by the growing mass appeal of socialism, an appeal that proved relatively immune to either appeasement, in the form of state socialism, or repression. Distress from the winter of 1916 / 1917 onwards, and defeat in 1918, led to the socialist uprisings of late 1918 and early 1919 and the establishment of Ebert’s republic. However, even in 1919 the appeal of unrequited nationalism was never far from the surface, as evidenced by the Freikorps and the emerging ‘stab-in-the-back’ theory. Examiners must be open to alternative approaches. If in doubt, they should consult their Team Leader.

Page 305: History A - The Bicester Schoolthebicesterschool.org.uk/wp-content/uploads/2015/09/Mark... · 2015-09-28 · History A Mark Schemes for the Units January 2009 H106/H506/MS/R/09J

F966/02 Mark Scheme June 2009

7

Theme 2: The Changing Nature of Warfare 1792 - 1945 4 ‘The concept of ‘total war’ should be applied only to conflicts in the twentieth

century.’ How far do you agree with this view of warfare in the period from 1792 to 1945? No set conclusions are to be expected, but candidates must answer the question and address the theme over the whole period. The two twentieth century conflicts we might expect to be addressed are the First and Second World Wars. Both easily fit the concept of total war very well due to their geographic spread, the economic and political commitment needed by the victorious powers, the scale of the military forces involved, the scale of the carnage, and the involvement of large percentages of the populations of the combatant states. There are many nineteenth and eighteenth century conflicts that might be used to counter the line advanced by the question. The Revolutionary and Napoleonic Wars might be argued to rival WWI and II in scale and all of the criteria for total war advanced above could be applied to these conflicts. The same argument can be applied to the ACW. The Wars of Unification in the middle part of the nineteenth century are less convincing candidates for total war due to their short duration and limited domestic impact. Alternatively the Russo-Japanese War also does not easily fit the concept of total war. Alternative explanations are possible and examiners must be open to alternative approaches. If in doubt, consult your Team Leader.

5 To what extent did developments in transport revolutionise warfare in the period

from 1792 to 1945? No set conclusions are to be expected, but candidates must answer the question and address the theme over the whole period. Transport will probably concentrate on the application of steam power in the form of railways (and steam ships where their use applies to land warfare – the Crimean War springs to mind) in the nineteenth century and the impact of the internal combustion engine in the twentieth. Candidates need to be aware, however, that railways also had a large impact on WWII. The response must use this knowledge in an analytical fashion with focus on the specific demands of the question set. Examples might be rapid strategic movement, the ability to mobilize large numbers of soldiers, its impact on concentration of force and movement across the battlefield. The obvious examples of the impact of railways are the 1866 Austro-Prussian War, the opening months of WW1 and the support of military activity on the Eastern Front in WWII. Better candidates might argue that the railway only took armies so far and once separated from rail networks soldiers moved as fast as their 18th century forebears had done. For the impact of the internal combustion engine in WW1 candidates might consider lorries, tanks and aircraft. The effect of this technology on WWII is obvious with true mechanised warfare dominating all European and North Africa fronts. For tanks, APCs and aircraft we might expect some discussion of use in battle. For pre-steam technologies use of waterways to transport troops or mass use of horse drawn carts were important. The former was a common feature of war in the later 18th and early 19th centuries, the latter was used on occasion by Napoleon, for example to move part of his army in 1805.

Page 306: History A - The Bicester Schoolthebicesterschool.org.uk/wp-content/uploads/2015/09/Mark... · 2015-09-28 · History A Mark Schemes for the Units January 2009 H106/H506/MS/R/09J

F966/02 Mark Scheme June 2010

8

Alternative positions might point to continuity across the period. For example, the limits of railways allowing troops to be massed at given points but then having to march to war as their forefathers did, this certainly applies to the wars of the mid-19th century and WW1 and arguably also applied on the Eastern Front in WWII. The limits to mechanisation of transport in WWI & II are possible alternative interpretations. On the Western Front in WWI the nature of trench warfare seriously hindered the successful application of transport innovations to war. In WWI the Eastern Front saw transport technology very similar to that of Napoleon’s wars. This is also the case on the Eastern Front in WWII where the number of fully mechanised formations was small and the bulk of the armies fought using transport technology that would be familiar to the armies of the Austro- or Franco-Prussian Wars. Some candidates may counter-argue that factors in addition to transport also revolutionised warfare. If so, technological developments, strategic ideas, and the introduction of conscription could be usefully assessed. A balanced and evaluative response is required. Alternative explanations are possible and examiners must be open to alternative approaches. If in doubt, consult your Team Leader.

6 ‘Victory in war was determined by superior economic power.’ To what extent do you

agree with this view of the period from 1792 to 1945? No set conclusions are to be expected, but candidates must answer the question and address the theme over the whole period. Responses will need a sound understanding of what is meant by economic power and its impact on conflict. Candidates need to be able to discuss the relative economic power of various combatant states during the period studied and its impact on the outcome of wars. Economic power might be encountered in many forms; industrialisation, commerce, agricultural output, etc. Britain’s commercial and emerging industrial strength might be contrasted with France’s in the Revolutionary and Napoleonic Wars. The chronic economic underdevelopment of Russia might be the main reason for allied victory in the Crimean War. The limited ability of the economy of states such as Prussia to sustain long term wars in the middle century might be discussed. On the other hand, the economic power of Prussia might be compared to her opponents in the wars of 1866 and 1870-71. The success of an economically weaker Japan in the Russo-Japanese War challenges the precept of the question. The First and Second World Wars are tailor made for the question. The ACW is also a case in point with the larger more sophisticated economy of the Union defeating the South but only after some time. Candidates might refer to the technology produced by the economies of combatant states but this must be linked to the specific demands of the question set. Also the development of the scale of warfare as the period went on and the impact on economic systems might be a worthwhile area for discussion. Alternative explanations are possible and examiners must be open to alternative approaches. If in doubt, consult your Team Leader.

Page 307: History A - The Bicester Schoolthebicesterschool.org.uk/wp-content/uploads/2015/09/Mark... · 2015-09-28 · History A Mark Schemes for the Units January 2009 H106/H506/MS/R/09J

F966/02 Mark Scheme June 2010

9

Theme 3: Britain and Ireland 1798 – 1921 7 ‘O’Connell was the most effective leader of constitutional nationalism in the period

from 1798 to 1921.’ How far do you agree with this view? No set conclusions are to be expected, but candidates must answer the question and address the theme over the whole period. Effective answers will be comparative in nature, considering O’Connell, Butt, Parnell, Redmond and Dillon. Comments on Butt and Dillon can be brief. The focus is expected to be on O’Connell, Parnell and Redmond, particularly their relative effectiveness in mobilising support and establishing clear policy and tactics. In O’Connell’s case the achievement of Emancipation in the 1820s, reform within the Union in the 1830s and repeal of the Union in the 1840s are key areas. In the first two he achieved much, far more in practice than later leaders although, like them, Repeal and Home Rule remained elusive. Unlike Parnell, O’Connell did not see the need for economic or land reform, other than the ending of religiously based tithes. Parnell was more effective here, forcing Gladstone to reconsider Land Reform in the 1880s (linking Davitt’s Land League to his cause). O’Connell’s Roman Catholic Association became a model for ‘constitutional’ agitation in the 1820s but with less success in the 1840s. The NRA mobilised large numbers but Peel had by then removed the Freeholders. He also had less success in creating a disciplined Parliamentary party than Parnell, whose aims were much focused on Westminster (‘obstructionism’). He gained complete control of the Irish political process outside Ulster. Both he and O’Connell became divorced from Ireland itself, O’Connell creating division over his ‘moderate’ tactics, whilst Parnell never recovered from the O’Shea Divorce case splitting his party. Nonetheless candidates could argue that O.’Connell. achieved more in association with the Whigs than Parnell did with Gladstone or Redmond with Campbell Bannerman and Asquith. Butt founded the Irish Nationalist party; Parnell gained a liberal commitment to Home Rule and substantive Land reform. Redmond secured Home Rule but not its implementation whilst Dillon ended up in alliance with Sinn Fein. Redmond could be accused of neglecting grass roots support in a way that O’Connell and Parnell never did. However O’Connell was never trusted in England whereas Parnell and Redmond were more effective in gaining the trust of Gladstone and Asquith. All three were broken, O’Connell by imprisonment in 1843, Parnell by the O’Shea divorce case (which lost him much support) and Redmond by the Great War and the Easter Rising. All depended on the fortune of Whiggery and Liberalism and all ignored Ulster, Redmond at great cost. Most candidates will see either O’Connell or Parnell as the most effective leaders although all could be considered failures, or merely effective in different ways. Alternative explanations are possible and examiners must be open to alternative approaches. If in doubt, consult your Team Leader. [60]

8 To what extent were British governments committed to the maintenance of the Act of Union in the period from 1800 to 1921? [60]

No set conclusions are to be expected, but candidates must answer the question and address the theme over the whole period. One view of this is that all British governments of whatever political complexion were fully committed to the Act of Union, only Lloyd George and his closest colleagues in 1920-21 reluctantly abandoned this position in the face of Collins’ tactics in the Anglo-Irish War. However it could be argued that governments came to regard it as having created more problems than it solved and sought to ditch various aspects. Better candidates will consider Pitt’s original aims when framing the Act of Union in 1800. He was determined to stabilise Ireland after the Wolfe Tone Rising and was convinced that the Irish parliament was a destabilising influence and had to go. The incorporation of Irish MPs and Peers to Westminster was vital for him. Governments stuck to this until 1886 when

Page 308: History A - The Bicester Schoolthebicesterschool.org.uk/wp-content/uploads/2015/09/Mark... · 2015-09-28 · History A Mark Schemes for the Units January 2009 H106/H506/MS/R/09J

F966/02 Mark Scheme June 2010

10

Gladstone introduced a Home Rule Bill that restored the Grattan Parliament with no Irish MPs at Westminster. However this was reversed in the Second 1893 Bill which would have restored representation at Westminster alongside that in Ireland. This arrangement was retained in the Third Bill of 1912 and in the Government of Ireland Act in 1920 (a Northern Ireland Parliament with Northern Irish MPs at Westminster). It could be argued that Pitt’s commitment to the Irish at Westminster was maintained by all (except for Gladstone in 1886), despite most Irish MPs forming their own grouping, distinct from Liberalism and Conservatism, from the 1870s onwards. One could also point to the restrictive powers of the proposed Home Rule Parliaments after 1886 and even to the idea of Dominion status in 1921 for the Free State. Defence fears were as evident in the Anglo-Irish Treaty as in the Act of Union. The Lord Lieutenant and Chief Secretary, the former exercising viceroy like powers throughout the period, remained the key to Irish executive government. There was less commitment to the religious aspects of the Union. Pitt had wanted Catholic Emancipation to accompany it but Peel did not. The latter reluctantly conceded it in 1829, the Whig governments making it more of a reality through catholic appointments. Peel went on to seek an accommodation with Catholicism via Maynooth, whilst Gladstone disestablished the Irish Church in 1869.Certainly Ulster Unionists feared that Governments would concede power to a Catholic Assembly in Dublin after 1886 but a commitment to Ulster was maintained despite the fears. Economically Pitt had high hopes for the Union, looking to wider market opportunities to ‘raise’ the Irish in the manner of the Scots. Few governments, despite the failure of such hopes, changed the nature of the Union in this respect, beyond some land concessions. Gladstone was careful not to concede economic control to a Dublin parliament and in 1921 the Free State was allowed continued access to mainland markets.

Alternative explanations are possible and examiners must be open to alternative approaches. If in doubt, consult your Team Leader.

9 How important were economic factors in explaining the development of Irish nationalism in the period from 1798 to 1921? [60]

No set conclusions are to be expected, but candidates must answer the question and address the theme over the whole period. This is a wide ranging question and exhaustive coverage is not expected. A broad comparative brush is required. A case could be made that economic factors were very important. Nationalists stressed that Irish land had been looted by English conquest, her peasantry oppressed with high rents and controlled by ruthless eviction. The Famine was seen as an English induced holocaust, whilst Ireland provided the cheap emigrant labour to build an industrialised society on the mainland. However few made much of this point before 1845. Tone and O’Connell stressed political and religious emancipation. Their successors too made little of it. Parnell and Redmond continued to stress a political nationalism whilst the Fenians and the IRB took the route of nationalist terrorism. Only Davitt and belatedly Parnell exploited economic agrarian grievance via the Land League, the latter ditching it once he scented the political prospect of Home Rule. Connolly’s Socialism made little headway outside Dublin, although Ulster nationalism used Belfast industry as a sign of divine approval for Protestantism, especially as a knee jerk protective reaction after 1886. There were few economic issues to exploit in the late Victorian and Edwardian periods and neither Sinn Fein nor Redmond used them. A more convincing case might be made for the importance of political, religious and cultural factors throughout. Better candidates will see the linkages between these. Protestant nationalism felt betrayed by the Act of Union, whilst Catholic nationalism was exploited by O’Connell and later used by De Valera. Sectarian divides fuelled the various nationalisms, reflected in the Ulster and Irish Volunteers which foreshadowed Partition post 1918. Both revolutionary and constitutional nationalism drew heavily on the political tradition of the enlightenment, liberalism and independence. Their leaders took a largely political line and adopted political methods to advance their cause (from Risings to Parliamentary

Page 309: History A - The Bicester Schoolthebicesterschool.org.uk/wp-content/uploads/2015/09/Mark... · 2015-09-28 · History A Mark Schemes for the Units January 2009 H106/H506/MS/R/09J

F966/02 Mark Scheme June 2010

11

obstruction). A case could also be made for the cultural impact of the Gaelic revival from the 1880s, particularly for those who pushed for independence. Language, poetry, theatre and sport rejected the Anglo-Irish tradition of Ascendancy nationalism and owed little to economic issues. It provided a ‘new’ national tradition built on powerful myths, including economic exploitation. Ulster was busy doing the same.

Alternative explanations are possible and examiners must be open to alternative approaches. If in doubt, consult your team Leader.

Page 310: History A - The Bicester Schoolthebicesterschool.org.uk/wp-content/uploads/2015/09/Mark... · 2015-09-28 · History A Mark Schemes for the Units January 2009 H106/H506/MS/R/09J

F966/02 Mark Scheme June 2010

12

Theme 4: Russia and its Rulers 1855 – 1964 10 How far do you agree that the October Revolution of 1917 was the most important

turning point in the development of Russian government in the period from 1855 to 1964? [60] No set conclusions are to be expected, but candidates must answer the question and address the theme over the full period. Candidates should focus on the phrase ‘most important turning point’ and ‘the development of Russian government’ in their answers. Candidates may argue either for or against the communist takeover in 1917 as the most important turning point, but must do so comparatively in the context of other turning points. What follows is not an exclusive list, but consideration could be given to defeat in the Crimean War in 1856, the assassination of Alexander II in 1881, the 1905 Revolution, the February Revolution of 1917, Stalin’s gaining total power by 1929 or Stalin’s death in 1953 and replacement by Khrushchev by 1956. For example, candidates might argue that the assassination of Alexander II in 1881 marked the end of any hope of meaningful reform from above by the Romanov dynasty, and set the Romanovs on course for revolution and their downfall. Candidates may argue that February 1917 was the most significant turning point as it ended the 304 year old Romanov dynasty, but may argue that ultimately this led to the replacement of ‘Romanov Tsars’ by ‘red Tsars’. Many candidates will undoubtedly argue that October 1917 and the triumph of Bolshevism was the most important turning point as it crushed all possibility that a liberal democracy might emerge in Russia and transformed Russia into the Soviet Union – the world’s first communist state. Candidates however may well consider that Lenin’s death in 1924 was the most significant turning point, perverting the true course of the Russian Revolution because Stalin succeeded Lenin. Candidates who argue this are likely to suggest that Stalin’s victory in the ensuing power struggle led Russia down a very different road than that being paved by Lenin. Other candidates may use a counter-argument based on more recent archival evidence to suggest that there was significant continuity between Lenin and Stalin and argue this. Candidates may argue that Khrushchev’s secret speech of 1956 and subsequent de-stalinisation marked a significant turning point in the development of Russian government. Examiners must be open to alternative approaches. If in doubt, they should consult their Team Leader.

11 Assess the view that Russia’s communist leaders did less than the Tsars to improve the lives of the working class in the period from 1855 to 1964. [60] No set conclusions are to be expected, but candidates must answer the question and address the theme over the full period. Candidates should focus on the living and working conditions of Russia’s industrial and urban working class. Candidates should compare the experience of the working class under the tsars with their experience under the communists. Candidates may wish to compare the expectations the working class had from their ‘Little Fathers’ with their expectations under the Bolsheviks. Some candidates may compare and contrast Marxist ideology with the actual experience of life in the USSR. Candidates are likely to compare the impact of industrialization on the lives of the people, both before and after 1917. Similarities could include the grim experience of industrialization experienced by the proletariat, both as a consequence of Witte’s ‘Great Spurt’ and Stalin’s Five Year Plans. Candidates may wish to compare the scale of the suffering under Lenin and Stalin with that experienced before 1917. Candidates may argue that the working peoples gained benefits from Soviet rule, for example in the sphere of education. Candidates are likely to limit their evaluation of life for the proletariat under the Tsars from around 1890 as there were comparatively few urban workers prior to Witte’s ‘Great Spurt’. However, some candidates may deal with the whole period from 1855 as there were serfs working in factories prior to Emancipation in 1861. Candidates may treat Russia’s peasants as part of their discussions as the growing proletariat consisted largely

Page 311: History A - The Bicester Schoolthebicesterschool.org.uk/wp-content/uploads/2015/09/Mark... · 2015-09-28 · History A Mark Schemes for the Units January 2009 H106/H506/MS/R/09J

F966/02 Mark Scheme June 2010

13

of urbanised peasants, but the main focus should involve a consideration of the experience of Russia’s industrial working class or proletariat and candidates who fail to discuss the industrial and urban workers should not be put into Levels (i) or (ii) or (iii) . Examiners must be open to alternative approaches. If in doubt, they should consult their Team Leader.

12 ‘Opposition to Russian governments was ineffective in the period from 1855 to 1964.’ How far do you agree with this view? [60]

No set conclusions are to be expected, but candidates must answer the question and address the theme over the full period. Candidates should focus on the phrase ‘opposition to Russian governments was ineffective’ in their answers. Candidates may argue that for much of the period this view can be justified. In support they are likely to cite examples such as Stalinist terror, Lenin’s victory in the Civil War and crushing of the Kronstadt Uprising and the silencing of opposition under Alexander III. Candidates may also argue that the crushing of the 1905 Revolution illustrates this view. However many candidates may argue that opposition under the Tsars was increasingly effective. This can be supported in the reign of Alexander II by the emergence of a wide range of opposition groups such as the Narodniks and the Peoples’ Will. His assassination may be used to illustrate effective opposition (although some candidates may argue that the only consequence was his replacement by a far more effective autocrat in Alexander III). Candidates may view the reign of Alexander III as a temporary setback to opposition and see the 1905 Revolution as a dress rehearsal and narrow escape for Nicholas II. Candidates may argue that by 1917 opposition groups such as the SRs, the Mensheviks and the Cadets were increasingly effective. Candidates are very likely to support this by reference to the abdication of Nicholas II and consequent end to the Romanov dynasty. Candidates are very likely to argue that in 1917 the Bolsheviks emerged as an extremely effective opposition group and are likely to support this by reference to the roles of Lenin and Trotsky. Candidates may argue that under communism opposition was much less effective. They are likely to understand that Lenin’s banning of factions and Stalin’s terror led to a situation when opposition even within the communist party was perilous! Candidates may argue that ‘the Thaw’ under Khrushchev led to a situation where within the party opposition could flourish; they may well use Khrushchev’s enforced resignation to support this argument. Examiners must be open to alternative approaches. If in doubt, they should consult their Team Leader.

Page 312: History A - The Bicester Schoolthebicesterschool.org.uk/wp-content/uploads/2015/09/Mark... · 2015-09-28 · History A Mark Schemes for the Units January 2009 H106/H506/MS/R/09J

F966/02 Mark Scheme June 2010

14

Theme 5: Civil rights in the USA 1865 – 1992 13 Assess the importance of the federal government in the advancement of African

American civil rights in the period from 1865 to 1992.

No set conclusions are to be expected, but candidates must answer the question and address the theme over the whole period. Most candidates will accept that the Federal government was crucial to the advancement of African American civil rights and observe that the 1950s and 1960s witnessed the most significant period of progress for the civil rights of African Americans with the passage by Congress of the Civil Rights Act of 1964 and the Voting Rights Act of 1965, both supported by the president, and that these advances occurred while the Supreme Court was also supportive (for example, in the Brown Case in 1954 and the Browder versus Gayle verdict in 1956 on the Montgomery bus boycott). They will possibly also refer to the period of Reconstruction (1865-1877) as another period in which Congress enacted significant advances (the 13th, 14th and 15th Amendments and the Civil Rights Act of 1866), with better candidates pointing out that these were not sustained because President Johnson and the Supreme Court were hostile. They might also contrast these periods with 1877 to 1941 when, without Federal government support, civil rights made little progress and the period from 1970 onwards when Republican presidents, Nixon, Ford, Reagan and Bush, gave at best, lukewarm support, and sometimes not even that. They will probably be aware of the changed attitude of the Supreme Court, contrasting the Plessy versus Ferguson verdict of 1896 with the Brown Case of 1954. Probably only the best candidates will discuss the implications of either the 1978 Bakke Case for affirmative action or the more conservative tone of the Rehnquist court after 1986. Better candidates will attempt to evaluate the importance of the Federal government in relation to other factors. They will be aware that, even when sympathetic to civil rights, the president and Congress were reluctant to act unless put under pressure by activists and protestors. They will point out that civil rights could be seen as making progress ahead of Federal government action because grass roots protest (and, in the post war period, a more sympathetic climate) were essential prerequisites to legislative change. They will also be aware that the huge economic and social changes brought about by the Depression and the Second World War undermined the ability of the Southern states to resist change. Examiners must be open to alternative approaches. If in doubt, they should consult their Team Leader.

14 ‘The actions of Native Americans themselves contributed nothing to the

advancement of their civil rights in the period from 1865 to 1992.’ To what extent do you agree with this view?

No set conclusions are to be expected, but candidates must answer the question and address the theme over the whole period.

Most candidates are likely to agree that the majority of Native Americans did little towards the attainment of their civil rights. They will argue that the Federal government did much more than Native Americans themselves to advance their civil rights. The weakest candidates are likely to resort to narrative, chronological exposition of Federal policy beginning with the period of forced assimilation that began with the 1887 Dawes Act. They will regard Collier and the Indian New Deal as a brief period of enlightened Federal policy which was followed by a return to forced assimilation under the policy of termination. Most candidates will be familiar with the high-profile activities of ‘Red Power’ in the late 1960s and early 1970s (the occupation of Alcatraz in 1969, the occupation of the BIA in 1972, and the protest at Wounded Knee in 1973). They may conclude that this Indian activism (together with the impact of the African American civil rights campaigns) resulted in the

Page 313: History A - The Bicester Schoolthebicesterschool.org.uk/wp-content/uploads/2015/09/Mark... · 2015-09-28 · History A Mark Schemes for the Units January 2009 H106/H506/MS/R/09J

F966/02 Mark Scheme June 2010

15

reluctant redress of some Native American grievances, though some candidates will argue that these actions did no more than raise the profile of Indians. Good candidates might consider the Indian Wars of the late 19th century as evidence of Native Americans attempting, though ultimately unsuccessfully, to defend their rights. High ability candidates will show the importance of this struggle in creating sympathy for Native Americans some 80 years later. Good candidates will also be aware that the significant contribution of Native Americans to the US war effort in the two world wars influenced subsequent Federal policy (the granting of US citizenship in 1924 and the formation of the Indian Claims Commission in 1946). The best candidates will be aware of the activities of the various Indian groups, the Society of American Indians (SAI, established in 1911), the National Congress of American Indians (NCAI, established in 1944) and the American Indian Movement (AIM, established in 1968) to preserve Indian customs and culture and exert pressure on Federal and state authorities. High ability candidates will also be able to evaluate the impact of these groups and show that, though they neither exercised much political power nor mobilised mass protest, their activities both influenced, and were encouraged by, sympathetic Federal officials (John Collier in the 1930s and 1940s) and policies (the Indian Claims Commission). They will also be able to show that Native Americans were able to exercise a significant, though limited, negative impact on Federal policy: both the Indian New Deal and the termination policy of the 1950s were abandoned in part because of Indian hostility. High ability candidates might also point out the lack of clarity about what Native American civil rights actually are. They will be aware of the impact of economic and social changes of the mid to late 20th century on Native Americans. Some wanted recognition of their separate culture and traditions while others, particularly those who have moved to the cities and/or intermarried, wished to assimilate into mainstream US culture and overcome de facto racial prejudice over such issues as employment and housing. Such candidates might also analyse the lack of consistency in Federal Indian policy and will be able to discuss the current legal and economic status of the reservations. Examiners must be open to alternative approaches. If in doubt, they should consult their Team Leader.

15 How far do you agree that divisions within the women’s movement were the most

important factor hindering the attainment of gender equality in the USA in the period from 1865 to 1992?

No set conclusions are to be expected, but candidates must answer the question and address the theme over the whole period.

Most candidates will be aware of the different, and sometimes competing, women’s organisations. They will refer to the different strategies adopted by the NWSA and the AWSA (both founded in 1869) and to the founding in 1913 of the militant Congressional Union. However, they may balance this by pointing out that the NWSA and the AWSA merged (as the NAWSA) in 1890 and may attribute the passage of the 19th Amendment in 1920 (which granted women the vote nationally) to the campaigning of the NAWSA and the Congressional Union. They might characterise the women’s suffrage movement (as well as the temperance campaigners and those agitating for Progressive Era social and economic reform) as essentially middle class and they should also be aware of divisions among women over race. Black women were excluded from the NAWSA (and so formed their own campaign group) and Elizabeth Stanton opposed allowing blacks to vote. White women were heavily represented in the membership of the revived KKK in the 1920s. Some women opposed the post-war civil rights movement for its failure to campaign for women’s issues. Most candidates should be aware of the divisions over abortion (Roe versus Wade, 1973) and the role of Phyllis Schlafly in opposing the Equal Rights Amendment.

Page 314: History A - The Bicester Schoolthebicesterschool.org.uk/wp-content/uploads/2015/09/Mark... · 2015-09-28 · History A Mark Schemes for the Units January 2009 H106/H506/MS/R/09J

F966/02 Mark Scheme June 2010

16

Weak candidates will probably not get much further than outlining the divisions within the women’s movement, but better candidates should be able to weigh up the importance of these divisions in relation to other factors. They could refer to the impact of economic and technological change (the development of factory work, the typewriter, the expansion of retail trade in the late 19th century and the development of domestic appliances in the 20th century) in changing women’s lives and prospects. They could also refer to the power of male opposition to gender equality, initially in resisting female suffrage, but also in trade unions. The ablest candidates may also be aware that some advances in gender equality had little to do with women’s agency. The granting of female suffrage in certain western states (eg Colorado in 1893, Idaho in 1896, Washington in 1910 and California in 1911) had more to do with encouraging westward migration than enlightened attitudes to women’s roles in society. Similarly, after the Second World War, some Southerners advocated women’s rights as a tactic to divide, and thus defeat, the civil rights campaign. High level answers might also discuss the nature of women’s rights and point out the lack of unity about what gender equality means in practice. Examiners must be open to alternative approaches. If in doubt, they should consult their Team Leader.

Page 315: History A - The Bicester Schoolthebicesterschool.org.uk/wp-content/uploads/2015/09/Mark... · 2015-09-28 · History A Mark Schemes for the Units January 2009 H106/H506/MS/R/09J

F966/02 Mark Scheme June 2010

17

Theme 6: The Development of Democracy in Britain 1868 – 1997 16 How far was the extension to the franchise in 1918 the most important factor in the

development of democracy from 1868 to 1997?

No set conclusions are to be expected, but candidates must answer the question and address the theme over the whole period.

Candidates could make a strong case for the Representation of the People’s Act of 1918 being the most important factor. Not only did women get the vote for the first time (over 8.4 million over the age of 30), but the total electorate was increased to over 21 million (all men over the age of 21 were given the vote). However, many responses are likely to indicate the limited nature of this change, in so far as many women were debarred as a result of the age limit. It might also be argued that the legislation was a logical (and inevitable?) development from franchise reforms that had occurred in the nineteenth century. Moreover, the weaknesses of 1918 were partly addressed by the Equal Franchise Act of 1928, when all women over 21 were given the vote, and the Representation of the People Act of 1969, which reduced the voting age of all to 18. Therefore, 1918 was part of a chain of events that led to a fairer and more equal democratic system.

Some candidates will make a case for other factors constituting major influences. These might include the Ballot Act (1872), the Corrupt and Illegal Practices Act (1883), the Redistribution of Seats Act (1885) and the Parliament Act (1911). Others might consider events of a more general nature, such as the formation of the Labour party, the emergence of general unions, the rise of the mass media and the influence of the Suffragettes. Such an approach is unlikely to be as successful as that which focuses on a particular reform or event, as it would be more difficult to measure the degree of change that occurred. Examiners must be open to alternative approaches. If in doubt, they should consult their Team Leader.

17 Assess why the Conservatives were more successful than any other party in staying

in power from 1868 to 1997.

No set conclusions are to be expected, but candidates must answer the question and address the theme over the whole period.

Candidates may argue that the answer relates primarily to ideological differences. The Conservatives consistently promoted law and order, the Church of England, paternalism, protectionism, property rights and nationalism, all of which had a general appeal to an ever increasing electorate. This seemed to marry well with a deep seated conservatism that remained part of the British psyche and culture throughout the period.

With the Liberals there seemed to be less ideological consistency and congruence, with a shift from so called ‘old liberalism’ to ‘new liberalism’ by the end of the nineteenth century. This was partly in response to an attempt to capture the votes of the newly enfranchised working classes. However, the emergence of the Labour party seemed to damage Liberal aspirations considerably, whilst the Conservatives maintained a steady level of appeal. However, Labour also went through major changes with respect to ideology and policy especially under Blair. This, of course, helped Labour achieve huge political success in the latter part of the period

Other factors should also be discussed, such as leadership differences, party organisation and unity, and contingency factors (eg wars, economic conditions, social changes). Better candidates will take heed of the command stem and ensure that they make a judgement

Page 316: History A - The Bicester Schoolthebicesterschool.org.uk/wp-content/uploads/2015/09/Mark... · 2015-09-28 · History A Mark Schemes for the Units January 2009 H106/H506/MS/R/09J

F966/02 Mark Scheme June 2010

18

about the relative importance of reasons for the differing fortunes of the main political parties. Examiners must be open to alternative approaches. If in doubt, they should consult their Team Leader.

18 ‘Trade Unions held back the development of democracy from 1868 to 1997.’ How far

do you agree?

No set conclusions are to be expected, but candidates must answer the question and address the theme over the whole period.

A case could be made for the role Trades Unions played in hindering attempts to increase freedom of choice (if that is how democracy is to be defined). There is likely to be a focus on the more frequent use of strikes and on a bigger scale, which resulted in restraint of trade. Examples to illustrate this could be taken from across the period and might include the late 1880s, 1912, 1926, 1972-74 and 1984-85. For many sectors of the population, restraint of trade meant restricted consumer choice, inconvenience and unnecessary hardship. There was also the more specific issue of the closed shop, which until the end of the period, could be seen as anti-democratic by restricting worker choice of who should best represent their interests in the work place. Many responses are likely to suggest that the existence of unions, particularly those that were ‘responsible’, enhanced the democratic process. They acted as successful pressure groups and a check on the power of central governments and their relationship with the Labour party aided more effective, appropriate and widespread representation of the interests of working people. Furthermore, they had a positive impact on economic and social policies (especially in the areas of health and education). The best responses should provide balance. Some candidates may comment on the Whiggish tone of the question and how it implies that democracy triumphed despite the odd obstruction. Examiners must be open to alternative approaches. If in doubt, they should consult their Team Leader.

Page 317: History A - The Bicester Schoolthebicesterschool.org.uk/wp-content/uploads/2015/09/Mark... · 2015-09-28 · History A Mark Schemes for the Units January 2009 H106/H506/MS/R/09J

Oxford Cambridge and RSA Examinations is a Company Limited by Guarantee Registered in England Registered Office; 1 Hills Road, Cambridge, CB1 2EU Registered Company Number: 3484466 OCR is an exempt Charity OCR (Oxford Cambridge and RSA Examinations) Head office Telephone: 01223 552552 Facsimile: 01223 552553 © OCR 2010

OCR (Oxford Cambridge and RSA Examinations) 1 Hills Road Cambridge CB1 2EU OCR Customer Contact Centre 14 – 19 Qualifications (General) Telephone: 01223 553998 Facsimile: 01223 552627 Email: [email protected] www.ocr.org.uk For staff training purposes and as part of our quality assurance programme your call may be recorded or monitored

Page 318: History A - The Bicester Schoolthebicesterschool.org.uk/wp-content/uploads/2015/09/Mark... · 2015-09-28 · History A Mark Schemes for the Units January 2009 H106/H506/MS/R/09J

Oxford Cambridge and RSA Examinations

GCE

History A

Unit F966/02: Historical Themes Option B: Modern 1789-1997

Advanced GCE

Mark Scheme for January 2011

Page 319: History A - The Bicester Schoolthebicesterschool.org.uk/wp-content/uploads/2015/09/Mark... · 2015-09-28 · History A Mark Schemes for the Units January 2009 H106/H506/MS/R/09J

OCR (Oxford Cambridge and RSA) is a leading UK awarding body, providing a wide range of qualifications to meet the needs of pupils of all ages and abilities. OCR qualifications include AS/A Levels, Diplomas, GCSEs, OCR Nationals, Functional Skills, Key Skills, Entry Level qualifications, NVQs and vocational qualifications in areas such as IT, business, languages, teaching/training, administration and secretarial skills. It is also responsible for developing new specifications to meet national requirements and the needs of students and teachers. OCR is a not-for-profit organisation; any surplus made is invested back into the establishment to help towards the development of qualifications and support which keep pace with the changing needs of today’s society. This mark scheme is published as an aid to teachers and students, to indicate the requirements of the examination. It shows the basis on which marks were awarded by Examiners. It does not indicate the details of the discussions which took place at an Examiners’ meeting before marking commenced. All Examiners are instructed that alternative correct answers and unexpected approaches in candidates’ scripts must be given marks that fairly reflect the relevant knowledge and skills demonstrated. Mark schemes should be read in conjunction with the published question papers and the Report on the Examination. OCR will not enter into any discussion or correspondence in connection with this mark scheme. © OCR 2011 Any enquiries about publications should be addressed to: OCR Publications PO Box 5050 Annesley NOTTINGHAM NG15 0DL Telephone: 0870 770 6622 Facsimile: 01223 552610 E-mail: [email protected]

Page 320: History A - The Bicester Schoolthebicesterschool.org.uk/wp-content/uploads/2015/09/Mark... · 2015-09-28 · History A Mark Schemes for the Units January 2009 H106/H506/MS/R/09J

F966/02 Mark Scheme January 2011

Maximum mark 120 for this unit. 2 answers: Each maximum mark 60

A01a A01b IA 18-20 36-40

IB 16-17 32-35

II 14-15 28-31

III 12-13 24-27

IV 10-11 20-23

V 8-9 16-19

VI 4-7 8-15

VII 0-3 0-7

Notes:

(i) Allocate marks to the most appropriate level for each AO.

(ii) If several marks are available in a box, work from the top mark down until the best fit has been found.

(iii) Many answers will not fall at the same level for each AO.

(iv) Candidates will demonstrate synoptic skills by drawing together appropriate techniques, knowledge and understanding to evaluate developments over the whole of the period

1

Page 321: History A - The Bicester Schoolthebicesterschool.org.uk/wp-content/uploads/2015/09/Mark... · 2015-09-28 · History A Mark Schemes for the Units January 2009 H106/H506/MS/R/09J

F966/02 Mark Scheme January 2011

AOs AO1a AO1b Total mark for each question = 60

Recall, select and deploy historical knowledge appropriately, and communicate knowledge and understanding of history in a clear and effective manner.

Demonstrate understanding of the past through explanation, analysis and arriving at substantiated judgements of: - key concepts such as causation, consequence, continuity, change and significance within an historical context; - the relationships between key features and characteristics of the periods studied

Level IA

Uses a wide range of accurate and relevant evidence

Accurate and confident use of appropriate historical terminology

Answer is clearly structured and coherent; communicates accurately and legibly.

18-20

Excellent understanding of key concepts (eg continuity and change) relevant to analysis in their historical context

Excellent synthesis and synoptic assessment

Answer is consistently and relevantly analytical with developed explanations and supported judgements

May make unexpected but substantiated connections over the whole period

36-40

Level IB

Uses accurate and relevant evidence Accurate use of a range of

appropriate historical terminology Answer is clearly structured and

mostly coherent; communicates accurately and legibly

16-17

Very good level of understanding of key concepts (eg continuity and change) in their historical context.

Answer is consistently focused on the question set

Very good level of explanation/ analysis, and provides supported judgements.

Very good synthesis and synoptic assessment of the whole period

32-35

Level II

Uses mostly accurate and relevant evidence

Generally accurate use of historical terminology

Answer is structured and mostly coherent; writing is legible and communication is generally clear

14-15

Good level of understanding of key concepts (eg continuity and change) in their historical context

Good explanation/ analysis but overall judgements may be uneven

Answer is focused on the issues in the question set

Good synthesis and assessment of developments over most of the period

28-31

2

Page 322: History A - The Bicester Schoolthebicesterschool.org.uk/wp-content/uploads/2015/09/Mark... · 2015-09-28 · History A Mark Schemes for the Units January 2009 H106/H506/MS/R/09J

F966/02 Mark Scheme January 2011

AOs AO1a AO1b Level III

Uses relevant evidence but there may be some inaccuracy

Answer includes relevant historical terminology but this may not be extensive or always accurately used

Most of the answer is structured and coherent; writing is legible and communication is generally clear

12-13

Shows a sound understanding of key concepts, especially continuity and change, in their historical context

Most of the answer is focused on the question set

Answers may be a mixture of analysis and explanation but also description and narrative, but there may also be some uneven overall judgements; OR answers may provide more consistent analysis but the quality will be uneven and its support often general or thin

Answer assesses relevant factors but provides only a limited synthesis of developments over most of the period

24-27 Level IV

There is deployment of relevant

knowledge but level/ accuracy will vary.

Some unclear and/or underdeveloped and/or disorganised sections

Mostly satisfactory level of communication

10-11

Satisfactory understanding of key concepts (eg continuity and change) in their historical context

Satisfactory focus on the question set Answer may be largely

descriptive/narratives of events, and links between this and analytical comments will typically be weak or unexplained

Makes limited synoptic judgements about developments over only part of the period

20-23 Level V

General and basic historical

knowledge but also some irrelevant and inaccurate material

Often unclear and disorganised sections

Adequate level of communication but some weak prose passages

8-9

General understanding of key concepts (eg continuity and change) in their historical context

Some understanding of the question but answers may focus on the topic and not address the question set OR provides an answer based on generalisation

Attempts an explanation but often general coupled with assertion, description/narrative

Very little synthesis or analysis and only part(s) of the period will be covered

16-19

3

Page 323: History A - The Bicester Schoolthebicesterschool.org.uk/wp-content/uploads/2015/09/Mark... · 2015-09-28 · History A Mark Schemes for the Units January 2009 H106/H506/MS/R/09J

F966/02 Mark Scheme January 2011

AOs AO1a AO1b Level VI Use of relevant evidence will be

limited; there will be much irrelevance and inaccuracy

Answers may have little organisation or structure

Weak use of English and poor organisation

4-7

Very little understanding of key concepts (eg continuity and change) in their historical context

Limited perhaps brief explanation Mainly assertion,

description/narrative Some understanding of the topic but

not the question’s requirements

8-15 Level VII Little relevant or accurate Knowledge

Very fragmentary and disorganised response

Very poor use of English and some incoherence

0-3

Weak understanding of key concepts (eg continuity and change) in their historical context

No explanation Assertion, description/narrative

predominate Weak understanding of the topic or of

the question’s requirements 0-7

4

Page 324: History A - The Bicester Schoolthebicesterschool.org.uk/wp-content/uploads/2015/09/Mark... · 2015-09-28 · History A Mark Schemes for the Units January 2009 H106/H506/MS/R/09J

F966/02 Mark Scheme January 2011

1 How far would you agree that Prussian ambitions undermined the aims of German nationalists in the period from 1789 to 1919? [60]

Candidates should focus on ‘the aims of German nationalists’ and evaluate whether these were either undermined or furthered by ‘Prussian ambitions’ in their answers. Candidates should compare the various aims and ideas of German nationalism with the developing impact that Prussia had on Germany, for example Prussia’s role in the defeat of Napoleon. Candidates may consider the impact that the Prussian Zollverein had on the development of nationalism. Candidates may discuss the role played by Prussia in the 1860s in forging the new Germany. Candidates may discuss Bismarck’s views about German liberalism and the impact of the 1871 Constitution. Candidates may demonstrate that they understand that the German Empire in 1871 represented Kleindeutschland and an enlarged Prussia rather than a united Germany. They may argue that it was a Prussian Empire rather than a German Empire. However not all German nationalists aspired to Grossdeutschland and it can be argued that the creation of the Second Reich was a crucial step forward for the aims of many German nationalists. Candidates could point to the mythical status of Bismarck in German history and/or to the popularity and mass appeal of increasingly radical nationalism in the reign of William II to argue that Prussia’s creation of the Second Reich was consistent with the aims of many German nationalists. Candidates might argue that the aims and ambitions of Prussian militarism were to ultimately set the German nation on course for disaster and humiliation by reference to Germany’s defeat in the First World War and her humiliation at Versailles. Candidates may well discuss other factors that undermined the aims of German nationalists, for example their own divisions. The 1848 Revolution may be seen as a pivotal moment by such candidates.

Examiners must be open to alternative approaches. If in doubt, they should consult their Team Leader.

2 Assess the view that the effects of industrialisation hindered the development of German nationalism in the period from 1789 to 1919. [60]

Candidates should focus on ‘the development of German nationalism’ and whether ‘the effects of industrialisation’ ‘hindered’ or encouraged it in their answers. Candidates should explain the impact of industrialisation on the development of German nationalism, for example the impact of the Zollverein after 1834 in developing Prussia’s economic strength and, consequently, Prussian leadership of Germany. This also had a limiting effect on the development of German nationalism as Prussia was able to exclude Austria, first from the Zollverein and then from Germany. This led to the creation of Kleindeutschland, thus thwarting the ambitions of those nationalists who aspired to Grossdeutschland. Candidates should understand how developments in the economy in the 1850s paved the way for the Prussian military victories of 1864, 1866 and 1870/71 and the creation of the Second Reich. Military strength depended upon industrialisation; ‘Coal & Iron’ rather than ‘Blood & Iron’ could be usefully debated. The impact of the extraordinary developments in the German economy after 1871 should be discussed. Candidates may well argue that the quickening pace of industrialisation led to urbanisation and the development of increasingly radical socialism. Socialism gave the working class an alternative loyalty to patriotism and nationalism. This in turn led Kaiser Wilhelm II to embark on a foreign policy aimed at distracting the workers from their grievances. It could be argued that this populist foreign policy fanned the flames of German nationalism.

Examiners must be open to alternative approaches. If in doubt, they should consult their Team Leader.

5

Page 325: History A - The Bicester Schoolthebicesterschool.org.uk/wp-content/uploads/2015/09/Mark... · 2015-09-28 · History A Mark Schemes for the Units January 2009 H106/H506/MS/R/09J

F966/02 Mark Scheme January 2011

3 Assess the importance of German nationalism in the creation and development of a united Germany in the period from 1789 to 1919. [60]

Candidates should focus on the ‘importance of German nationalism’ in the ‘creation and development’ of a ‘united Germany’. Candidates should show that they understand that German nationalism was not the sole factor determining the creation and development of the German Empire in this period. The ‘importance of German nationalism’ should be evaluated against the other dominant factors in the creation and development of the German Empire during this period. Candidates may argue that the emergence of romantic nationalism, during and after the Napoleonic period, had an impact on the creation and development of Germany. The development and impact of ideas on the emergence of nationalism may usefully be explored as may the impact of these ideas on the creation and development of the German Empire. Candidates may argue that the divisions within the nationalist movement weakened the impact of nationalism in the creation of a united Germany. Candidates should be able to explain the importance of economic factors on the creation of a united Germany, for example the impact of the Zollverein after 1834 in developing Prussia’s economic strength and on Prussian leadership of Germany. The leadership role played by Bismarck in determining the shape, extent and development of the German Empire may usefully be discussed. Candidates could argue that Bismarck’s achievements owed little or nothing to nationalism and that therefore the contribution of nationalism to the creation of the Empire was minimal. The impact of Kaiser Wilhelm II from 1888 was arguably as significant on the further development and fortunes of the Empire.

Examiners must be open to alternative approaches. If in doubt, they should consult their Team Leader.

6

Page 326: History A - The Bicester Schoolthebicesterschool.org.uk/wp-content/uploads/2015/09/Mark... · 2015-09-28 · History A Mark Schemes for the Units January 2009 H106/H506/MS/R/09J

F966/02 Mark Scheme January 2011

4 ‘A strong alliance was the most important reason for a country to succeed in war.’ To what extent do you agree with this view of the period from 1792 to 1945? [60]

There are two key concepts to be evaluated here, firstly the importance of alliances in warfare; secondly, if the significance of the role of alliances changed across the period.

Strong alliances tended to have more importance at a time when Europe was in a state of general warfare and conflicts took place over a longer period of time. These examples will allow candidates to chart success/failure in war and link that to the state of given alliances. In the Revolutionary and Napoleonic periods coalitions of European powers were vital to containing and eventually defeating France. A key point would have been the need to form strong alliances when faced with the demographic and economic power of France. Candidates may also argue that France forged alliances of sorts with states in the wake of military victories. Examples would be Bavaria and other German minor states, the alliances with Austria and Prussia between the middle part of the Napoleonic Wars and the Russian campaign of 1812. The First World War is tailor-made for the question as it entailed two strong alliances fighting with each other. In the case of the Central Powers the relationship of a dominant partner – Germany – with its allies might be a worthwhile area for discussion. The Second World War saw a large powerful alliance of Britain, the USA and the USSR successfully defeating a far weaker alliance, Germany – again very much the dominant partner – Italy, Romania and other Axis minor allies. Candidates could discuss the formation of the Western/Soviet alliance over time and relate this to varying success on the battlefield.

In other periods strong alliances were less important and alliances tended to be used to diplomatically isolate powers rather than create powerful military and economic blocs. Good examples are the Wars of Unification and the Crimean War. The American Civil War might also be used in this context, candidates arguing that the Confederacy could only win with a strong alliance with a third power, an event that did not take place and/or that the Union was capable of winning the war without an ally. Other examples of countries winning without the aid of a strong alliance might be Austria against Piedmont in 1848 & 1849, Japan defeating Russia in 1904-5 and the many examples that come from the colonial conflicts of the period, especially where the colonial power was defeated by indigenous forces. Some candidates may place a greater emphasis on factors other than alliances eg the quality of leadership, strategy and tactics, economic resources and technological developments. Better responses should therefore set the impact of ‘strong alliances’ against a range of factors.

Examiners must be open to alternative approaches. If in doubt, consult your Team Leader.

7

Page 327: History A - The Bicester Schoolthebicesterschool.org.uk/wp-content/uploads/2015/09/Mark... · 2015-09-28 · History A Mark Schemes for the Units January 2009 H106/H506/MS/R/09J

F966/02 Mark Scheme January 2011

5 ‘Battlefield tactics remained unchanged.’ How far do you agree with this view of the period from 1792 to 1945?

The question invites candidates to discuss change and continuity in battle tactics. Candidates must be aware that the question is specifically about tactics. Some leeway, however, must be allowed as the period goes on and the division between strategy and tactics starts to blur, a good example would be a large WWI battle like the Somme. There are two clear lines of debate that might be established. The first would be to challenge the question. Given the developments in military organisation, command, control and technology across the period battlefield tactics obviously changed. A simple example would be to compare a WWII to a Napoleonic battlefield. Such an essay might then link synoptically the factors that caused such changes to take place to the line of debate.

Alternatively, the response might support the proposition in the question pointing to consistency in certain principles of warfare. Examples might be drawn from the military theorists in the specification, economy of force, maintenance of aim, concentration of force, etc. These concepts could then be linked to specific examples such as the Crimean War, the Wars of Unification or WWI where it is possible to argue that linear tactics were used throughout this period, any differences in the nature of battle being superficial.

Examiners must be open to alternative approaches. If in doubt, consult your Team Leader.

6 To what extent did public opinion influence the conduct of war in the period from

1792 to 1945?

It is expected that candidates will discuss the conduct of war in the light of the pressures of domestic public opinion. Definitions of public opinion can be expected and examiners need to be aware that political pressure on elites can come in many forms. Candidates might link developments in state structures – for example, the development of more democratic forms of government – and/or more effective forms of media that informed public opinion as the period developed. Arguments that public opinion did influence military decisions might include the early part of the Revolutionary Wars where the French Republic was fighting perhaps an ideological war for its existence. Candidates might point to the concept of the ‘nation in arms’ or the execution of generals for political reasons. It might be argued that the Ancien Regime powers fought for the same reasons, ie to protect their own political establishment from potential opposition from below. Napoleon might be used both ways, for example the need for military victory and its links to political stability. The Crimean War is a good example where military decisions were influenced by public opinion placing pressure on military decisions via the political demands on the French and British governments. Popular reaction within Russia to this conflict might also be discussed. With regard to the Wars of Unification, an example of the impact of public opinion might be the entry of France into the Italian War of 1859. The impact of rising nationalism on all of the wars of this period provides many obvious examples for candidates to use in support of analysis. Candidates might link the outcome of the Russo-Japanese war to the 1905 Revolution. Colonial conflicts in the latter part of the period, for example the Boer war, are candidates for discussion. Both WWI and II have a lot of potential with regard to the question with discussions of propaganda, different forms of media and censorship. Conversely, candidates might argue that military decisions were made with no regard to public opinion in autocratic states or that factors other than public opinion were of greater importance eg the quality of leadership, strategy and tactics, economic resources and technological developments. Better responses should therefore argue for and against the influence of public opinion upon the conduct of war.

Examiners must be open to alternative approaches. If in doubt, consult your Team Leader.

8

Page 328: History A - The Bicester Schoolthebicesterschool.org.uk/wp-content/uploads/2015/09/Mark... · 2015-09-28 · History A Mark Schemes for the Units January 2009 H106/H506/MS/R/09J

F966/02 Mark Scheme January 2011

7 ‘Irish cultural nationalism was the most important factor in undermining the links with Britain.’ How far do you agree with this view of the period from 1798 to 1921?

[60]

Candidates will need to be able to define cultural nationalism as a changing phenomenon and, whilst comparing with other factors that undermined links and the Union, will need to keep a reasonable focus on the issue of cultural nationalism. It is possible to argue that it was vital in undermining the links with Britain. In the late 18th century it was arguably important in providing the impetus both for the Parliamentarianism and Home Rule of Grattan’s Patriot followers amongst the literate and educated Protestant ascendancy and the revolutionary nationalists like Wolfe Tone whose culture was that of the rights of man and the liberty and equality of the French Revolution. Candidates could usefully examine the rising of 1798 in this context. Such enlightened ideas also informed the campaigns of O’Connell for both Emancipation and Home Rule. The Romantic view of the ‘nation’ became an important force in the 1840s for Young Ireland but it was the failure of Parnell and Home Rule in the 1880s that led to the emergence of a more distinct sense of an Irish cultural identity. This owed less to Protestantism, Catholicism or the rights of man than to other aspects of Irishness, albeit a romanticised one of ancient identity distinct from Englishness, increasingly seen as alien in every respect (Church, property laws, culture etc) and above all to language (the Irish language was fast dying). The Gaelic League (1893) waged war on ‘complete Anglicization’ from games to literary traditions for the rest of the period and was to find expression in the declaration of the republic in 1916 which, it has been argued, had its origins in the Gaelic revival, and Pearse’s School to raise the young in an Irish spirit. It was a bridge into the IRB, revived in the early 1900s and, via Griffith, into Sinn Fein. By the 1910s therefore it could be argued that Ireland had created its own history, language and culture in opposition to the mainland and that political events from 1916 played into this ‘myth’ and broke the bonds with a state that used illegitimate repression. However candidates could question its importance by stressing its minority appeal, particularly obvious in the earlier periods when any mass support was economically, religiously and socially rather than culturally driven and the emphasis was firmly political and religious. Even from the 1880s, after half a century of basic education in English, it remained largely middle and lower middle class in appeal, and would only gradually have a wider impact (imposed by the later Republic) and never in Ulster. It specifically renounced politics as a mere response to the English ‘game’. The Irish literary movement of Yeats used English. Amongst revolutionary nationalists there was only a very small minority who were active. The IRB was to be as much inspired by the Fenianism of Tom Clark as by any new cultural Irishness. Nationalists were politically inspired and 1916 had little to do with it. Events like the diaspora following the Famine probably had more impact on undermining links than home grown cultural nationalism. Catholicism, Disestablishment, the faltering of the Protestant ascendancy, local government reform in 1898, the First World War (1916 and conscription), the Land revolution, government mistakes and the failure of repression after 1916 were probably of much greater importance than cultural nationalism in undermining the Union. Examiners must be open to alternative approaches. If in doubt, they should consult their team Leader.

9

Page 329: History A - The Bicester Schoolthebicesterschool.org.uk/wp-content/uploads/2015/09/Mark... · 2015-09-28 · History A Mark Schemes for the Units January 2009 H106/H506/MS/R/09J

F966/02 Mark Scheme January 2011

8 ‘Tory and Conservative governments were the most successful in maintaining a stable Union with Ireland.’ How far do you agree with this view of the period from 1800-1921? [60]

Candidates could usefully point to the fact that all governments desired a stable Union and tended to implement changes in reaction to opposition and to recreate equilibrium. The case for Tory and later Conservative success lies with first Pitt’s creation of the Union itself (which survived until 1921), born out of the reaction to rebellion, followed by relative stability in the Lord Liverpool period and then the handling by Peel of Catholic Emancipation (its concession with safeguards), the Famine and his proposed reforms to defuse O’Connell’s Repeal Association’s campaign in the 1840s. This was combined with frequent recourse to coercion and attempts to restrict the activities of O’Connell, unsuccessfully in the 1820s, successfully in the early 1840s (arrest and trial). Tories also had frequent recourse to military repression (in 1798 and afterwards, notably ‘Bloody Balfour in the later 1880s). Later in the century the Conservatives made considerable economic concessions, building on Peel’s land proposals, via the Balfour and Wyndham Land Purchase Acts after 1886 and the Congested Boards Act which stabilised Ireland and oversaw huge changes in land ownership in the face of O’Brien’s United Irish League western campaign. It could be argued that this went much further than just an attempt to kill Home Rule through kindness. There were also political attempts at stabilisation, from the viceroyalty of Marlborough under Disraeli (genuinely popular in contrast to his predecessors?) to extensive local government reform in the 1890s, although the latter was to see the nationalists entrench themselves in the new county councils. These have generally been regarded as successful in that Ireland appeared stable in the post 1886 period, despite O’Brien and nationalist developments, the latter imbued with perhaps greater significance than was due given later developments. Candidates may be more equivocal on the 1916-21 period (Conservative dominated Coalition governments). They could be blamed for an over-reaction post 1916, for disastrous decisions on conscription and the use of the Black and Tans but equally measures to attempt stabilisation such as the Home Rule for both North and South in the Government of Ireland Act in 1920 and the Anglo Irish Treaty of 1921 which stabilised the North if not the South (Civil War). Alternatively candidates could argue that the Whig/Liberals were more successful, having advocated Catholic emancipation, preferring concession to coercion (the 1830s governments moving towards more patronage sharing and municipal government reform), and to the years of relative stability under Palmerston in the mid century. The Gladstone period could be seen in both lights – a successful stabilisation via Land Reform, Disestablishment and the attempt at Home Rule or a mishandling of Parnell and his party, a resort to coercion in 1881-3, inappropriate Land reforms and repeated failure over Home Rule. However constitutional nationalists found it easier to deal and cooperate with the Whig Liberals than the Conservatives and it could be argued that this aided stability and integration within the Union. The 3rd Home Rule crisis is a good example of a Liberal lack of success in achieving stability. Some candidates could conclude that ‘success’ was occasioned less by the political complexion of governments than by developments in Ireland. Both reacted firmly to risings and terrorism, both made considerable political, economic and religious concessions. Both contained individuals who took different views on how best to stabilise Ireland, eg Chamberlain (Liberal and then Unionist). Examiners must be open to alternative approaches. If in doubt, they should consult their Team Leader.

10

Page 330: History A - The Bicester Schoolthebicesterschool.org.uk/wp-content/uploads/2015/09/Mark... · 2015-09-28 · History A Mark Schemes for the Units January 2009 H106/H506/MS/R/09J

F966/02 Mark Scheme January 2011

9 How far was the issue of Home Rule in 1886 the main turning point in British attitudes to the Union with Ireland in the period from 1800 to 1921? [60]

Candidates will need to compare Home Rule with other approaches towards the Union –religious reforms (Catholic Emancipation, Maynooth and Disestablishment), local government reform, economic issues and land and other political approaches such as Partition. The question is concerned with the British point of view – did ‘1886’ change government thinking more radically than other events? The argument that it was a turning point might refer to its impact on the Liberals who split on the issue but remained formally committed to a form of autonomy for Ireland up to the Government of Ireland Act in 1920, which created separate Home Rule for both North and South. All Liberal governments attempted versions of it (1886, 1893, and 1912) except for Campbell Bannerman’s. Gladstone argued that the Union had no moral force without autonomy. Before 1886 all governments either coerced to maintain the Union as it stood or adopted the approach of reform within the Union. After Peel’s Catholic Emancipation in 1829 Britain seemed prepared to make religious and land concessions to Ireland but took a firm stance against any suggestion of a return to a Grattan style Dublin parliament, however constituted. That changed in 1886, its significance underlined by Chamberlain’s opposition and the creation of the Liberal Unionists and by continued Conservative commitment to the Union (and opposition to Home Rule) and to Ulster under Balfour and Bonar Law, at least until 1919-20. ‘1886’ thus opened up a key split in British attitudes. Against this view candidates might look more closely at the Home Rule Bills and stress that they were simply another way of reserving power at Westminster whilst granting control over minor matters to a Dublin Parliament. The 1886 Bill removed the now ‘awkward’ and obstructionist Irish Nationalist MPs from Westminster, kept even economic issues under central control and would have handed over only very marginal powers to the ‘conservative’ political machine of Parnell and later Redmond. The Second Bill in 1893 was also, arguably, another fudge by Gladstone in that it was largely about extending local self government, something the Conservatives and Chamberlain had argued for and were to deliver in 1898. It could be argued that Home Rule had more to do with maintaining power at Westminster than any change in attitude to the Union. 1886 (the ‘Union of Hearts’) and 1912 could be seen as tactics designed to secure the Irish Nationalist vote. Some candidates may see Catholic Emancipation as the more important turning point in that it heralded the beginning of a reluctant acceptance of a Catholic nation’s right to political office holding and represented a key change in approach to the role of the Protestant Ascendancy as Ireland’s local ruling class and the attitude of the Catholic Church to Irish nationalism. Similar importance could also be accorded to Land reform from the 1830s onwards. Together these changes in approach, largely consensual in the second half of the century, provided a successor group, the Catholic tenant farmer, to the Protestant Ascendancy. Alternatively candidates could point to the radical changes in attitude which occurred in 1919-21 as the more important turning point in that it accepted Partition, either in the Home Rule sense (1920) or as an absolute divide (1921) as the answer to relations with Ireland. Examiners must be open to alternative approaches. If in doubt, they should consult their Team Leader.

11

Page 331: History A - The Bicester Schoolthebicesterschool.org.uk/wp-content/uploads/2015/09/Mark... · 2015-09-28 · History A Mark Schemes for the Units January 2009 H106/H506/MS/R/09J

F966/02 Mark Scheme January 2011

Key Theme: Russia and its Rulers 1855 – 1964 10 Assess the view that the 1905 Revolution changed Russian government more than

other events in the period from 1855 to 1964. [60]

Candidates should focus on ‘the 1905 Revolution’ and the extent to which it led to changes in Russian government in their answers. Candidates may argue either for or against the importance of the 1905 Revolution but must do so comparatively in the context of other events. What follows is not an exclusive list, but consideration could be given to the effects of the Crimean War, the reforms of Alexander II, in particular the establishment of Zemstva, the assassination of Alexander II in 1881, the Statute of State Security and Land Captains introduced by Alexander III, the First World War, the February Revolution of 1917, the October Revolution of 1917, Stalin’s victory in the 1920s power struggle and his death in 1953. Examiners must not expect to find reference to all these examples in candidate answers and candidates may select other events in their answers. Candidates may argue that the 1905 Revolution changed Russian government because Bloody Sunday was a pivotal moment when Russians lost faith in their Tsar. They may argue that the main impact of the Revolution was the issue of the October Manifesto and the consequent abandonment of autocracy through elections to the Duma. Candidates may however argue that the Fundamental Laws and the rigging of the elections to the 3rd and 4th Dumas suggest that little of substance really changed. Candidates may argue that the First World War was the event with the most important impact on the development of Russian government because it was the horrific impact of the war both at the front and at home that sealed the fate of the Romanovs and, in turn, the Provisional Government. Arguably the appeal of the Bolsheviks in 1917 and the triumph of Lenin were directly related to the impact of the First World War. Candidates may argue that the end of over 300 years of Romanov rule in February 1917 was the event with the most important impact as it ended the 304 year old Romanov dynasty. It could also be argued that ultimately this led to the replacement of ‘Romanov Tsars’ by ‘Red Tsars’. Many candidates will undoubtedly argue that October 1917 and the triumph of Bolshevism was the event with the most important impact as it crushed all possibility that a liberal democracy might emerge in Russia and transformed Russia into the Soviet Union – the world’s first communist state. Some candidates may well consider that Lenin’s death in 1924 was the event with the most important impact, perverting the true course of the Russian Revolution because Stalin succeeded Lenin. Candidates who argue this are likely to suggest that Stalin’s victory in the ensuing power struggle led Russia down a very different road than that being paved by Lenin. Other candidates may use a counter-argument based on more recent archival evidence to suggest that there was significant continuity between Lenin and Stalin and argue accordingly. Candidates may argue that Khrushchev’s secret speech of 1956 and subsequent de-stalinisation was the event with the most important impact on the development of Russian government though others may argue that the continuation of communism way beyond 1964 somewhat negates that view.

Examiners must be open to alternative approaches. If in doubt, they should consult their Team Leader.

11 ‘Communists and Tsars ruled Russia in the same way.’ How far do you agree with

this view of the period from 1855 to 1964? [60]

Candidates should focus on the similarities and differences between the ways in which the Communists and the Tsars ruled Russia in this period. Candidates may well choose to concentrate predominantly on a comparison between the Tsars and the communists as rulers, but the most successful answers may involve comparisons between the individual rulers within each period. Arguments in favour of overall similarity might include autocratic/dictatorial government, the use of terror, centralized control of the economy and brief periods of reform. Comparisons could also be made at a personal level, for example

12

Page 332: History A - The Bicester Schoolthebicesterschool.org.uk/wp-content/uploads/2015/09/Mark... · 2015-09-28 · History A Mark Schemes for the Units January 2009 H106/H506/MS/R/09J

F966/02 Mark Scheme January 2011

between Alexander III and Stalin. Both regimes tended only to reform under pressure. The regimes ruled in similar ways, but there were considerable differences of scale (eg in terms of economic progress, urbanisation & the use of terror). A case could be made for arguing that the communists were a more ruthless and efficient twentieth century variant of Russian authoritarianism. In terms of arguing that there was more difference than similarity in the ways in which Russia was governed by the Tsars and the communists, arguments might include the very different ideologies, the fate of the old elite & the attitudes towards religion and the Orthodox Church. In terms of comparisons between the individual rulers within each period there was little in terms of continuity in terms of how Alexander II governed Russia from 1855 (beyond his desire to uphold the principle of autocracy) and Alexander III’s approach to government which was in significant contrast to his father’s. On the other hand there was a great deal of continuity between the reigns of Alexander III and Nicholas II in intent (though rather less in terms of outcome!). In terms of the communist period, the most interesting debate for candidates is the extent to which Stalinism was Leninism’s baby; did Stalin take the government of Russia down different paths to those being paved by Lenin? Candidates may consider the period of Lenin’s rule and to what extent he aimed to set up a harsh dictatorial regime. And of course, particular reforms implemented by Khrushchev after 1956 may be usefully contrasted with the ethos of centralisation which had coloured Stalin’s later years in power.

Examiners must be open to alternative approaches. If in doubt, they should consult their Team Leader.

12 Assess the view that economic change in Russia was more successful under Stalin

than any other ruler in the period from 1855 to 1964. [60]

Candidates should focus on ‘economic change’ and whether it was more successful under Stalin in their answers. Candidates may focus on the reasons for the economic development of Russia from 1855 to 1964. A relative comparison of Russia’s economic development under the Tsars from 1855 – 1917 and under communism may be undertaken. In support of the view in the question candidates are likely to focus on the achievements of the Five Year Plans both before and after the Second World War. Candidates may argue that Russia’s victory in the Second World War (as opposed to their defeats in most other wars during this period) and Russia’s emergence as a global super-power in the Cold War are adequate testament to the significance of Stalin’s role in Russian industrialisation. Any answers that are limited to the importance of Stalin’s role in terms of economic change within Russia are likely to be imbalanced. Candidates should compare and contrast the roles of others, for example Alexander II, Lenin and Witte (in the reigns of Alexander III and Nicholas II) in order to fully answer the question. For example, candidates may well argue that Alexander II’s Emancipation Edict of 1861 enabled much of the economic change that followed to happen, though others may argue that its economic impact on the lives of most peasants was insignificant. Witte’s ‘Great Spurt’ accomplished a great deal in terms of modernizing the economy in the 1890s. The NEP made important progress in terms of development after the economic low point of 1921. Candidates may also choose to argue that economic change under Stalin was only successful in terms of industrial might and that the consequences of collectivisation and the Five Year Plans were dreadful for many Russians. Some candidates may make a case for the economic reforms and achievements of Khrushchev and refer to his Five Year Plan, Seven Year Plan and Virgin Lands Scheme.

Examiners must be open to alternative approaches. If in doubt, they should consult their Team Leader.

13

Page 333: History A - The Bicester Schoolthebicesterschool.org.uk/wp-content/uploads/2015/09/Mark... · 2015-09-28 · History A Mark Schemes for the Units January 2009 H106/H506/MS/R/09J

F966/02 Mark Scheme January 2011

13 To what extent was the presidency of Lyndon Johnson (1963-69) the most important turning-point in the development of African American civil rights in the period from 1865 to 1992? [60]

Candidates who support this view might refer to the Civil Rights Acts of 1964 and 1968 and the Voting Rights Act of 1965 as the most significant legislative milestones because they ended the Jim Crow era in the South, and to the Great Society programmes which helped poor inner-city African Americans. Some candidates might see the LBJ presidency as a negative turning-point and refer to the Black Power movement, the Black Panthers, the urban rioting, the assassinations of Malcolm X and Martin Luther King and the damage done to LBJ’s Great Society programme by the Vietnam War. Better candidates might also be aware that, after 1965, the Civil Rights campaign achieved only mixed success – the emergence of a successful black middle class and increased political participation contrasted with the continued deprivation, discrimination and low aspirations of many poor, especially inner-city, African Americans. They may also refer to the increasingly acrimonious debates in the 1970s and 1980s over key aspects of civil rights: school bussing, affirmative action, judicial appointments and welfare provision.

Alternative turning-points candidates might consider include,

1 The Reconstruction period (1865-77) when important constitutional amendments were passed (1865: 13th abolishing slavery; 1868: 14th guaranteeing civil rights; 1870: 15th guaranteeing the right to vote) and there was significant African American participation in politics, supported by the Radical Republicans in Congress. The Freedman’s Bureau (until 1872) provided some legal assistance for former slaves and helped to establish schools.

2 The 1890s when the Jim Crow system was established in the South, taking advantage of the end of Reconstruction and a series of Supreme Court judgments (1873 Slaughterhouse Case; 1883 Civil Rights Cases; 1896 Plessy v Ferguson) which undermined the 14th and 15th amendments.

3 The two world wars when the expansion of the economy provided job opportunities for African American workers and led to migration northwards.

4 The Second World War, in particular, because the impact on society and the economy, and the number of African Americans serving in the armed forces were so great. The war also stimulated the expansion of grass-roots activism (the founding of CORE in 1942, the creation of the FEPC in response to Randolph’s threatened march on Washington, the ‘double V’ campaign) and discredited racism (Hitler’s camps and Japanese victories over Europeans).

5 The events of the 1950s when the Brown decision of 1954 overturned Plessy v Ferguson and the non-violent campaigns of Martin Luther King gave the civil rights movement publicity, significant white support and – arguably – irresistible momentum.

The best candidates will be able to make a judgment about the relative importance of the various turning-points. If arguing for the LBJ presidency they might observe that both the legislative changes and the bifurcation of the 1960s have proved permanent. Those selecting the Second World War might show how (building on the New Deal) the relationship between federal and state governments changed permanently, making it increasingly difficult for state governments to ignore federal legislation and Supreme Court rulings. They might also emphasise the importance of the transformation wrought by the war. If selecting the 1950s, they might point out that the Cold War competition between the West and the USSR for influence in the Third World made the retention of institutionalised racism in the USA impossible, especially as the war had destroyed any notion of the implicit superiority of white European culture. They might observe that, without these changes, the achievements of the LBJ era could not have occurred.

Examiners must be open to alternative approaches. If in doubt, they should consult their Team Leader.

14

Page 334: History A - The Bicester Schoolthebicesterschool.org.uk/wp-content/uploads/2015/09/Mark... · 2015-09-28 · History A Mark Schemes for the Units January 2009 H106/H506/MS/R/09J

F966/02 Mark Scheme January 2011

14 ‘The Federal government hindered rather than helped the development of trade union and labour rights.’ How far do you agree with this view of the period from 1865 to 1992? [60]

Candidates will probably argue that this was true of the period up to the Great Depression but that the Federal government was supportive during the New Deal. They will probably be aware that, after World War Two, the Federal government became less supportive of trade union rights but (under Democratic administrations at least) supportive of workers’ rights.

Examples of Federal hostility before the New Deal include 1 Laissez-faire assumptions about the role of government which helped to create a

climate of hostility to organised labour, especially during the ‘red scare’ that followed each world war.

2 President Cleveland’s use of federal troops to suppress the 1894 Pullman Strike. 3 1895 Supreme Court decision upholding the use of injunctions against trade unions

under the Sherman Anti-Trust Act. 4 1905 Lochner Case (striking down a New York law setting a maximum number of

working hours for bakery workers). 5 1908 Adair Case (upholding ‘yellow dog’ contracts by which workers were prevented

from joining a union). 6 Use of the 1917 Espionage Act to suppress ‘The Wobblies’ (Industrial Workers of the

World). 7 1921 ruling declaring unconstitutional the 1914 Clayton Act (which aimed to

guarantee workers’ rights to organize, bargain collectively, strike, boycott and picket).

Examples of the post-war climate unsympathetic to organised labour include 1 1943 Smith-Connally Act preventing strike action in industrial plant producing war

materials. 2 1947 Taft-Hartley Act allowing states to pass ‘right to work’ laws banning the 'closed

shop'. 3 1959 Landrum-Griffin Act banning secondary picketing. 4 The impact on trade union power, membership and strike activity of Reagan’s 1981

defeat of the Air Traffic Controllers’ strike.

In dealing with Federal support, candidates might refer to 1 Attempts during the presidencies of Theodore Roosevelt and Woodrow Wilson to

legislate on such matters as working conditions, consumer protection, housing and education.

2 The key New Deal measures: the National Labour Relations Act of 1935 (Wagner Act), the NIRA (1933), the Social Security Act (1935) and the Fair Labour Standards Act (1938). Better candidates might also point out that FDR’s response to the labour militancy of 1936-7 contrasts sharply with that of earlier administrations.

3 The creation during WW2 of the FEPC and the National War Labour Board.

Good candidates may also be aware of the support given, especially by Democratic presidents, to workers. Truman’s ‘Fair Deal’, JFK’s 'New Frontier' and, in particular, LBJ’s 'Great Society' programmes aimed to build on FDR’s New Deal and brought definite benefits to working people: support for a minimum wage, economic regeneration measures, improved housing and medical care and better work opportunities. Candidates can also refer to Nixon’s support for affirmative action and Carter’s extension of the minimum wage as examples of presidential support for labour rights. In contrast, they may also be aware that Reagan’s policies of lower taxes and business deregulation were part of a deliberate rejection of the New Deal philosophy in the 1980s. They might also show that the three branches of the federal government were not always in harmony: the Supreme Court struck down Congressional legislation in 1921 (the Clayton Act) and in 1935 the Schechter Case

15

Page 335: History A - The Bicester Schoolthebicesterschool.org.uk/wp-content/uploads/2015/09/Mark... · 2015-09-28 · History A Mark Schemes for the Units January 2009 H106/H506/MS/R/09J

F966/02 Mark Scheme January 2011

ended FDR’s NIRA. Both the Smith-Connally Act and the Taft-Hartley Act were passed over presidential vetoes.

Examiners must be open to alternative approaches. If in doubt, they should consult their Team Leader.

15 ‘It was the activism of women themselves that was the most important factor in advancing their civil rights.’ How far do you agree with this judgement on the period from 1865 to 1992? [60]

From the Progressive era candidates can discuss campaigners for improved regulation of working conditions and for health and housing reform such as Jane Addams, Florence Kelley and the National Consumers’ League. They might also discuss Ida Wells’s anti-lynching crusade and the role of the National Association of Colored Women (1896). On the suffrage issue, some might refer back to the Seneca Falls Convention of 1848 as the start of organised women’s campaigning and go on to refer to the work of Elizabeth Cady Stanton and Susan B Anthony and of the more genteel Lucy Stone. They will probably be aware of their respective pressure groups, the NWSA and the AWSA (both 1869, but merged as the NAWSA, 1890) as well as Alice Paul’s more radical campaign and the role of the National Women’s Party (1917). Candidates may mention women’s role in the temperance movement and refer to the work of Frances Willard and the WCTU (1874) as well as the Anti-Saloon League (1893). Better candidates may also be aware of women’s role in the repeal of prohibition and of Pauline Sabin and WONPR (1929). Some might refer to Margaret Sanger’s lengthy campaign for family planning and birth control. In the post-war period, candidates will probably discuss the impact of Betty Friedan’s campaign to liberate women from domesticity and the role of NOW (1966) in pressing for women’s equality and its impact on the Roe versus Wade judgement and the ERA. Some may wish to discuss Phyllis Schlafly and the importance of her campaign against the ERA.

The weaker candidates will probably do little more than outline the activities of some or all of these campaigns. Better candidates will attempt some judgement about their importance. They may be aware that women were not the only campaigners during the Progressive era which was a reaction to changing economic and social conditions brought about by the rapid industrialisation of the USA in the late 19th century and that legislative and judicial progress depended on male politicians in the White House and Congress and male justices in the Supreme Court. They may also be aware that the 19th amendment (1920) came after many states had already given women the vote and that western territories (eg Wyoming 1869, Utah 1870) did so to encourage emigration westwards and hasten statehood. They may point out that prohibition was part of the wider Progressive agenda and as much a product of protestant religious zeal as female activism. Repeal owed as much to the perceived failure of prohibition and the need to revive (and tax) the drinks industry during the Depression as to women’s campaigning. Better candidates will probably also be aware of the controversial nature of modern feminism and that issues such as pornography, abortion, the ERA and women’s role in the economy have divided both the women’s movement and male opinion.

The best candidates might also assess how far divisions in the women’s movement lessened the effectiveness of their campaigns (eg the differing aims of black and white, middle class and poor). They may also be able to evaluate the importance of women’s activism in relation to other factors and balance the importance of women’s pressure on politicians with the impact of social, cultural and economic change bringing women more opportunities in education and employment. They may wish to analyse how far social and legislative change was caused by, or followed, women’s activism. Finally, they might assess how far there is a consensus on what women’s civil rights are.

Examiners must be open to alternative approaches. If in doubt, they should consult their Team Leader.

16

Page 336: History A - The Bicester Schoolthebicesterschool.org.uk/wp-content/uploads/2015/09/Mark... · 2015-09-28 · History A Mark Schemes for the Units January 2009 H106/H506/MS/R/09J

F966/02 Mark Scheme January 2011

16 ‘Changes made by the Liberals between 1868 and 1893 were the most important factors in the development of democracy.’ How far do you agree with this view of the period from 1868 to 1997? [60]

Candidates will need to compare the importance of the party and constitutional changes of the Liberal Gladstone period against other factors such as the achievement of universal male suffrage in 1918, the enfranchisement of women 1918-28, the curbing of the Lords’ veto in 1911, the creation of a literate nation by 1914, the inclusion of Youth in the later 20th century, the mobilisation and involvement of key groups throughout the period (Nonconformists, Catholics, Trade Unions, regional groups in Wales, Scotland and Ireland), and the provision of political information by popular newspapers, radio and television. It could be argued that the 1868-93 changes - Irish Disestablishment (1869) the Secret Ballot Act (1872), Forster’s Education Act (1870), the Birmingham Caucus and the National Liberal Federation, the Corrupt Practices Act (1883) the Third Reform Act and Redistribution (1884 and 1885) and Irish Home Rule (1886 and 1893) – created the possibility of democracy by the end of the 19th century. It furthered religious equality, extended to Wales and Scotland in the early 20th century, and at least seriously proposed a federal and devolved constitutional structure for the UK that was achieved for part of Ireland by 1921. The ending of patronage and corruption via expense limits and secret voting in 1883 and 1872 ended nomination and private corruption, something that required a more democratic type of voter mobilisation pioneered by Chamberlain in the Birmingham Caucus and the ensuing National Liberal Federation (more accountability, political involvement and canvassing etc.). With the extension of universal household suffrage to the Counties and the end of the county/borough distinction a majority of men (60%) now had the vote and ‘population’ could have due effect via equal constituencies. The skilled Trade Unions were recognised and integrated into society. Artisan democracy had been achieved. However it could be argued that the Church in England was never disestablished and even Liberal politicians saw no reason to enfranchise the residuum of the working class until the 1910s. Property remained the determinant of voting rights. Female suffrage was not on the agenda until the early 20th century and even then wider female issues were ignored until the 1970s by middle class male and arguably unrepresentative MPs. Devolution proved a cul de sac up to 1997 (except for Northern Ireland, with considerable consequences for the rights of Catholics there until the 1970s and beyond). Education remained limited to the basics and was essentially primary until the developments of 1918-45. The Trade Unions were to feel unrepresented by both Liberals and Conservatives by the turn of the century and moved to cooperate in the founding of their own party post 1900. It could be argued that it was this and the slow acceptance of Labour between 1900 and 1931 that was more important in achieving democracy. Similarly the aristocratic and establishment veto was not removed until 1911 and the Lords remained an undemocratic voice and power up to 1997.The arrival of a populist press by the 1890s, largely Conservative, and the role of the BBC, could also be seen as vital in popularising politics and, in the case of the Reithian BBC, in consciously educating people in respectable democratic modes. The consideration of minorities (not least women) awaited the later 20th century. Thus democracy was a patchy development throughout the period. Examiners must be open to alternative approaches. If in doubt, they should consult their Team Leader.

17

Page 337: History A - The Bicester Schoolthebicesterschool.org.uk/wp-content/uploads/2015/09/Mark... · 2015-09-28 · History A Mark Schemes for the Units January 2009 H106/H506/MS/R/09J

F966/02 Mark Scheme January 2011

17 Assess the view that a two party political system dominated British politics in the period from 1868 to 1997.

The argument that it did could be supported by reference to the dominance of the Liberal and Conservative parties between 1868 and 1918 followed by the dominance of the Conservative and Labour parties from 1918 to 1997. This was aided by a first past the post system that tended to militate against coalitions and supported parties that could draw upon money, voluntary effort and the support of key groups and organisations. It was also reinforced by the development of class based politics as the franchise was increased, something that helped explain the transition from Liberal to Labour as the other main party. The ‘swing of the pendulum’ provided ‘choice’ in a democratising society and the ambitious gravitated towards one or the other of the two parties. Their control of politics and parliament made constitutional change to widen choice difficult if not impossible, as the Liberal commitment to PR and the attempted breakthrough of the SDP in the 1980s demonstrated in the second half of the 20th century. This dominance was also reinforced by the media in its focus on the two parties and the less air - time allocated to minority parties. The periods 1868 - 1915 and 1945-1997 seem to support the contention in the question. However this would be to ignore periods when it could be argued that two-party dominance was a myth. All parties tended to be coalitions and groups moved to independence and association with others throughout the period. The Whigs moved from Liberal to Conservative in the Salisbury period (from the late 1870s), Lib-Labs moved into association with the Socialist Societies from the 1880s onwards, the Liberal Unionists moved to the Conservatives in the late 1880s and the Irish Nationalists were a crucial third party from the late 1870s until 1918, being replaced by Sinn Fein who refused to work with any mainland party. The Nationalists arguably determined the fate of governments in 1886 and again after 1910. The 1918 – 45 period could be seen as a period when a two party system broke down with the fragmentation of the Liberals (Asquithians v the followers of Lloyd George), Labour during the 1st World War and after the split of 1931 and with divisions in the Conservatives occasioned by Tariff reform 1904-6 and by the Lloyd George Coalition 1918-22. Coalitions dominated the period – 1915-22 (Liberal, Conservative and, until 1917, Labour); 1924 and 1929 -45 (Labour with Liberal support 1929-31, followed by Conservatives with National Liberal and Labour support 1931-1940, then an all party wartime government 1940-45). Social changes, war, economic crisis and political infighting led to a breakdown in a two party system. Since 1945 the Liberals, SDP and the Liberal Democrats have maintained a reasonably high profile (polling large numbers of votes after 1970 – up to 25%) with much talk about breaking the mould in the 1980s when Labour was subject to internecine feuding and restructuring as its traditional electorate fractured and declined. Plaid Cymru and the SNP have also maintained a presence in their respective regions, despite a tendency for Labour to dominate, whilst within the political parties there have always been groupings and ‘caves’ where those of a particular persuasion have made their presence felt eg the Ulster Unionists within the Tories. It could also be pointed out that the two main parties have been subject to considerable change both in terms of policies, structures and support. It could also be pointed out that, in periods of two-party dominance, (post 1945), there have been times when elections were won by the same party consecutively - between 1951 and 1964 Labour lost three elections in a row. There is a case for Britain having a dominant party system from 1916 - the Conservatives. Labour failed to achieve two full term consecutive periods of office in the 1918 – 1997 period. Examiners must be open to alternative approaches. If in doubt, they should consult their Team Leader.

18

Page 338: History A - The Bicester Schoolthebicesterschool.org.uk/wp-content/uploads/2015/09/Mark... · 2015-09-28 · History A Mark Schemes for the Units January 2009 H106/H506/MS/R/09J

F966/02 Mark Scheme January 2011

19

18 How far did economic change influence the development of democracy in the period from 1868 to 1997?

The argument for a crucial role for economic change is that parties have responded to it by moving in a democratic manner. Thus the franchise consciously caught up with the rise of the upper working class after 1867 and this reinforced the dominance of Liberalism at the beginning of the period, which was urban based. It also led to the rise of the lower middle class as a tertiary sector grew to service a mature industrial society and this helped to secure the late Victorian dominance of the Conservatives (Mr Pooter) who were keen to cooperate over single member constituencies in order to give due weight to the suburbs. A mature industrial society also accommodated female employment in primary and tertiary industry and their importance in the 1st World War was thus recognised in 1918 and 1928 through suffrage equality with men, although unemployment and slump in the Inter-War years delayed any emancipation and gender and economic equality until much later in the 20th century. Trade Unions as an expression of economic change (Model Unionism in the 1860s and 1870s; New Unionism in the 1880s) also resulted in a Labour party post 1900 that threatened to put a different economic agenda before government and both conservative and Liberal agendas adapted to a more progressive one, the Liberals before 1914 and the Conservatives intermittently in the 1920s and 1930s and after 1945. Wartime economic controls and the move to a command economy in the 1910s and 1940s put democratic issues (homes, health, education, full employment and insurance) firmly to the fore. Since 1945 economic change has had less effect on democracy given that a democratic arena had been achieved earlier, although candidates could point to the increasing role of middle class women in the economy and the Equal Opportunities Acts of the 1970s that resulted and educational developments that sought to respond to economic demand. The main democratic argument has been the position and power of the industrial unions in a de-industrialising society, fought out over Union legislation between 1969 and 1993 (especially in the 1980s – limiting the position achieved in the 1870s via the Employment Act of 1980, the Trade Union Act of 1984 and other employment acts in 1982, 1986, 1988, 1990 and 1993). These Acts reflected the economic decline of manufacturing industry and the rise of service industries characterised by part-time, flexibility and low wages. It could be argued that in terms of democracy fewer chose to vote, becoming marginalised as a unrepresented under-class, socially excluded and dependent on state benefits. As in the 1920s and 1930s unemployment was an issue for governments which also became more interested in middle class and middle aged issues, ie those who voted. It has also had an impact on the regions, the argument being that the North and the Celtic Fringe, recognised in the 2nd half of the 19th century, have lost out economically and politically to the South and East. However candidates can equally argue that economic change has had little importance in the development of democracy in comparison to party competition, the impact of pressure groups, war, education or the manoeuvrings of the political elite. It was rare that economic factors were cited as the reason for any specific change. Similarly they could argue that economic change was more important in some periods (1868-1914) than others (1945-1997). Much depends on whether economic change is considered the determinant of all changes (in a Marxist sense), just some change, or simply the general backdrop to democratic development. Examiners must be open to alternative approaches. If in doubt, they should consult their Team Leader.

Page 339: History A - The Bicester Schoolthebicesterschool.org.uk/wp-content/uploads/2015/09/Mark... · 2015-09-28 · History A Mark Schemes for the Units January 2009 H106/H506/MS/R/09J

OCR (Oxford Cambridge and RSA Examinations) 1 Hills Road Cambridge CB1 2EU OCR Customer Contact Centre 14 – 19 Qualifications (General) Telephone: 01223 553998 Facsimile: 01223 552627 Email: [email protected] www.ocr.org.uk For staff training purposes and as part of our quality assurance programme your call may be recorded or monitored

Oxford Cambridge and RSA Examinations is a Company Limited by Guarantee Registered in England Registered Office; 1 Hills Road, Cambridge, CB1 2EU Registered Company Number: 3484466 OCR is an exempt Charity OCR (Oxford Cambridge and RSA Examinations) Head office Telephone: 01223 552552 Facsimile: 01223 552553 © OCR 2011

Page 340: History A - The Bicester Schoolthebicesterschool.org.uk/wp-content/uploads/2015/09/Mark... · 2015-09-28 · History A Mark Schemes for the Units January 2009 H106/H506/MS/R/09J

Oxford Cambridge and RSA Examinations

GCE

History A Advanced GCE

Unit F966/02: Historical Themes Option B: Modern 1789-1997

Mark Scheme for June 2011

Page 341: History A - The Bicester Schoolthebicesterschool.org.uk/wp-content/uploads/2015/09/Mark... · 2015-09-28 · History A Mark Schemes for the Units January 2009 H106/H506/MS/R/09J

OCR (Oxford Cambridge and RSA) is a leading UK awarding body, providing a wide range of qualifications to meet the needs of pupils of all ages and abilities. OCR qualifications include AS/A Levels, Diplomas, GCSEs, OCR Nationals, Functional Skills, Key Skills, Entry Level qualifications, NVQs and vocational qualifications in areas such as IT, business, languages, teaching/training, administration and secretarial skills. It is also responsible for developing new specifications to meet national requirements and the needs of students and teachers. OCR is a not-for-profit organisation; any surplus made is invested back into the establishment to help towards the development of qualifications and support which keep pace with the changing needs of today’s society. This mark scheme is published as an aid to teachers and students, to indicate the requirements of the examination. It shows the basis on which marks were awarded by Examiners. It does not indicate the details of the discussions which took place at an Examiners’ meeting before marking commenced. All Examiners are instructed that alternative correct answers and unexpected approaches in candidates’ scripts must be given marks that fairly reflect the relevant knowledge and skills demonstrated. Mark schemes should be read in conjunction with the published question papers and the Report on the Examination. OCR will not enter into any discussion or correspondence in connection with this mark scheme. © OCR 2011 Any enquiries about publications should be addressed to: OCR Publications PO Box 5050 Annesley NOTTINGHAM NG15 0DL Telephone: 0870 770 6622 Facsimile: 01223 552610 E-mail: [email protected]

Page 342: History A - The Bicester Schoolthebicesterschool.org.uk/wp-content/uploads/2015/09/Mark... · 2015-09-28 · History A Mark Schemes for the Units January 2009 H106/H506/MS/R/09J

F966/02 Mark Scheme June 2009

AOs AO1a AO1b Total mark for each question = 60

Recall, select and deploy Demonstrate understanding of the past historical knowledge through explanation, analysis and arriving at appropriately, and communicate substantiated judgements of: knowledge and understanding of - key concepts such as causation, history in a clear and effective consequence, continuity, change and manner. significance within an historical context;

- the relationships between key features and characteristics of the periods studied

Level IA Uses a wide range of accurate

and relevant evidence Accurate and confident use of

appropriate historical terminology

Answer is clearly structured and coherent; communicates accurately and legibly.

18-20

Excellent understanding of key concepts (eg continuity and change) relevant to analysis in their historical context

Excellent synthesis and synoptic assessment

Answer is consistently and relevantly analytical with developed explanations and supported judgements

May make unexpected but substantiated connections over the whole period

36-40

Level IB Level IB

Uses accurate and relevant evidence

Accurate use of a range of appropriate historical terminology

Answer is clearly structured and mostly coherent; communicates accurately and legibly

16-17

Very good level of understanding of key

concepts (eg continuity and change) in their historical context.

Answer is consistently focused on the question set

Very good level of explanation/ analysis, and provides supported judgements.

Very good synthesis and synoptic assessment of the whole period

32-35

Level II Uses mostly accurate and relevant evidence

Generally accurate use of historical terminology

Answer is structured and mostly coherent; writing is legible and communication is generally clear

14-15

Good level of understanding of key concepts (eg continuity and change) in their historical context

Good explanation/ analysis but overall judgements may be uneven

Answer is focused on the issues in the question set

Good synthesis and assessment of developments over most of the period

28-31

1

Page 343: History A - The Bicester Schoolthebicesterschool.org.uk/wp-content/uploads/2015/09/Mark... · 2015-09-28 · History A Mark Schemes for the Units January 2009 H106/H506/MS/R/09J

F966/02 Mark Scheme June 2011

Level III Uses relevant evidence but there may be some inaccuracy

Answer includes relevant historical terminology but this may not be extensive or always accurately used

Most of the answer is structured and coherent; writing is legible and communication is generally clear

12-13

Shows a sound understanding of key concepts, especially continuity and change, in their historical context

Most of the answer is focused on the question set

Answers may be a mixture of analysis and explanation but also description and narrative, but there may also be some uneven overall judgements; OR answers may provide more consistent analysis but the quality will be uneven and its support often general or thin

Answer assesses relevant factors but provides only a limited synthesis of developments over most of the period

24-27

Level IV

There is deployment of

relevant knowledge but level/ accuracy will vary.

Some unclear and/or underdeveloped and/or disorganised sections

Mostly satisfactory level of communication

10-11

Satisfactory understanding of key concepts

(eg continuity and change) in their historical context

Satisfactory focus on the question set Answer may be largely

descriptive/narratives of events, and links between this and analytical comments will typically be weak or unexplained

Makes limited synoptic judgements about developments over only part of the period

20-23

Level V

General and basic historical

knowledge but also some irrelevant and inaccurate material

Often unclear and disorganised sections

Adequate level of communication but some weak prose passages

8-9

General understanding of key concepts (eg continuity and change) in their historical context

Some understanding of the question but answers may focus on the topic and not address the question set OR provides an answer based on generalisation

Attempts an explanation but often general coupled with assertion, description/narrative

Very little synthesis or analysis and only part(s) of the period will be covered

16-19

2

Page 344: History A - The Bicester Schoolthebicesterschool.org.uk/wp-content/uploads/2015/09/Mark... · 2015-09-28 · History A Mark Schemes for the Units January 2009 H106/H506/MS/R/09J

F966/02 Mark Scheme June 2011

Level VI Use of relevant evidence will be limited; there will be much irrelevance and inaccuracy

Answers may have little organisation or structure

Weak use of English and poor organisation

4-7

Very little understanding of key concepts (eg continuity and change) in their historical context

Limited perhaps brief explanation Mainly assertion, description/narrative Some understanding of the topic but not

the question’s requirements 8-15

Level VII Little relevant or accurate

knowledge Very fragmentary and

disorganised response Very poor use of English and

some incoherence 0-3

Weak understanding of key concepts (eg continuity and change) in their historical context

No explanation Assertion, description/narrative

predominate Weak understanding of the topic or of the

question’s requirements 0-7

3

Page 345: History A - The Bicester Schoolthebicesterschool.org.uk/wp-content/uploads/2015/09/Mark... · 2015-09-28 · History A Mark Schemes for the Units January 2009 H106/H506/MS/R/09J

F966/02 Mark Scheme June 2011

1 Assess the view that German nationalism lacked popular appeal in the period from 1789 to 1919. [60]

Candidates should focus on ‘German nationalism’ and ‘popular appeal’ in their answers in

an attempt to assess the appeal of nationalism in this period. Candidates should evaluate the extent to which nationalism had popular appeal and demonstrate awareness that such appeal was not uniform but fluctuated. Candidates may demonstrate that concepts of romantic nationalism had a limited intellectual appeal. Candidates could consider the extent to which nationalism appealed or failed to appeal to the people at various points, for example from 1789 – 1815, 1815 – 1848, from 1871 - 1914 and in 1918 - 19. Candidates may well demonstrate that they understand that Wilhelmine Germany increasingly looked to exploit nationalist yearnings and the mass appeal of German nationalism, pursuing a populist foreign policy to distract the masses from social discontent. Candidates might choose to demonstrate that the mass appeal of nationalism may be compared to the mass appeal of other philosophies. For example the growing industrialization of Prussia and the German Empire was mirrored by the growing mass appeal of socialism, an appeal that proved relatively immune to either appeasement, in the form of state socialism, or repression. Distress from the winter of 1916 / 1917 onwards, and defeat in 1918, led to the socialist uprisings of late 1918 and early 1919 and the establishment of Ebert’s republic. However, even in 1919 the appeal of unrequited nationalism was never far from the surface, as evidenced by the Freikorps and the emerging ‘stab-in-the-back’ theory.

Examiners must be open to alternative approaches. If in doubt, they should consult their

Team Leader. 2 ‘Bismarck’s appointment as minister president of Prussia in 1862 was the most

important turning-point in the course of German nationalism.’ How far do you agree with this view of the period from 1789 to 1919? [60]

Candidates should focus on the phrase ‘most important turning point’ in their answers.

Candidates may argue either for or against the appointment of Bismarck as minister president of Prussia in 1862 as the most important turning point, but must do so comparatively in the context of other turning points. Any answers that are limited to the importance of the appointment of Bismarck as minister president of Prussia in 1862, however full and accurate, are likely to be imbalanced. In assessing the significance of the appointment of Bismarck as minister president of Prussia in 1862 candidates are likely to stress his impact on events in the period from 1862 to 1870 and the consequent domination of Germany by Prussia. What follows is not an exclusive list of other potential turning-points, but obvious consideration could be given to

1792 The start of the Revolutionary Wars 1813 Napoleon’s defeat at the battle of Leipzig (the Battle of the Nations) 1815 Congress of Vienna – formation of German Confederation 1848-49 Defeat of the Revolution(s) 1866 Seven Weeks War – the defeat of Austria 1870/71 The Franco – Prussian War and formation of the Second Reich (German

Empire) 1888 Accession of Wilhelm II 1890 Sacking of Bismarck 1914 Start of the First World War 1918 Defeat in the First World War

4

Page 346: History A - The Bicester Schoolthebicesterschool.org.uk/wp-content/uploads/2015/09/Mark... · 2015-09-28 · History A Mark Schemes for the Units January 2009 H106/H506/MS/R/09J

F966/02 Mark Scheme June 2011

Clearly answers of the very highest quality can be written without considering all of these potential turning points, but the most able candidates will demonstrate a breadth of vision and a good understanding of the moments that shaped the destiny of German nationalism. Candidates may also choose to compare Bismarck with other individuals.

Examiners must be open to alternative approaches. If in doubt, they should consult their Team Leader.

3 Assess the view that the German nation was just as divided from 1871 to 1919 as it

was from 1789 to 1870. [60] Candidates should focus on the extent to which the German people were more united in

the period following the creation of the Second Reich than they were up to 1870. There are many possible approaches to this question as candidates may consider unity territorially and / or politically and / or economically and / or socially. Candidates may argue both for and against this proposition. Candidates may understand that the Prussian Empire in 1871 represented Kleindeutschland and was an enlarged Prussia. They may argue that it was a Prussian Empire rather than a German Empire; it certainly did not unite all the German people even geographically. The exclusion of Austria from the process of German unification may be dealt with, though candidates may refer to Bismarck’s creation of the Dual Alliance as significant. Divisions within the German Nation after 1871 might be illustrated through the Kulturkampf and the rise of socialism, or the domination of the Reich by the elites. However, candidates may also argue that territorial boundaries rarely exactly match where the people of that nationality live and that divisions within a nation based on class or culture do not necessarily define the unity or otherwise of that nation. All modern nations have exhibited such divisions. They could certainly argue that the German Empire from 1871 physically united the majority of Germans. Candidates may argue that the German nation achieved a form of unity under the domination of Napoleon and through the creation of the Confederation of the Rhine. Germany was then certainly very divided from 1815 as a consequence of decisions taken at the Congress of Vienna, though it could also be argued that the German Confederation from 1815 loosely bound most Germans into a Confederation with a Diet. Candidates may also make the point that from 1866 the vast majority of German states were already united behind the leadership of Prussia. Candidates may argue that the development of more radical nationalism in the late Nineteenth and early Twentieth Century demonstrates an emergence of greater national unity amongst the German people during this period. Candidates may argue that the First World War united the German nation at first but that divisions soon arose and were entrenched by 1918. Similarly, whilst Versailles divided the nation geographically it united the nation in condemnation and bitterness of the ‘diktat’.

Examiners must be open to alternative approaches. If in doubt, they should consult their

Team Leader.

5

Page 347: History A - The Bicester Schoolthebicesterschool.org.uk/wp-content/uploads/2015/09/Mark... · 2015-09-28 · History A Mark Schemes for the Units January 2009 H106/H506/MS/R/09J

F966/02 Mark Scheme June 2011

4 ‘Armies were led by incompetent generals.’ To what extent do you agree with this view of the period from 1792 to 1945? [60]

Examples of incompetent generals in the period are legion but specific examples might be

Mack, Brunswick in 1806, Raglan, McClellan, Benedek or Bazaine. Examples of competent generals that might be used to challenge the proposition are

Napoleon, Wellington, Lee, Grant, Sherman, Moltke the Elder, Montgomery, Eisenhower, von Manstein, Guderian, Zhukov or Slim. Some candidates may refer to generals who have been the subject of revision; an obvious example might be Haig.

We might expect discussion of quality of leadership: control and application of strategy and

tactics, an ability to respond positively to the changing demands of warfare in the period (new technologies, transport systems, etc.), the skill to motivate other officers and men, broad vision, willingness to delegate and to be flexible. Other factors may be included in the essay but the key prompt of the question must be at the core of the response. Where other factors are linked directly and intelligently to the specific wording of the question then credit is due. Candidates might discuss the role of luck, fog of war, problems created by incompetent subordinates. The incompetence or competence of a general might be discussed in the context of the changing nature of warfare and, indeed, this might be the mark of a high quality response.

Examiners must be open to alternative approaches. If in doubt, consult your Team Leader. 5 ‘The development of the Prussian general staff during the Wars of German

Unification was the main turning point in the conduct of war.’ How far do you agree with this view in the period from 1792 to 1945? [60]

Candidates may argue for the turning point because it was only in the wars of 1866 and

1870-1 that the Prussians had fully developed their general staff concept and applied it to the planning and control of warfare. Key ideas for discussion might be the development of a meritocratic, professional and properly trained officer corps, higher army organisation, the mobilisation and control of mass armies, the development of general staffs, the planning of campaigns (military concentration, speed of movement, control of corps etc.), and the search for rapid and decisive victory. This list is not exhaustive.

A positive answer might concentrate on the rise of Prussian styles of warfare and would

use evidence from the Austro-Prussian and Franco-Prussian Wars. Candidates might argue that this Prussian style of higher command then dominated warfare until the end of the period. Alternative turning points might be the Revolutionary and Napoleonic periods with the development of Napoleon’s headquarters and allied attempts to copy them during those conflicts; the planning & preparation by the Great Powers for WWI. The American Civil War could be used either way, pointing to American armies copying & developing European styles of war or pointing to a general state of disorder. Candidates might point to the links between technology and command and control in WWII as the real turning point.

Alternatively, the concept of turning point might be rejected altogether, rather planning and

preparation for war was part of an ongoing process. Examiners must be open to alternative approaches. If in doubt, consult your Team Leader.

6

Page 348: History A - The Bicester Schoolthebicesterschool.org.uk/wp-content/uploads/2015/09/Mark... · 2015-09-28 · History A Mark Schemes for the Units January 2009 H106/H506/MS/R/09J

F966/02 Mark Scheme June 2011

6 ‘Developments in weaponry revolutionised warfare.’ To what extent do you agree with this view of the period from 1792 to 1945? [60]

Candidates should focus on the concept of revolution in the development of warfare. Two common lines of argument might be expected, either developments in weaponry did

revolutionise warfare or did not. A third position might be to question the extent of such a revolution at different points in the period, perhaps arguing that warfare changed far more as weapons development increased in tempo.

The first line of argument might point to the development of longer range and more rapid

firing weapons and the impact this had on battlefield formations, the deployment and manoeuvre of bodies of troops. Candidates might use examples of developments in weaponry across a very short period of time, for example the use of breech loading needle guns against muzzle loading muskets in the 1866 war, or compare one end of the period to the other. A variety of weaponry could be deployed in support of the debate, artillery, small arms etc, but the synoptic nature of the assessment should be at the core of the argument.

A counter argument might concentrate on the nature of warfare and an argument that at a

very basic level there was considerable continuity in warfare. This type of response might use military theory as the basis for the debate, for example the use of concentration of force, surprise, etc.

Alternatively, candidates might argue that battle tactics did not keep up with developments

in weapons technology. Indeed, much of the modern literature points to essentially Napoleonic tactics being used until very late in the period. The American Civil War fits neatly into this debate.

Examiners must be open to alternative approaches. If in doubt, consult your Team Leader.

7

Page 349: History A - The Bicester Schoolthebicesterschool.org.uk/wp-content/uploads/2015/09/Mark... · 2015-09-28 · History A Mark Schemes for the Units January 2009 H106/H506/MS/R/09J

F966/02 Mark Scheme June 2011

7 How effective was revolutionary nationalism in Ireland in the period from 1798 to 1921? [60]

Candidates may argue that revolutionary nationalism was mainly ineffective until 1916, and

that many republicans even viewed the years from 1917 to 1921 as a failure since Ireland experienced civil war, an imposed Anglo-Irish Treaty and partition. However they will need to establish criteria for effectiveness, not least the achievement of aims. Here they failed throughout the period but they were able to mobilise large numbers on occasions (in 1798, during the Tithe War in the 1830s, in the agrarian outrages and land war of the 1880s, in the western parts of Ireland in the 1890s – O’Brien, and post 1916). They certainly changed British government policy after the Wolfe Tone Rising of 1798, after 1867, 1881 and again after the Easter Rising of 1916. They staged rebellions throughout the period, maintaining an effective ‘blood sacrifice’ (Emmet in 1803, Young Ireland in the 1840s, the Fenians and the IRB). Candidates could assess their leadership and may well conclude that it was ineffective prior to 1916, although Davitt successfully harnessed agrarian violence to nationalist ends and Tone, Emmet and O’Brien all managed heroic personal stands. Candidates could usefully evaluate any of the risings to illustrate inadequate arming, poor organisation and lack of an over-arching plan. Wolfe Tone or the Easter Rising of 1916 are good examples of this. More effective were De Valera and Collins post 1916. Much would be dependent on externalities like war (Napoleonic and 1st World War), foreign aid and support (France and Germany), Famine, the impact of the Diaspora (the Fenians and the US), agrarian crisis and the impact of Parliamentary Reform (in 1850, 1884-5 and 1918). The resolution of such events could rob the revolutionary nationalists of effectiveness. This could also be conditioned by the strategies adopted, usually based on a rising, which were poorly organised and easily put down, although violence could put Ireland onto the British agenda (the Fenians in 1867). Up to 1918 the secrecy adopted robbed them of the ability to organise large numbers, unless they used agrarian issues (Davitt and O’Brien). In contrast to Constitutional Nationalism an electoral strategy was only fully adopted in 1918 which, combined with absenteeism from Westminster was very effective. This political strategy was combined with a much more effective military one; not the single heroic rising of the 19th century but owing more to agrarian wars, in combination with passive resistance, boycotts and rural guerrilla warfare. This targeted the police and British Intelligence (Bloody Sunday) and provoked an unacceptable reaction from the British authorities (the Black and Tans and Croke Park). It used assassinations, terrorising local communities into non assistance and effectively using local government to set up a ‘state within a state’ to control land and administration outside the large towns. From the 1890s the Gaelic revival was used by some to achieve cultural independence (Pearse). British governments also determined, through their response, how effective revolutionary nationalists could be eg their over- reaction post 1916, in contrast to 1798 (very bloody but in a different context) and the 19th century.

Examiners must be open to alternative approaches. If in doubt, they should consult their

Team Leader.

8

Page 350: History A - The Bicester Schoolthebicesterschool.org.uk/wp-content/uploads/2015/09/Mark... · 2015-09-28 · History A Mark Schemes for the Units January 2009 H106/H506/MS/R/09J

F966/02 Mark Scheme June 2011

8 How far did support for the Union change within Ireland in the period from 1800 to 1921? [60]

The focus is on support in Ireland, although reference can be made, where appropriate, to

Britain given the links between the Protestant Ascendancy and Ulster Unionism and mainland political parties. British governments certainly hoped to build Irish support for the Union and indeed sought to govern Ireland through the Ascendancy until at least the 1880s. The Ascendancy was their main instrument in the first half of the 19th century, controlling patronage, politics, land and religion. The mass of the people may have been indifferent to the Union post 1800, except in a religious and economic sense, presenting little threat to the Ascendancy. The latter initially had mixed feelings about the Union, feeling the loss of Grattan’s Patriot Parliament. Their Protestant Irish nationalism began the 19th century on the back foot but they adjusted and took full advantage of Westminster, supporting coercion in the 1820s and 1830s. Emancipation challenged its political and religious dominance in the long term, although both the Whigs and Peel were reassuring in practice, punishing freeholders who had rebelled electorally in the late 1820s. There was little change here until the 1860s when Butt began to organise a separate political identity amongst Irish MPs to push for a return of a Dublin Parliament. The other area of support was Ulster but its nature and nationalism was different – middle and working class rather than aristocratic and gentry based, more confident, based on Presbyterianism rather than Anglicanism, boosted by industrial expansion and proceeding via the exclusion of Catholics from employment. It was less high profile before the 1880s but it too worried about Whig deals with O’Connell and Peelite and Gladstonian attempts to woo the Catholic Church, the mid century revival of which put Union Anglicanism and Presbyterianism on the defensive. The reaction of some of the Ascendancy was to back the Home Rule initiatives of Butt and Parnell, both from the Ascendancy, or to support devolved power like Plunkett towards the end of the century. Candidates may well argue that the 1880s were the turning point in changing the main base of support for the Union from the Ascendancy towards the Ulster Unionists. The former turned to Home Rule within the Union as their preferred solution but were undermined by government, economically through Land reform and the Agricultural Depression and politically through local government devolution in the 1880s and 1890s. They were replaced by a neutral or more pro nationalist class of Catholic Tenant Farmers whose loyalties were there to be won. Trenchant support for the Union passed to Ulster which took a hard line on resistance to Home Rule and defence of the status quo. It developed strong links with the Conservatives from 1886 (Churchill and the Orange Card) and organised along sectarian lines (Carson and Craig, the Solemn League and Covenant and the UVF), especially in the 1910s. Such Union support was of a very different type – sectarian, radical in organisation and willing to defend the Union, or at least Ulster, along para-military lines against nationalists and British governments and, given its regional rather than national base, prepared ultimately to accept Partition and Home Rule for itself 1918-21. There is thus a considerable change in the class basis and regional nature of support for the Union over the period and in the tactics adopted at any given time (support and/or opposition to governments and their Irish initiatives).

Examiners must be open to alternative approaches. If in doubt, they should consult their

Team Leader.

9

Page 351: History A - The Bicester Schoolthebicesterschool.org.uk/wp-content/uploads/2015/09/Mark... · 2015-09-28 · History A Mark Schemes for the Units January 2009 H106/H506/MS/R/09J

F966/02 Mark Scheme June 2011

9 To what extent did the Famine of 1845-9 change the Irish economy in the period from 1798 to 1921? [60]

Candidates will need to address the broad direction of Irish economic development over

the period to assess the extent of change occasioned by the Famine. Economically did it markedly change the Irish export of labour, the tendency to subsistence agriculture alongside larger estates, the decline of the Cottier class and the rise of a more substantial Catholic tenant farmer and the development of Industries in the North East (linen, shipbuilding and engineering)? Economically famines were not unusual (severe in the 1810s) and more substantial tenant farmers were already emerging. The Devon Commission, called before the Famine, concluded small plots were unsustainable. It could be argued the Famine merely hastened the land shake-up. Similarly emigration and labour export was well underway before the famine (1½ million leaving between 1815 and 1845). The West remained poor both before and after the famine. However some may stress change, particularly in reference to the scale of the disaster, citing the large drop in population and the arrest in the burgeoning figures of the 1770-1840 period (3 to 8 to 6 million). The Cottier class was wiped out along with seasonal unemployment. The potato declined in relative importance. Larger tenant farmers slowly began to modernise, move towards pastoral farming and government became more interventionist via public works, workhouses, Land sales, (Encumbered Estates Act 1849), and ultimately effective land reform. Evictions increased and tenant rights became politically more central in Anglo –Irish relationships. Ulster was less affected by the famine and the cities were more affected by the Cholera epidemic of 1849-50. Emigration was of the poorer type and it remained high throughout the period (2½ million 1850-1921). Arguably the Agricultural Depression was of similar gravity.

Examiners must be open to alternative approaches. If in doubt, they should consult their

Team Leader.

10

Page 352: History A - The Bicester Schoolthebicesterschool.org.uk/wp-content/uploads/2015/09/Mark... · 2015-09-28 · History A Mark Schemes for the Units January 2009 H106/H506/MS/R/09J

F966/02 Mark Scheme June 2011

10 Assess the view that the condition of the peasantry in Russia was transformed in

the period from 1855 to 1964. [60] Candidates should focus on the similarities and differences between the condition of the

peasantry and the treatment that the peasants received, both before and after 1917. Transformed is the key word in this title; candidates may well consider how valid this premise is. Candidates may well argue against the condition of the peasantry being transformed. It could be argued that the peasantry made little progress in many ways during this period and that predominantly their living and working conditions remained bleak. Peasants were only serfs under the Romanovs, but some candidates may argue that there was little real improvement and / or that collectivization was a ‘second serfdom’. Before and after 1917 there was harsh treatment of the peasantry by both regimes; in both periods they were ‘squeezed dry’ to finance industrialization. Famine hit, e.g. 1891, 1921 & 1932, regardless of regime, although arguably Stalin’s denial of the famine of the 1930s made its impact worse. Control over their lives, whether exercised through the Mir, the Land Captains or the Kolkhoz was a common feature, although distinctions may clearly be made. However there were periods of reform both before and after 1917 that should enable candidates to successfully support the view in the question. The peasants were given glimpses of reform, e.g. Emancipation in 1861, the Peasants Land Bank from the 1880s, the Decree on Land in 1917 and the NEP from 1921. All of these changes led to improvements, although some were temporary, in their living and working conditions. Both regimes had a temporary Kulak policy under Stolypin from 1906 & under the NEP from 1921-28 as peasants were encouraged to ‘enrich themselves’. Arguably the communists did much more to introduce social reform, for example in the sphere of education, than the Tsars. Candidates may argue that whilst some peasants suffered dreadfully under Stalin because of collectivisation and de-kulakisation the survivors had significantly better health care and education than their predecessors. And their prospects were further enhanced by Khrushchev’s Virgin Lands Scheme.

Examiners must be open to alternative approaches. If in doubt, they should consult their

Team Leader. 11 ‘The communist rulers were effective autocrats; the Tsars were not.’ How far do you

agree with this view of Russian government in the period from 1855 to 1964? [60] Candidates should focus on the relative effectiveness of the communists and the Romanov

Tsars as autocratic or dictatorial rulers of Russia. Candidates are likely to define their criteria for assessing the effectiveness of Russian rulers in this period and will then judge the rulers against them. The following list is not exclusive but obvious criteria might include the extent of the ruler’s personal power and authority and how effectively they dealt with opposition, the extent to which they successfully implemented their policies or the extent to which they were able to develop the power and international standing of the Russian state. Candidates can be expected to refer to the Okhrana, OGPU, NKVD, KGB and other secret services. Candidates who do not restrict themselves to a narrow definition of effectiveness are likely to be more successful! Candidates may well choose predominantly to concentrate on a comparison between the Tsars and the communists as rulers, but candidates may make comparisons between the individual rulers within each period. When arguing in support of the view in the question, candidates are likely to argue that Lenin and Stalin were ‘effective autocrats’. Lenin seized power in 1917 and successfully defended his revolution during the Civil War. Other parties were all banned, as were factions within the Communist Party. Candidates could argue that Stalin was even more effective, for example arguing that his economic policies in the 1930s enabled the USSR to successfully survive Barbarossa and emerge victorious in the Great Fatherland War. Candidates may argue that his ‘effectiveness’ was achieved at horrific expense and with needless brutality. Candidates are also likely to argue in support of the view in the question that neither Alexander II nor Nicholas II were ‘effective autocrats’ – the former was faced with a rising

11

Page 353: History A - The Bicester Schoolthebicesterschool.org.uk/wp-content/uploads/2015/09/Mark... · 2015-09-28 · History A Mark Schemes for the Units January 2009 H106/H506/MS/R/09J

F966/02 Mark Scheme June 2011

tide of opposition from the early 1870s whilst the latter was forced to abdicate in 1917 and butchered the following year with the rest of his family. Candidates may argue that Alexander II was effective because of the successful implementation of sweeping reforms, for example emancipation of the serfs, in the 1860s. When arguing against the view in the question, candidates are likely to be able to differentiate between the relative effectiveness of the individual Tsars. Candidates may well see Alexander III as an effective autocrat, although it can be argued that his repressive and reactionary policies were effective in the short-term but, as Trotsky put it, ‘bequeathed a revolution’ to his son and successor, Nicholas II. Candidates may argue that Alexander III achieved very little for Russia or his dynasty despite his apparent reassertion of autocratic control. Candidates may also argue that Khrushchev was far from an ‘effective autocrat’ using his overthrow and forced retirement in 1964 and the failure of policies such as the Virgin Lands scheme as obvious examples.

Examiners must be open to alternative approaches. If in doubt, they should consult their

Team Leader. 12 ‘All Russia’s rulers tried to modernise Russia.’ How far do you agree with this view

of the period from 1855 to 1964? [60] Candidates should focus on the phrases ‘all Russia’s rulers’ and ‘tried to modernise

Russia’ in their answers. ‘All’ and ‘tried’ are key words that most successful answers are likely to address. Candidates should argue both for and against the view in the question. Candidates may argue that all of Russia’s rulers modernised Russia using examples such as those that follow:

Alexander II Emancipation of the serfs and other reforms (e.g. Zemstva and Trial by Jury) Alexander III Peasants Land Bank / appointed Witte (start of his ‘Great Spurt’) Nicholas II Witte’s Great Spurt / the October Manifesto of 1905 / Stolypin’s Reforms Provisional Gov. Planned democratisation Lenin Decree on Land / War Communism / NEP Stalin Five Year Plans and Collectivisation Khrushchev Secret Speech / Virgin Lands / Space Race

Some candidates may focus on the social changes such as education, health, housing,

religion and the position of women in society. Clearly answers of the very highest quality can be written without considering all of these

events, but the most able candidates will demonstrate a breadth of vision and a good understanding of the ways in which most rulers tried to modernise Russia. However, the assertion that all rulers tried to modernise Russia will be challenged by most candidates. When arguing against this view, candidates may argue that neither Alexander II nor Nicholas II tried to modernise Russia. They are likely to focus on their determination to uphold autocracy, the influence of Pobeodonotsev and his desire to keep ‘Russia in a frozen state’. They may argue that the appointment of Witte by Alexander IlI and the continuation of the Great Spurt under Nicholas II were purely because of the need to modernise the Russian Armed forces and that any other aspects of modernisation were unintended by-products. Candidates may argue that some rulers were at times forced into modernising policies because of adverse circumstances, eg the Crimean War or the Russo-Japanese War, or to ensure their regime’s survival. Candidates may argue that Nicholas II was not trying to modernise Russia when introducing reforms from 1905; he was simply trying to keep his throne. Some candidates may also challenge the motives of other rulers when modernising; for example they may argue that the maintenance of autocracy was a key driver in Alexander II’s decision to emancipate the serfs and that most of his other social reforms had to be implemented once the serfs were no longer under the jurisdiction of the landowners. Some candidates may argue that the short-lived Provisional

12

Page 354: History A - The Bicester Schoolthebicesterschool.org.uk/wp-content/uploads/2015/09/Mark... · 2015-09-28 · History A Mark Schemes for the Units January 2009 H106/H506/MS/R/09J

F966/02 Mark Scheme June 2011

Government may have intended to modernise Russia but fell because it singularly failed to do so.

Examiners must be open to alternative approaches. If in doubt, they should consult their

Team Leader. 13 Assess the view that the quality of leadership shown by African American

campaigners was the most important factor in the advancement of their civil rights in the period from 1865 to 1992. [60]

Candidates will probably endorse the view that leadership was decisive and focus their

attention on the inspirational, non-violent campaigns of Martin Luther King and his ability to win mass (including white) support, national media attention and international recognition. They will probably refer to the bus boycott in Montgomery, Alabama in 1956, his leadership of the SCLC, his campaign to desegregate Birmingham, Alabama in 1963 which influenced JFK to introduce the Civil Rights Bill, his speech at the Lincoln Memorial in the Washington March in 1963 and his Selma march in 1965 to pressure LBJ into persuading Congress to pass the Voting Rights Act.

Candidates may also refer to the value of the leadership of Booker T Washington in

championing African American economic rights and the importance of the Tuskegee Institute in providing education and training. Candidates may wish to contrast him with his critic, WEB du Bois, and outline the latter’s importance in asserting uncompromising opposition to black inferiority and his importance in helping to establish the NAACP. Better answers might analyse the crucial importance of the NAACP strategy of challenging segregation in the courts and explain the importance of the 1954 Brown decision. Candidates might refer to Marcus Garvey’s influence in helping African Americans to rediscover and take pride in their heritage. A. Philip Randolph’s campaign to end discrimination in federal employment in the 1940s and Malcolm X’s inspirational work with the Nation of Islam in the 1950s and early 1960s may also be mentioned. Some candidates may suggest that, after the death of Luther King in 1968, African-Americans lacked effective leadership, though they might discuss Jesse Jackson’s unsuccessful bids to win the Democratic presidential nomination in 1984 and 1988.

Better candidates should be aware of the need to evaluate the importance of leadership in

relation to other factors. They may be aware that leadership could only be effective when there were large numbers of followers willing to risk their livelihoods, and even their lives, by campaigning for civil rights. The best answers may point out that the absence of such grass-roots activism helps to explain why Booker T Washington or WEB du Bois could achieve less than Luther King, he had his leadership failures, for example at Albany in 1961-2. Candidates might also be aware that, to achieve legislative change, the Civil Rights campaign needed the support of the Federal government, pointing not only to the Civil Rights Act (1964) and the Voting Rights Act (1965) as evidence, but also observing that Civil Rights made less progress under Republican presidents Nixon, Reagan and Bush.

The best candidates will perhaps be aware that leadership was one of a number a crucial

factors necessary to the advancement of civil rights: an intellectual climate sympathetic to African-American aspirations, support from the three branches of the Federal government, a strong grass-roots movement and a clear, obtainable set of goals. They may be aware that these five factors came together only in the early 1960s. The absence of one, or more, of these factors explains the relative lack of progress of African-American civil rights in the years before 1960 and after 1965.

Examiners must be open to alternative approaches. If in doubt, they should consult their

Team Leader.

13

Page 355: History A - The Bicester Schoolthebicesterschool.org.uk/wp-content/uploads/2015/09/Mark... · 2015-09-28 · History A Mark Schemes for the Units January 2009 H106/H506/MS/R/09J

F966/02 Mark Scheme June 2011

14 ‘Gaining the vote in Federal elections in 1920 was the most important turning-point in the campaign for gender equality in the USA’. How far do you agree with this view of the period from 1865 to 1992? [60]

The 19th Amendment can be seen as a turning-point in allowing women full participation in

political life. Candidates might contrast 1865, when women could not vote, with the end of the period when, in 1984, Geraldine Ferraro was the vice-presidential Democratic candidate and there were significant (though small) numbers of women in Congress. They may refer to the “flappers” of the 1920s as evidence of a greater sense of liberation among women (from the middle class, at least) perhaps attributable to the achievement of the vote and point out that Frances Perkins became the first female member of the Cabinet as Secretary of Labor only 13 years after the 19th Amendment and that FDR’s New Deal legislation advanced a number of women’s causes. They might regard the 19th Amendment as evidence of the success of women’s activism and see it as inspiring later campaigns on other issues.

Some candidates will wish to challenge whether 1920 can be seen as a turning-point,

perhaps pointing out that the vote had been granted in nearly half (20/48) of the states before 1920. They may also observe that the 19th Amendment had no impact on black women in the Jim Crow south and that gaining the vote had been an objective of mainly middle class women. The better candidates may observe that the vote made no real difference to women’s other, arguably more important, concerns such as education, job opportunities, working conditions, and earnings and point out that, after 1920, opinion among women over the ERA become more polarised.

Candidates should compare 1920 with other potential turning-points. These might include

either of the two world wars which expanded women’s employment and earning power. Even if these benefits lasted only for the duration of the war, the economic expansion that occurred in each war widened women’s opportunities and horizons and, arguably, contributed to long-term change in social attitudes. Candidates might also consider the 1960s which saw the expansion of university education and welfare provision, two important legislative milestones (1963 Equal Pay Act, 1964 Civil Rights Act), the feminist movement spearheaded by Betty Friedan and the marketing of the contraceptive pill. Some candidates might argue for the Roe versus Wade Supreme Court decision on abortion as the turning-point which began the culture wars of the 1970s and 1980s and stimulated the women’s backlash against the ERA led by Phyllis Schlafly.

The best candidates will weigh up the relative merits of the various turning-points. They

may conclude that the relatively limited impact of the 19th Amendment disqualifies it as the most important turning-point. They might argue that the changes brought about by the wars were more significant because they affected a wider range of issues and people. Others will opt for the 1960s as the most significant because the changes were legislative, educational, economic and social. Furthermore, they were arguably more divisive than the changes brought about by other turning-points.

Examiners must be open to alternative approaches. If in doubt, they should consult their

Team Leader.

14

Page 356: History A - The Bicester Schoolthebicesterschool.org.uk/wp-content/uploads/2015/09/Mark... · 2015-09-28 · History A Mark Schemes for the Units January 2009 H106/H506/MS/R/09J

F966/02 Mark Scheme June 2011

15 ‘The policies of the Federal government failed to support the civil rights of Native Americans.’ To what extent do you agree with this view of the period from 1865 to 1992? [60]

Many candidates will wish to agree with this view and discuss the unsympathetic treatment

Native Americans have received from the Federal government. They might mention the period of the Indian Wars (1860s to 1890) when the aim of the Federal government was to destroy Native American opposition to white westward expansion, defeat them as a military threat and subjugate them onto reservations. They may also be familiar with the Dawes Severalty Act of 1887 which aimed to break up the reservations and turn Native Americans into self-sufficient farmers. Candidates may also know about the 1903 Supreme Court decision (Lone Wolf v. Hitchcock) that Congress could dispose of Indian land without gaining the consent of the Indians involved. In this period reformers and educators aimed to destroy Native American culture and separate identity by ‘civilising’ them – by converting them to Christianity and teaching their children in special schools, modelled on the Carlisle Indian School. Some will be aware that, although the 1924 Citizenship Act granted US citizenship to all Native Americans who had not already acquired it, Indians were denied the vote in many Western states by much the same methods as African Americans were disenfranchised in the South. Candidates will perhaps regard the policy known as ‘termination’ from 1948 to 1961 as another unsympathetic period when the Federal government sought to end Federal supervision of the reservations and to ‘liberate’ Native Americans by encouraging them to relocate to the cities and assimilate into mainstream US society. Candidates may well interpret this as another cynical government ploy to seize Native American land.

In challenging the view that the Federal government was consistently unsympathetic,

candidates may wish to discuss FDR’s Indian commissioner, John Collier, who ended the assimilation policy. The Indian Reorganisation Act of 1934 ended the policy of allotment, banned the further sale of Indian land and decreed that any unallotted land not yet sold should be returned to tribal control. It also granted Indian communities a measure of governmental and judicial autonomy. The IRA was important in arresting the loss of Indian resources and Collier successfully encouraged a renewed respect for Native American culture and traditions. Candidates may also wish to argue that, from the 1960s, Federal policy has been much more supportive of Native American rights. They might point to the impact of the Red Power movement in drawing national attention to the plight of Indians and the largely sympathetic response in a series of measures including the 1968 Indian Civil Rights Act, the 1972 Indian Education Act, the 1975 Indian Self-Determination and Education Assistance Act and the 1978 American Indian Religious Freedom Act. They may also be aware of the decisions of the Indian Claims Commission and the Supreme Court in granting compensation for lost Indian lands and the social, economic and social impact these have had on some reservations.

The best answers may well be able to point out that Federal policy is complicated by a lack

of clarity about what Native American rights should be and that there is no agreement among Native Americans about this either. They might point out that some Native Americans are only too anxious to assimilate, as the opposition of many to the IRA, their willingness to enlist (especially in World War Two) and the degree of support for termination demonstrates. Some candidates may point out that Federal policy has been consistently characterised by paternalism. Both the policy of Federal supervision of the reservations and the attempts to assimilate Native Americans are founded on the assumption that WASP culture is superior.

Examiners must be open to alternative approaches. If in doubt, they should consult their

Team Leader.

15

Page 357: History A - The Bicester Schoolthebicesterschool.org.uk/wp-content/uploads/2015/09/Mark... · 2015-09-28 · History A Mark Schemes for the Units January 2009 H106/H506/MS/R/09J

F966/02 Mark Scheme June 2011

16 ‘The decade from 1910 to 1920 was the most important turning point for the development of mass democracy.’ How far do you agree with this view of the period from 1868 to 1997? [60]

Candidates could point to the problem of defining mass democracy, certainly in the first

half of the period when it was only partially in place – a universal franchise, popular electoral methods and systems, a bi- cameral system (one house unelected throughout the period), recognition of mass numbers, gender issues and minority representation, regional control, class politics and a recognition of the role of pressure groups. There could be opposition to some, acceptance of others at various times. Thus by the 1880s all accepted the need to change electoral methods – ballot and restrictions on patronage. However property rights remained the basis of suffrage, Liberals and Conservatives alike fearing the franchise reaching the residuum. The argument for the importance of the 1910s is that before this only Socialists and Labour politicians uniformly argued for mass male suffrage, yet in 1918 the vote was given to all men over 21. On female suffrage the Conservatives, who had opposed up to 1916, converted, as did the Liberals and Labour, both formerly divided. Against the 1910s is that most still favoured the unequal situation on male/female voting that existed up to 1928. Only a rebalancing of the population and a perception that women could be conservative led to equality (1928) and a case could be made for the 1920s as being a more important decade for female suffrage. Reform of the Lords also occurred in 1911. Conservatives had always opposed this, whilst many Liberals were far from anxious to follow Gladstone over removal of the Lords’ veto from 1893 until the 1906 -11 experience. Since 1911 all parties have been reluctant to reform the power and composition of the Lords, content to see a gradual change from hereditary aristocrats to Life Peers up to 1997. Changes to the Commons (Payment of MPs and the move towards professional rather than amateur representation) were also accepted in the 1910s; Labour stood most to gain from such measures. Others may argue that mass democracy was a matter for the 1940s, thanks to the democratisation of war, rationing, educational change, health and the high turn-out involved in the 1945 election. A case could also be made for the 1960s and 1970s as the most important turning point if one’s criteria are youth, gender and race. Youth were included in the franchise in 1969, whilst minority rights (sexual and gender) had been opposed by all before the 1960s, Labour politicians being acutely aware of Union gender prejudice. They took the lead on a more inclusive approach to gay and female rights, backed by Liberals, in the 1960s and 1970s, although in some areas there was a Conservative backlash in the 1980s. There was also the debatable extent of Trade Union power, the 1980s seeing a concern to restrict what was seen as their excessive power. Candidates should attempt some overall judgement. For most it is likely that the 1910s were the crucial decade (franchise and institutional change). On wider and mass democratic issues the picture is more complex (the 1940s), although the 1960s and 1970s could be seen as crucial for minorities and women.

Examiners must be open to alternative approaches. If in doubt, they should consult their

Team Leader.

16

Page 358: History A - The Bicester Schoolthebicesterschool.org.uk/wp-content/uploads/2015/09/Mark... · 2015-09-28 · History A Mark Schemes for the Units January 2009 H106/H506/MS/R/09J

F966/02 Mark Scheme June 2011

17 How far did the role of the House of Commons change in the period from 1868 to 1997? [60]

In some respects candidates could argue that the Commons remained in its format much

the same throughout the period but there were significant changes to its composition as a result of reform, up to 1918, of the franchise redistribution, methods and patronage. In 1868 most MPs were landed and propertied. They were unpaid until 1911 and inevitably came from backgrounds that could afford to be a part time MP. Most came from rural areas or were men of traditional business, whether Tory or Liberal. From the 1880s business, banking, finance and the professions grew and, from 1900 Trade Union sponsored MPs, working men rising through Union administration or educated professionals also diluted an aristocratic and gentry based Commons. Throughout the 20th century MPs have been predominantly upper middle class, increasingly from a legal background or financial background and male. Women had a minor impact in the 1918-60 periods but then declined until 1997. In terms of its influence on law and government it could be argued that it first had to share power with the Lords until 1911, often seeing its legislation delayed or vetoed (1893; 1903-1910), but was also powerful in relation to governments who were reluctant to confront it (as Gladstone did over Home Rule in 1886). This remained the case, outside war time, until 1945, after which governments increasingly controlled timetables and business, whilst legislation increased and the powers of the parties came to restrict the more independent MPs of the 19th and early 20th centuries (the whip system). Nonetheless governments throughout the period were adept at seeking to control the Commons. The growth of government however gave it the opportunity to develop more ministerial office and thus the chance to offer promotion to party members. MPs also became more accountable to their constituency parties (rigorous panel interviews and selection committees) and, in the case of many Labour MPs, their Trade Union sponsors. MPs from the 1880s became ever more bound by their party manifestos and the need to campaign amongst a widening electorate. From 1945 they have stood for and won under a party label and discipline. In 1868 many seats remained uncontested but by 1918 almost all were contested, despite their un-winability. MPs could switch allegiance at the beginning of the period but by 1945 this could spell the end of a political career and even Churchill was frowned on in the interwar period for this. Much legislation in 1868 was initiated by individual MPs but this declined steadily throughout the period and had all but disappeared by 1997. There was little institutional reform in the period. A guillotine motion was introduced in the early 1880s to stop Irish obstructionism and parliamentary sessions became longer but in procedural terms little changed (a rebuilt Commons post 1945 still lacked seating for all and voting was still in person and done through the lobbies). Until 1995 Debates still started in the afternoon, although the Committee system developed to cope with the increasingly technical nature of legislation. However standing committees have an inbuilt government majority; only select committees can scrutinise governments effectively and their introduction in 1979 was a major reform. The work load is now much greater than before 1945 and debate since the 1880s has become ever more controlled. The issue of Commons corruption has remained a constant, from electoral scandals in the 1880s to the Cash for Questions case in 1994.

Examiners must be open to alternative approaches. If in doubt, they should consult their

Team Leader.

17

Page 359: History A - The Bicester Schoolthebicesterschool.org.uk/wp-content/uploads/2015/09/Mark... · 2015-09-28 · History A Mark Schemes for the Units January 2009 H106/H506/MS/R/09J

F966/02 Mark Scheme June 2011

18

18 ‘The mass media played the most important role in the development of democracy’. How far do you agree with this view of the period from 1868 to 1997? [60]

The argument that it was the most important factor was that its development preceded

many of the key developments such as party organisation and franchise reform. It could be argued that governments noted its impact and sought to exploit it politically or moved to include groups affected by it. Thus the emergence of a powerful provincial middle class press in the 1860s was evidence of political commitment, especially nonconformist and artisan, and franchise and electoral reform followed. The Times could ruin the career of Parnell in the late 1880s. However the more populist press of the 1890s, with its sensationalism, could be seen to have a contrary effect. Balfour and Lord Salisbury were scathing in their view of its readership and it helped to convince them that further democracy should be resisted. The mass media would appear to have little effect on Labour politics and indeed remained its enemy, with the exception of the Daily Herald and later the Daily Mirror until the 1990s. Governments could easily use the press to influence and control public opinion, most obviously during the World Wars, the General Strike in 1926, the Falklands War and the Troubles in Northern Ireland. However it could also hold government to account, as it did in the 1980s (Ponting) and over Suez. However by the 20th century it was rare for democratic change to be pushed by the press on anything major, like gender or minority issues, PR and reform of the Lords or Monarchy. It preferred a single, often personal issue – a government scalp or bureaucratic waste. Nonetheless the rise of the tabloid press in the 1960s was widely held by governments to be crucial during elections, the ‘Sun’ in particular. Press barons were considered important whether they be Harmsworth, Beaverbrook or Murdoch and the concentration of ownership in the hands of a few is questionable in a democratic sense. Cinema tended to be non-political, its newsreels largely conformist in both war and peace. The radio, and later TV, given its BBC origins in the early 1920s, was more consciously moulding of what it conceived to be British democratic values – fair play, educative in a highbrow sense, informative and grave, at least until the 1950s. Its peculiar Corporative position and monopoly was open to abuse, as in 1926 when Labour and the Archbishop of Canterbury were refused the chance to broadcast. However it was important in creating a sense that Britain upheld democracy in the 1930s and 1940s. In the 1960s and 1970s it and ITV could campaign for the underprivileged (Plays for Today) and it became a crucial medium for democratic politicians to master, although the broadcasting media arguably remained in a cosy relationship with the State and was criticised as such by some. It could also be said that by presenting news in a package it contributed, in the later 20th century, to less political activism than in the pre 1945 period. Nonetheless Channel 4 was established in the 1980s to cater for minority interests, arguably following democratic trends rather than initiating them. Both the BBC and ITC are required to maintain political impartiality. Candidates could argue that other factors were of greater importance, downplaying the media as an influence (it rarely took a mobilising or libertarian role and had always been subject to the tradition of public secrecy, reinforced by acts dating back to 1911) and stressing pressure groups, party competition, education, economic and social factors and war as the determinants in democratic change.

Examiners must be open to alternative approaches. If in doubt, they should consult their

Team Leaders.

Page 360: History A - The Bicester Schoolthebicesterschool.org.uk/wp-content/uploads/2015/09/Mark... · 2015-09-28 · History A Mark Schemes for the Units January 2009 H106/H506/MS/R/09J

OCR (Oxford Cambridge and RSA Examinations) 1 Hills Road Cambridge CB1 2EU OCR Customer Contact Centre 14 – 19 Qualifications (General) Telephone: 01223 553998 Facsimile: 01223 552627 Email: [email protected] www.ocr.org.uk For staff training purposes and as part of our quality assurance programme your call may be recorded or monitored

Oxford Cambridge and RSA Examinations is a Company Limited by Guarantee Registered in England Registered Office; 1 Hills Road, Cambridge, CB1 2EU Registered Company Number: 3484466 OCR is an exempt Charity OCR (Oxford Cambridge and RSA Examinations) Head office Telephone: 01223 552552 Facsimile: 01223 552553 © OCR 2011

Page 361: History A - The Bicester Schoolthebicesterschool.org.uk/wp-content/uploads/2015/09/Mark... · 2015-09-28 · History A Mark Schemes for the Units January 2009 H106/H506/MS/R/09J

Oxford Cambridge and RSA Examinations

GCE

History A

Unit F966/02: Historical Themes Option B: Modern 1789-1997

Advanced GCE

Mark Scheme for January 2012

Page 362: History A - The Bicester Schoolthebicesterschool.org.uk/wp-content/uploads/2015/09/Mark... · 2015-09-28 · History A Mark Schemes for the Units January 2009 H106/H506/MS/R/09J

OCR (Oxford Cambridge and RSA) is a leading UK awarding body, providing a wide range of qualifications to meet the needs of candidates of all ages and abilities. OCR qualifications include AS/A Levels, Diplomas, GCSEs, OCR Nationals, Functional Skills, Key Skills, Entry Level qualifications, NVQs and vocational qualifications in areas such as IT, business, languages, teaching/training, administration and secretarial skills. It is also responsible for developing new specifications to meet national requirements and the needs of students and teachers. OCR is a not-for-profit organisation; any surplus made is invested back into the establishment to help towards the development of qualifications and support, which keep pace with the changing needs of today’s society. This mark scheme is published as an aid to teachers and students, to indicate the requirements of the examination. It shows the basis on which marks were awarded by examiners. It does not indicate the details of the discussions which took place at an examiners’ meeting before marking commenced. All examiners are instructed that alternative correct answers and unexpected approaches in candidates’ scripts must be given marks that fairly reflect the relevant knowledge and skills demonstrated. Mark schemes should be read in conjunction with the published question papers and the report on the examination. OCR will not enter into any discussion or correspondence in connection with this mark scheme. © OCR 2012 Any enquiries about publications should be addressed to: OCR Publications PO Box 5050 Annesley NOTTINGHAM NG15 0DL Telephone: 0870 770 6622 Facsimile: 01223 552610 E-mail: [email protected]

Page 363: History A - The Bicester Schoolthebicesterschool.org.uk/wp-content/uploads/2015/09/Mark... · 2015-09-28 · History A Mark Schemes for the Units January 2009 H106/H506/MS/R/09J

F966/02 Mark Scheme January 2012 Subject-specific Marking Instructions

Distribution of marks for each level that reflects the Unit’s AOs 2 answers: Each maximum mark 60

A01a A01b IA 18-20 36-40

IB 16-17 32-35

II 14-15 28-31

III 12-13 24-27

IV 10-11 20-23

V 8-9 16-19

VI 4-7 8-15

VII 0-3 0-7

1

Page 364: History A - The Bicester Schoolthebicesterschool.org.uk/wp-content/uploads/2015/09/Mark... · 2015-09-28 · History A Mark Schemes for the Units January 2009 H106/H506/MS/R/09J

F966/02 Mark Scheme January 2012

Notes: (i) Allocate marks to the most appropriate level for each AO. (ii) If several marks are available in a box, work from the top mark down until the best fit has been found. (iii) Many answers will not fall at the same level for each AO. (iv) Candidates will demonstrate synoptic skills by drawing together appropriate techniques, knowledge and understanding to evaluate developments over the whole of the period

AOs AO1a AO1b

Total mark for each question = 60

Recall, select and deploy historical knowledge appropriately, and communicate knowledge and understanding of history in a clear and effective manner.

Demonstrate understanding of the past through explanation, analysis and arriving at substantiated judgements of: - key concepts such as causation, consequence, continuity, change

and significance within an historical context; - the relationships between key features and characteristics of the

periods studied Level IA

Uses a wide range of accurate and relevant evidence

Accurate and confident use of appropriate historical terminology

Answer is clearly structured and coherent; communicates accurately and legibly.

18-20

Excellent understanding of key concepts (eg continuity and change) relevant to analysis in their historical context

Excellent synthesis and synoptic assessment Answer is consistently and relevantly analytical with developed

explanations and supported judgements May make unexpected but substantiated connections over the whole

period 36-40

Level IB

Level IB Uses accurate and relevant evidence Accurate use of a range of appropriate historical

terminology Answer is clearly structured and mostly

coherent; communicates accurately and legibly 16-17

Very good level of understanding of key concepts (eg continuity and change) in their historical context.

Answer is consistently focused on the question set Very good level of explanation/analysis, and provides supported

judgements. Very good synthesis and synoptic assessment of the whole period

32-35

2

Page 365: History A - The Bicester Schoolthebicesterschool.org.uk/wp-content/uploads/2015/09/Mark... · 2015-09-28 · History A Mark Schemes for the Units January 2009 H106/H506/MS/R/09J

F966/02 Mark Scheme January 2012

Level II

Uses mostly accurate and relevant evidence Generally accurate use of historical terminology Answer is structured and mostly coherent;

writing is legible and communication is generally clear

14-15

Good level of understanding of key concepts (eg continuity and change) in their historical context

Good explanation/analysis but overall judgements may be uneven Answer is focused on the issues in the question set Good synthesis and assessment of developments over most of the

period 28-31

Level III Uses relevant evidence but there may be some inaccuracy

Answer includes relevant historical terminology but this may not be extensive or always accurately used

Most of the answer is structured and coherent; writing is legible and communication is generally clear

12-13

Shows a sound understanding of key concepts, especially continuity and change, in their historical context

Most of the answer is focused on the question set Answers may be a mixture of analysis and explanation but also

description and narrative, but there may also be some uneven overall judgements; OR answers may provide more consistent analysis but the quality will be uneven and its support often general or thin

Answer assesses relevant factors but provides only a limited synthesis of developments over most of the period

24-27 Level IV

There is deployment of relevant knowledge but level/accuracy will vary.

Some unclear and/or underdeveloped and/or disorganised sections

Mostly satisfactory level of communication

10-11

Satisfactory understanding of key concepts (eg continuity and change) in their historical context

Satisfactory focus on the question set Answer may be largely descriptive/narratives of events, and links

between this and analytical comments will typically be weak or unexplained

Makes limited synoptic judgements about developments over only part of the period

20-23

3

Page 366: History A - The Bicester Schoolthebicesterschool.org.uk/wp-content/uploads/2015/09/Mark... · 2015-09-28 · History A Mark Schemes for the Units January 2009 H106/H506/MS/R/09J

F966/02 Mark Sche 12

4

me January 20

General understanding of key concepts (eg continuity and change) in their historical context

Level V

General and basic historical knowledge but also some irrelevant and inaccurate material

Often unclear and disorganised sections Adequate level of communication but some weak

prose passages

8-9

Some understanding of the question but answers may focus on the topic and not address the question set OR provides an answer based on generalisation

Attempts an explanation but often general coupled with assertion, description/narrative

Very little synthesis or analysis and only part(s) of the period will be covered

16-19 Level VI Use of relevant evidence will be limited; there will

be much irrelevance and inaccuracy Answers may have little organisation or structure Weak use of English and poor organisation

4-7

Very little understanding of key concepts (eg continuity and change) in their historical context

Limited perhaps brief explanation Mainly assertion, description/narrative Some understanding of the topic but not the question’s requirements

8-15 Level VII Little relevant or accurate knowledge

Very fragmentary and disorganised response Very poor use of English and some incoherence

0-3

Weak understanding of key concepts (eg continuity and change) in their historical context

No explanation Assertion, description/narrative predominate Weak understanding of the topic or of the question’s requirements

0-7

Page 367: History A - The Bicester Schoolthebicesterschool.org.uk/wp-content/uploads/2015/09/Mark... · 2015-09-28 · History A Mark Schemes for the Units January 2009 H106/H506/MS/R/09J

F966/02 Mark Scheme January 2012

Question Answer Marks Guidance 1 Candidates should focus on the dominant intellectual ideas and the importance of

the role they played in the creation and development of German nationalism during this period. Candidates might discuss the origins and growth of German nationalism from 1789, the impact of romanticism on national ideology, the surge of German nationalism stimulated by the experience of Germans in the Napoleonic period and as a consequence of the impact on Germany of the Congress of Vienna. For example, candidates might discuss the Burschenschaft student organizations or popular demonstrations such as those held at Wartburg Castle in October 1817 and explain how these contributed to a growing sense of unity among the German speakers of Central Europe. Candidates are likely to show knowledge of developments in intellectual nationalism in the first half of the Nineteenth Century. Liberalism offered an intellectual basis for unification by challenging the status quo and absolutism; German liberals emphasised the linguistic and cultural unity of German peoples. Candidates are likely to demonstrate understanding of the debate about Grossdeutschland or Kleindeutschland in the period 1815 – 1871. Candidates may discuss the impact of the 1848/49 revolutions and the Frankfurt Parliament on German liberalism. Candidates are likely to discuss the reasons for the development of more radical nationalism in the remainder of the period and the reasons for the divergence between German liberals and other nationalists from 1870 in Imperial Germany. Candidates may discuss the development of mass-nationalism and its appeal in the late nineteenth and early twentieth centuries. Candidates must however show that they understand that intellectual forces were not the sole factors determining the creation and development of German nationalism in this period. Economic factors undeniably contributed to Prussia’s domination of Germany from 1866. Events also determined the development of German nationalism, for example the defeat of Austria in 1866. The impact of people should be explored. For example, Bismarck’s opportunistic and skilful leadership clearly had a significant impact on German nationalism as did the accession to the throne of Wilhelm II on the development of mass-nationalism.

60 Candidates are expected to demonstrate understanding of the issues in each of their selected questions over a period of at least a hundred years (unless an individual question specifies a slightly shorter period.) Candidates are reminded of the synoptic nature of the Unit. Answers are required to demonstrate understanding of the processes of historical continuity, development and change across the full breadth of the period studied. Assessors must be open to alternative approaches. If in doubt, they should consult their Team Leader.

5

Page 368: History A - The Bicester Schoolthebicesterschool.org.uk/wp-content/uploads/2015/09/Mark... · 2015-09-28 · History A Mark Schemes for the Units January 2009 H106/H506/MS/R/09J

F966/02 Mark Scheme January 2012

Question Answer Marks Guidance 2 Candidates will undoubtedly be more successful if they define ‘effective

management’ in their answer. Candidates might define the ways in which the three were (or were not) effective: for example in controlling, harnessing or using nationalism. Candidates who focus on the aims of the respective leaders, how each manipulated nationalism, how they dealt with crises and finally their legacy will be more likely to be able to make an effective judgement on their relative success. Clearly all three had different aims and different circumstances, which could enable candidates to make convincing cases for all of them. In discussing the case for William II candidates are likely to argue that his search for world power was undoubtedly populist, building on the development of radical nationalism. Arguably mass-nationalism distracted sufficient Germans from social, economic and political issues and represented the effective management of German nationalism to control the German people. However, this search for world power placed Germany in a vulnerable, dangerous position. The ultimate outcome of William II’s policies was defeat in the Great War and humiliation at Versailles. In discussing the case for Metternich candidates are likely to argue that because of his effective management of German nationalism by 1848/49 no leader of the nationalist movement with mass appeal had emerged. From 1815 to 1848 the nationalist movement was too weak to effectively challenge the Metternich System: arguably this demonstrates Metternich’s effective control over German nationalists. However Metternich fled Vienna in 1848, although his downfall owed little to German nationalism. Many candidates may argue in favour of Bismarck because of his critical role in the 1860s in the creation of the Second Reich; candidates may argue that he managed German nationalism by hijacking the nationalist cause for Prussia’s ends. The crucial role he played in the unification and development of Imperial Germany may well be considered to be effective management of German nationalism.

60 Candidates are expected to demonstrate understanding of the issues in each of their selected questions over a period of at least a hundred years (unless an individual question specifies a slightly shorter period.) Candidates are reminded of the synoptic nature of the Unit. Answers are required to demonstrate understanding of the processes of historical continuity, development and change across the full breadth of the period studied. Assessors must be open to alternative approaches. If in doubt, they should consult their Team Leader.

6

Page 369: History A - The Bicester Schoolthebicesterschool.org.uk/wp-content/uploads/2015/09/Mark... · 2015-09-28 · History A Mark Schemes for the Units January 2009 H106/H506/MS/R/09J

F966/02 Mark Scheme January 2012

Question Answer Marks Guidance 3 Candidates should argue both for and against this proposition. Candidates may

well argue that the German people became more united from 1866 and then after 1870 under the leadership of Prussia. They could certainly argue that the German Empire from 1871 physically united the majority of Germans. Candidates may also make the point that from 1866 the vast majority of German states were already united behind the leadership of Prussia. However, candidates may also understand that the Prussian Empire in 1871 represented Kleindeutschland and was an enlarged Prussia rather than a united Germany. They may argue that it was a Prussian Empire rather than a German Empire; it certainly did not unite all the German people even geographically. The exclusion of Austria from the process of German unification may be dealt with, though candidates may refer to Bismarck’s creation of the Dual Alliance as significant. Divisions within the German Nation after 1871 might be illustrated through the Kulturkampf and the rise of socialism or by the domination of the Reich by the elites. However, candidates may also argue that territorial boundaries rarely exactly match where the people of that nationality live and that divisions within a nation based on class or culture do not necessarily define the unity or otherwise of that nation. All modern nations have exhibited such divisions. Candidates may argue that the development of more radical nationalism in the late Nineteenth and early Twentieth Century demonstrates an emergence of greater national unity amongst the German people during this period. Candidates may argue that earlier in the period the German nation achieved a form of unity under the domination of Napoleon and through the creation of the Confederation of the Rhine. Germany was then certainly very divided from 1815 as a consequence of decisions taken at the Congress of Vienna, though it could also be argued that the German Confederation from 1815 loosely bound most Germans into a Confederation with a Diet. Candidates may argue that the First World War united the German nation at first but that divisions soon arose and were entrenched by 1918. Similarly, whilst Versailles divided the nation geographically it united the nation in condemnation and bitterness of the ‘diktat’.

60 Candidates are expected to demonstrate understanding of the issues in each of their selected questions over a period of at least a hundred years (unless an individual question specifies a slightly shorter period.) Candidates are reminded of the synoptic nature of the Unit. Answers are required to demonstrate understanding of the processes of historical continuity, development and change across the full breadth of the period studied. Assessors must be open to alternative approaches. If in doubt, they should consult their Team Leader.

7

Page 370: History A - The Bicester Schoolthebicesterschool.org.uk/wp-content/uploads/2015/09/Mark... · 2015-09-28 · History A Mark Schemes for the Units January 2009 H106/H506/MS/R/09J

F966/02 Mark Scheme January 2012

Question Answer Marks Guidance 4 Better responses might define ‘more important’; there are several possible ways

this might be done. Strategic as opposed to tactical advantage might be discussed, for example better trained and/or veteran soldiers tend to move faster strategically before the advent of the steam engine. Tactically ‘quality’ gave soldiers many advantages, ability to fire faster with muzzle loading gunpowder weapons, the use of small unit tactics, better use of ground and such like. Quality might produce higher morale allowing units to maintain cohesion and continue to fight for longer period of time.

Examples of ‘quality’ might be the French Grande Armée in the period 1805-07, the British army of the Napoleonic Wars, the French army in the 1859 Italian and of the Franco-Prussian Wars, many of the Confederate armies of the American Civil War, the BEF in both world wars, the German army in World War II until, probably, 1943. Virtually all of the armies of the period had ‘quality’ formations within their overall structure and candidates might use them as examples to support debate. Examples might be elite formations such as the French Imperial Guard in the Grande Armée of 1813 and 1814 or the armoured formations of the German army in the later part of World War II. Armies that relied on quantity to achieve success are very common in the period, for example the French armies of the early stages of Revolutionary Wars and the Russian armies of 1812, 1914-17 and 1941-45.

The question might also engage the changing nature of the composition of armies in the period with the greater use of poorly trained conscripts. Indeed, the question of balancing quality with quantity was an ongoing problem for military planners in the period. Better candidates might link the specific manpower issue with other parts of the specification, for example quantity gave greater advantage except when opposed by armies with superior weapons technology or leadership. The link with the manpower issue is, however, at the core of the question and links with these alternate factors must be explicit to gain full merit.

The question implies that ‘quantity’ is accompanied by lack of quality and we might assume that is how most candidates will approach the question. There is no reason why better candidates might not point to examples of combinations of quality and quantity creating advantage but such responses will be rare. An example might be the German army of 1914 which mixed reserve with regular formations in its order of battle.

60 Candidates are expected to demonstrate understanding of the issues in each of their selected questions over a period of at least a hundred years (unless an individual question specifies a slightly shorter period.) Candidates are reminded of the synoptic nature of the Unit. Answers are required to demonstrate understanding of the processes of historical continuity, development and change across the full breadth of the period studied. Assessors must be open to alternative approaches. If in doubt, they should consult their Team Leader.

8

Page 371: History A - The Bicester Schoolthebicesterschool.org.uk/wp-content/uploads/2015/09/Mark... · 2015-09-28 · History A Mark Schemes for the Units January 2009 H106/H506/MS/R/09J

F966/02 Mark Scheme January 2012

Question Answer Marks Guidance 5 The most obvious response expected is one that discusses technological change

in relation to military theory. At different points in the period technology caused significant changes in the nature of war to which some armies had difficulty adapting. Candidates might argue that in such circumstances armies fought using a theoretical model that was out of date making success in war more difficult. Candidates might discuss the ability of armies to modify their military theory faced with technological change and the speed and effectiveness that this was applied to military operations. Candidates might discuss the impact of technology on development of mass warfare, the application of the states’ economy to war and the development of total war in relation to military theory. Changes in the organisation of armies might also be used in the same way. In general terms examiners should expect candidates to agree with the question but once again they should be aware of possible counter arguments, for example the ability of British military theorists to adapt to changes in warfare prior to World War I or their German or Soviet counterparts’ development of the theory of mechanised warfare prior to World War II. Candidates will probably define military theory as the work of writers such as Jomini, Clausewitz, Ardent du Picq and Liddell Hart. They might also discuss military theory in the context of the military doctrines of the various armies of the period. The American Civil War and Russo-Japanese War are certainly useful to candidates as wars that did not meet the expectations of mainstream European theorists and did not prompt changes in theory in the years leading up to World War I.

60 Candidates are expected to demonstrate understanding of the issues in each of their selected questions over a period of at least a hundred years (unless an individual question specifies a slightly shorter period.) Candidates are reminded of the synoptic nature of the Unit. Answers are required to demonstrate understanding of the processes of historical continuity, development and change across the full breadth of the period studied. Assessors must be open to alternative approaches. If in doubt, they should consult their Team Leader.

9

Page 372: History A - The Bicester Schoolthebicesterschool.org.uk/wp-content/uploads/2015/09/Mark... · 2015-09-28 · History A Mark Schemes for the Units January 2009 H106/H506/MS/R/09J

F966/02 Mark Scheme January 2012

Question Answer Marks Guidance 6 Candidates might accept the premise of the question on the grounds that superior

organisational structures emerged in the course of the Napoleonic Wars in the form of, firstly, permanent divisions and, secondly, corps composed of multiple armies. They might also point to even larger formations such as the army groups used by Napoleon in Russia in 1812. These developments originated in the French army and by 1815 had been adopted by all major European powers. Candidates might also examine command and control systems – such as the French general staff – in relation to the control of these new organisational structures. This type of approach would argue that essentially the organisational form of European and North American armies had it roots firmly in the opening two decades of the period. An alternative turning point linked to the development of army corps might be the reform of the organisation of the Prussian army in the middle part of the nineteenth century. Candidates might argue for a turning point in the 20th century due to the sheer scale of warfare from World War I onwards. This argument is more convincing when applied to World War II than World War I. The armies of the latter war had organisational structures based on corps whereas the armies of World War II were certainly organised at levels higher than corps into army groups – in the case of the Allied and German armies – or Soviet fronts.

60 Candidates are expected to demonstrate understanding of the issues in each of their selected questions over a period of at least a hundred years (unless an individual question specifies a slightly shorter period.) Candidates are reminded of the synoptic nature of the Unit. Answers are required to demonstrate understanding of the processes of historical continuity, development and change across the full breadth of the period studied. Assessors must be open to alternative approaches. If in doubt, they should consult their Team Leader.

10

Page 373: History A - The Bicester Schoolthebicesterschool.org.uk/wp-content/uploads/2015/09/Mark... · 2015-09-28 · History A Mark Schemes for the Units January 2009 H106/H506/MS/R/09J

F966/02 Mark Scheme January 2012

Question Answer Marks Guidance 7 It is important for candidates to attempt to find patterns or a lack of pattern in the

nature of revolutionary nationalism. Candidates may wish to comment on the high and fervent idealism of revolutionary nationalism from Tone to the Young Ireland Movement and then on to the leaders of the Easter Rising, for example the role of blood sacrifice. On the other hand they may wish to comment on the changing inspiration of revolutionary nationalism from revolutionary France to later forms of revolutionary nationalism, drawn from cultural nationalism and socialism and ethnic and linguistic awareness, responding to, and inspired by, nationalist trends elsewhere. Here they might comment on the changing role of religion from Tone’s non-sectarianism to the more sectarian stance of later revolutionaries. Perhaps the pattern of taking advantage of serious crises in British or Irish affairs or of promises of foreign aid might be mentioned – the revolutionary wars of the 1790s and promises of French aid, the famine of the 1840s and the First World War and attempted German aid. On the other hand revolutionary tactics might be considered and assessed, such as the use of rebellion, conspiracy and secrecy. The organised risings of 1798 and 1916, for example, might be considered alongside the tactics of outrage, terror and assassination used, for example, by the Fenians in both Ireland and Britain, eg the Phoenix Park murders.

60 Candidates are expected to demonstrate understanding of the issues in each of their selected questions over a period of at least a hundred years (unless an individual question specifies a slightly shorter period.) Candidates are reminded of the synoptic nature of the Unit. Answers are required to demonstrate understanding of the processes of historical continuity, development and change across the full breadth of the period studied. Assessors must be open to alternative approaches. If in doubt, they should consult their Team Leader.

11

Page 374: History A - The Bicester Schoolthebicesterschool.org.uk/wp-content/uploads/2015/09/Mark... · 2015-09-28 · History A Mark Schemes for the Units January 2009 H106/H506/MS/R/09J

F966/02 Mark Scheme January 2012

Question Answer Marks Guidance 8 It is vital for candidates to engage with the concept of turning point to some

degree. Essays that agree with the proposition should be appropriately rewarded according to the degree and accuracy of the evidence provided to sustain the case and the quality and sophistication of analytical argument, though the expectation behind the question is that candidates will consider and assess the impact of Catholic Emancipation against the merits of other potential turning points during the period. There is much to support the notion – the final abolition of the ‘confessional state’ in both Britain and Ireland, the success of O’Connell’s movement encouraging further protest, the attempts by Whig and Tory governments to make further reforms after Catholic Emancipation, eg the reform of the Irish Church in the 1830s and the Maynooth Grant in the 1840s and perhaps on to other reforms in Gladstone’s first administration. On the other hand the strengths and weaknesses of the proposition might well be tested against other, possibly more plausible, turning points – for example, the impact of the First World War 1914, 1845-9, the First Home Rule bill and its defeat, the Easter Rising or the 1918 election.

60 Candidates are expected to demonstrate understanding of the issues in each of their selected questions over a period of at least a hundred years (unless an individual question specifies a slightly shorter period.) Candidates are reminded of the synoptic nature of the Unit. Answers are required to demonstrate understanding of the processes of historical continuity, development and change across the full breadth of the period studied. Assessors must be open to alternative approaches. If in doubt, they should consult their Team Leader.

12

Page 375: History A - The Bicester Schoolthebicesterschool.org.uk/wp-content/uploads/2015/09/Mark... · 2015-09-28 · History A Mark Schemes for the Units January 2009 H106/H506/MS/R/09J

F966/02 Mark Scheme January 2012

Question Answer Marks Guidance 9 It is likely that candidates will attempt to assess the harmful effects of the Union on

Ireland’s economic development, such as the ending of protection for Ireland’s trade and industry after the Union and the impact of free trade policies adopted by the Westminster government and parliament, particularly after 1841. They might also consider whether or not British and imperial interests disadvantaged Irish ones. On the other hand candidates might consider other factors that hindered Ireland’s economic development, such as Ireland’s rapidly increasing population before the Great Famine, the subdivision of tenancies, the impact of an aggressive and often absentee landlordism, the lack of the necessary natural resources in an age in which technology and economic progress was driven by coal, iron and steam and the impact of mass emigration during and after the famine. Answers might attempt a counter-argument showing that the economic impact of the Union was not uniformly negative, They might point to the industrial development of Ulster during the nineteenth century, particularly the growth of a mechanised, mass producing linen industry and ship-building in Belfast. On the other hand they might point out that this affected only part of Ulster and that the other provinces of Ireland, particularly in the West were untouched by large-scale industrialisation. Candidates might also assess the degree to which Ireland acquired a modern transport infrastructure with the building of canals and railways and the benefits accruing through Ireland’s access to British capital and financial institutions. They might also consider the benefits to the Irish economy of its ports being on the routes of imperial shipping lines. The economic impact of the telegraph and British garrison and naval bases might also be considered.

60 Candidates are expected to demonstrate understanding of the issues in each of their selected questions over a period of at least a hundred years (unless an individual question specifies a slightly shorter period.) Candidates are reminded of the synoptic nature of the Unit. Answers are required to demonstrate understanding of the processes of historical continuity, development and change across the full breadth of the period studied. Assessors must be open to alternative approaches. If in doubt, they should consult their Team Leader.

13

Page 376: History A - The Bicester Schoolthebicesterschool.org.uk/wp-content/uploads/2015/09/Mark... · 2015-09-28 · History A Mark Schemes for the Units January 2009 H106/H506/MS/R/09J

F966/02 Mark Scheme January 2012

Question Answer Marks Guidance 10 Candidates may well argue that Lenin was the most successful because he cut a

swath through the other parties that aspired to power in 1917 and successfully defended his revolution during the Civil War. He created the world’s first communist state and died with his party securely in power. Other parties were all banned, as were factions within the Communist Party. However, candidates must also consider whether other rulers dealt with opposition more successfully than Lenin did. Candidates who adopt a comparative approach and demonstrate synthesis throughout the essay are likely to be most successful. Most candidates are likely to concentrate their alternative arguments on Alexander III and Stalin when considering whether Lenin was the most successful ruler at dealing with opposition. Candidates may well see Alexander III and Stalin as more successful at dealing with opposition than either Alexander II (who faced a growing tide of opposition and was ultimately assassinated) or Nicholas II (under whom the Romanov dynasty ended) or Khrushchev (who was forced to retire by the Central Committee in 1964) or Prince Lvov/Kerensky who were swept aside in 1917. Candidates who choose to differentiate between dealing with opponents and dealing with the reasons for opposition may see Alexander II in a different light. They may wish to argue that the granting of concessions was a more successful way of dealing with opposition than ruthless repression. It can be argued that Alexander III’s imposition of ‘the reaction’ from 1881 bequeathed Nicholas II a revolution. Stalin defeated all of his rivals during the power struggle with consummate skill and exterminated real and imagined opponents with bloodcurdling efficiency for the next 25 years and his chilling terror may well lead candidates to argue that he, rather than Lenin, was the most successful ruler at dealing with opposition.

60 Candidates are expected to demonstrate understanding of the issues in each of their selected questions over a period of at least a hundred years (unless an individual question specifies a slightly shorter period.) Candidates are reminded of the synoptic nature of the Unit. Answers are required to demonstrate understanding of the processes of historical continuity, development and change across the full breadth of the period studied. Assessors must be open to alternative approaches. If in doubt, they should consult their Team Leader.

14

Page 377: History A - The Bicester Schoolthebicesterschool.org.uk/wp-content/uploads/2015/09/Mark... · 2015-09-28 · History A Mark Schemes for the Units January 2009 H106/H506/MS/R/09J

F966/02 Mark Scheme January 2012

Question Answer Marks Guidance 11 Candidates may argue that the development of Russian government was

influenced more by war than any other factor using a variety of evidence. The horrific impact of the First World War, both at the front and at home, sealed the fate of the Romanovs and, in turn, the Provisional Government in 1917. Arguably the appeal of the Bolsheviks in 1917 and the triumph of Lenin were directly related to the impact of the First World War. War can therefore be viewed as the prime cause of the end of autocratic government and the failure of the temporary move towards constitutional government. Candidates may argue that October 1917 and the triumph of Bolshevism crushed all possibility that a liberal democracy might emerge in Russia and transformed Russia into the Soviet Union – the world’s first communist state. Defeat in the Crimean War can be seen as the trigger for Alexander II’s programme of reform and the introduction of Zemstva as a new system of local government post-Emancipation. Similarly the Russo-Japanese War led to Nicholas II’s announcement of the October Manifesto and the formation of the Duma. In a pure sense, this was the abandonment of absolutism. Arguably, victory in the Second World War entrenched Stalin’s dictatorial power and had a brutal impact on the government of many of the outlying ‘republics’ of the USSR.

However, candidates may choose to argue that these developments in Russian Government had other causes. The impact of the First World War was not the only cause of either of the revolutions of 1917 for example. Candidates may choose to argue that the revolutions themselves were multi-causal and that they rather than war had the most important impact on the development of Russian government in this period. The personality of Nicholas II and the tactics of Lenin also played their part. Indeed, candidates are likely to argue that a variety of other people had a significant impact on developments of Russian Government, for example from Alexander II to Lenin, Stalin and Khrushchev. Some candidates may well consider that Stalin’s rise to power had a very important influence on Russian government. Candidates who argue this are likely to suggest that Stalin led Russia down a very different road than that being paved by Lenin. Other candidates may use a counter-argument based on more recent archival evidence to suggest that there was significant continuity between Lenin and Stalin. Candidates may argue that Khrushchev’s secret speech of 1956 and subsequent de-stalinisation had an important influence on the development of Russian government though the continuation of communism way beyond 1964 somewhat negates that view.

60 Candidates are expected to demonstrate understanding of the issues in each of their selected questions over a period of at least a hundred years (unless an individual question specifies a slightly shorter period.) Candidates are reminded of the synoptic nature of the Unit. Answers are required to demonstrate understanding of the processes of historical continuity, development and change across the full breadth of the period studied. Assessors must be open to alternative approaches. If in doubt, they should consult their Team Leader.

15

Page 378: History A - The Bicester Schoolthebicesterschool.org.uk/wp-content/uploads/2015/09/Mark... · 2015-09-28 · History A Mark Schemes for the Units January 2009 H106/H506/MS/R/09J

F966/02 Mark Scheme January 2012

Question Answer Marks Guidance What follows is not an exclusive list, but consideration could be given to the impact of key individuals (for example rulers/ministers) or key events (for example assassinations/revolutions/introduction of significant policy changes). Candidates must focus on how their chosen factors influenced the development of government. Economic reforms such as emancipation of the peasantry, collectivisation and five-year plans only become relevant when they are linked to political, administrative and ideological methods and changes in government. Candidates who discuss aspects of Russian government such as reform and repression, the fate of opposition, changes in ideology, the absence of democracy, the one party state and compare the relative influence of war and other factors on these developments are most likely to be successful. Examiners must not expect to find reference to all these aspects in candidates’ answers and candidates may select other factors in their answers.

16

Page 379: History A - The Bicester Schoolthebicesterschool.org.uk/wp-content/uploads/2015/09/Mark... · 2015-09-28 · History A Mark Schemes for the Units January 2009 H106/H506/MS/R/09J

F966/02 Mark Scheme January 2012

Question Answer Marks Guidance 12 Candidates are likely to focus on the main economic and social changes during the

period; weaker responses are likely to give particular attention to developments after 1917 whereas better responses should present a balanced assessment of the whole period. How candidates define ‘lost’ and ‘gained’ in their assessment and the variety of examples used to illustrate ‘Russian people’ could determine the quality of the essay. Some candidates may adopt a chronological approach, which will need frequent cross-referencing while others, who assess the effects of economic and social changes on different groups of Russians, are likely to produce a more effective synthesis. Most candidates are likely to discuss the emancipation of the serfs but better responses should assess the extent to which people were advantaged and disadvantaged by the changes from 1861 to 1917. The impact of industrial developments on urban and rural people, particularly resulting from Witte’s ‘Great Spurt’, may appear in some essays and some candidates may also consider the minority nationalities in the Russian Empire, most of whom endured consistent suffering for much of the period in question. References to War Communism, NEP, Five Year Plans, Collectivisation, Seven Year Plans and the Virgin Land policy may figure in most essays to underline the extent that Russians both gained and lost in the period from 1917 to 1964. Considerable emphasis is likely to be put on changes during Stalin’s regime and candidates may argue that any material gains were often at the expense of personal liberty. Some candidates will examine how far different social and economic groups benefited under the communists, perhaps assessing peasants, industrial and urban workers, merchants and landowners, and fluctuations in people’s standard of living and working conditions. Better responses might examine how far women gained after 1917, consider the way in which religious groups were affected, and discuss developments in education, particularly under Stalin and Khrushchev. The best essays are likely to suggest that some people gained and some people lost as a result of economic and social changes, and that beneficial experiences were not uniform and often short-lived. For example, the kulaks gained under Nicholas II and Lenin but lost a great deal under Stalin; and many city and urban workers gained materially during the 1930s but rural workers on the kolkhoz suffered intermittent famine and persistent hardship. Candidates are likely to conclude that while a minority of people ‘gained’ at some stage during the period, most Russians ‘lost’ rather more as a result of economic and social changes.

60 Candidates are expected to demonstrate understanding of the issues in each of their selected questions over a period of at least a hundred years (unless an individual question specifies a slightly shorter period.) Candidates are reminded of the synoptic nature of the Unit. Answers are required to demonstrate understanding of the processes of historical continuity, development and change across the full breadth of the period studied. Assessors must be open to alternative approaches. If in doubt, they should consult their Team Leader.

17

Page 380: History A - The Bicester Schoolthebicesterschool.org.uk/wp-content/uploads/2015/09/Mark... · 2015-09-28 · History A Mark Schemes for the Units January 2009 H106/H506/MS/R/09J

F966/02 Mark Scheme January 2012

Question Answer Marks Guidance 13 The weakest answers are likely to dismiss the notion that Booker T Washington

was the most important African American leader and concentrate on outlining the career and achievements of Martin Luther King. They are also likely to have little, if anything, to say about any African American leader after 1968. Modest answers will show awareness of Washington’s achievements and will probably refer to his slave background, his education at the Hampton Institute, and his drive and enterprise in establishing the Tuskagee Institute in 1861 and the Negro Business League in 1900. They will be aware of his relationship with Teddy Roosevelt and his importance in gaining the backing and support of influential white philanthropic entrepreneurs such as Andrew Carnegie and in providing African Americans with a sense of dignity, purpose and training in practical skills to maximise their economic and educational opportunities. They may well criticize his Atlanta Compromise of 1895 for accepting racial segregation, although better answers may be able to place this in context and argue that, given the circumstance of his time, compromising with the white establishment over political rights in order to promote African Americans’ participation in US capitalism perhaps made good sense. Candidates should consider the contributions made by some or all of the following: William Du Bois, Marcus Garvey, Malcolm X, Stokely Carmichael and Jesse Jackson, though they are not obliged to write about them all. Better answers will compare these leaders effectively to Washington, though weaker answers will merely outline their achievements and perhaps offer a comparative analysis only in the final paragraph. The best answers, although they are still likely to regard Martin Luther King as the most important African American leader, will argue comparatively throughout. They could point out that Washington’s personal sensitivity to criticism made him a less inspirational leader than Garvey, Martin Luther King, Malcolm X, Carmichael or Jackson, and that each of these was a more inspirational speaker than Washington. They could also argue that only by overturning Jim Crow could civil rights advance and thus Du Bois’ establishment of the NAACP to challenge discrimination in the courts was more significant. In Washington’s defence they

60 Candidates are expected to demonstrate understanding of the issues in each of their selected questions over a period of at least a hundred years (unless an individual question specifies a slightly shorter period.) Candidates are reminded of the synoptic nature of the Unit. Answers are required to demonstrate understanding of the processes of historical continuity, development and change across the full breadth of the period studied. Assessors must be open to alternative approaches. If in doubt, they should consult their Team Leader.

18

Page 381: History A - The Bicester Schoolthebicesterschool.org.uk/wp-content/uploads/2015/09/Mark... · 2015-09-28 · History A Mark Schemes for the Units January 2009 H106/H506/MS/R/09J

F966/02 Mark Scheme January 2012

Question Answer Marks Guidance may argue that his organisational skills and his ability to appeal to white opinion

rivalled that of King, that his Atlanta Compromise rivalled that of King, and that his career coincided with the establishment of the Jim Crow regime in the South (reinforced by the 1896 Supreme Court decision in Plessy v Ferguson) and the dominance of Social Darwinist views about racial hierarchy which made a challenge to segregation unrealistic, whereas King led a highly-motivated African American protest movement with widespread support, media attention and sympathetic Supreme Court judgments at a time when the Cold War made legalised racism difficult to maintain. They might suggest that Washington’s long-term significance exceeds that of Garvey, Malcolm X, Carmichael and Jackson as these men alienated white support (and some black) as much as they galvanised followers.

19

Page 382: History A - The Bicester Schoolthebicesterschool.org.uk/wp-content/uploads/2015/09/Mark... · 2015-09-28 · History A Mark Schemes for the Units January 2009 H106/H506/MS/R/09J

F966/02 Mark Scheme January 2012

Question Answer Marks Guidance 14 Most candidates will endorse this view and point out that the New Deal’s alphabet

agencies aimed to get the unemployed back to work and that FDR’s administration gave trade unions and workers the support of the Federal government for the first time, referring to the NIRA of 1933, the Wagner Act of 1935 and the Fair Labor Standards Act of 1938. They may also be aware that trade union membership tripled between 1933 and 1939 and that FDR and most state officials refused to intervene on the side of management. Some will point out that these gains were sustained during the Second World War when an unprecedented expansion of American industry to meet the demands of war production gave considerable bargaining power to workers. Better answers will show awareness that attitudes to workers’ rights were not completely transformed by the New Deal – the NIRA was declared unconstitutional in 1935, there were serious, and sometimes violent, industrial disputes in 1934 and 1937 and some major employers (such as Ford) resisted recognising unions until the war. Candidates should contrast the New Deal with some or all of the other potential turning points of the late 19th and early 20th centuries. These include the Haymarket bombing of 1886, the Homestead steel strike in 1892 and the treatment of Coxey’s ‘Army’ and the Pullman strike in 1894. Candidates may point out that, in contrast to the New Deal, trade unions in this period faced legal obstacles and government hostility to strike action. The 1890 Sherman Anti-Trust Act was used to gain Federal injunctions against strikers and Supreme Court rulings that unions were ‘illegal combinations’. On several occasions troops were deployed to break strikes and most presidents took a tough line. Some candidates might contrast the advances of the New Deal with the largely negative 1920s when, despite the expansion of industry, racial discrimination continued and trade unions made little, if any, progress. Major strikes were unsuccessful and strikers faced the full might of the law and a hostile federal government. A variety of post-war turning points might be offered. Candidates might be aware that in the immediate post-war period the New Deal gains were, to some degree, clawed back by Congress by the 1947 Taft-Hartley Act and the 1959 Landrum-Griffin Act. Despite the AFL-CIO merger in 1955, structural changes in the US economy (the decline of heavy industry and blue collar work) gradually eroded

60 Candidates are expected to demonstrate understanding of the issues in each of their selected questions over a period of at least a hundred years (unless an individual question specifies a slightly shorter period.) Candidates are reminded of the synoptic nature of the Unit. Answers are required to demonstrate understanding of the processes of historical continuity, development and change across the full breadth of the period studied. Assessors must be open to alternative approaches. If in doubt, they should consult their Team Leader.

20

Page 383: History A - The Bicester Schoolthebicesterschool.org.uk/wp-content/uploads/2015/09/Mark... · 2015-09-28 · History A Mark Schemes for the Units January 2009 H106/H506/MS/R/09J

F966/02 Mark Scheme January 2012

Question Answer Marks Guidance union power. The legislation of the New Frontier and Great Society programmes of

the 1960s might be offered as a positive turning point for workers. Most candidates will refer to the defeat of the PATCO strike in 1981 as a major negative turning point for unions since it led to a decline in membership and in the number of strikes and signalled the hostility of the Federal government to organised labour. Better answers will place this in the context of changes in the US economy with the growth of the service sector and non-unionised, low-paid, part-time and immigrant labour. Weaker answers will probably describe their selected turning points in sequence and perhaps confine their comparative evaluation to a concluding paragraph. Better answers will argue comparatively throughout.

21

Page 384: History A - The Bicester Schoolthebicesterschool.org.uk/wp-content/uploads/2015/09/Mark... · 2015-09-28 · History A Mark Schemes for the Units January 2009 H106/H506/MS/R/09J

F966/02 Mark Scheme January 2012

Question Answer Marks Guidance 15 Weaker answers will probably adopt a chronological approach and outline the

gains made by women, confining their analytical comments to a concluding paragraph. Better answers will analyse a variety of significant themes and the strongest may challenge, at least in part, the assumption in the question and argue that, for many women, equality remains elusive or that progress began well before 1941. The best answers might be expected to show how the factors discussed inter-relate and analyse their relative importance. Many candidates, taking their cue from the question, are likely to discuss the impact of the Second World War. Women’s opportunities and horizons were expanded by joining the armed forces, working in war industries and earning higher wages. These wartime experiences challenged many assumptions about women’s role in society, though for most women, these gains were temporary. Good answers might regard long-term changes to the US economy as more significant. They will refer to the expansion of employment opportunities for women in the post-war period, especially in middle-class careers such as medicine, law, the civil service and politics, perhaps pointing out that some of these trends had already begun in the inter-war years. Stronger answers might argue that for black, Hispanic and white working class women the situation did not change dramatically in the post-war period since such women remained in low-paid, part-time, non-unionised jobs. The expansion of education, especially at university level, in the post-war period gave women the qualifications to aspire to middle-class careers and the intellectual assurance to challenge long-held assumptions about their role in society. Many candidates will be aware how both male and female attitudes to women’s roles changed from the ‘separate spheres’ to the presumption of equality of opportunity. Some might successfully make a case for the importance of changes to family size, the widespread availability and acceptability of birth control techniques (especially the development of ‘the pill’ in the 1960s), the increased rate of divorce and the importance of the Roe versus Wade Supreme Court judgment in 1973 in accelerating gender equality. Better answers will be able to show that most of these developments pre-date 1941 and will be able to explain why these trends developed more quickly after 1945 and especially from the 1960s.

60 Candidates are expected to demonstrate understanding of the issues in each of their selected questions over a period of at least a hundred years (unless an individual question specifies a slightly shorter period.) Candidates are reminded of the synoptic nature of the Unit. Answers are required to demonstrate understanding of the processes of historical continuity, development and change across the full breadth of the period studied. Assessors must be open to alternative approaches. If in doubt, they should consult their Team Leader.

22

Page 385: History A - The Bicester Schoolthebicesterschool.org.uk/wp-content/uploads/2015/09/Mark... · 2015-09-28 · History A Mark Schemes for the Units January 2009 H106/H506/MS/R/09J

F966/02 Mark Scheme January 2012

Question Answer Marks Guidance Candidates may well analyse the role of the Federal government, referring to the

Equal Pay Act of 1963 and the Civil Rights Act of 1964 although, again, better candidates will be aware of the New Deal legislation that assisted women. Some candidates might suggest that women’s activism became more widespread and effective in the post-war period, referring to Betty Friedman and NOW as well as the campaign for the Equal Rights Amendment. Better answers will show awareness that women’s campaigning alienated many, and will also point out that women had campaigned for the vote and over prohibition in the pre-war period. They might suggest that extensive media coverage and greater awareness of minority rights as a result of the black civil rights movement helped the effectiveness of women’s campaigning.

23

Page 386: History A - The Bicester Schoolthebicesterschool.org.uk/wp-content/uploads/2015/09/Mark... · 2015-09-28 · History A Mark Schemes for the Units January 2009 H106/H506/MS/R/09J

F966/02 Mark Scheme January 2012

Question Answer Marks Guidance 16 Candidates should pick up on the phrase ‘most important’ by showing some

degree of ability in assessing the strengths and weaknesses of the validity of the proposition. Weaker answers might consider just one side of the question either by agreeing with the proposition entirely or completely disagreeing with it. Candidates might consider the ideology of the Labour Party and emphasise that, with the successive extensions of the male franchise from 1867 and female franchise from 1918, there could only be representative democracy in Britain during the twentieth century if the working class was represented by a party devoted to their interests. On the other hand some might take the view that with its support from much of the Trades Union Movement, its commitment to public ownership through Clause IV of its constitution to 1994 and its socialist ideology and foundation upon class divisions, its democratic credentials were debateable. They might consider the changes to the electoral system undertaken by various Labour governments, including the first use of referendums and assess whether these had contributed significantly to the development of mass democracy. On the other hand it is possible to question the party’s achievements eg the failure to reform the second chamber until after 1997. Candidates might wish to compare the achievements of other parties in the development of democracy in Britain – the Conservatives’ role in the passing of the Second Reform Act before the 1868 election, the changes to the franchise in 1928; the Liberals’ part in extending the franchise in the Third Reform Act and the constitutional and electoral changes of 1911. They might point out that female suffrage was first achieved by coalition government. They might question whether mass democracy was already well developed before the establishment of the party or, at least, before it became a party of government in 1924.

60 Candidates are expected to demonstrate understanding of the issues in each of their selected questions over a period of at least a hundred years (unless an individual question specifies a slightly shorter period.) Candidates are reminded of the synoptic nature of the Unit. Answers are required to demonstrate understanding of the processes of historical continuity, development and change across the full breadth of the period studied. Assessors must be open to alternative approaches. If in doubt, they should consult their Team Leader.

24

Page 387: History A - The Bicester Schoolthebicesterschool.org.uk/wp-content/uploads/2015/09/Mark... · 2015-09-28 · History A Mark Schemes for the Units January 2009 H106/H506/MS/R/09J

F966/02 Mark Scheme January 2012

Question Answer Marks Guidance 17 This question is designed to test whether regional politics had a significant impact

on party fortunes. Candidates are expected to weigh up other factors of electoral success against regional politics and assess whether larger issues were always more important than regional politics, power bases and patterns of support. For example, candidates might consider whether the national, and, at times, imperial, issues of Unionism in the 1880s, tariff reform in 1906, the desire for welfare reform in 1945 to implement the Beveridge Report or the hopes of New Labour and ‘a Third Way’ in 1997 were more important than regional patterns of support and politics in securing electoral victories for the successful parties. They might ponder whether regional politics and power bases were more important in ensuring the survival, rather than the success, of parties at times of electoral defeat. For example, they might assess whether the Liberal party simply survived on its ‘Celtic fringes’ after it fell from being a party of government from 1922 until the 1970s and 1980s or whether Labour survived the traumas of Thatcherism and the emergence of a Social Democrat party through its strengths and solid base in Scotland, Wales and its old industrial bases in England. Candidates might consider the patterns of support in the regions for the parties when at their most successful as well as when they hit their nadirs. They might explain the patterns of Conservative support and whether it has always been essentially an ‘English’ party, particularly of rural constituencies and leafy suburbs, and if so, account for seeming exceptions; for example, ‘Tory Lancashire’, Macmillan’s success in winning Stockton-on-Tees in the 1920s and Thatcher’s electoral dominance 1979-87.

60 Candidates are expected to demonstrate understanding of the issues in each of their selected questions over a period of at least a hundred years (unless an individual question specifies a slightly shorter period.) Candidates are reminded of the synoptic nature of the Unit. Answers are required to demonstrate understanding of the processes of historical continuity, development and change across the full breadth of the period studied. Assessors must be open to alternative approaches. If in doubt, they should consult their Team Leader.

25

Page 388: History A - The Bicester Schoolthebicesterschool.org.uk/wp-content/uploads/2015/09/Mark... · 2015-09-28 · History A Mark Schemes for the Units January 2009 H106/H506/MS/R/09J

F966/02 Mark Scheme January 2012

26

Question Answer Marks Guidance 18 Candidates should recognise that cabinet government adjusted itself and

developed through contingent circumstances such as the character, ambitions, programmes and style of the prime minister and his/her power within the party and at the polls. These points should be supported by a range of examples over a hundred year period of prime ministers shaping the development of their respective cabinets, though examiners should not demand exhaustive and extended illustration beyond what can be reasonably expected given the time constraints upon candidates. Perhaps candidates will try and counter the proposition by pointing to the influence of other powerful personalities within the cabinet, apart from the prime minister, again using a range of examples during the period, eg Joseph Chamberlain, Lloyd George before 1916 etc. Other candidates might recognise that other contingent factors have shaped the character of cabinet, above all the issues and needs of the times (for example, economic crisis, coalition, war). Candidates might consider the degree to which the principles of cabinet government were already established by 1868, for example collective responsibility, as well as developments since 1868, and that these precedents constrained prime ministers to act within a determined framework, despite the extensive prerogative powers held by a prime minister. They might explore the idea that cabinet government developed through the unintentional impact of prime ministerial behaviour, eg the impact on later cabinets of Macmillan’s ‘night of the long knives’ and of Thatcher’s fall in 1990. They might wish to comment on the size of cabinets and look for trends, commenting on factors that caused it to change, eg the growing complexity of government in response to social and economic change and the growth of government intervention, the growth of a welfare state and the proliferation of ministries and expansion of the Civil Service and the degree to which these trends were independent of prime ministers; that the cabinet was driven by the times rather than by individuals. They may also comment on the use of ‘inner cabinets’, the adoption of more presidential styles of government before Blair, again in the light of prime ministerial action and trends within modern political life.

60 Candidates are expected to demonstrate understanding of the issues in each of their selected questions over a period of at least a hundred years (unless an individual question specifies a slightly shorter period.) Candidates are reminded of the synoptic nature of the Unit. Answers are required to demonstrate understanding of the processes of historical continuity, development and change across the full breadth of the period studied. Assessors must be open to alternative approaches. If in doubt, they should consult their Team Leader.

Page 389: History A - The Bicester Schoolthebicesterschool.org.uk/wp-content/uploads/2015/09/Mark... · 2015-09-28 · History A Mark Schemes for the Units January 2009 H106/H506/MS/R/09J

OCR (Oxford Cambridge and RSA Examinations) 1 Hills Road Cambridge CB1 2EU OCR Customer Contact Centre Education and Learning Telephone: 01223 553998 Facsimile: 01223 552627 Email: [email protected] www.ocr.org.uk For staff training purposes and as part of our quality assurance programme your call may be recorded or monitored

Oxford Cambridge and RSA Examinations is a Company Limited by Guarantee Registered in England Registered Office; 1 Hills Road, Cambridge, CB1 2EU Registered Company Number: 3484466 OCR is an exempt Charity OCR (Oxford Cambridge and RSA Examinations) Head office Telephone: 01223 552552 Facsimile: 01223 552553 © OCR 2012

Page 390: History A - The Bicester Schoolthebicesterschool.org.uk/wp-content/uploads/2015/09/Mark... · 2015-09-28 · History A Mark Schemes for the Units January 2009 H106/H506/MS/R/09J

GCE

Oxford Cambridge and RSA Examinations

Unit F966/02: Historical Themes Option B: Modern 1789-1997

Advanced GCE

History A

Mark Scheme for June 2012

Page 391: History A - The Bicester Schoolthebicesterschool.org.uk/wp-content/uploads/2015/09/Mark... · 2015-09-28 · History A Mark Schemes for the Units January 2009 H106/H506/MS/R/09J

OCR (Oxford Cambridge and RSA) is a leading UK awarding body, providing a wide range of qualifications to meet the needs of candidates of all ages and abilities. OCR qualifications include AS/A Levels, Diplomas, GCSEs, OCR Nationals, Functional Skills, Key Skills, Entry Level qualifications, NVQs and vocational qualifications in areas such as IT, business, languages, teaching/training, administration and secretarial skills. It is also responsible for developing new specifications to meet national requirements and the needs of students and teachers. OCR is a not-for-profit organisation; any surplus made is invested back into the establishment to help towards the development of qualifications and support, which keep pace with the changing needs of today’s society. This mark scheme is published as an aid to teachers and students, to indicate the requirements of the examination. It shows the basis on which marks were awarded by examiners. It does not indicate the details of the discussions which took place at an examiners’ meeting before marking commenced. All examiners are instructed that alternative correct answers and unexpected approaches in candidates’ scripts must be given marks that fairly reflect the relevant knowledge and skills demonstrated. Mark schemes should be read in conjunction with the published question papers and the report on the examination. OCR will not enter into any discussion or correspondence in connection with this mark scheme. © OCR 2012 Any enquiries about publications should be addressed to: OCR Publications PO Box 5050 Annesley NOTTINGHAM NG15 0DL Telephone: 0870 770 6622 Facsimile: 01223 552610 E-mail: [email protected]

Page 392: History A - The Bicester Schoolthebicesterschool.org.uk/wp-content/uploads/2015/09/Mark... · 2015-09-28 · History A Mark Schemes for the Units January 2009 H106/H506/MS/R/09J

F966/02 Mark Scheme June 2012 Subject-specific Marking Instructions

Distribution of marks for each level that reflects the Unit’s AOs 2 answers: Each maximum mark 60

A01a A01b IA 18-20 36-40

IB 16-17 32-35

II 14-15 28-31

III 12-13 24-27

IV 10-11 20-23

V 8-9 16-19

VI 4-7 8-15

VII 0-3 0-7

1

Page 393: History A - The Bicester Schoolthebicesterschool.org.uk/wp-content/uploads/2015/09/Mark... · 2015-09-28 · History A Mark Schemes for the Units January 2009 H106/H506/MS/R/09J

F966/02 Mark Scheme June 2012 Notes: (i) Allocate marks to the most appropriate level for each AO. (ii) If several marks are available in a box, work from the top mark down until the best fit has been found. (iii) Many answers will not fall at the same level for each AO. (iv) Candidates will demonstrate synoptic skills by drawing together appropriate techniques, knowledge and understanding to evaluate developments over the whole of the period

AOs AO1a AO1b

Total mark for each question = 60

Recall, select and deploy historical knowledge appropriately, and communicate knowledge and understanding of history in a clear and effective manner.

Demonstrate understanding of the past through explanation, analysis and arriving at substantiated judgements of: - key concepts such as causation, consequence, continuity, change

and significance within an historical context; - the relationships between key features and characteristics of the

periods studied Level IA

Uses a wide range of accurate and relevant evidence

Accurate and confident use of appropriate historical terminology

Answer is clearly structured and coherent; communicates accurately and legibly.

18-20

Excellent understanding of key concepts (eg continuity and change) relevant to analysis in their historical context

Excellent synthesis and synoptic assessment Answer is consistently and relevantly analytical with developed

explanations and supported judgements May make unexpected but substantiated connections over the whole

period 36-40

Level IB

Level IB Uses accurate and relevant evidence Accurate use of a range of appropriate historical

terminology Answer is clearly structured and mostly

coherent; communicates accurately and legibly 16-17

Very good level of understanding of key concepts (eg continuity and change) in their historical context.

Answer is consistently focused on the question set Very good level of explanation/analysis, and provides supported

judgements. Very good synthesis and synoptic assessment of the whole period

32-35

2

Page 394: History A - The Bicester Schoolthebicesterschool.org.uk/wp-content/uploads/2015/09/Mark... · 2015-09-28 · History A Mark Schemes for the Units January 2009 H106/H506/MS/R/09J

F966/02 Mark Scheme June 2012

AOs AO1a AO1b

Level II

Uses mostly accurate and relevant evidence Generally accurate use of historical terminology Answer is structured and mostly coherent;

writing is legible and communication is generally clear

14-15

Good level of understanding of key concepts (eg continuity and change) in their historical context

Good explanation/analysis but overall judgements may be uneven Answer is focused on the issues in the question set Good synthesis and assessment of developments over most of the

period 28-31

Level III

Uses relevant evidence but there may be some inaccuracy

Answer includes relevant historical terminology but this may not be extensive or always accurately used

Most of the answer is structured and coherent; writing is legible and communication is generally clear

12-13

Shows a sound understanding of key concepts, especially continuity and change, in their historical context

Most of the answer is focused on the question set Answers may be a mixture of analysis and explanation but also

description and narrative, but there may also be some uneven overall judgements; OR answers may provide more consistent analysis but the quality will be uneven and its support often general or thin

Answer assesses relevant factors but provides only a limited synthesis of developments over most of the period

24-27 Level IV

There is deployment of relevant knowledge but level/accuracy will vary.

Some unclear and/or underdeveloped and/or disorganised sections

Mostly satisfactory level of communication

10-11

Satisfactory understanding of key concepts (eg continuity and change) in their historical context

Satisfactory focus on the question set Answer may be largely descriptive/narratives of events, and links

between this and analytical comments will typically be weak or unexplained

Makes limited synoptic judgements about developments over only part of the period

20-23

3

Page 395: History A - The Bicester Schoolthebicesterschool.org.uk/wp-content/uploads/2015/09/Mark... · 2015-09-28 · History A Mark Schemes for the Units January 2009 H106/H506/MS/R/09J

F966/02 Mark Scheme June 2012

4

AOs AO1a AO1b

Level V

General and basic historical knowledge but also some irrelevant and inaccurate material

Often unclear and disorganised sections Adequate level of communication but some weak

prose passages

8-9

General understanding of key concepts (eg continuity and change) in their historical context

Some understanding of the question but answers may focus on the topic and not address the question set OR provides an answer based on generalisation

Attempts an explanation but often general coupled with assertion, description/narrative

Very little synthesis or analysis and only part(s) of the period will be covered

16-19 Level VI Use of relevant evidence will be limited; there will

be much irrelevance and inaccuracy Answers may have little organisation or structure Weak use of English and poor organisation

4-7

Very little understanding of key concepts (eg continuity and change) in their historical context Limited perhaps brief explanation Mainly assertion, description/narrative Some understanding of the topic but not the question’s requirements

8-15 Level VII Little relevant or accurate knowledge

Very fragmentary and disorganised response Very poor use of English and some incoherence

0-3

Weak understanding of key concepts (eg continuity and change) in their historical context No explanation Assertion, description/narrative predominate Weak understanding of the topic or of the question’s requirements

0-7

Page 396: History A - The Bicester Schoolthebicesterschool.org.uk/wp-content/uploads/2015/09/Mark... · 2015-09-28 · History A Mark Schemes for the Units January 2009 H106/H506/MS/R/09J

F966/02 Mark Scheme June 2012

Question Answer Marks Guidance 1 Candidates should focus on the dominant ideas and aims of German nationalists,

from the aims of the growing emergent nationalist movement from 1789 to the aims of more radical nationalists in the late nineteenth and early twentieth Centuries. Arguably the core aim of German nationalists, the creation and development of a united German nation, remained the same throughout this period. However, different strands within the nationalists had differing aims. Candidates might well demonstrate understanding of the debate about Grossdeutschland or Kleindeutschland in the period 1815 – 1871 and the reasons for the development of more radical nationalism in the remainder of the period. Candidates may argue that the aims of nationalists were changed by the impact of events. Candidates may argue that the common fight of people from different German states against the French, especially in 1813, gave strong impulses to nationalism. A few intellectuals consequently demanded the unification of all German-speaking lands, although they represented a minority. Candidates are likely to show knowledge of developments in intellectual nationalism in the first half of the Nineteenth Century. Candidates might explain the importance of economic developments on the changing aims of German nationalism, for example the impact of the Zollverein after 1834. The impact of the foundation of the Second Reich from 1871 clearly had a profound effect on the development of nationalism during the latter part of this period. Candidates are likely to discuss the reasons for the development of more radical nationalism in the remainder of the period and the reasons for the divergence between German liberals and other nationalists from 1870 in Imperial Germany. The impact of people on the aims of nationalism may also be explored. For example, Bismarck’s opportunistic and skilful leadership clearly had a significant impact on the development of German nationalism as did the accession to the throne of Wilhelm II. The change of heart from 1866 when liberals became national liberals may well be stressed by some candidates. Candidates may argue that the Great War left Germany broken and half-starved with the aims of German nationalists in tatters.

60 Candidates are expected to demonstrate understanding of the issues in each of their selected questions over a period of at least a hundred years (unless an individual question specifies a slightly shorter period.) Candidates are reminded of the synoptic nature of the Unit. Answers are required to demonstrate understanding of the processes of historical continuity, development and change across the full breadth of the period studied. Assessors must be open to alternative approaches. If in doubt, they should consult their Team Leader.

5

Page 397: History A - The Bicester Schoolthebicesterschool.org.uk/wp-content/uploads/2015/09/Mark... · 2015-09-28 · History A Mark Schemes for the Units January 2009 H106/H506/MS/R/09J

F966/02 Mark Scheme June 2012

Question Answer Marks Guidance 2 Candidates may well argue that the development of trade and transport through

industrialisation played a key role in the unification of Germany. Candidates are likely to assess key economic factors, such as trade, industry and communications, and evaluate their impact on the development of German nationalism. Candidates are likely to understand that the impact of the Napoleonic period was ruinous to the early developments of the industrial economy and that this placed severe constraints on the practical ambitions of German nationalists. Candidates are likely to explain the impact of industrialisation in the early nineteenth century on the development of German nationalism, for example the impact of the Zollverein after 1834 in developing Prussia’s economic strength and, consequently, Prussian leadership of Germany. This also had a limiting effect on the development of German nationalism as Prussia was able to exclude Austria, first from the Zollverein and then from Germany. This led to the creation of Kleindeutschland, thus thwarting the ambitions of those nationalists who aspired to Grossdeutschland. Candidates should understand how developments in the economy in the 1850s paved the way for the Prussian military victories of 1864, 1866 and 1870/71 and the creation of the Second Reich. Military strength depended upon industrialisation: ‘Coal & Iron’ rather than ‘Blood & Iron’ could be usefully debated. The impact of the extraordinary developments in the German economy after 1871 should be discussed. Candidates should however show that they understand that economic factors were not the sole factors determining the fortunes of German nationalism in this period. Candidates may well argue that the development of German nationalism owed much to reactions to domination by France in the Napoleonic period and the development of ideas in the first half of the Nineteenth Century. The development and impact of ideas could be explored. Economic factors undeniably contributed to Prussia’s domination of Germany from 1866, but opportunistic and skilful leadership, both for and against German nationalism, should not be overlooked. Events also determined the development of German nationalism, for example the failure of the 1848 Revolution. The Great War left Germany broken and half-starved despite the German economic domination of continental Europe in 1914.

60 Candidates are expected to demonstrate understanding of the issues in each of their selected questions over a period of at least a hundred years (unless an individual question specifies a slightly shorter period.) Candidates are reminded of the synoptic nature of the Unit. Answers are required to demonstrate understanding of the processes of historical continuity, development and change across the full breadth of the period studied. Assessors must be open to alternative approaches. If in doubt, they should consult their Team Leader.

6

Page 398: History A - The Bicester Schoolthebicesterschool.org.uk/wp-content/uploads/2015/09/Mark... · 2015-09-28 · History A Mark Schemes for the Units January 2009 H106/H506/MS/R/09J

F966/02 Mark Scheme June 2012

Question Answer Marks Guidance 3 Candidates should focus on the phrase ‘most important turning point’ in their

answers. Candidates may argue either for or against the Revolutions of 1848 – 1849 as the most important turning point, but should do so comparatively in the context of other turning points. Any answers that are limited to the importance of the Revolutions of 1848 – 1849, however full and accurate, are likely to be unbalanced. In assessing the significance of the 1848 – 49 Revolutions candidates are likely to stress German liberalism’s missed opportunity and the significance for Austria and Germany of the fall of Metternich. Arguably after 1849 the course of German nationalism was less likely to be shaped by liberal constitutionalism than it had been in the previous years. What follows is not an exclusive list of other potential turning-points, but obvious consideration could be given to 1792 The start of the Revolutionary Wars 1809 Metternich became Minister of State in Austria 1813 Napoleon’s defeat at the battle of Leipzig (the Battle of the Nations) 1815 Congress of Vienna – formation of German Confederation 1862 Bismarck’s appointment as Minister-President of Prussia 1866 Seven Weeks War – the defeat of Austria 1870/71 The Franco – Prussian War and formation of the Second Reich (Germany) 1888 Accession of William II 1890 Sacking of Bismarck 1914 Start of the First World War 1918 Defeat in the First World War Clearly answers of the very highest quality can be written without considering all of these potential turning points, but the most able candidates will demonstrate a breadth of vision and a good understanding of the moments that shaped the destiny of German nationalism. Candidates who adopt a fully comparative approach and demonstrate synthesis throughout the essay are likely to be most successful.

60 Candidates are expected to demonstrate understanding of the issues in each of their selected questions over a period of at least a hundred years (unless an individual question specifies a slightly shorter period.) Candidates are reminded of the synoptic nature of the Unit. Answers are required to demonstrate understanding of the processes of historical continuity, development and change across the full breadth of the period studied. Need for comparative synthesis, not just a series of paragraphs that are free standing. Assessors must be open to alternative approaches. If in doubt, they should consult their Team Leader.

7

Page 399: History A - The Bicester Schoolthebicesterschool.org.uk/wp-content/uploads/2015/09/Mark... · 2015-09-28 · History A Mark Schemes for the Units January 2009 H106/H506/MS/R/09J

F966/02 Mark Scheme June 2012

Question Answer Marks Guidance 4 The candidates need a clear understanding of what constitutes state involvement

in the organisation and conduct of war, although some leeway might be expected on the part of examiners given the potential scope of the question. Good responses will set down criteria and then evaluate them in relation to the evidence. The mobilization of resources in the Revolutionary and Napoleonic periods might concentrate on the French Republic and Empire, its successful conscription of manpower in the military and economic infrastructures. Napoleon’s organisation of France and her empire would be a good example. The industrial and financial power of Britain and her empire throughout the period might be a good example for investigation, although the Crimean War might well be part of a negative argument. The mobilization of the state in the support of war in Bismarck’s Prussia might be contrasted with the more haphazard effort of both France and Austria in the wars of Unification. The First and Second World Wars with mass mobilization of military manpower, labour and resources, etc is an obvious candidate for discussion. In order to meet the synoptic elements of the mark scheme candidates might chart the different reactions of states to war across the period arguing that effectiveness depended when the question was being applied between 1792 and 1945. Candidates wishing to use the American Civil War might cite the Union as an example of the state being ineffective in meeting the demands of war at the start of the conflict but becoming more effective as the war went on. The Confederacy reacted to the demands of war in the opposite fashion, initially being very effective but becoming ineffective as the long term effects of the conflict took hold.

60 Candidates are expected to demonstrate understanding of the issues in each of their selected questions over a period of at least a hundred years (unless an individual question specifies a slightly shorter period.) Candidates are reminded of the synoptic nature of the Unit. Answers are required to demonstrate understanding of the processes of historical continuity, development and change across the full breadth of the period studied. Assessors must be open to alternative approaches. If in doubt, they should consult their Team Leader. Keep an open mind about how candidates define ‘state’.

8

Page 400: History A - The Bicester Schoolthebicesterschool.org.uk/wp-content/uploads/2015/09/Mark... · 2015-09-28 · History A Mark Schemes for the Units January 2009 H106/H506/MS/R/09J

F966/02 Mark Scheme June 2012

Question Answer Marks Guidance 5 Strategy is defined as the art of planning and directing overall military operations

as opposed to tactics, the control of armies in battle. Better responses might define ‘principles of strategy’ such as concentration of force, maintenance of aim, manoeuvre on a strategic level and the like. Responses might link strategy to other elements of the specification which emphasise developments in warfare and engage the question by interweaving the two. Thus, candidates may argue that developments in weapons technology did not influence strategy whilst those in transport and communications did. It is possible for responses to include battle tactics but only as an outcome of strategy, an example might be – due to the developments in weapons technology French strategy in the opening rounds of the Franco-Prussian War placed emphasis on positioning armies in strong defensive positions to fight battles successfully. The synoptic element of the mark scheme might be engaged by discussing the question in the light of the changing nature of warfare across the period. A possible response might be to agree with the question in the earlier part of the period, perhaps to the Russo-Japanese War and challenge it later. For example, it is reasonable to argue that WWI demanded new principles of strategy on the Western Front.

60 Candidates are expected to demonstrate understanding of the issues in each of their selected questions over a period of at least a hundred years (unless an individual question specifies a slightly shorter period.) Candidates are reminded of the synoptic nature of the Unit. Answers are required to demonstrate understanding of the processes of historical continuity, development and change across the full breadth of the period studied. Assessors must be open to alternative approaches. If in doubt, they should consult their Team Leader.

9

Page 401: History A - The Bicester Schoolthebicesterschool.org.uk/wp-content/uploads/2015/09/Mark... · 2015-09-28 · History A Mark Schemes for the Units January 2009 H106/H506/MS/R/09J

F966/02 Mark Scheme June 2012

Question Answer Marks Guidance 6 The Austro-Prussian War of 1866 might be seen as a turning point in the

application of industrialisation to war because of the increased use of technology, such as railways and more advanced weapons such as breech loading rifles and artillery, in war. This technology required an advanced industrial base, hence the turning point. The advantages held by the increasingly industrialised Prussia in the war might also support the question. One might expect an evaluation of the extent of the impact of industrialisation on warfare as part of the response. Alternatively candidates might argue for later turning points; both of the World Wars are obvious candidates. Candidates might be expected to discuss the relative impact of industrialisation on warfare as criteria for reaching a given conclusion. Thus, for example, the Austro-Prussian War of 1866, WWI and WWII are possible turning points but WWI better fits the description because the impact of industrialisation on warfare was more profound. Earlier turning points might be less common due to the nature of the development of industrial economies in Europe and North America. It is certainly the case that the Crimean War could be argued as a turning point. The Revolutionary and Napoleonic Wars might also be identified due to Britain’s role in the wars as the world’s first emerging industrial power. Some candidates might also identify Napoleonic France as possessing a nascent industrial base and apply this example to the question. The American Civil War might be used and examiners should note that it is contemporaneous with the Austro-Prussian War.

60 Candidates are expected to demonstrate understanding of the issues in each of their selected questions over a period of at least a hundred years (unless an individual question specifies a slightly shorter period.) Candidates are reminded of the synoptic nature of the Unit. Answers are required to demonstrate understanding of the processes of historical continuity, development and change across the full breadth of the period studied. Assessors must be open to alternative approaches. If in doubt, they should consult their Team Leader. There must be clear evidence of knowledge of the Austro-Prussian War. Just a mention is not enough.

10

Page 402: History A - The Bicester Schoolthebicesterschool.org.uk/wp-content/uploads/2015/09/Mark... · 2015-09-28 · History A Mark Schemes for the Units January 2009 H106/H506/MS/R/09J

F966/02 Mark Scheme June 2012

Question Answer Marks Guidance 7 Perhaps many candidates will broadly agree with the proposition. The obvious

point is that a self-governing Ireland emerged in the 1920s from rebellion, revolution and civil war. Attempts to achieve Home Rule through parliamentary legislation either failed in the process of law making or through the outcome of events – the First World War, the Easter Rising, the elections of 1918 and so on. Some candidates might wish to concentrate on analysing the strength of the difficulties constitutionalist nationalists faced in attempting to fulfil their aims in order to argue that the odds were always stacked against them, pointing to, for example, the strength of support for the union within Britain’s political system, British imperial interests, the rise of popular Unionism and the ‘Ulster problem’, the tradition and significance of revolutionary nationalism, the very special circumstances as a result of the constitutional crisis of 1911 and the elections of 1910 enabling the passing of the Home Rule Act, the dependence of constitutional nationalists on Liberal governments. Others might wish to point to the achievements of constitutionalist nationalism and suggest that it achieved at least some of its aims – the achievement of Catholic Emancipation, the adoption by the Liberal Party of the principle of Home Rule, the promotion of Land Reform either a direct or indirect consequence of their actions, the successes of Redmond before 1914 and the impact of unforeseeable events that destroyed his achievement.

60 Candidates are expected to demonstrate understanding of the issues in each of their selected questions over a period of at least a hundred years (unless an individual question specifies a slightly shorter period.) Candidates are reminded of the synoptic nature of the Unit. Answers are required to demonstrate understanding of the processes of historical continuity, development and change across the full breadth of the period studied. Assessors must be open to alternative approaches. If in doubt, they should consult their Team Leader. Candidates who write ‘reasons why it failed’ are drifting away from assessing to explaining.

11

Page 403: History A - The Bicester Schoolthebicesterschool.org.uk/wp-content/uploads/2015/09/Mark... · 2015-09-28 · History A Mark Schemes for the Units January 2009 H106/H506/MS/R/09J

F966/02 Mark Scheme June 2012

Question Answer Marks Guidance 8 The purpose of the question is to encourage candidates to consider the factors that

encouraged continuity rather than change. After all, the Union lasted for over 120 years, longer than many other political systems. Candidates might be tempted to dismiss the proposition and to emphasise other factors that contributed more to its survival such as the determination of British Governments and interests, especially imperial interests, to uphold it, even Liberal Governments who supported Home Rule, the weaknesses, divisions and limitations of the opponents of the Union within Ireland and the ability of British governments to exploit them and the degree of force and ruthlessness used against them. Better candidates should give more careful consideration to the proposition. They might consider the various strengths and advantages of the Union to Ireland resulting in the establishment of various vested interests, both religious and secular. They will probably refer to the surviving influence of the Protestant Ascendancy to the 1870s and rise, strengths and political potential of popular unionism, but they might also consider the landed, commercial, industrial, academic, army and naval interests (and others) that had stakes in the survival of the Union. They might also consider the relatively late development of cultural nationalism and its impact.

60 Candidates are expected to demonstrate understanding of the issues in each of their selected questions over a period of at least a hundred years (unless an individual question specifies a slightly shorter period.) Candidates are reminded of the synoptic nature of the Unit. Answers are required to demonstrate understanding of the processes of historical continuity, development and change across the full breadth of the period studied. Assessors must be open to alternative approaches. If in doubt, they should consult their Team Leader.

12

Page 404: History A - The Bicester Schoolthebicesterschool.org.uk/wp-content/uploads/2015/09/Mark... · 2015-09-28 · History A Mark Schemes for the Units January 2009 H106/H506/MS/R/09J

F966/02 Mark Scheme June 2012

Question Answer Marks Guidance 9 An informed essay and demonstration that Ireland was/was not economically

undeveloped should be rewarded. Better answers will address ‘head on’ the ‘how’ part of the question and, perhaps, even better ones will give thought to the concept of ‘undeveloped’. Candidates are likely to consider the significance of subsistence agriculture, land subdivision, seasonal employment and underemployment, rack renting, the poorly run estates of often absentee landowners, the dependence on the potato as a staple leading to the disaster of the famine, workshop production, lack of capital accumulation, mass emigration and so on as evidence of a lack of economic development. It is likely that candidates will emphasise the limited impact of the Industrial Revolution on Ireland, the lack of the necessary raw materials, above all coal, to respond to the changing technologies of the late eighteenth and nineteenth centuries, the impact of the Union and free trade on Irish commerce and industry and its undeveloped agriculture. On the other hand candidates might temper these comments by pointing to the industrialisation of Ulster, Ireland’s access to coal by sea, the development of its canal and rail networks, the degree and characteristics of its urbanisation, regional differences and impact of land reform on agriculture with assessment of the degree to which landlords and trends in eviction and absenteeism changed during the period. Candidates might point to the impact of the Union with Britain, linking it to one of the most developed and dynamic parts of the world, in a positive way, providing access to capital and expertise.

60 Candidates are expected to demonstrate understanding of the issues in each of their selected questions over a period of at least a hundred years (unless an individual question specifies a slightly shorter period.) Candidates are reminded of the synoptic nature of the Unit. Answers are required to demonstrate understanding of the processes of historical continuity, development and change across the full breadth of the period studied. Assessors must be open to alternative approaches. If in doubt, they should consult their Team Leader.

13

Page 405: History A - The Bicester Schoolthebicesterschool.org.uk/wp-content/uploads/2015/09/Mark... · 2015-09-28 · History A Mark Schemes for the Units January 2009 H106/H506/MS/R/09J

F966/02 Mark Scheme June 2012

Question Answer Marks Guidance 10 Candidates may argue either for or against Stalin’s victory in the power struggle

after Lenin’s death as the most important turning point in the development of Russian government, but must do so comparatively in the context of other turning points. When considering the importance of turning points, the ways in which they impacted on the development of Russian government must be analysed. Economic reforms such as emancipation of the peasantry, collectivisation and five-year plans only become relevant when they are linked to political, administrative and ideological methods and changes in government. Candidates who discuss aspects of Russian government such as reform and repression, the fate of opposition, changes in ideology, the absence of democracy, the one party state and compare the relative influence of war and other factors on these developments are most likely to be successful. Candidates who adopt a comparative approach and demonstrate synthesis throughout the essay are likely to be most successful.

Candidates may well consider that Stalin’s rise to power was the most significant turning point in the development of Russian government, perverting the true course of the Russian Revolution because Stalin succeeded Lenin. Candidates who argue this are likely to suggest that Stalin’s victory in the ensuing power struggle led Russia down a very different road than that being paved by Lenin and imposed brutal totalitarianism on Russia. The impact of terror, the purges and the Show Trials on Russian Government is likely to be discussed. Candidates are likely to discuss the 1936 Constitution, though most are likely to consider it an exercise in propaganda. Candidates who disagree with the view that Stalin’s rise to power was the most important turning point in the development of Russian government may use a counter-argument based on more recent archival evidence to suggest that there was significant continuity between Lenin and Stalin. In choosing alternative turning points candidates are likely to select from the assassination of Alexander II in 1881, the 1905 Revolution, the February Revolution of 1917, the October Revolution of 1917 or Stalin’s death in 1953 and replacement by Khrushchev by 1956. For example, candidates might argue that the assassination of Alexander II in 1881 marked the end of any hope of meaningful reform from above by the Romanov dynasty, and set the Romanovs on course for revolution and their downfall. Candidates may argue that February 1917 was the most significant turning point as it ended the 304-year-old Romanov dynasty, but may argue that ultimately this led to the replacement of ‘Romanov Tsars’ by ‘red Tsars’.

60 Candidates are expected to demonstrate understanding of the issues in each of their selected questions over a period of at least a hundred years (unless an individual question specifies a slightly shorter period.) Candidates are reminded of the synoptic nature of the Unit. Answers are required to demonstrate understanding of the processes of historical continuity, development and change across the full breadth of the period studied. Assessors must be open to alternative approaches. If in doubt, they should consult their Team Leader. ‘Turning point’ needs to include other issues but must still focus on Russian government.

14

Page 406: History A - The Bicester Schoolthebicesterschool.org.uk/wp-content/uploads/2015/09/Mark... · 2015-09-28 · History A Mark Schemes for the Units January 2009 H106/H506/MS/R/09J

F966/02 Mark Scheme June 2012

Question Answer Marks Guidance Many candidates will undoubtedly argue that October 1917 and the triumph of

Bolshevism was the most important turning point as it crushed all possibility that a liberal democracy might emerge in Russia and transformed Russia into the Soviet Union – the world’s first communist one-party state. Candidates may choose to deal with 1917 as a single, and most important turning point, in the development of Russian government but are likely to be more successful if able to make comparisons between the two revolutions of that momentous year. Candidates may argue that Khrushchev’s secret speech of 1956 and subsequent de-stalinisation marked a significant turning point in the development of Russian government though the continuation of communism and the one-party state way beyond 1964 somewhat negates that view.

15

Page 407: History A - The Bicester Schoolthebicesterschool.org.uk/wp-content/uploads/2015/09/Mark... · 2015-09-28 · History A Mark Schemes for the Units January 2009 H106/H506/MS/R/09J

F966/02 Mark Scheme June 2012

Question Answer Marks Guidance 11 Candidates should argue both for and against the assertion that the rulers of

Russia were ‘reluctant reformers’ in this period. Candidates who consider reform by type, for example political, social, economic and military may argue that all rulers were reluctant to introduce certain types of reform, but equally keen to introduce others. Candidates who contend that rulers were never reluctant to introduce the reforms that matched their aims are likely to be successful. Candidates who adopt a comparative approach and demonstrate synthesis throughout the essay are likely to be most successful.

Candidates may argue that the Romanov Tsars were usually reluctant reformers but that the communists wanted to change everything. However many candidates will argue that Alexander II was far from a reluctant reformer, citing his intention to introduce ‘reform from above’ from 1856. Candidates may also argue that the default position of some of the communist rulers, most typically Stalin was repressive rather than reformist. When arguing in favour of the assertion in the question, candidates may draw on examples from across the period, from Alexander III’s imposition of the Reaction from 1881 and the influence of Pobedonostev to the repression of Lenin (eg Red Terror) and Stalin (eg Gulags, purges and Show Trials). Candidates may argue that some of the rulers only introduced reform under duress, for example Nicholas II in 1905 and Lenin in 1921. Candidates, however, may also argue that rulers were at times forced into the adoption of repressive policies because of adverse circumstance, or to ensure their regime’s survival (eg Lenin during the Civil War). However, when arguing against the assertion candidates can also draw on a wide range of evidence; examples could include Alexander II’s reforms, Lenin’s post-revolutionary reforms and the reforms initiated by Khrushchev towards the end of this period. Stalin may have valued repression but was arguably equally determined to impose change on the USSR. Candidates may well cite various examples of Stalin (perhaps with particular reference to the Five Year Plans) doing much to change Russia.

60 Candidates are expected to demonstrate understanding of the issues in each of their selected questions over a period of at least a hundred years (unless an individual question specifies a slightly shorter period.) Candidates are reminded of the synoptic nature of the Unit. Answers are required to demonstrate understanding of the processes of historical continuity, development and change across the full breadth of the period studied. Assessors must be open to alternative approaches. If in doubt, they should consult their Team Leader. Key is motives of reformers rather than outcome of reforms.

16

Page 408: History A - The Bicester Schoolthebicesterschool.org.uk/wp-content/uploads/2015/09/Mark... · 2015-09-28 · History A Mark Schemes for the Units January 2009 H106/H506/MS/R/09J

F966/02 Mark Scheme June 2012

Question Answer Marks Guidance 12 Candidates should focus on the similarities and differences between the treatment

of the peoples of the Russian Empire, both before and after 1917. Candidates are likely to compare and contrast the repression of the peoples and the harsh treatment of opponents both before and after 1917. Candidates who adopt a comparative approach and demonstrate synthesis throughout the essay are likely to be most successful.

One possible line of argument would be to suggest that the communists ruled Russia much more repressively than their tsarist predecessors. A case could be made for arguing that the Russians ‘exchanged’ an increasingly ineffectual and superannuated form of authoritarianism for a more ruthless and efficient twentieth century variant. However, other candidates may argue that some rulers from both periods were ultra-repressive, typically Alexander III, Lenin and Stalin, whereas the others were less repressive or less capable of effective repression either due to their context or their personality. Candidates are likely to make comparisons based on the experience of the peasants, the proletariat, and the minority nations under Russia’s various rulers in this period. Equally they are likely to look at the agencies, and scale, of repression, and the existence of/fate of opposition groups, under each ruler in order to make judgements. Candidates are likely to argue that as opposition flourished during the reigns of Alexander II and Nicholas II and during the Provisional Government the peoples of Russia were not consistently repressed by their rulers. Similarly, de-Stalinisation under Khrushchev, and his enforced retirement in 1964, is likely to be used by candidates to argue that the peoples of Russia were not consistently repressed during the communist period. Candidates who consider the scale of repression are likely to argue that repression reached its zenith under Stalin, citing the terror, purges, Show Trials and gulags. Candidates may use recent archival evidence to suggest that there was significant continuity between Lenin and Stalin, whereas others may draw distinctions between repression under these two rulers.

60 Candidates are expected to demonstrate understanding of the issues in each of their selected questions over a period of at least a hundred years (unless an individual question specifies a slightly shorter period.) Candidates are reminded of the synoptic nature of the Unit. Answers are required to demonstrate understanding of the processes of historical continuity, development and change across the full breadth of the period studied. Assessors must be open to alternative approaches. If in doubt, they should consult their Team Leader. ‘Peoples’ is a comprehensive phrase and includes the people governed by Russia. This may be illustrated in the better answers.

17

Page 409: History A - The Bicester Schoolthebicesterschool.org.uk/wp-content/uploads/2015/09/Mark... · 2015-09-28 · History A Mark Schemes for the Units January 2009 H106/H506/MS/R/09J

F966/02 Mark Scheme June 2012

Question Answer Marks Guidance 13 The divisions within the trade union and labour movement were mainly over

membership, organisation and tactics. The Knights of Labour (1869) were organised by industry rather than craft, disliked strikes as tacit recognition of the wage system, and were prepared in some assemblies to admit women and black workers. The American Federation of Labor (1886) were organised as craft associations, aimed to improve wages, conditions etc within the capitalist system but admitted few women or blacks and disliked immigrants from south and east Europe. The socialist United Mine Workers (1890) allowed black workers and new immigrants to join. The so-called ‘Molly Maguires’ (formed in 1865 by Pennsylvania coal miners) were secretive, radical and (allegedly) violent. The Industrial Workers of the World (founded by western miners in 1905 and known as ‘The Wobblies’) aimed to unite all workers in one big union, were committed to revolution but never enjoyed large membership and recruited only the poorest, most isolated workers. In 1935 the Committee for Industrial Organisation was established within the AFL to recruit unskilled workers, organise them by industry and adopt more radical tactics. In 1938, as the Congress for Industrial Organisation, it broke away from the AFL. But in 1955 the CIO merged with the AFL. The AFL-CIO brought a measure of unity to the US trade union movement, although powerful unions such as the Teamsters retained considerable autonomy and the unpopularity of the PATCO strikers in 1981 showed that divisions remained. Weaker answers are likely to outline the divisions within the trade union and labour movement and perhaps offer only comparative analysis of alternative factors only in the final paragraph. Better answers are likely to evaluate the importance of internal divisions against the lack of support for trade union and workers’ rights from the government, at both Federal and state level, referring perhaps to some or all of the following: the Haymarket bombing (1886), the Homestead steel strike (1892), the treatment of Coxey’s ‘Army’ and the Pullman strike (1894), the use of the 1890 Sherman Anti-Trust Act to gain injunctions against strikers and Supreme Court rulings that unions were ‘illegal combinations’, the 1921 Supreme Court decision that the 1914 Clayton Act was unconstitutional, the 1947 Taft-Hartley Act, the 1959 Landrum-Griffin Act, and Reagan’s defeat of the PATCO strikers in 1981.

60 Candidates are expected to demonstrate understanding of the issues in each of their selected questions over a period of at least a hundred years (unless an individual question specifies a slightly shorter period.) Candidates are reminded of the synoptic nature of the Unit. Answers are required to demonstrate understanding of the processes of historical continuity, development and change across the full breadth of the period studied. Assessors must be open to alternative approaches. If in doubt, they should consult their Team Leader.

18

Page 410: History A - The Bicester Schoolthebicesterschool.org.uk/wp-content/uploads/2015/09/Mark... · 2015-09-28 · History A Mark Schemes for the Units January 2009 H106/H506/MS/R/09J

F966/02 Mark Scheme June 2012

Question Answer Marks Guidance Good answers will also show awareness that, for much of this period, trade unions

were operating in a hostile environment in which co-operative action by workers was regarded as tantamount to socialism, or even communism, especially in the two ‘red scares’ that followed each of the world wars. They will also be aware that the extremism and avowed radicalism of some unions reinforced these assumptions and alienated potential sympathisers. Good answers will refer to the readiness of some employers to resort to strong-arm tactics to break strikes (for example Pinkertons in 1892 and Ford’s battle with the UAW in 1937) and the use of ‘yellow-dog’ contracts by which workers were prevented from joining unions. Perhaps only the best answers will analyse the divisions in US society (between black and white, male and female, Catholic and Protestant, old immigrant and new immigrant workers). They may also be able to analyse change over time and argue that divisions became less important after 1945 and that, as union influence declined, workers looked increasingly to the Federal government for the protection of their rights.

19

Page 411: History A - The Bicester Schoolthebicesterschool.org.uk/wp-content/uploads/2015/09/Mark... · 2015-09-28 · History A Mark Schemes for the Units January 2009 H106/H506/MS/R/09J

F966/02 Mark Scheme June 2012

Question Answer Marks Guidance 14 The case for 1924 as a significant turning point for Native Americans rests on the

importance of citizenship. Hitherto, their status derived from Chief Justice Marshall’s 1831 decision that they were ‘domestic dependent nations’ whose relationship with the US ‘resembles that of a ward to his guardian’. Thus the Act was an important step in achieving civil rights because, without citizenship, Native Americans could not claim the rights guaranteed by the US constitution. However the Act granted citizenship to all Native Americans who did not have it already (estimated to be about 125000 of the 300000 Native Americans then living in the USA). Native Americans who were already citizens were those who had taken land allotments under the 1887 Dawes Act, the Five Civilised Tribes of Native American Territory (ie Oklahoma) who were granted citizenship in 1901, and those who served in the First World War (estimates vary from 8000 to 16000) who gained it in 1919. In practical terms, citizenship did not mean much. Most states with large numbers of Native Americans disenfranchised them in the same ways as African Americans were denied the vote in the south – by imposing literacy tests and requiring them to be taxpayers. Most candidates will regard other turning points as more significant. These include the 1887 Dawes Act, the 1890 massacre at Wounded Knee, the 1934 Indian Reorganisation Act, the Second World War, the 1953 Congressional resolution launching termination, and the Nixon presidency. Weaker answers will describe each (or some) of these in turn, confining their analysis to a final paragraph. Better answers will endeavour to explain why they regard one (or more) of them as more significant than the 1924 Act. The Dawes Act and the Wounded Knee massacre signalled the start of a policy of forced assimilation. Arguably this affected more Native Americans in more significant ways than 1924, but should be seen as regressive. Most candidates will select the 1934 IRA as the most important turning point, pointing out that the Indian New Deal replaced forced assimilation with respect for Native American culture and customs which arguably has persisted to the present. Furthermore, the IRA prevented the alienation of more tribal land and granted Native American communities a measure of governmental and judicial autonomy. The best answers will perhaps show awareness of the limitations of IRA and the degree of opposition to it (for different reasons) in Congress and among Native Americans.

60 Candidates are expected to demonstrate understanding of the issues in each of their selected questions over a period of at least a hundred years (unless an individual question specifies a slightly shorter period.) Candidates are reminded of the synoptic nature of the Unit. Answers are required to demonstrate understanding of the processes of historical continuity, development and change across the full breadth of the period studied. Assessors must be open to alternative approaches. If in doubt, they should consult their Team Leader.

20

Page 412: History A - The Bicester Schoolthebicesterschool.org.uk/wp-content/uploads/2015/09/Mark... · 2015-09-28 · History A Mark Schemes for the Units January 2009 H106/H506/MS/R/09J

F966/02 Mark Scheme June 2012

Question Answer Marks Guidance Some candidates will want to argue for the Second World War. It effectively

destroyed the Indian New Deal and made the termination policy almost inevitable. 25000 Indians served in the armed forces and a further 40000 worked in war-related industries, many of whom permanently relocated to the cities and sought to assimilate. A pressure group was formed in 1944 – the National Congress of American Indians (NCAI) and in 1946 Congress set up the Indian Claims Commission to hear Native American claims for their lost lands. Those arguing for the start or end of termination as the most significant turning point will be able to compare it to the post-Dawes Act period of forced assimilation when Indians were encouraged to leave the reservations and adopt the mainstream white lifestyle and culture. Some candidates will opt for the Nixon presidency when Red Power activism drew attention to Native American issues and the president himself was sympathetic to their cause. This began a phase in which Congress passed a series of Acts to improve conditions on Native American reservations and extend Indian rights and autonomy. The Supreme Court also handed down a series of landmark judgments about compensation for lost lands and Native American rights.

21

Page 413: History A - The Bicester Schoolthebicesterschool.org.uk/wp-content/uploads/2015/09/Mark... · 2015-09-28 · History A Mark Schemes for the Units January 2009 H106/H506/MS/R/09J

F966/02 Mark Scheme June 2012

Question Answer Marks Guidance 15 Many candidates will argue that, for middle-class women, changes in the economy

have been fundamental to the breakdown of ‘separate spheres’ assumptions which circumscribed their aspirations at the start of the period. The periods in which the USA has experienced its most significant economic developments (late 19th and early 20th centuries, the 1920s, the Second World War and post-war boom, the 1980s) have coincided with the most fundamental transformations of women’s role and attitudes to their status. The periods of economic expansion have provided greater access to white collar employment, educational opportunities and consumer goods. They have also transformed attitudes to marriage, divorce, sex, child-rearing and fashion. Arguably the challenge to feminism and the campaign against the Equal Rights Amendment coincided with, and was partly the product of, the economic conditions of the 1980s which rejected the Keynsianism and liberalism of the New Deal and Great Society. Probably only the better answers will identify the continuity in the lives and experiences of poor women, especially those from ethnic minorities (African American, Hispanic American, Native American) whose rights, it can be argued, have not advanced very much. Arguably their economic role has not changed very much either, since they are predominantly doing low-paid, part-time, unskilled and/or menial jobs without the protection of trade unions. Candidates will need to evaluate other factors which have influenced the development of women’s rights. These include the expanded access to good education (especially at university), the impact of the two world wars, the role of the Federal government, social and attitudinal changes and the impact of women’s activism. The best answers will either show how these factors are influenced by economic changes or will explain convincingly how and why they were more important in the development of women’s rights than economic changes. They might, for example, suggest that the Federal legislation of the early 1960s was more a response to the civil rights campaign than to the needs of the economy. Alternatively they might suggest that scientific development, especially with regard to birth control and abortion, was more influential than economic change in accelerating women’s rights. High level answers might question how far women’s rights have advanced and point to continuities in conservative assumptions about a woman’s place in society.

60 Candidates are expected to demonstrate understanding of the issues in each of their selected questions over a period of at least a hundred years (unless an individual question specifies a slightly shorter period.) Candidates are reminded of the synoptic nature of the Unit. Answers are required to demonstrate understanding of the processes of historical continuity, development and change across the full breadth of the period studied. Assessors must be open to alternative approaches. If in doubt, they should consult their Team Leader.

22

Page 414: History A - The Bicester Schoolthebicesterschool.org.uk/wp-content/uploads/2015/09/Mark... · 2015-09-28 · History A Mark Schemes for the Units January 2009 H106/H506/MS/R/09J

F966/02 Mark Scheme June 2012

Question Answer Marks Guidance 16 Candidates are expected to consider the impact of changes to party organisations

against other factors that promoted mass democracy In Britain. Candidates are likely to consider responses to the extension of the Franchise in the Second and Third Reform Acts, especially to attract the votes of the working man and develop democracy. For example, they might refer to Disraelian Conservatism and the promotion of ‘One Nation’ Toryism, the establishment of the Primrose League (1883), the setting up of Liberal and Reform Clubs, Liberal endorsement of Trade Union candidates and the gradual emergence of a Labour Movement and, eventually, a Labour Party, aiming to represent the working class through democratic means. Candidates might refer to the attempts of parties to reform their organisation and reinvent themselves in order to expand and broaden their electoral appeal. Candidates might cite, for example, the promotion of New Liberalism from the publication of the Newcastle Programme of 1891 onwards, the revival of ‘One Nation Toryism’ and a commitment to steady social reform under Baldwin in the 1920s, the promotion of ‘Socialism and Peace’ by the Labour Party in 1934, the conservative reforms and response to their defeat at the polls in 1945, especially in the Industrial Charter of 1947, Jo Grimond’s reforms of the Liberal Party from 1957, the rise of ‘pavement politics’ and targeting of winnable seats, the emergence of the Social Democratic Party in 1983, the changing roles of Trades Unions in the organisation of the labour party, the reforms of the Labour Party under Kinnock in the 1980s and the emergence of New Labour and ‘a Third Way’ in the 1990s and the abolition of Clause IV in 1995 – all attempts to provide the enfranchised with a democratic voice and programme. Against changes to party organisations candidates might argue that other factors were more important, such as changes to the Franchise from 1867, especially female suffrage, or the impact of social and economic changes brought on by two World Wars and the Great Depression or the consumer-led prosperity from the 1950s or the growth of libertarianism from the 1960s. It is moreover possible for candidates to argue that changes to party organisation were as much the consequence of, and a political response to, the development of democracy in Britain as the cause.

60 Candidates are expected to demonstrate understanding of the issues in each of their selected questions over a period of at least a hundred years (unless an individual question specifies a slightly shorter period.) Candidates are reminded of the synoptic nature of the Unit. Answers are required to demonstrate understanding of the processes of historical continuity, development and change across the full breadth of the period studied. Assessors must be open to alternative approaches. If in doubt, they should consult their Team Leader.

23

Page 415: History A - The Bicester Schoolthebicesterschool.org.uk/wp-content/uploads/2015/09/Mark... · 2015-09-28 · History A Mark Schemes for the Units January 2009 H106/H506/MS/R/09J

F966/02 Mark Scheme June 2012

Question Answer Marks Guidance 17 Candidates should recognise that the fortunes of the Liberals varied considerably

during the period and most are likely to see the question as a game of two halves: a party of office before 1922 and a party out of office thereafter. However, they should also recognise changes of fortunes within these two periods. Some might follow a chronological approach and examiners should take care to distinguish between narrative and chronological analysis within such a structure. Other candidates might define themes to explain the changing fortunes of the Liberals within the chronology: the strengths and weaknesses of Gladstonian Liberalism; the impact of key individuals (Gladstone, Asquith, Lloyd George, Thorpe, Steel and Ashdown), the splits within the party caused by the issue of Home Rule for Ireland; the issue of tariff reform 1903-6; the rise of New Liberalism; the divisions encouraged by constitutional crisis and the First World War 1909-22; the impact of Nonconformity and its decline; social and economic change, the rise of organised labour and the emergence of the Labour Party; the ability to find a distinctive voice rather than votes in the 1930s onwards; the reasons for the revival of the party in the last decades of the twentieth century, for example splits within Labour 1979-83, the emergence of the Social Democratic Party and the pact of the ‘two Davids’ and the emergence of the Liberal Democrats, Thatcherism, the decline of organised labour and rise of a new middle class and a new intelligentsia in 1970s along with the decline of Marxism as an ideology.

60 Candidates are expected to demonstrate understanding of the issues in each of their selected questions over a period of at least a hundred years (unless an individual question specifies a slightly shorter period.) Candidates are reminded of the synoptic nature of the Unit. Answers are required to demonstrate understanding of the processes of historical continuity, development and change across the full breadth of the period studied. Assessors must be open to alternative approaches. If in doubt, they should consult their Team Leader.

24

Page 416: History A - The Bicester Schoolthebicesterschool.org.uk/wp-content/uploads/2015/09/Mark... · 2015-09-28 · History A Mark Schemes for the Units January 2009 H106/H506/MS/R/09J

F966/02 Mark Scheme June 2012

25

Question Answer Marks Guidance 18 Candidates should consider the idea that the Lords were an obstacle to democratic

change and not an obstacle to any sort of change. They might agree that the House of Lords remained a bulwark of the remnants of old, landed, aristocratic order and ‘the establishment’ (Church of England bishops and senior law officers,) and some might show that new creations to the peerage were often unrepresentative of contemporary society throughout the period to 1997. They might consider particular examples of the House proving an obstacle, particularly the treatment of Liberal governments of the 1890s and from 1906, resulting in the constitutional crisis of 1909-11. Some might counter the argument by pointing out that the House of Lords had either not obstructed or had failed to obstruct significant democratic changes before 1911, such as the Second and Third Reform Acts and the changes to local government, eg the County Councils Act and the Parish Councils Act, though their partiality towards Conservative-backed measures might also be noted. Others might argue the House unintentionally pushed forward democratic change, for example, the changes of 1911 promoted by the hostile reaction of the Lords to liberal measures. They might consider that as a revising chamber after 1911 the House of Lords was so neutered that it could no longer act as a barrier, hence, in part, the success of women gaining the vote in 1918 and 1928. Perhaps its weakness was exemplified by the Salisbury Convention after the Tory electoral collapse in 1945. On the other hand some candidates might wish to use examples to 1997 of the Lords acting as a restraining influence on what might be considered a powerful executive in control of a cowed first chamber, eg the role of the Lords during the premiership of Margaret Thatcher. Another line of argument might be to consider the democratisation of the membership of the House of Lords, especially its active membership, with comments on the decline of the wealth and significance of the landed aristocracy, the ennoblement of new money and industrialists, the slow impact of a more socially, broadly based meritocracy, for example on the bench of bishops, the creation of life peerages from the late 1950s. By the 1990s some members of the Lords were from humble origins.

60 Candidates are expected to demonstrate understanding of the issues in each of their selected questions over a period of at least a hundred years (unless an individual question specifies a slightly shorter period.) Candidates are reminded of the synoptic nature of the Unit. Answers are required to demonstrate understanding of the processes of historical continuity, development and change across the full breadth of the period studied. Assessors must be open to alternative approaches. If in doubt, they should consult their Team Leader.

Page 417: History A - The Bicester Schoolthebicesterschool.org.uk/wp-content/uploads/2015/09/Mark... · 2015-09-28 · History A Mark Schemes for the Units January 2009 H106/H506/MS/R/09J

OCR (Oxford Cambridge and RSA Examinations) 1 Hills Road Cambridge CB1 2EU OCR Customer Contact Centre Education and Learning Telephone: 01223 553998 Facsimile: 01223 552627 Email: [email protected] www.ocr.org.uk For staff training purposes and as part of our quality assurance programme your call may be recorded or monitored

Oxford Cambridge and RSA Examinations is a Company Limited by Guarantee Registered in England Registered Office; 1 Hills Road, Cambridge, CB1 2EU Registered Company Number: 3484466 OCR is an exempt Charity OCR (Oxford Cambridge and RSA Examinations) Head office Telephone: 01223 552552 Facsimile: 01223 552553 © OCR 2012

Page 418: History A - The Bicester Schoolthebicesterschool.org.uk/wp-content/uploads/2015/09/Mark... · 2015-09-28 · History A Mark Schemes for the Units January 2009 H106/H506/MS/R/09J

Oxford Cambridge and RSA Examinations

GCE

History A Advanced GCE

Unit F966/02: Historical Themes Option B: Modern 1789–1997

Mark Scheme for January 2013

Page 419: History A - The Bicester Schoolthebicesterschool.org.uk/wp-content/uploads/2015/09/Mark... · 2015-09-28 · History A Mark Schemes for the Units January 2009 H106/H506/MS/R/09J

OCR (Oxford Cambridge and RSA) is a leading UK awarding body, providing a wide range of qualifications to meet the needs of candidates of all ages and abilities. OCR qualifications include AS/A Levels, Diplomas, GCSEs, Cambridge Nationals, Cambridge Technicals, Functional Skills, Key Skills, Entry Level qualifications, NVQs and vocational qualifications in areas such as IT, business, languages, teaching/training, administration and secretarial skills. It is also responsible for developing new specifications to meet national requirements and the needs of students and teachers. OCR is a not-for-profit organisation; any surplus made is invested back into the establishment to help towards the development of qualifications and support, which keep pace with the changing needs of today’s society. This mark scheme is published as an aid to teachers and students, to indicate the requirements of the examination. It shows the basis on which marks were awarded by examiners. It does not indicate the details of the discussions which took place at an examiners’ meeting before marking commenced. All examiners are instructed that alternative correct answers and unexpected approaches in candidates’ scripts must be given marks that fairly reflect the relevant knowledge and skills demonstrated. Mark schemes should be read in conjunction with the published question papers and the report on the examination. OCR will not enter into any discussion or correspondence in connection with this mark scheme. © OCR 2013

Page 420: History A - The Bicester Schoolthebicesterschool.org.uk/wp-content/uploads/2015/09/Mark... · 2015-09-28 · History A Mark Schemes for the Units January 2009 H106/H506/MS/R/09J

F966/02 Mark Scheme January 2013

1

Subject-specific Marking Instructions Distribution of marks for each level that reflects the Unit’s AOs 2 answers: Each maximum mark 60

AO1a AO1b IA 18–20 36–40

IB 16–17 32–35

II 14–15 28–31

III 12–13 24–27

IV 10–11 20–23

V 8–9 16–19

VI 4–7 8–15

VII 0–3 0–7

Page 421: History A - The Bicester Schoolthebicesterschool.org.uk/wp-content/uploads/2015/09/Mark... · 2015-09-28 · History A Mark Schemes for the Units January 2009 H106/H506/MS/R/09J

F966/02 Mark Scheme January 2013

2

Notes: (i) Allocate marks to the most appropriate level for each AO. (ii) If several marks are available in a box, work from the top mark down until the best fit has been found. (iii) Many answers will not fall at the same level for each AO. (iv) Candidates will demonstrate synoptic skills by drawing together appropriate techniques, knowledge and understanding to evaluate developments over the whole of the period.

AOs AO1a AO1b Total mark for each question = 60

Recall, select and deploy historical knowledge appropriately, and communicate knowledge and understanding of history in a clear and effective manner.

Demonstrate understanding of the past through explanation, analysis and arriving at substantiated judgements of: - key concepts such as causation, consequence, continuity, change

and significance within an historical context; - the relationships between key features and characteristics of the

periods studied.

Level IA

Uses a wide range of accurate and relevant evidence

Accurate and confident use of appropriate historical terminology

Answer is clearly structured and coherent; communicates accurately and legibly.

18–20

Excellent understanding of key concepts (eg continuity and change) relevant to analysis in their historical context

Excellent synthesis and synoptic assessment Answer is consistently and relevantly analytical with developed

explanations and supported judgements May make unexpected but substantiated connections over the

whole period.

36–40 Level IB

Level IB Uses accurate and relevant evidence Accurate use of a range of appropriate historical

terminology Answer is clearly structured and mostly

coherent; communicates accurately and legibly.

16–17

Very good level of understanding of key concepts (eg continuity and change) in their historical context

Answer is consistently focused on the question set Very good level of explanation/analysis, and provides supported

judgements Very good synthesis and synoptic assessment of the whole period.

32–35

Page 422: History A - The Bicester Schoolthebicesterschool.org.uk/wp-content/uploads/2015/09/Mark... · 2015-09-28 · History A Mark Schemes for the Units January 2009 H106/H506/MS/R/09J

F966/02 Mark Scheme January 2013

3

AOs AO1a AO1b Level

II Uses mostly accurate and relevant evidence Generally accurate use of historical

terminology Answer is structured and mostly coherent;

writing is legible and communication is generally clear.

14–15

Good level of understanding of key concepts (eg continuity and change) in their historical context

Good explanation/analysis but overall judgements may be uneven Answer is focused on the issues in the question set Good synthesis and assessment of developments over most of the period.

28–31 Level

III Uses relevant evidence but there may be

some inaccuracy Answer includes relevant historical

terminology but this may not be extensive or always accurately used

Most of the answer is structured and coherent; writing is legible and communication is generally clear.

12–13

Shows a sound understanding of key concepts, especially continuity and change, in their historical context

Most of the answer is focused on the question set Answers may be a mixture of analysis and explanation but also description

and narrative, but there may also be some uneven overall judgements; OR answers may provide more consistent analysis but the quality will be uneven and its support often general or thin

Answer assesses relevant factors but provides only a limited synthesis of developments over most of the period.

24–27

Level IV

There is deployment of relevant knowledge but level/accuracy will vary

Some unclear and/or underdeveloped and/or disorganised sections

Mostly satisfactory level of communication.

10–11

Satisfactory understanding of key concepts (eg continuity and change) in their historical context

Satisfactory focus on the question set Answer may be largely descriptive/narratives of events, and links between

this and analytical comments will typically be weak or unexplained Makes limited synoptic judgements about developments over only part of

the period.

20–23

Page 423: History A - The Bicester Schoolthebicesterschool.org.uk/wp-content/uploads/2015/09/Mark... · 2015-09-28 · History A Mark Schemes for the Units January 2009 H106/H506/MS/R/09J

F966/02 Mark Scheme January 2013

4

AOs AO1a AO1b Level V

General and basic historical knowledge but also some irrelevant and inaccurate material

Often unclear and disorganised sections Adequate level of communication but some

weak prose passages.

8–9

General understanding of key concepts (eg continuity and change) in their historical context

Some understanding of the question but answers may focus on the topic and not address the question set OR provides an answer based on generalisation

Attempts an explanation but often general coupled with assertion, description/narrative

Very little synthesis or analysis and only part(s) of the period will be covered.

16–19

Level VI

Use of relevant evidence will be limited; there will be much irrelevance and inaccuracy

Answers may have little organisation or structure

Weak use of English and poor organisation.

4–7

Very little understanding of key concepts (eg continuity and change) in their historical context

Limited perhaps brief explanation Mainly assertion, description/narrative Some understanding of the topic but not the question’s requirements.

8–15

Level VII

Little relevant or accurate knowledge Very fragmentary and disorganised response Very poor use of English and some

incoherence.

0–3

Weak understanding of key concepts (eg continuity and change) in their historical context

No explanation Assertion, description/narrative predominate Weak understanding of the topic or of the question’s requirements.

0–7

Page 424: History A - The Bicester Schoolthebicesterschool.org.uk/wp-content/uploads/2015/09/Mark... · 2015-09-28 · History A Mark Schemes for the Units January 2009 H106/H506/MS/R/09J

F966/02 Mark Scheme January 2013

5

Question Answer Marks Guidance 1 Candidates should focus on how effectively Bismarck managed German nationalism.

Candidates will undoubtedly be more successful if they define ‘effectively’ in their answer. Candidates might define the ways in which Bismarck was (or was not) effective: for example in controlling, harnessing or using nationalism. Many candidates may argue in favour of the assertion in the question because of Bismarck’s critical role in the 1860s in the creation of the Second Reich; candidates may argue that he managed German nationalism by hijacking the nationalist cause for Prussia’s ends. This could certainly be considered effective management of German nationalism. However, any answers that only focus on Bismarck and his management of German nationalism are significantly flawed. To be successful candidates must compare Bismarck’s management of German nationalism with others. They may conclude that Bismarck was or was not the only effective manager of German nationalism but they must do so on the basis of having given consideration to the claims of others. Most candidates are likely to draw comparisons between Bismarck, Metternich and William II. Clearly all three had different aims and different circumstances, which could enable candidates to make convincing cases for all of them as effective managers of German nationalism. From 1815 to 1848 the nationalist movement was too weak to effectively challenge the Metternich System: arguably this demonstrates Metternich’s effective control over German nationalists. By 1848/49 no leader of the nationalist movement with mass appeal emerged. Although Metternich fled Vienna in 1848, his downfall was hardly dominated by German nationalism. Wilhelm II’s search for world power was undoubtedly populist, mirroring the development of radical nationalism; arguably he too managed German nationalism effectively. However, the ultimate outcome of his policies was defeat in the Great War and humiliation at Versailles and his own abdication.

60 Candidates are expected to demonstrate understanding of the issues in each of their selected questions over a period of at least a hundred years (unless an individual question specifies a slightly shorter period.) Candidates are reminded of the synoptic nature of the Unit. Answers are required to demonstrate understanding of the processes of historical continuity, development and change across the full breadth of the period studied. Assessors must be open to alternative approaches. If in doubt, they should consult their Team Leader.

Page 425: History A - The Bicester Schoolthebicesterschool.org.uk/wp-content/uploads/2015/09/Mark... · 2015-09-28 · History A Mark Schemes for the Units January 2009 H106/H506/MS/R/09J

F966/02 Mark Scheme January 2013

6

Question Answer Marks Guidance 2 Candidates should focus on to what extent German nationalism was popular in their

answers in an attempt to evaluate the mass appeal of nationalism in this period. Candidates should evaluate the extent to which nationalism was popular and demonstrate awareness that popularity was not uniform throughout the period. Candidates could consider the extent to which nationalism was popular at various points, for example from 1789–1815, 1815–1848, from 1871–1914 and in 1918. Candidates may well argue that nationalism gained appeal during the Napoleonic Wars but that consequently concepts of romantic nationalism had a limited intellectual appeal. Similarly candidates may argue that in the revolutions of 1848 nationalism gained considerable popularity, especially in the early days, but that this was not embedded, partly at least due to the actions/inactions of the revolutionaries. Candidates are likely to argue that Prussia under Bismarck did a great deal to popularise nationalism and that victories in the wars against Austria and France certainly fanned those flames as did the establishment of the Reich. Candidates may well demonstrate that they understand that Wilhelmine Germany increasingly looked to exploit nationalist yearnings and the mass appeal of German nationalism, pursuing a populist foreign policy to distract the masses from social discontent. Candidates might choose to demonstrate that the popularity of nationalism may be compared to the appeal of other philosophies. For example the growing industrialization of Prussia and the German Empire was mirrored by the growing popularity of socialism, an appeal that proved relatively immune to either appeasement, in the form of state socialism, or repression. Distress from the winter of 1916/1917 onwards, and defeat in 1918, led to the socialist uprisings of late 1918 and early 1919 and the establishment of Ebert’s republic. However, even in 1919 the appeal of unrequited nationalism was never far from the surface, as evidenced by the Freikorps and the emerging ‘stab-in-the-back’ theory.

60 Candidates are expected to demonstrate understanding of the issues in each of their selected questions over a period of at least a hundred years (unless an individual question specifies a slightly shorter period.) Candidates are reminded of the synoptic nature of the Unit. Answers are required to demonstrate understanding of the processes of historical continuity, development and change across the full breadth of the period studied. Assessors must be open to alternative approaches. If in doubt, they should consult their Team Leader.

Page 426: History A - The Bicester Schoolthebicesterschool.org.uk/wp-content/uploads/2015/09/Mark... · 2015-09-28 · History A Mark Schemes for the Units January 2009 H106/H506/MS/R/09J

F966/02 Mark Scheme January 2013

7

Question Answer Marks Guidance 3 Candidates should focus on the relative importance of political factors within the

development of German Nationalism in this period. Candidates must show that they understand that the development of German nationalism was not solely dependent on political factors in this period. The importance of political factors should be evaluated against the other dominant factors in the development of German nationalism during this period, for example economic factors, the role played by wars and the role played by significant individuals such as Metternich, Bismarck and Kaiser Wilhelm II. In discussing political factors candidates might consider the management of German nationalism, in particular they might discuss the role of Bismarck and both his internal and external policies, particularly his diplomacy during the 1860s, this might also lead to a consideration of European political factors, such as the role of Napoleon III. However, some candidates might consider earlier political developments and look back to the legacy of the French Revolution and Metternich, or the Revolutions of 1848-9. In order to cover the whole period, candidates might also consider the importance of Kaiser Wilhelm II. It is possible that many candidates will link political factors to military developments. Candidates may explain the impact of industrialisation on the development of German nationalism, for example the impact of the founding and development of the Krupp Gusstahlfabrik (Cast Steel Works) from 1811 and the impact of the Zollverein after 1834. Some candidates may argue that Prussia’s economic dominance led to their dominance over Germany and in turn had a limiting effect on the development of German nationalism through the establishment of a Kleindeutschland in 1871. Candidates are likely to understand how developments in the economy in the 1850s paved the way for the military victories of 1864, 1866 and 1870/71. Military strength depended upon economic strength however. ‘Coal & Iron’ rather than ‘Blood & Iron’ could be usefully debated. The development of the railways may be seen as significant. The impact of the extraordinary developments in the German economy after 1871 should be discussed. For example, the Great War left Germany broken and half-starved despite the German economic domination of continental Europe in 1914. The development and impact of ideas on the emergence and development of intellectual nationalism may be usefully explored. Candidates may argue that initially this provided the impetus or springboard for later developments and that, in the Napoleonic period, it was the common fight of people from different German states against their French enemy that gave strong impulses to nationalism.

60 Candidates are expected to demonstrate understanding of the issues in each of their selected questions over a period of at least a hundred years (unless an individual question specifies a slightly shorter period.) Candidates are reminded of the synoptic nature of the Unit. Answers are required to demonstrate understanding of the processes of historical continuity, development and change across the full breadth of the period studied. Assessors must be open to alternative approaches. If in doubt, they should consult their Team Leader.

Page 427: History A - The Bicester Schoolthebicesterschool.org.uk/wp-content/uploads/2015/09/Mark... · 2015-09-28 · History A Mark Schemes for the Units January 2009 H106/H506/MS/R/09J

F966/02 Mark Scheme January 2013

8

Question Answer Marks Guidance 4 Candidates will need to show a clear understanding of the concepts of tactics and

strategy, many are likely to treat the two together and show little or no discrimination, but stronger answers should treat them separately. Some may argue that both remained the same, whilst others may argue that one chaged or both changed; any approach is acceptable, provided the answer is supported. Given the developments in military organisation, command, control and technology across the period battlefield tactics obviously changed. A simple example would be to compare a WWII to a Napoleonic battlefield. Such an essay might then link synoptically the factors that caused such changes to take place to the line of debate. Alternatively, the response might support the proposition in the question pointing to consistency in certain principles of warfare. Examples might be drawn from the military theorists in the specification, economy of force, maintenance of aim, concentration of force, etc. These concepts could then be linked to specific examples such as the Crimean War, the Wars of Unification or WWI where it is possible to argue that linear tactics were used throughout this period, any differences in the nature of battle being superficial. In discussing strategy some may argue that it changed and point to the development of the organisation of the state and development of the concept of total war, whilst others might point to the use of bombing at the end of the period. However, some may argue that there was little change as total war was a feature, to a greater or lesser degree throughout the period. Some may argue that planning remained the same and that planners continued to look for at sweeping offensive actions against their opponents as was the case under Napoleon through to Blitzkrieg. Some may suggest that in order to breakthrough this did lead to a change in the latter period as, faced with fortifications and modern weapons it required a massive build up of men, armaments and strategic railways.

60 Candidates are expected to demonstrate understanding of the issues in each of their selected questions over a period of at least a hundred years (unless an individual question specifies a slightly shorter period.) Candidates are reminded of the synoptic nature of the Unit. Answers are required to demonstrate understanding of the processes of historical continuity, development and change across the full breadth of the period studied. Assessors must be open to alternative approaches. If in doubt, they should consult their Team Leader.

Page 428: History A - The Bicester Schoolthebicesterschool.org.uk/wp-content/uploads/2015/09/Mark... · 2015-09-28 · History A Mark Schemes for the Units January 2009 H106/H506/MS/R/09J

F966/02 Mark Scheme January 2013

9

Question Answer Marks Guidance 5 Alliances tended to have more importance when Europe was in a state of

general warfare and conflicts took place over a long period of time with grand objectives such as the domination of the entire continent. Conflicts such as the Revolutionary and Napoleonic Wars, the First and Second World Wars are such examples and allow candidates to chart success/failure in war in the light of strong or weak alliances. In shorter wars alliances were less important and tended to be used to isolate powers rather than create powerful military coalitions. Good examples are all of the Wars of Unification and the Crimean War. The American Civil War might be used, candidates arguing that the Confederacy could only win with an alliance with a third power, an event that did not take place. Thus, the response might focus on the context of war determining the importance – or not – of alliances in determining a successful outcome. Alternatively candidates might argue that other factors were more important in determining the successful outcome of war: manpower, leadership, economic power, etc. Such responses are valid but the core issue posed by the question must be addressed and rejected in a synoptic manner before the candidate embarks on an alternative explanation. Better responses might advance an alternative explanation or explanations for success in war but interweave such explanations into the response in a synoptic fashion. An example might be manpower; a candidate might argue that it is the manpower produced by a large alliance that was the key factor in success rather than the alliance per se. The opportunities for such a response are legion and would meet the synoptic requirements of the mark scheme.

60 Candidates are expected to demonstrate understanding of the issues in each of their selected questions over a period of at least a hundred years (unless an individual question specifies a slightly shorter period.) Candidates are reminded of the synoptic nature of the Unit. Answers are required to demonstrate understanding of the processes of historical continuity, development and change across the full breadth of the period studied. Assessors must be open to alternative approaches. If in doubt, they should consult their Team Leader.

Page 429: History A - The Bicester Schoolthebicesterschool.org.uk/wp-content/uploads/2015/09/Mark... · 2015-09-28 · History A Mark Schemes for the Units January 2009 H106/H506/MS/R/09J

F966/02 Mark Scheme January 2013

10

Question Answer Marks Guidance 6 Responses will need a sound understanding of what is meant by economic

strength and its impact on war. Thus, candidates might examine the impact of industrial power, man power, agricultural wealth etc on the outcome of wars agreeing or disagreeing with the basic precept of the question as the response develops. Examples of economically strong states being successful in war might be Britain in the Napoleonic Wars, the coalition against Russia in the Crimean Wars or the USA in WWII. Examples of economically strong states being unsuccessful might be France in the Napoleonic Wars or Austria in the 1866 war against Prussia. In both of these cases the nature of economic strength might be examined, industrial strength as opposed to stronger manpower and/or agricultural strength. A clearer example of a more economically strong state losing to a weaker one might be the Russo-Japanese War. There are more complex situations where the relative economic strength of the combatant powers change depending on which point of a conflict is being discussed, for example the Axis powers in WWII. The important quality to identify in scripts is the application of the synoptic element of the mark scheme to the question set in a focused manner. Weaker scripts might agree or disagree with the question – or indeed do both – and then list wars to prove the case. Examiners might even encounter scripts that accept and reject the question producing two lists with relevant reasoning, effectively two mini essays. Better responses will engage the question in a synoptic manner and produce a thematic response focused on aspects of economic strength and the impact of such factors on the outcome of wars.

60 Candidates are expected to demonstrate understanding of the issues in each of their selected questions over a period of at least a hundred years (unless an individual question specifies a slightly shorter period.) Candidates are reminded of the synoptic nature of the Unit. Answers are required to demonstrate understanding of the processes of historical continuity, development and change across the full breadth of the period studied. Assessors must be open to alternative approaches. If in doubt, they should consult their Team Leader.

Page 430: History A - The Bicester Schoolthebicesterschool.org.uk/wp-content/uploads/2015/09/Mark... · 2015-09-28 · History A Mark Schemes for the Units January 2009 H106/H506/MS/R/09J

F966/02 Mark Scheme January 2013

11

Question Answer Marks Guidance 7 Candidates will need to stress that all British political parties agreed that the

Union should be maintained and even between 1798 and 1800 it was only the Whigs that argued for maintenance of Grattan’s Dublin parliament. Disagreement came over reform within and of the Union. The extent of that disagreement forms the focus of the question. Candidates may stress that up to 1868 disagreement was relatively minor with both Whig/Liberals and Tories/Conservatives preferring to maintain the protestant ascendancy, the Irish Church and the land settlement. Both revolutionary and constitutional nationalism was resisted, often through coercion. This was particularly so from 1800–1829 and again from 1846–1868. After 1868 Gladstone’s ‘mission to pacify Ireland’ polarised policy, the Liberals epitomising reform within the Union to 1885 (Disestablishment, two Land Acts and University reform) and then reform of the Union by proposing Home Rule in 1886, although this prompted a split, Liberal Unionists (who saw Irish reform as tied to both Empire and social reform) moving across to the Conservatives. Post Gladstone (1895) the Liberal party remained wedded to Home Rule as the answer to Ireland’s problems through to 1920. The Conservatives initially opposed Liberal reform under Gladstone and were strengthened in this by Whig defections in the 1880s. Their policy remained a defence of the Irish Church and they questioned interference with landlord rights and concessions to Parnell and the Land League. They identified with both the Liberal Unionists and the Ulster Unionists in opposing Liberal Home Rule. Between 1910 and 1914 they took this opposition very far (Bonar Law’s Blenheim Palace speech) by backing Ulster’s mobilisation to resist the imposition of Home Rule. However in the Salisbury Balfour period, 1886–1905, they too were prepared to continue with liberal type reform within the Union, particularly by facilitating land purchase and moving towards ratepayer democracy at a local level in Ireland. Both Liberal and Conservative administrations conceded much in the post Parnell period. Disagreement was also marked in the 1916–21 period, although coalition government blurred matters. Liberals, both inside and out of the Coalition disliked the post Easter Rising coercion of the Conservative dominated administrations of Lloyd George, but supported Home Rule for North and South. All parties reluctantly agreed to the end of the Union in the Anglo-Irish treaty. However there is a case that the pre 1868 period is less consensual than it seems.

60 Candidates are expected to demonstrate understanding of the issues in each of their selected questions over a period of at least a hundred years (unless an individual question specifies a slightly shorter period.) Candidates are reminded of the synoptic nature of the Unit. Answers are required to demonstrate understanding of the processes of historical continuity, development and change across the full breadth of the period studied. Assessors must be open to alternative approaches. If in doubt, they should consult their Team Leader.

Page 431: History A - The Bicester Schoolthebicesterschool.org.uk/wp-content/uploads/2015/09/Mark... · 2015-09-28 · History A Mark Schemes for the Units January 2009 H106/H506/MS/R/09J

F966/02 Mark Scheme January 2013

12

Question Answer Marks Guidance The Whigs instinctively disliked coercion, managing to live without it between

1835 and 1841, and individual liberal figures supported considerable reform at key points. The Lichfield House Compact saw a willingness to do deals with O’Connell. Whigs supported Catholic Emancipation; the Tories on the whole did not, despite being confronted with it in 1829. Some useful comparisons can be made between Peel’s Tories and the Whigs. In practice both pursued reform – emancipation, patronage, police, tithe and land, and with relations with the Catholic Church (Peel and Maynooth). Disagreement was not always clear cut.

Page 432: History A - The Bicester Schoolthebicesterschool.org.uk/wp-content/uploads/2015/09/Mark... · 2015-09-28 · History A Mark Schemes for the Units January 2009 H106/H506/MS/R/09J

F966/02 Mark Scheme January 2013

13

Question Answer Marks Guidance 8 The question focuses on the relative importance of factors in determining the

fate of constitutional nationalism. Candidates will need to assess not only the role and attitude of the Catholic Church but at other, possibly more important factors, such as the leadership of O’Connell, Butt, Parnell and Redmond; the tactics of the parliamentary party; the state of Westminster politics; agitation in Ireland (the Catholic Association, the Repeal Association; links to the Land League) etc. The argument that success was determined by the Catholic Church could be demonstrated by an examination of the 1820s and the Catholic Association where O’Connell used the Church to organise, mobilise, fund (the catholic rent) and support what was a catholic issue – emancipation. The revival of Catholicism in the mid and later 19th century made it a powerful force. It gained from educational developments, from emancipation and, under O’Connell, became identified with the causes of constitutional nationalism, to the extent that Young Ireland left in 1846. Its attitudes helped to determine the fate of Gladstone’s reforms and its withdrawal of support for Parnell, after formal backing in 1884 (Parnell agreed to back its educational demands) in the late 1880s sealed his fate. As the franchise widened its hold on the catholic voter increased. Revolutionary nationalism lost from the Church’s disapproval. Redmond’s loss of Church support during the 1st World War was one factor in the failure of 1918. By identification with the Church constitutional nationalism failed to maintain a secular and unitary policy, losing the initial support it had in Ulster. By the 1910s the Church was beginning to support and do deals with Sinn Fein, backing it after 1916. However it is possible to overstate the importance of the Church, especially in the later Parnell and Redmond periods, when the Irish Parliamentary Party focused on Westminster tactics and alliances with the Land League (the Church was socially conservative) and the Liberals to achieve its agenda (Land reform and Home Rule). The fate of Home Rule lay at Westminster and in the politics and events of the 1st World War. Failure in the 1910s was determined by British governments and how tactically Redmond responded to the World War, not by the attitudes of the Church. Even before this Parnell was dependent on Gladstone and on the fate of the Liberal party after 1886. Land issues could either help or hinder constitutional leaders, as the Tithe war (hindered) and the Land War (helped) demonstrated.

60 Candidates are expected to demonstrate understanding of the issues in each of their selected questions over a period of at least a hundred years (unless an individual question specifies a slightly shorter period.) Candidates are reminded of the synoptic nature of the Unit. Answers are required to demonstrate understanding of the processes of historical continuity, development and change across the full breadth of the period studied. Assessors must be open to alternative approaches. If in doubt, they should consult their Team Leader.

Page 433: History A - The Bicester Schoolthebicesterschool.org.uk/wp-content/uploads/2015/09/Mark... · 2015-09-28 · History A Mark Schemes for the Units January 2009 H106/H506/MS/R/09J

F966/02 Mark Scheme January 2013

14

Question Answer Marks Guidance The Church was never as explicitly active after the 1820s and 1830s and

preferred, under Cardinal Cullen, to cultivate British governments at a higher level. More effective answers here might make distinctions between the ordinary clergy, whose activism notably helped constitutional nationalism, and the hierarchy, more aware of government.

Page 434: History A - The Bicester Schoolthebicesterschool.org.uk/wp-content/uploads/2015/09/Mark... · 2015-09-28 · History A Mark Schemes for the Units January 2009 H106/H506/MS/R/09J

F966/02 Mark Scheme January 2013

15

Question Answer Marks Guidance 9 Candidates will need to compare the reasons for the relative success or lack of it

of the various nationalist revolutionary groups. Until 1916–17 it could well be argued that Sinn Fein too lacked mass support in its early years, sharing the fate of earlier groups. Its initial stress was on cultural nationalism, as Young Ireland’s had been in the early 1840s. This had limited mass appeal. The focus for earlier groups, 1798–1848, was martyrdom and rising. This proved a failed tactic for early revolutionary nationalists, productive of a blood sacrifice only tradition (Emmet in 1803 and ‘1848’). Risings lacked wider planning to mobilise the masses and foreign support (from France) proved ineffective. Emmet’s rising in 1803 had as its basis a Dublin coup that, if it failed, could not trigger a wider response. They failed to mobilise and use the Catholic Church or to use Land issues (the Tithe war of the 1830s) to garner support. However it could be argued that revolutionary nationalists of the mid century (Fenians) were more effective in finding support – funding, money and political direction from the expat community in the US, literary and press contributions and in forging alliances with more mainstream groups (Butt and especially Parnell). However it too indulged in rather unproductive and unfocussed terrorism (the 1867 rising and the mainland bombing campaign in 1867–8) until Davitt’s Land League was able to tap into mass agrarian grievance and anti-landlordism, from 1873 to the 1880s, and link to Parnell in 1879 (New Departure). The Fenians were in some respects much more successful in the 1870s and 1880s than Sinn Fein in the early 1900s. However the links with constitutional nationalism would breakdown after the mid 1880s. The key to Sinn Fein’s later success was to exploit key issues – the Boer War (out of which SF emerged in 1905) and especially the 1st World War. Earlier groups, despite trying in 1798 and 1848 had not been able to effectively exploit contemporary issues. Crucial was to create a political party to rival Redmond, a tactic not used before and to explicitly reject association with constitutional nationalism and its links to the Liberal party. Griffiths tried to broaden SF’s appeal but it was not until 1913 that a party political approach linked to a military one in response to the UVF – the creation of the Irish Volunteers and then the IRA with whom SF associated post 1916. In contrast to the French wars the German War created opportunities and led to government mistakes (martial law, the extension of conscription to Ireland and the overreaction to the Easter Rising, from martial law to martyrdom).

60 Candidates are expected to demonstrate understanding of the issues in each of their selected questions over a period of at least a hundred years (unless an individual question specifies a slightly shorter period.) Candidates are reminded of the synoptic nature of the Unit. Answers are required to demonstrate understanding of the processes of historical continuity, development and change across the full breadth of the period studied. Assessors must be open to alternative approaches. If in doubt, they should consult their Team Leader.

Page 435: History A - The Bicester Schoolthebicesterschool.org.uk/wp-content/uploads/2015/09/Mark... · 2015-09-28 · History A Mark Schemes for the Units January 2009 H106/H506/MS/R/09J

F966/02 Mark Scheme January 2013

16

Question Answer Marks Guidance The Rising was, in practice, another failed Irish rising but SF moved to link

constitutional methods with organised military ones (Collins). It challenged Redmond, choosing De Valera as President who courted international sympathy and contested the first universal suffrage election in 1918, a tactic less open to its predecessors who had to accept a limited electorate that supported constitutional nationalism. It could then effectively declare an independent and democratic Dail Eireann in the South and master mind a very effective local challenge to the traditional state (police, justice, administration) using guerrilla warfare. It was then able to negotiate, not without division, the Anglo-Irish Treaty in 1921. Candidates should focus on the key differences between 1916–21 and 1798–1915, in the light of the above.

Page 436: History A - The Bicester Schoolthebicesterschool.org.uk/wp-content/uploads/2015/09/Mark... · 2015-09-28 · History A Mark Schemes for the Units January 2009 H106/H506/MS/R/09J

F966/02 Mark Scheme January 2013

17

Question Answer Marks Guidance 10 Candidates should compare the impact of war and revolutions on the

development of Russian government. Their impact on economic reforms such as emancipation of the peasantry, collectivisation and five-year plans only become relevant when they are linked to political, administrative and ideological methods and changes in government. Candidates who discuss aspects of Russian government such as reform and repression, the fate of opposition, changes in ideology, the absence of democracy, the one party state and compare the relative influence of war and revolutions on these developments are most likely to be successful. Examiners must not expect to find reference to all these aspects in candidate answers. Candidates may argue that the development of Russian government was influenced more by war than any other factor using a variety of evidence. The horrific impact of the First World War, both at the front and at home, sealed the fate of the Romanovs and, in turn, the Provisional Government in 1917. Arguably the appeal of the Bolsheviks in 1917 and the triumph of Lenin were directly related to the impact of the First World War. War can therefore be viewed as the prime cause of the end of autocratic government and the failure of the temporary move towards constitutional government. Defeat in the Crimean War can be seen as the trigger for Alexander II’s programme of reform and the introduction of Zemstva as a new system of post-Emancipation local government. Similarly the Russo-Japanese War led to Nicholas II’s announcement of the October Manifesto and the formation of the Duma. In a pure sense, this was the abandonment of absolutism. Arguably, victory in the Second World War entrenched Stalin’s dictatorial power and had a brutal impact on the government of many of the outlying ‘republics’ of the USSR. However, candidates may argue that revolution had a greater impact on the development of Russian government than war. The impact of the First World War was not the only cause of either the October or the February revolutions of 1917. Candidates may choose to argue that the revolutions themselves were multi-causal and that they rather than war had the most important impact on the development of Russian government in this period. October 1917 and the triumph of Bolshevism crushed all possibility that a liberal democracy might emerge in Russia and transformed Russia into the Soviet Union – the world’s first communist state.

60 Candidates are expected to demonstrate understanding of the issues in each of their selected questions over a period of at least a hundred years (unless an individual question specifies a slightly shorter period.) Candidates are reminded of the synoptic nature of the Unit. Answers are required to demonstrate understanding of the processes of historical continuity, development and change across the full breadth of the period studied. Assessors must be open to alternative approaches. If in doubt, they should consult their Team Leader.

Page 437: History A - The Bicester Schoolthebicesterschool.org.uk/wp-content/uploads/2015/09/Mark... · 2015-09-28 · History A Mark Schemes for the Units January 2009 H106/H506/MS/R/09J

F966/02 Mark Scheme January 2013

18

Question Answer Marks Guidance The February Revolution ended over 300 years of Romanov rule and the

establishment of the Provisional Government intent on bringing constitutional government to Russia. The 1905 revolution may have been triggered by the Russo-Japanese War but had many other causes and arguably it was fear of being overthrown that led to Nicholas II accepting the suggestion of the October Manifesto and the introduction of the Duma. However, candidates may argue that this made little significant change to how Nicholas II wielded power. Some candidates may argue that whilst the revolutions of 1917 swept aside the Romanovs and introduced Bolshevism, they had a limited impact on the development of Russian government as one form of autocracy was replaced by another.

Page 438: History A - The Bicester Schoolthebicesterschool.org.uk/wp-content/uploads/2015/09/Mark... · 2015-09-28 · History A Mark Schemes for the Units January 2009 H106/H506/MS/R/09J

F966/02 Mark Scheme January 2013

19

Question Answer Marks Guidance 11 Candidates should focus on the similarities and differences between the

condition of the peasantry and the treatment that the peasants received, both before and after 1917. Candidates may well argue that neither the Tsars nor the communist leaders consistently improved the lives of the peasants. The peasantry made little sustained progress in many ways during this period and that predominantly their living and working conditions remained bleak. However to answer this question effectively it is essential that a synoptic comparison is made. To support the assertion in the question candidates are likely to focus on the Emancipation of the serfs in 1861. Candidates may well choose to identify different aspects of peasant life and compare the Tsars and the communists on each one. These aspects might include land ownership, food supply and famine, financial burdens, requisition and taxation, social reform, regulation and control or the pressure to leave the countryside for the towns. However candidates are also likely to argue that emancipation had only a limited impact on improving the lives of the peasants because of the terms. Candidates may further support the assertion by reference to the establishment of a Peasants Land Bank in the 1880s and improvements in their lives made possible by Stolypin’s Kulak policy after 1905. However, many candidates may argue in favour of the Romanovs largely because they feel that even less was done to improve their lives by the communist leaders. Those candidates are likely to support their arguments with the consequences of War Communism under Lenin and collectivisation and de-kulakisation under Stalin. Candidates may argue that collectivisation was a ‘second serfdom’. Before and after 1917 there were famines, eg 1891, 1921 & 1932, regardless of regime, but candidates may argue that Stalin’s denial of the famine of the 1930s made its impact worse then. To argue against the assertion in the question candidates are likely to suggest that emancipation was so flawed that the lives of the peasants did not improve and, as a consequence of the rapidly rising population arguably deteriorated through time. They are likely to view the appointment of Land Captains under Alexander III as marking a significant deterioration in the lives of peasants who at the same time were being squeezed dry by taxation to finance Witte’s ‘Great Spurt’. Candidates are likely to argue that Lenin’s Decree on Land in 1917 marked a definite improvement and may suggest that War Communism was simply a temporary deterioration made essential by the Civil War. These candidates are likely to assert that the

60 Candidates are expected to demonstrate understanding of the issues in each of their selected questions over a period of at least a hundred years (unless an individual question specifies a slightly shorter period.) Candidates are reminded of the synoptic nature of the Unit. Answers are required to demonstrate understanding of the processes of historical continuity, development and change across the full breadth of the period studied. Assessors must be open to alternative approaches. If in doubt, they should consult their Team Leader.

Page 439: History A - The Bicester Schoolthebicesterschool.org.uk/wp-content/uploads/2015/09/Mark... · 2015-09-28 · History A Mark Schemes for the Units January 2009 H106/H506/MS/R/09J

F966/02 Mark Scheme January 2013

20

Question Answer Marks Guidance NEP from 1921 marked a return to their lives improving as they were

encouraged to become Kulaks and enrich themselves. Certainly the communists did much more to introduce social reform, for example in the sphere of education, than the Tsars. Candidates may argue that whilst some peasants suffered dreadfully under Stalin because of collectivisation and de-kulakisation the survivors had significantly better health care and education than their predecessors. Candidates are likely to argue that Khrushchev’s development of more consumer goods also did improve the lives of the peasants, though his Virgin Lands policy is likely to be judged an ill-considered mistake.

Page 440: History A - The Bicester Schoolthebicesterschool.org.uk/wp-content/uploads/2015/09/Mark... · 2015-09-28 · History A Mark Schemes for the Units January 2009 H106/H506/MS/R/09J

F966/02 Mark Scheme January 2013

21

Question Answer Marks Guidance 12 Candidates should focus on whether all the Russian rulers had the same aims in

their answers. To be successful candidates should focus on aims, what each ruler was trying to achieve. Their policies and outcomes are only relevant if candidates refer to them in the context of what the ruler was aiming to achieve. Reference to policies and outcomes should only be rewarded where they are used to illustrate and confirm the aims of rulers. Candidates are likely to want to argue both for and against the view in this question. Candidates may well argue that retention of power, whether autocratic or dictatorial, and the crushing of opposition were priorities for all the rulers even if some were singularly unsuccessful in achieving those goals. Candidates may well argue that the modernisation of Russia was an aim for all the rulers, though candidates are likely to differentiate between rulers such as Alexander III and Stalin in terms of motives and extent. Candidates are also likely to argue that commonly the aim of modernisation was to improve Russia’s military strength in order to either win or not be defeated in future wars. Candidates may however wish to argue that the communist rulers had very different core priorities to the Tsars in terms of political ideology and social priorities; others may contend that this should have been the case but that rulers, especially Stalin (though some will also indict Lenin) used Marxism as little more than a fig-leaf to cover their rampant desire for personal power. Candidates may argue that the Tsars were not uniform in their core aims; they are likely to see Alexander II as having different priorities to his successors, citing emancipation and the other reforms of the 1860s in support. Candidates may also argue that the communist rulers were not uniform in their core aims either; they are likely to argue that Khrushchev had very different priorities to Stalin, citing de-stalinisation as support. Candidates may well understand that whether Lenin and Stalin had similar aims is subject to significant historical debate. However while knowledge of this debate should be rewarded, it is not required for candidates to achieve the highest level or mark. Candidates are likely to argue that the leaders of the Provisional Government had some significantly different aims to all the other rulers, citing their plans for a constituent assembly for example as evidence of them aiming to establish a more democratic Russia, but they may see some continuity in aims with Tsar Nicholas II because of their continued involvement in the First World War.

60 Candidates are expected to demonstrate understanding of the issues in each of their selected questions over a period of at least a hundred years (unless an individual question specifies a slightly shorter period.) Candidates are reminded of the synoptic nature of the Unit. Answers are required to demonstrate understanding of the processes of historical continuity, development and change across the full breadth of the period studied. Assessors must be open to alternative approaches. If in doubt, they should consult their Team Leader.

Page 441: History A - The Bicester Schoolthebicesterschool.org.uk/wp-content/uploads/2015/09/Mark... · 2015-09-28 · History A Mark Schemes for the Units January 2009 H106/H506/MS/R/09J

F966/02 Mark Scheme January 2013

22

Question Answer Marks Guidance 13 Key developments of the Eisenhower administration include:

1954 Brown v Board of Education Supreme Court decision ordering desegregation of schools; Murder of Emmett Till, 1955; emergence of King as leader of the non-violent mass protest in the Montgomery Bus boycott of 1954–56; formation of the SCLC in 1957 and SNCC in 1960; Civil Rights Acts of 1957 and 1960; Eisenhower’s role in the Little Rock crisis of 1957; Sit-ins of 1960. Alternative turning points include: Reconstruction period (1865–77): important constitutional amendments: 1865, 13th abolishing slavery; 1868, 14th guaranteeing civil rights; 1870, 15th guaranteeing the right to vote; significant African American participation in politics, supported by the Radical Republicans in Congress; Freedman’s Bureau (until 1872) in providing legal assistance for former slaves and helping to establish schools. 1890s: establishment of Jim Crow in the South, taking advantage of the end of Reconstruction and a series of Supreme Court judgments (1873 Slaughterhouse Case; 1883 Civil Rights Cases; 1896 Plessy v Ferguson) which undermined the 14th and 15th amendments. Second World War: expansion of the economy providing job opportunities for AA workers and migration northwards; number of AAs serving in the armed forces; expansion of grass-roots activism – founding of CORE in 1942, the creation of the FEPC in response to Randolph’s threatened march on Washington, the ‘double V’ campaign; discrediting of racism – Hitler’s camps and Japanese victories over Europeans; Nuremberg trials, the UNO, and the development of culture of human rights. 1960s Civil Rights Acts of 1964 and 1968 and the Voting Rights Act of 1965 – legislative milestones ending Jim Crow; Great Society help for poor inner-city African Americans; the Black Power movement, the activities of the Black Panthers, the urban rioting, the assassinations of Malcolm X and Martin Luther King and the damage done to LBJ’s Great Society programme by the Vietnam War.

60 Candidates are expected to demonstrate understanding of the issues in each of their selected questions over a period of at least a hundred years (unless an individual question specifies a slightly shorter period.) Candidates are reminded of the synoptic nature of the Unit. Answers are required to demonstrate understanding of the processes of historical continuity, development and change across the full breadth of the period studied. Assessors must be open to alternative approaches. If in doubt, they should consult their Team Leader.

Page 442: History A - The Bicester Schoolthebicesterschool.org.uk/wp-content/uploads/2015/09/Mark... · 2015-09-28 · History A Mark Schemes for the Units January 2009 H106/H506/MS/R/09J

F966/02 Mark Scheme January 2013

23

Question Answer Marks Guidance Issues which might be covered from the 1970–92 period include:

Civil Rights campaigners forced onto the defensive to maintain legislative achievements and welfare programmes; bussing, affirmative action programmes controversial; emergence of successful black middle class and increased political participation contrasted with the continued deprivation, discrimination and low aspirations of many poor AAs; Reagan cuts funding for enforcement agencies and welfare programmes; Supreme Court less supportive (1974: Milliken v Bradley, 1978: Bakke v University of California, 1984: Grove City v Bell, 1992: Freeman v Pitts); Conservative appointments to SC – 1986: Rehnquist Chief Justice, 1991: Clarence Thomas (black conservative) appointed; but Democrat dominated Congress largely supportive – 1983: MLK birthday as public holiday; 1986: Economic sanctions imposed on South Africa over Reagan’s veto; 1987: Rejection of Bork’s nomination to SC; 1988 Civil Rights Restoration Act, reversing SC judgement in Grove City v Bell, passed over Reagan’s veto; Reagan fails to overturn LBJ’s Executive Order 11246 requiring federal employers to implement affirmative action; Jesse Jackson’s bids for Democratic presidential nominations in 1984 and 1988.

Page 443: History A - The Bicester Schoolthebicesterschool.org.uk/wp-content/uploads/2015/09/Mark... · 2015-09-28 · History A Mark Schemes for the Units January 2009 H106/H506/MS/R/09J

F966/02 Mark Scheme January 2013

24

Question Answer Marks Guidance 14 Candidates will probably argue that this was not true of the period up to the

Great Depression but that the Federal government was supportive during the New Deal. They will probably be aware that, after World War Two, the Federal government became less supportive of trade union rights but (under Democratic administrations at least) supportive of workers’ rights. Examples of Federal hostility before the New Deal include Laissez-faire assumptions about the role of government which helped to

create a climate of hostility to organised labour, especially during the ‘red scare’ that followed each world war

President Cleveland’s use of federal troops to suppress the 1894 Pullman Strike

1895 Supreme Court decision upholding the use of injunctions against trade unions under the Sherman Anti-Trust Act

1905 Lochner Case (striking down a New York law setting a maximum number of working hours for bakery workers)

1908 Adair Case (upholding ‘yellow dog’ contracts by which workers were prevented from joining a union)

Use of the 1917 Espionage Act to suppress ‘The Wobblies’ (Industrial Workers of the World)

1921 ruling declaring unconstitutional the 1914 Clayton Act (which aimed to guarantee workers’ rights to organize, bargain collectively, strike, boycott and picket)

The best candidates might also show that the three branches of the federal government were not always in harmony: in 1921 the Supreme Court struck down the Clayton Act and in 1935 the Schechter Case ended FDR’s NIRA. Both the Smith-Connally Act and the Taft-Hartley Act were passed over presidential vetoes.

Examples of the post-war climate unsympathetic to organised labour include 1943 Smith-Connally Act preventing strike action in industrial plant

producing war materials 1947 Taft-Hartley Act allowing states to pass ‘right to work’ laws banning

the 'closed shop'  

60 Candidates are expected to demonstrate understanding of the issues in each of their selected questions over a period of at least a hundred years (unless an individual question specifies a slightly shorter period.) Candidates are reminded of the synoptic nature of the Unit. Answers are required to demonstrate understanding of the processes of historical continuity, development and change across the full breadth of the period studied. Assessors must be open to alternative approaches. If in doubt, they should consult their Team Leader.

Page 444: History A - The Bicester Schoolthebicesterschool.org.uk/wp-content/uploads/2015/09/Mark... · 2015-09-28 · History A Mark Schemes for the Units January 2009 H106/H506/MS/R/09J

F966/02 Mark Scheme January 2013

25

Question Answer Marks Guidance 1959 Landrum-Griffin Act banning secondary picketing

The impact on trade union power, membership and strike activity of Reagan’s 1981 defeat of the PATCO strike

Reagan’s policies of lower taxes and business deregulation were part of a deliberate rejection of the New Deal philosophy in the 1980s.

In dealing with Federal support, candidates might refer to Attempts during the presidencies of Theodore Roosevelt and Woodrow

Wilson to legislate on such matters as working conditions, consumer protection, housing and education

The key New Deal measures: the National Labour Relations Act of 1935 (Wagner Act), the NIRA (1933), the Social Security Act (1935) and the Fair Labour Standards Act (1938). Better candidates might also point out that FDR’s response to the labour militancy of 1936–7 contrasts sharply with that of earlier administrations

The creation during WW2 of the FEPC and the National War Labour Board

The support given, especially by post-war Democratic presidents, to workers: Truman’s ‘Fair Deal’, JFK’s 'New Frontier' and LBJ’s 'Great Society' programmes aimed to build on FDR’s New Deal and brought benefits to working people – support for a minimum wage, economic regeneration measures, improved housing and medical care and better work opportunities

Nixon’s support for affirmative action and Carter’s extension of the minimum wage are examples of presidential support for labour rights.

Page 445: History A - The Bicester Schoolthebicesterschool.org.uk/wp-content/uploads/2015/09/Mark... · 2015-09-28 · History A Mark Schemes for the Units January 2009 H106/H506/MS/R/09J

F966/02 Mark Scheme January 2013

26

Question Answer Marks Guidance 15 Evidence of Native American activism includes

Indian Wars of the late 19th century – unsuccessful defence of their rights NA contribution to the US war effort in the two world wars influencing

subsequent federal policy – the granting of US citizenship in 1924 and the formation of the Indian Claims Commission in 1946

Society of American Indians (SAI, established in 1911), the National Congress of American Indians (NCAI, established in 1944), American Indian Movement (AIM, established in 1968) to preserve Indian customs and culture and exert pressure on federal and state authorities

Opposition to termination policy of 1950s helped to end it ‘Red Power’ in the late 1960s and early 1970s (the occupation of Alcatraz

in 1969, the occupation of the BIA in 1972, and the protest at Wounded Knee in 1973)

Actions by individual Indian nations to gain greater economic wealth through court action against the US Federal government (eg Passamaquaddy Indians of Maine in the 1970s) or in developing their reservations (Mohawks of New York or Jicarilla Apaches of New Mexico).

Most candidates are likely to argue that the federal government did much more than NAs themselves to advance their civil rights. They might mention the following 1934 Indian Reorganisation Act inaugurating the Indian New Deal –

recognition of Indian separate identity and right to self-government under the BIA; allotment ended, some land restored

1946 Indian Claims Commission – some recognition of illegal land loss in 19th century

1968 Indian Civil Rights Act – guaranteed individual rights under US Constitution to Indians (some able candidates might interpret this as limiting tribal collective rights)

1975 Indian Self-Determination and Education Assistance Act – Indians given increased responsibility for educational and other programmes previously administered for them.

60 Candidates are expected to demonstrate understanding of the issues in each of their selected questions over a period of at least a hundred years (unless an individual question specifies a slightly shorter period.) Candidates are reminded of the synoptic nature of the Unit. Answers are required to demonstrate understanding of the processes of historical continuity, development and change across the full breadth of the period studied. Assessors must be open to alternative approaches. If in doubt, they should consult their Team Leader.

Page 446: History A - The Bicester Schoolthebicesterschool.org.uk/wp-content/uploads/2015/09/Mark... · 2015-09-28 · History A Mark Schemes for the Units January 2009 H106/H506/MS/R/09J

F966/02 Mark Scheme January 2013

27

Question Answer Marks Guidance The best answers might point out the lack of clarity about what Native American

civil rights actually are and discuss the impact of economic and social changes of the mid to late 20th century on Native Americans. Some want recognition of their separate culture and traditions while others, particularly those who have moved to the cities and/or intermarried, wish to assimilate into mainstream US culture and overcome de facto racial prejudice over such issues as employment and housing. Candidates might also analyse the lack of consistency in federal Indian policy and will be able to discuss the current legal and economic status of the reservations.

Page 447: History A - The Bicester Schoolthebicesterschool.org.uk/wp-content/uploads/2015/09/Mark... · 2015-09-28 · History A Mark Schemes for the Units January 2009 H106/H506/MS/R/09J

F966/02 Mark Scheme January 2013

28

Question Answer Marks Guidance 16 Candidates should examine the reasons for democratic change, particularly

whether it occurred because of reforming governments and party fighting in parliament and outside or whether outside pressure groups and popular movements were the key to moving governments and MPs in a democratic direction. Pressure groups could range from Trade Unions, to single cause groups like the Ballot Society and the Suffragettes. Candidates should examine key moments of change (the franchise in the 1880s, 1910s, 1928; local government in the 1880s; Lords reform in 1910/11; Welfare Reforms in the 1940s; equal rights legislation in the 1960s and 1970s). Did these occur because of pressure groups and their ability to mobilise opinion or because of parliamentary and political pressures or general shifts in public opinion? The Suffragettes could be a good example to take – did they help or hinder the cause of votes for women? Other test cases selected might be the relative importance of the Dagenham women strikers in the 1970s in persuading the Labour government to move on equal pay issues. Was it already on Labour’s agenda? The 1918 Reform Act could be seen as the product of Liberal and Labour pressure, the propaganda of wartime cohesion or simply the political calculations of Lloyd George’s Coalition government? Was outside pressure an issue at this point? Earlier examples could be Joe Chamberlain’s Education League or Nonconformist pressure on a variety of civil equality issues. The Trade Unions would be a useful example to take across the period – from their role in pushing for Union rights in the 1870s, their 1st World War deals with government over labour practices, their pressure for mining reform in the 1920s, the Jarrow Marchers attempt to persuade the National Government to change economic and social policy and Trades Union pressure in general on economic and social policy from 1926 to the 1980s. It might be noted that they were able to apply pressure both within and through parliament given their sponsorship of Labour MPs from 1900.

60 Candidates are expected to demonstrate understanding of the issues in each of their selected questions over a period of at least a hundred years (unless an individual question specifies a slightly shorter period.) Candidates are reminded of the synoptic nature of the Unit. Answers are required to demonstrate understanding of the processes of historical continuity, development and change across the full breadth of the period studied. Assessors must be open to alternative approaches. If in doubt, they should consult their Team Leader.

Page 448: History A - The Bicester Schoolthebicesterschool.org.uk/wp-content/uploads/2015/09/Mark... · 2015-09-28 · History A Mark Schemes for the Units January 2009 H106/H506/MS/R/09J

F966/02 Mark Scheme January 2013

29

Question Answer Marks Guidance 17 The argument that regional politics were less important after 1918 could be

made with reference to the importance of Wales, Scotland and Ireland to the Liberal party and its dominance in the pre 1918 period (1868–1886 and 1905–1916) and to the Labour party after 1918. Ulster was important to the Conservatives from 1886–1905 and again in the 1910 period and beyond. Candidates could examine why those areas became identified with the two parties – in Wales, from 1868, a rebellion against Conservative squire and parson which Gladstone’s plural vision embraced; in Scotland a continuing Whig tradition which rejected Conservatism as an English party; in Ireland a Liberal association with the Irish Nationalists under Parnell and Redmond which saw them as the handmaidens of Liberalism throughout the 1868–1918 period, despite occasional dalliances with the Conservatives in 1885–6. In English terms the nonconformist hold on many northern towns helped the Liberals (local press; temperance etc) and local government via the caucus was useful for whosoever could exploit it (Chamberlain’s Birmingham). However it could be argued that regional politics remained important after 1918. Scotland became important for the Conservatives until Thatcher as it did for Labour until Blair. Wales became a Labour stronghold from 1908. Ulster could usually be relied upon for the Conservatives. In much diminished form Scotland retained some important seats for the Liberals, as did the West Country. Again candidates could identify the reasons for this. The argument that regional politics was less important (both before and after 1918) is that national parties had emerged by 1868 and that English dominance in the Commons would always determine electoral outcome. The rise of class allegiance over regional identity was gradual but very obvious post 1900. It explained why Labour came to have such a hold in the Scottish industrial lowlands (Red Clydeside), the South Welsh coalmines and the industrial cities of Lancashire and Yorkshire (minus Liverpool before 1945) and why Conservatism dominated the suburbs, rural areas and southern England. The loss of Ireland outside Ulster from 1918 helps to explain Liberal collapse, as does the decline of Northern nonconformity. The rise of a national press as opposed to a regional one, a national radio and TV service after 1922 and 1945 respectively also eroded regional political loyalties.

60 Candidates are expected to demonstrate understanding of the issues in each of their selected questions over a period of at least a hundred years (unless an individual question specifies a slightly shorter period.) Candidates are reminded of the synoptic nature of the Unit. Answers are required to demonstrate understanding of the processes of historical continuity, development and change across the full breadth of the period studied. Assessors must be open to alternative approaches. If in doubt, they should consult their Team Leader.

Page 449: History A - The Bicester Schoolthebicesterschool.org.uk/wp-content/uploads/2015/09/Mark... · 2015-09-28 · History A Mark Schemes for the Units January 2009 H106/H506/MS/R/09J

F966/02 Mark Scheme January 2013

30

Question Answer Marks Guidance 18 A variety of arguments are possible here. It is likely that many candidates will

argue that Unions helped democracy up to the early 20th century (to 1926? to 1945?) but then were perceived by more and more as an unelected power challenging elected democratic government, particularly from 1950 to the early 1980s. Up to 1926 they pushed for legal equality, were seen as forces for stability and the socialisation of the working man (but not the working woman), campaigned for better conditions, shorter hours and better pay in a widening set of industries (from skilled engineers to the New Unions of the 1880s) and became involved in politics via Liberalism and then independent labour in the 1890s. They had become a major force by the Triple Alliance of 1913 and negotiated with government in the 1st World War. After the war they continued to champion fairness of sacrifice in the 1920s and 1930s (mines and then railways) and played a huge role in the 2nd World War. At that point the TUC negotiated directly with Conservative and Labour governments on national pay (beer and sandwiches in Number 10). Their control of key industries (mining, transport, iron and steel and cars) via shop stewards and the use of strikes, primary and secondary, gave them great power in an era of mid century economic growth and labour shortage that was only half heartedly challenged by Barbara Castle in the 1970s and by Heath in the early 1970s. Thatcher saw them as obstacles to economic democracy in the 1980s and acted accordingly. However candidates can challenge this view using examples from across the period. Pre 1914 the Unions remained reluctant to get involved in democratic issues and Labour politics. They were led by men who were reluctant to do anything about the unskilled residuum (as Tillet pointed out) and women (as Besant pointed out). Unions remained undemocratic in their structures. Post 1918 the Unions conspired to remove women from work and there is an alternative view of the General Strike – that of the government and much middle class opinion – that the Miners sought political and syndicalist conflict. Post 1945 and especially in the 1980s there were conflicting views on how far the Unions attempted to dragoon their members and how far some deliberately sought conflict with the Conservative governments of Heath and Thatcher. Certainly the Union reforms of the 1980s removed many of the privileges/rights gained in the 1870s on the grounds that they were undemocratic.

60 Candidates are expected to demonstrate understanding of the issues in each of their selected questions over a period of at least a hundred years (unless an individual question specifies a slightly shorter period.) Candidates are reminded of the synoptic nature of the Unit. Answers are required to demonstrate understanding of the processes of historical continuity, development and change across the full breadth of the period studied. Assessors must be open to alternative approaches. If in doubt, they should consult their Team Leader.

Page 450: History A - The Bicester Schoolthebicesterschool.org.uk/wp-content/uploads/2015/09/Mark... · 2015-09-28 · History A Mark Schemes for the Units January 2009 H106/H506/MS/R/09J

Oxford Cambridge and RSA Examinations is a Company Limited by Guarantee Registered in England Registered Office; 1 Hills Road, Cambridge, CB1 2EU Registered Company Number: 3484466 OCR is an exempt Charity OCR (Oxford Cambridge and RSA Examinations) Head office Telephone: 01223 552552 Facsimile: 01223 552553 © OCR 2013

OCR (Oxford Cambridge and RSA Examinations) 1 Hills Road Cambridge CB1 2EU OCR Customer Contact Centre Education and Learning Telephone: 01223 553998 Facsimile: 01223 552627 Email: [email protected] www.ocr.org.uk For staff training purposes and as part of our quality assurance programme your call may be recorded or monitored

Page 451: History A - The Bicester Schoolthebicesterschool.org.uk/wp-content/uploads/2015/09/Mark... · 2015-09-28 · History A Mark Schemes for the Units January 2009 H106/H506/MS/R/09J

Oxford Cambridge and RSA Examinations

GCE

History A Advanced GCE

Unit F966/02: Historical Themes Option B: Modern 1789–1997

Mark Scheme for June 2013

Page 452: History A - The Bicester Schoolthebicesterschool.org.uk/wp-content/uploads/2015/09/Mark... · 2015-09-28 · History A Mark Schemes for the Units January 2009 H106/H506/MS/R/09J

OCR (Oxford Cambridge and RSA) is a leading UK awarding body, providing a wide range of qualifications to meet the needs of candidates of all ages and abilities. OCR qualifications include AS/A Levels, Diplomas, GCSEs, Cambridge Nationals, Cambridge Technicals, Functional Skills, Key Skills, Entry Level qualifications, NVQs and vocational qualifications in areas such as IT, business, languages, teaching/training, administration and secretarial skills. It is also responsible for developing new specifications to meet national requirements and the needs of students and teachers. OCR is a not-for-profit organisation; any surplus made is invested back into the establishment to help towards the development of qualifications and support, which keep pace with the changing needs of today’s society. This mark scheme is published as an aid to teachers and students, to indicate the requirements of the examination. It shows the basis on which marks were awarded by examiners. It does not indicate the details of the discussions which took place at an examiners’ meeting before marking commenced. All examiners are instructed that alternative correct answers and unexpected approaches in candidates’ scripts must be given marks that fairly reflect the relevant knowledge and skills demonstrated. Mark schemes should be read in conjunction with the published question papers and the report on the examination. OCR will not enter into any discussion or correspondence in connection with this mark scheme. © OCR 2013

Page 453: History A - The Bicester Schoolthebicesterschool.org.uk/wp-content/uploads/2015/09/Mark... · 2015-09-28 · History A Mark Schemes for the Units January 2009 H106/H506/MS/R/09J

F966/02 Mark Scheme June 2013

1

Subject-specific Marking Instructions

Distribution of marks for each level that reflects the Unit’s AOs 2 answers: Each maximum mark 60

AO1a AO1b IA 18–20 36–40

IB 16–17 32–35

II 14–15 28–31

III 12–13 24–27

IV 10–11 20–23

V 8–9 16–19

VI 4–7 8–15

VII 0–3 0–7

Page 454: History A - The Bicester Schoolthebicesterschool.org.uk/wp-content/uploads/2015/09/Mark... · 2015-09-28 · History A Mark Schemes for the Units January 2009 H106/H506/MS/R/09J

F966/02 Mark Scheme June 2013

2

Notes: (i) Allocate marks to the most appropriate level for each AO. (ii) If several marks are available in a box, work from the top mark down until the best fit has been found. (iii) Many answers will not fall at the same level for each AO. (iv) Candidates will demonstrate synoptic skills by drawing together appropriate techniques, knowledge and understanding to evaluate developments over the whole of the period

AOs AO1a AO1b Total mark for each question = 60

Recall, select and deploy historical knowledge appropriately, and communicate knowledge and understanding of history in a clear and effective manner.

Demonstrate understanding of the past through explanation, analysis and arriving at substantiated judgements of: - key concepts such as causation, consequence, continuity, change

and significance within an historical context; - the relationships between key features and characteristics of the

periods studied Level IA

Uses a wide range of accurate and relevant evidence

Accurate and confident use of appropriate historical terminology

Answer is clearly structured and coherent; communicates accurately and legibly.

18–20

Excellent understanding of key concepts (eg continuity and change) relevant to analysis in their historical context

Excellent synthesis and synoptic assessment Answer is consistently and relevantly analytical with developed

explanations and supported judgements May make unexpected but substantiated connections over the whole

period. 36–40

Level IB

Level IB Uses accurate and relevant evidence Accurate use of a range of appropriate historical

terminology Answer is clearly structured and mostly

coherent; communicates accurately and legibly. 16–17

Very good level of understanding of key concepts (eg continuity and change) in their historical context

Answer is consistently focused on the question set Very good level of explanation/analysis, and provides supported

judgements Very good synthesis and synoptic assessment of the whole period.

32–35

Page 455: History A - The Bicester Schoolthebicesterschool.org.uk/wp-content/uploads/2015/09/Mark... · 2015-09-28 · History A Mark Schemes for the Units January 2009 H106/H506/MS/R/09J

F966/02 Mark Scheme June 2013

3

AOs AO1a AO1b Level II

Uses mostly accurate and relevant evidence Generally accurate use of historical terminology Answer is structured and mostly coherent;

writing is legible and communication is generally clear.

14–15

Good level of understanding of key concepts (eg continuity and change) in their historical context

Good explanation/analysis but overall judgements may be uneven Answer is focused on the issues in the question set Good synthesis and assessment of developments over most of the

period. 28–31

Level III Uses relevant evidence but there may be some inaccuracy

Answer includes relevant historical terminology but this may not be extensive or always accurately used

Most of the answer is structured and coherent; writing is legible and communication is generally clear.

12–13

Shows a sound understanding of key concepts, especially continuity and change, in their historical context

Most of the answer is focused on the question set Answers may be a mixture of analysis and explanation but also

description and narrative, but there may also be some uneven overall judgements; OR answers may provide more consistent analysis but the quality will be uneven and its support often general or thin

Answer assesses relevant factors but provides only a limited synthesis of developments over most of the period.

24–27 Level IV

There is deployment of relevant knowledge but level/accuracy will vary

Some unclear and/or underdeveloped and/or disorganised sections

Mostly satisfactory level of communication.

10–11

Satisfactory understanding of key concepts (eg continuity and change) in their historical context

Satisfactory focus on the question set Answer may be largely descriptive/narratives of events, and links

between this and analytical comments will typically be weak or unexplained

Makes limited synoptic judgements about developments over only part of the period.

20–23

Page 456: History A - The Bicester Schoolthebicesterschool.org.uk/wp-content/uploads/2015/09/Mark... · 2015-09-28 · History A Mark Schemes for the Units January 2009 H106/H506/MS/R/09J

F966/02 Mark Scheme June 2013

4

AOs AO1a AO1b Level V

General and basic historical knowledge but also some irrelevant and inaccurate material

Often unclear and disorganised sections Adequate level of communication but some weak

prose passages

8–9

General understanding of key concepts (eg continuity and change) in their historical context

Some understanding of the question but answers may focus on the topic and not address the question set OR provides an answer based on generalisation

Attempts an explanation but often general coupled with assertion, description/narrative

Very little synthesis or analysis and only part(s) of the period will be covered.

16–19 Level VI Use of relevant evidence will be limited; there will

be much irrelevance and inaccuracy Answers may have little organisation or structure Weak use of English and poor organisation.

4–7

Very little understanding of key concepts (eg continuity and change) in their historical context

Limited perhaps brief explanation Mainly assertion, description/narrative Some understanding of the topic but not the question’s

requirements. 8–15

Level VII Little relevant or accurate knowledge Very fragmentary and disorganised response Very poor use of English and some incoherence.

0–3

Weak understanding of key concepts (eg continuity and change) in their historical context

No explanation Assertion, description/narrative predominate Weak understanding of the topic or of the question’s requirements.

0–7

Page 457: History A - The Bicester Schoolthebicesterschool.org.uk/wp-content/uploads/2015/09/Mark... · 2015-09-28 · History A Mark Schemes for the Units January 2009 H106/H506/MS/R/09J

F966/02 Mark Scheme June 2013

5

Question Answer Marks Guidance 1 Candidates should focus on ‘the development of German nationalism’ and ‘the

impact of industrialisation’ in their answers. The impact of industrialisation may be evaluated against the other dominant factors in the development of German nationalism during this period, for example the role played by German nationalists and by significant individuals such as Bismarck and Kaiser Wilhelm II. However candidates may also only analyse the impact of industrialisation, and either approach is acceptable. In support of the view in the question candidates are likely to explain the impact of industrialisation on the development of German nationalism, for example the impact of the founding and development of the Krupp Gusstahlfabrik (Cast Steel Works) from 1811 and the impact of the Zollverein after 1834. Candidates are likely to demonstrate an understanding of the link between industrialisation, Prussia’s growing economic power and Prussia’s pivotal role in the creation and development of the German Empire. The development of the railways may be seen as significant. Candidates are likely to understand how developments in the economy in the 1850s paved the way for the Prussian military victories of 1864, 1866 and 1870/71 and the creation of the Second Reich. Military strength depended upon industrialisation: ‘Coal & Iron’ rather than ‘Blood & Iron’ could be usefully debated. This though also had a limiting effect on the development of German nationalism as Prussia was able to exclude Austria, first from the Zollverein and then from Germany. This led to the creation in 1871 of Kleindeutschland, thus thwarting the ambitions of those nationalists who aspired to Grossdeutschland. The impact of the extraordinary developments in the German economy after 1871 should be discussed. By the late 1880s the manufacture of armaments represented around 50% of Krupp's total output, for example. Candidates may well argue that the quickening pace of industrialization led to urbanization and the development of increasingly radical socialism. Socialism gave the working class an alternative loyalty to patriotism and nationalism. Arguably this had a negative impact on the development of German nationalism. This in turn led Kaiser Wilhelm II to embark on a foreign policy aimed at distracting the workers from their grievances. It could be argued that this populist foreign policy fanned the flames of German nationalism. Candidates may also argue that this radical nationalism had a significantly negative impact on the development of German nationalism as entry into the First World War left the German nation defeated and deeply divided by 1919.

60 Candidates are expected to demonstrate understanding of the issues in each of their selected questions over a period of at least a hundred years (unless an individual question specifies a slightly shorter period.) Candidates are reminded of the synoptic nature of the Unit. Answers are required to demonstrate understanding of the processes of historical continuity, development and change across the full breadth of the period studied. Assessors must be open to alternative approaches. If in doubt, they should consult their Team Leader.

Page 458: History A - The Bicester Schoolthebicesterschool.org.uk/wp-content/uploads/2015/09/Mark... · 2015-09-28 · History A Mark Schemes for the Units January 2009 H106/H506/MS/R/09J

F966/02 Mark Scheme June 2013

6

Question Answer Marks Guidance 2 Candidates should compare the importance of the aims and ideas of German

nationalism with the impact that Prussia had on the development of a united Germany. Candidates should focus on ‘more important’ in their answers. Candidates may well draw comparisons between the emergence of German nationalism in the Napoleonic period with Prussia’s role in the defeat of Napoleon. Candidates are likely to show knowledge of developments in intellectual nationalism in the first half of the Nineteenth Century. Candidates may consider the impact that the Prussian Zollverein had on the development of nationalism, but may argue that in the period between 1815 and 1862 the emerging ideas of German nationalism were more important to the development of a united Germany than Prussian ambitions. Candidates may contrast this with the role played by Prussia in the 1860s in forging the new Germany. Candidates may discuss the impact of the 1871 Constitution. Candidates may demonstrate that they understand that the German Empire in 1871 represented Kleindeutschland and an enlarged Prussia rather than a united Germany. They may argue that it was a Prussian Empire rather than a German Empire. However not all German nationalists aspired to Grossdeutschland and it can be argued that the creation of the Second Reich was a crucial step forward for the aims of many German nationalists. Candidates could point to the mythical status of Bismarck in German history and/or to the popularity and mass appeal of increasingly radical nationalism in the reign of William II to argue that Prussia’s creation of the Second Reich was consistent with the aims of many German nationalists. Candidates may well discuss factors that undermined the aims of German nationalists, for example their own divisions. The 1848 Revolution may be seen as a pivotal moment by such candidates. Candidates might argue that the aims and ambitions of Prussian militarism were to ultimately set the German nation on course for disaster and humiliation by reference to Germany’s defeat in the First World War and her humiliation at Versailles.

60 Candidates are expected to demonstrate understanding of the issues in each of their selected questions over a period of at least a hundred years (unless an individual question specifies a slightly shorter period.) Candidates are reminded of the synoptic nature of the Unit. Answers are required to demonstrate understanding of the processes of historical continuity, development and change across the full breadth of the period studied. Assessors must be open to alternative approaches. If in doubt, they should consult their Team Leader.

Page 459: History A - The Bicester Schoolthebicesterschool.org.uk/wp-content/uploads/2015/09/Mark... · 2015-09-28 · History A Mark Schemes for the Units January 2009 H106/H506/MS/R/09J

F966/02 Mark Scheme June 2013

7

Question Answer Marks Guidance 3 Candidates should focus on whether wars were the main factor in the unification

of the German people and evaluate whether this was accurate throughout the period from 1789 to 1919 in their answers. Candidates must show that they understand that wars were not the sole factor in the unification of the German people in this period. The importance of wars should be evaluated against the other dominant factors in the unification of the German people during this period, for example the economy and the role played by Bismarck. In support of the view in the question, candidates may argue that the common fight of people from different German states against the French, especially in 1813, gave strong impulses to nationalism and united the German people against a common foe. They may argue that the consequences of the Napoleonic Wars, the decisions about Germany taken at the Congress of Vienna, reinforced the desire of some Germans to unite the German nation. However candidates may argue that this had limited impact as subsequently only a minority of Germans, a few intellectuals, demanded the unification of all German-speaking people. Candidates are likely to demonstrate understanding of the debate about Grossdeutschland or Kleindeutschland in the period 1815–1871 to illustrate that all German nationalists were far from united in this period and that their views were hardly representative of the German masses. Candidates are likely to argue that after 1815 Austria had a controlling role over the other German states through the German Confederation and that, especially up to 1848, Austria was successful in ensuring that the German people remained divided. Arguably therefore, the consequence of the Napoleonic Wars had a divisive influence on Germany and the German people at least through to the 1860s. Candidates are likely to argue that the wars in the 1860s were an extremely important unifying influence on Germany. Candidates may well stress the importance of the victory over Austria in 1866 in enabling Prussia to overcome a major obstacle to unification. However, some candidates may stress that this was a war between the German peoples and that one consequence was to ensure that only a Kleindeutschland would emerge by 1871. Candidates are likely to view the Franco-Prussian War as of pivotal importance in the unification of Germany, though some are likely to argue that this, and the consequences beyond 1871 and the establishment of the German Empire, can be viewed more as a take-over of Germany by Prussia than as a process of unification.

60 Candidates are expected to demonstrate understanding of the issues in each of their selected questions over a period of at least a hundred years (unless an individual question specifies a slightly shorter period.) Candidates are reminded of the synoptic nature of the Unit. Answers are required to demonstrate understanding of the processes of historical continuity, development and change across the full breadth of the period studied. Assessors must be open to alternative approaches. If in doubt, they should consult their Team Leader.

Page 460: History A - The Bicester Schoolthebicesterschool.org.uk/wp-content/uploads/2015/09/Mark... · 2015-09-28 · History A Mark Schemes for the Units January 2009 H106/H506/MS/R/09J

F966/02 Mark Scheme June 2013

8

Question Answer Marks Guidance Candidates may argue that although wars united the German Empire by 1871

they left the German people divided geographically. Candidates are also likely to argue that the continued divisions between the German people, for example on religious or social grounds, illustrate the fact that the wars of the 1860s may have been a unifying influence on Germany but they failed to unite the German people. Candidates may argue that the First World War united the German nation at first but that divisions soon arose and were entrenched by 1918. However, whilst Versailles divided the nation geographically it united the nation in condemnation and bitterness of the ‘diktat’.

Page 461: History A - The Bicester Schoolthebicesterschool.org.uk/wp-content/uploads/2015/09/Mark... · 2015-09-28 · History A Mark Schemes for the Units January 2009 H106/H506/MS/R/09J

F966/02 Mark Scheme June 2013

9

Question Answer Marks Guidance 4 The question addresses the importance of the organisation of armies in the

period and the potential advantages that superior organisational structures gave in military operations. Candidates might address the development of permanent divisional structures in the course of the Napoleonic Wars and the development of corps composed of multiple arms. This organisational structure remained the basic system used by armies throughout the period. Early in the period Napoleonic France pioneered such organisational structures and, arguably, this is one of the reasons why they had the advantage over their opponents until their European enemies followed suit.

Candidates might discuss the development of larger organisations such as the army group which first appears in the Russian campaign of 1812. Candidates might focus on the advantages of such larger groupings of soldiers demonstrated in the later wars of the 19th and 20th centuries. Candidates might also examine command and control systems – such as the French general staff – in relation to these organisational structures. This type of approach would argue that organisational efficiency gave rise to better command control and hence more effective war fighting capabilities on the part of the armies of the period.

Alternatively, candidates might opt for another ‘main factor’ in determining the outcome of wars. Such responses must, however, address and challenge the basic precept of the question. Better forms of this type of response will engage the question in a synoptic fashion interweaving alternative ‘main factors’ into the basic precept of the question. For example, weapons technology was the ‘main factor’ but it was used more effectively by armies which had better organisational structures.

60 Candidates are expected to demonstrate understanding of the issues in each of their selected questions over a period of at least a hundred years (unless an individual question specifies a slightly shorter period.) Candidates are reminded of the synoptic nature of the Unit. Answers are required to demonstrate understanding of the processes of historical continuity, development and change across the full breadth of the period studied. Assessors must be open to alternative approaches. If in doubt, they should consult their Team Leader.

Page 462: History A - The Bicester Schoolthebicesterschool.org.uk/wp-content/uploads/2015/09/Mark... · 2015-09-28 · History A Mark Schemes for the Units January 2009 H106/H506/MS/R/09J

F966/02 Mark Scheme June 2013

10

Question Answer Marks Guidance 5 Modes of communication might be dispatches delivered by men either on foot or

horseback, the visual telegraph of the Napoleonic era, the electric telegraph, the telephone and radio. In a given conflict more than one of these modes of communication might be used and the nature, sophistication and efficiency of methods of communication developed across the period as technology became more advanced.

Weaker responses might describe methods of communication and link them to developments in warfare. Better responses must, however, address the question in a synoptic manner evaluating the impact – or not – of communications on various aspects of war.

Possible themes are battle management, the application of strategy, the coordination of armies at all levels. Better responses might discuss these themes in the light of developments in the nature of warfare across the period. With regard the nature and extent of the battlefield comparing the Napoleonic Wars and WWII. In the former runners might have been totally adequate, in the latter advanced communications such as radio were vital. More advanced communications helped the application of strategy and the co-ordination of armies across larger theatres of operations. Of course, candidates might point to the failings of strategy despite better communications technology.

Another possible theme might be to discuss the effectiveness of communications in different military situations, for example the static warfare of the western front in WWII compared to more fluid styles of warfare.

Transport will probably concentrate on the application of steam power in the form of railways (and steam ships where their use applies to land warfare – the Crimean War springs to mind) in the nineteenth century and the impact of the internal combustion engine in the twentieth. Candidates need to be aware, however, that railways also had a large impact on WWII. The response must use this knowledge in an analytical fashion with focus on the specific demands of the question set. Examples might be rapid strategic movement, the ability to mobilize large numbers of soldiers, its impact on concentration of force and movement across the battlefield.

60 Candidates are expected to demonstrate understanding of the issues in each of their selected questions over a period of at least a hundred years (unless an individual question specifies a slightly shorter period.) Candidates are reminded of the synoptic nature of the Unit. Answers are required to demonstrate understanding of the processes of historical continuity, development and change across the full breadth of the period studied. Assessors must be open to alternative approaches. If in doubt, they should consult their Team Leader.

Page 463: History A - The Bicester Schoolthebicesterschool.org.uk/wp-content/uploads/2015/09/Mark... · 2015-09-28 · History A Mark Schemes for the Units January 2009 H106/H506/MS/R/09J

F966/02 Mark Scheme June 2013

11

Question Answer Marks Guidance The obvious examples of the impact of railways are the 1866 Austro-Prussian

War, the opening months of WW1 and the support of military activity on the Eastern Front in WWII. Better candidates might argue that the railway only took armies so far and once separated from rail networks soldiers moved as fast as their 18th century forebears had done. For the impact of the internal combustion engine in WW1 candidates might consider lorries, tanks and aircraft. The effect of this technology on WWII is obvious with true mechanised warfare dominating all European and North Africa fronts. For tanks, APCs and aircraft we might expect some discussion of use in battle. For pre-steam technologies use of waterways to transport troops or mass use of horse drawn carts were important. The former was a common feature of war in the later 18th and early 19th centuries, the latter was used on occasion by Napoleon, for example to move part of his army in 1805.

Page 464: History A - The Bicester Schoolthebicesterschool.org.uk/wp-content/uploads/2015/09/Mark... · 2015-09-28 · History A Mark Schemes for the Units January 2009 H106/H506/MS/R/09J

F966/02 Mark Scheme June 2013

12

Question Answer Marks Guidance 6 The question invites candidates to evaluate battlefield tactics – as opposed to

strategy – in the light of developments in weaponry. The question can be accepted, challenged or a compromise position adopted on an evaluation contingent on the point within the period being studied. No set conclusions are expected.

Candidates who challenge the question might argue that battlefield tactics were always behind developments in weaponry. For example, the increasingly effective firepower of infantry weapons throughout the 19th century to at least the Russo-Japanese war and perhaps the opening phases of WWI were not reflected in the tactics of the period which remained linear and ignored the need to disperse soldiers and to utilise cover. Candidates might use types of weaponry as themes.

Alternatively, the question might be accepted. Candidates might argue that weaponry dictated battlefield tactics and armies which entered wars with inappropriate tactics were forced to modify them swiftly in order to survive. A good example might be the Wars of Unification in the middle 19th century or WWI.

A compromise position would be to assume both sides of the argument but specify the relationship of tactics to weaponry at a specific point within the time frame of the topic.

Another possible type of response might be to argue that battlefield tactics were dictated by other factors, for example terrain, tactical situation or generalship. Alternatively, candidate might argue that tactics were dictated by the concepts encountered in the military theorists – concentration or force, maintenance of aim and the like – and were applied regardless of weapons technology.

60 Candidates are expected to demonstrate understanding of the issues in each of their selected questions over a period of at least a hundred years (unless an individual question specifies a slightly shorter period.) Candidates are reminded of the synoptic nature of the Unit. Answers are required to demonstrate understanding of the processes of historical continuity, development and change across the full breadth of the period studied. Assessors must be open to alternative approaches. If in doubt, they should consult their Team Leader.

Page 465: History A - The Bicester Schoolthebicesterschool.org.uk/wp-content/uploads/2015/09/Mark... · 2015-09-28 · History A Mark Schemes for the Units January 2009 H106/H506/MS/R/09J

F966/02 Mark Scheme June 2013

13

Question Answer Marks Guidance 7 Candidates will need to establish some criteria for success in pacifying Ireland –

law and order; winning hearts and minds for the Union; dealing with specific threats (rebellion; the problems posed by the Catholic association, the NRA, the Land League and Land issues, the nationalist challenge, obstructionism in Parliament, the threat of Civil War etc). It may be argued that earlier governments achieved pacification through coercion, being more prepared to do this than later ones, as the period 1798–1835 and the 1840s demonstrated. The Union itself was one such mechanism of pacification and there were few challenges to this until the 1820s. The Protestant Ascendency soon settled into a Westminster mode. Candidates might point to Pitt’s handling of Ireland up to 1806 as an example of successful and original pacification – suppressing Wolfe Tone but then completely changing the basis of Anglo-Irish affairs by the Act of Union seeking to deal with economic, religious and reform issues (many Irish rotten boroughs were abolished). Wellington and Peel in 1829 succeeded in ending O’Connell’s Repeal Association and his campaigns of the 1820s by concession, with safeguards. The Whigs passed a Coercion Act in 1833, but in association with O’Connell presided over more catholic patronage, and significant reform of municipal, poor law, education and the Tithe. Peel’s government in the 1840s similarly faced down the NRA and dealt reasonably with the early stages of the Famine. In contrast post 1867 governments found land issues difficult to deal with, particularly Gladstone. The Land League and an organised Parnellite Irish Nationalist party at Westminster, noticeably embryonic or absent in the earlier period, proved especially taxing, with both a Land war in Ireland in the 1880s and obstructionism at Westminster to deal with, especially by Liberal governments that sought to avoid coercion. Home Rule, never considered by earlier governments, proved divisive politically after 1886 and was frequently damaging electorally. Ulster had not been an issue before 1886, except possibly in 1798, but emerged as an issue when Home Rule was posed as a possibility. It proved difficult for Liberal governments to deal with, especially Asquith in the 1910s. Much could also be made of government handling both of the Easter Rising and the ensuing Anglo-Irish war from 1918–1921. However candidates could equally argue that there is either continuity throughout the period 1798 to 1921 or that the contrast between pre 1867 and post 1867 should be reversed.

60 Candidates are expected to demonstrate understanding of the issues in each of their selected questions over a period of at least a hundred years (unless an individual question specifies a slightly shorter period.) Candidates are reminded of the synoptic nature of the Unit. Answers are required to demonstrate understanding of the processes of historical continuity, development and change across the full breadth of the period studied. Assessors must be open to alternative approaches. If in doubt, they should consult their Team Leader.

Page 466: History A - The Bicester Schoolthebicesterschool.org.uk/wp-content/uploads/2015/09/Mark... · 2015-09-28 · History A Mark Schemes for the Units January 2009 H106/H506/MS/R/09J

F966/02 Mark Scheme June 2013

14

Question Answer Marks Guidance All governments could be seen as enjoying a measure both of success and

failure, or that the balance of success lies more with the later period. Certainly some issues were solved post 1880 (land for example). In the earlier period there was much failure – rebellion, 1798–1803, that could only be put down by military force. Pitt failed to include emancipation in his Act of Union. This was only reluctantly conceded by Wellington and Peel in 1829 and the Whigs were faced with a Tithe war in the early 1830s, again only resolved by concessions and deals with O’Connell. Much that Peel attempted failed (land and higher educational reform) and Ireland contributed to his downfall. Post 1846 Russell’s Whigs conspicuously failed to deal with the Famine. Gladstone’s liberal governments of 1868–1895 attempted pacification along Peelite lines, via land and religious reform, but faced intractable problems, sometimes misdiagnosing what was required. Home Rule could be seen either as a Gladstonian gesture that merely raised expectations or a genuine attempt to deal with Irish issues that merely polarised Ireland even further. A case could be made for Salisbury and Balfour’s governments being successful in terms of land reform and the democratisation of local affairs (killing Home Rule by kindness) whilst also cracking down on law and order. Lloyd George’s Coalition of 1916–22 dealt with rebellion and civil war by partition and the abandonment of the South to civil war. Was this a success? The question as a whole is open to a wide variety of interpretations as to government success in pacification.

Page 467: History A - The Bicester Schoolthebicesterschool.org.uk/wp-content/uploads/2015/09/Mark... · 2015-09-28 · History A Mark Schemes for the Units January 2009 H106/H506/MS/R/09J

F966/02 Mark Scheme June 2013

15

Question Answer Marks Guidance 8 Answers need to be comparative in nature, considering Grattan, O’Connell, Butt,

Parnell, Redmond and Dillon. An examination of the pros and the cons of leaders like these should form the basis of an effective answer. The focus is expected to be on O’Connell, Parnell and Redmond, particularly their relative effectiveness in mobilising support and establishing clear policy and tactics for attaining the ends of constitutional nationalism – emancipation, reform of the Union and ultimately its repeal and a restoration of a Dublin parliament. In O’Connell’s case the achievement of Emancipation in the 1820s and reform within the Union in the 1830s are key areas. In the first two he achieved much, far more in practice than later leaders although, like them, Repeal and Home Rule remained elusive, arguably because, as Young Ireland argued, he was too tied into the Westminster process. Unlike Parnell, O’Connell did not see the need for economic or land reform, other than the ending of religiously based tithes. This meant an overreliance on the Catholic Church, any victory being seen as a blow to non sectarianism and the original vision of a united Ireland. Parnell was more effective on land issues, forcing Gladstone to reconsider this in the 1880s (linking Davitt’s Land League to his cause). O’Connell’s Roman Catholic Association became a model for ‘constitutional’ agitation in the 1820s but had less success in the 1840s. Like the Catholic Association in the 1820s the NRA mobilised large numbers but Peel had by then removed the Freeholders and O’Connell’s tactics in the 1840s involved mere persuasion through the hint of a violence he was not prepared to carry out. He also had less success in creating a disciplined Parliamentary party than Parnell, whose aims were less focused on cooperation at Westminster than its ‘obstruction’. He gained complete control of the Irish political process outside Ulster. However both he and O’Connell became divorced from Ireland itself, O’Connell creating division over his ‘moderate’ tactics, whilst Parnell never recovered from smears and the O’Shea Divorce case, splitting his party after 1890 as O ‘Connell had done in 1846. Nonetheless candidates could argue that O’Connell achieved more in association with the Whigs than Parnell did with Gladstone or Redmond with Campbell Bannerman and Asquith. Butt founded the Irish Nationalist party and began cooperation with the IRB, a fruitful policy. Some may consider him wrongly marginalised as a leader; Parnell gained a liberal commitment to Home Rule and substantive Land reform.

60 Candidates are expected to demonstrate understanding of the issues in each of their selected questions over a period of at least a hundred years (unless an individual question specifies a slightly shorter period.) Candidates are reminded of the synoptic nature of the Unit. Answers are required to demonstrate understanding of the processes of historical continuity, development and change across the full breadth of the period studied. Assessors must be open to alternative approaches. If in doubt, they should consult their Team Leader.

Page 468: History A - The Bicester Schoolthebicesterschool.org.uk/wp-content/uploads/2015/09/Mark... · 2015-09-28 · History A Mark Schemes for the Units January 2009 H106/H506/MS/R/09J

F966/02 Mark Scheme June 2013

16

Question Answer Marks Guidance Redmond finally secured Home Rule from 1910 but not its implementation whilst

Dillon ended up in alliance with Sinn Fein. Redmond could be accused of neglecting grass roots support in a way that O’Connell and Parnell rarely did. However O’Connell was never trusted in England or by PMs whereas Parnell and Redmond were more effective in gaining the trust of Gladstone and Asquith. All three were broken, O’Connell by imprisonment in 1843, Parnell by the O’Shea divorce case (which lost him much support) and Redmond by the Great War and the Easter Rising. Redmond made mistakes in overly committing Ireland to the prosecution of the War. All depended on the fortune of firstly Whiggery and then Liberalism and all ignored Ulster, Redmond at great cost. Most candidates will see either O’Connell or Parnell as the most effective leaders although all could be considered failures, or merely effective in different ways.

Page 469: History A - The Bicester Schoolthebicesterschool.org.uk/wp-content/uploads/2015/09/Mark... · 2015-09-28 · History A Mark Schemes for the Units January 2009 H106/H506/MS/R/09J

F966/02 Mark Scheme June 2013

17

Question Answer Marks Guidance 9 Although one element of land reform was to preserve the Protestant Ascendancy

(PA) the original land settlement of the late 17th and 18th century which underpinned the Ascendency was progressively eroded after the 1830s. The Tithe Act placed landlords in the eye of the storm instead of the Church and anti landlord sentiment increased in the middle of the century, culminating in the Land League’s tactics. Gladstone’s land reforms were resisted by the Ascendancy as weighting the landlord-tenant relationship too much in favour of the tenant through conceding the three ‘F’s. However for many the increasing unprofitability of land meant that land reforms from 1870, and especially the later reforms culminating in Wyndham’s Land Act of 1903, provided a financial safety net, enabling a tenant buy-out at subsidised rates and retirement to England. However it could be argued that the turning point economically was a failure to exploit industry and minerals like their mainland counterparts or the disaster of the Famine where a poor rate crippled some and made it difficult to adapt to a pastoral economy. Candidates will need to compare land reform with other factors determining Ascendancy decline. Politically they lost both their parliament in 1800 and many of their seats in the corrupt Irish boroughs of pre 1800. Those who transferred as MPs and Lords to Westminster lost a political base in Ireland that was vulnerable once emancipation was conceded along with parliamentary reform, especially in 1850 (restoration of the Freeholders, mostly catholic), 1867, 1884 and 1918. The rise of catholic freeholders and Catholic democracy, a replacement ascendency after 1830, was to be fatal when combined with mainland governments that were prepared to undermine the ascendency in the face of catholic or Ulster protest. One tactic was to run with the hounds, a minority in 1798 joining the United Irishmen and many more joining with constitutional nationalism under Parnell, himself from the ascendency class. However local government reform at the end of the century undermined ascendency control of the localities (the RMs) and was to be serious in the crises of the 1910s which saw terrorist attacks on the great estates. Nonetheless the Ascendancy remained powerful in the army (the Curragh mutiny), many losing their sons in the 1st World War. Religious emancipation in 1829 and Disestablishment in 1869 undermined their minority Anglican Church of Ireland, the former contributing to a gradual loss of office and patronage, although the police remained under their control until the 1890s. Another factor was the challenge of Dissenting Ulster nationalism.

60 Candidates are expected to demonstrate understanding of the issues in each of their selected questions over a period of at least a hundred years (unless an individual question specifies a slightly shorter period.) Candidates are reminded of the synoptic nature of the Unit. Answers are required to demonstrate understanding of the processes of historical continuity, development and change across the full breadth of the period studied. Assessors must be open to alternative approaches. If in doubt, they should consult their Team Leader.

Page 470: History A - The Bicester Schoolthebicesterschool.org.uk/wp-content/uploads/2015/09/Mark... · 2015-09-28 · History A Mark Schemes for the Units January 2009 H106/H506/MS/R/09J

F966/02 Mark Scheme June 2013

18

Question Answer Marks Guidance The Ascendency’s basis was in the South and it increasingly lacked influence on

Ulster after the 1870s. Ulster politicians became more influential with governments – Craig and Carson. It lost out in organisational and military terms to Ulster, especially in the 1910s. Much will depend on whether candidates see the PA’s decline as economically or politically driven. Its basis in legal privilege was vulnerable to reform on the mainland and its political defence of these would be reliant on mainland parties.

Page 471: History A - The Bicester Schoolthebicesterschool.org.uk/wp-content/uploads/2015/09/Mark... · 2015-09-28 · History A Mark Schemes for the Units January 2009 H106/H506/MS/R/09J

F966/02 Mark Scheme June 2013

19

Question Answer Marks Guidance 10 Candidates should focus on ‘the October Revolution of 1917’ and the extent to

which it led to changes in Russian government in their answers. In their answers candidates need to refer to the ways in which the October Revolution and any other chosen events changed Russian government. Changes to other aspects, for example the economy or society can only be rewarded if related to changes in government. Candidates who discuss aspects of Russian government such as reform and repression, the fate of opposition, changes in ideology, the absence of democracy, the one party state or the role of secret police are most likely to be successful. Candidates may argue either for or against the importance of the October Revolution but must do so comparatively in the context of other events. What follows is not an exclusive list, but consideration could be given to defeat in the Crimean War, the assassination of Alexander II in 1881, the 1905 Revolution, the First World War, the February Revolution of 1917, Stalin’s victory in the 1920s power struggle and his death in 1953. Examiners must not expect to find reference to all these examples in candidate answers and candidates may select other events in their answers. Candidates may argue that the October Revolution of 1917 changed Russian government more than any other in the period as it represented the triumph of Bolshevism and the establishment of communism. It was the event that crushed all possibility that a liberal democracy might emerge in Russia and transformed Russia into the Soviet Union. Candidates may argue that the end of over 300 years of Romanov rule in February 1917 was the event with the most important impact as it ended the 304 year old Romanov dynasty. Candidates may argue that the First World War was the event with the most important impact on the development of Russian government because it was the horrific impact of the war both at the front and at home that sealed the fate of both the Romanovs and, in turn, the Provisional Government. Arguably the appeal of the Bolsheviks in 1917 and the triumph of Lenin were directly related to the impact of the First World War. Candidates may argue that the 1905 Revolution changed Russian government because Bloody Sunday was a pivotal moment when Russians lost faith in their Tsar. They may argue that the main impact of the Revolution was the issue of the October Manifesto and the consequent abandonment of autocracy through elections to the Duma.

60 Candidates are expected to demonstrate understanding of the issues in each of their selected questions over a period of at least a hundred years (unless an individual question specifies a slightly shorter period.) Candidates are reminded of the synoptic nature of the Unit. Answers are required to demonstrate understanding of the processes of historical continuity, development and change across the full breadth of the period studied. Assessors must be open to alternative approaches. If in doubt, they should consult their Team Leader.

Page 472: History A - The Bicester Schoolthebicesterschool.org.uk/wp-content/uploads/2015/09/Mark... · 2015-09-28 · History A Mark Schemes for the Units January 2009 H106/H506/MS/R/09J

F966/02 Mark Scheme June 2013

20

Question Answer Marks Guidance Candidates may however argue that the Fundamental Laws and the rigging of

the elections to the 3rd and 4th Dumas suggest that little of substance really changed. Some candidates may well consider that Lenin’s death in 1924 was the event with the most important impact, perverting the true course of the Russian Revolution because Stalin succeeded Lenin. Candidates who argue this are likely to suggest that Stalin’s victory in the ensuing power struggle led Russia down a very different road than that being paved by Lenin. Other candidates may use a counter-argument based on more recent archival evidence to suggest that there was significant continuity between Lenin and Stalin and argue this. Candidates may argue that Khrushchev’s secret speech of 1956 and subsequent de-Stalinisation was the event with the most important impact on the development of Russian government though others may argue that the continuation of communism way beyond 1964 somewhat negates that view.

Page 473: History A - The Bicester Schoolthebicesterschool.org.uk/wp-content/uploads/2015/09/Mark... · 2015-09-28 · History A Mark Schemes for the Units January 2009 H106/H506/MS/R/09J

F966/02 Mark Scheme June 2013

21

Question Answer Marks Guidance 11 Candidates should focus on ‘the impact’ Russia’s rulers had on Russia’s

economy and society’. Candidates may argue either for or against Lenin but must do so comparatively in the context of other rulers. Candidates should argue both for and against the assertion in the question. Some candidates may more successfully differentiate between the impact rulers had on the economy and on society. When arguing in favour of Lenin candidates are likely to argue that his introduction of communism had the most profound impact on society of any ruler in this period. They may also argue that Lenin had a significant impact on the economy from his early decrees, to the imposition of War Communism in 1918 and the introduction of the New Economic Policy in 1921. When arguing against Lenin it is likely that candidates will consider the claims of Alexander II, Alexander III (though some candidates may struggle to differentiate between his reign and that of Nicholas II in terms of the economy as Witte’s ‘Great Spurt’ straddles both reigns), Stalin and Khrushchev. Alexander II certainly made significant changes to society through Emancipation and the consequent legal and administrative reforms. Candidates are likely to assert that he did less that impacted on the economy. Alexander III’s impact on society is likely to be seen as negative and reactionary whereas candidates are likely to credit him with the appointment of Witte as Minister of Finance and the consequent rapid industrialisation and urbanisation. Stalin, however, will be seen by many candidates as having had the greatest impact on the economy because of collectivisation and the Five Year Plans (both before and after the Second World War). Candidates may also argue that his impact on society was profound. Many may stress the negative impact of terror and the purges but others may balance this with consideration of reforms in areas such as health and education. Khrushchev is likely to be credited for impacting on society through his ‘secret speech’ and subsequent de-stalinisation and on the economy through his introduction of more consumer goods and flawed Virgin Lands Scheme. Some candidates may choose to conclude that one ruler had the most impact on society whereas another had most impact on the economy. Some candidates may choose to consider specific aspects of the economy and society in turn, assessing Lenin’s impact against that of the others in each case. These aspects could include agriculture, industry, living conditions, working conditions, education or health.

60 Candidates are expected to demonstrate understanding of the issues in each of their selected questions over a period of at least a hundred years (unless an individual question specifies a slightly shorter period.) Candidates are reminded of the synoptic nature of the Unit. Answers are required to demonstrate understanding of the processes of historical continuity, development and change across the full breadth of the period studied. Assessors must be open to alternative approaches. If in doubt, they should consult their Team Leader.

Page 474: History A - The Bicester Schoolthebicesterschool.org.uk/wp-content/uploads/2015/09/Mark... · 2015-09-28 · History A Mark Schemes for the Units January 2009 H106/H506/MS/R/09J

F966/02 Mark Scheme June 2013

22

Question Answer Marks Guidance 12 Candidates may argue either for or against Alexander III but must do so

comparatively. Candidates who adopt a comparative approach and demonstrate synthesis throughout the essay are likely to be most successful. Candidates may well see Alexander III, Lenin and Stalin as more successful at dealing with opposition than either Alexander II (who faced a growing tide of opposition and was ultimately assassinated) or Nicholas II (under whom the Romanov dynasty ended) or Khrushchev (despite his evident success in the power struggle after Stalin’s death as he was forced to retire by the Central Committee in 1964) or Prince Lvov/Kerensky (who were swept aside in 1917). Candidates may well argue that Alexander III was very successful at dealing with opposition. He came to the throne after the assassination of his father, whose reign had seen opposition spiralling out of control. His imposition of ‘the Reaction’ drove opponents underground or abroad. Candidates who choose to differentiate between dealing with opponents and dealing with the reasons for opposition may see Alexander III in a different light. They may wish to argue that the granting of concessions was a more successful way of dealing with opposition than ruthless repression. It can be argued that his imposition of ‘the reaction’ from 1881 bequeathed Nicholas II a revolution. Candidates may well compare his achievements with the failure of both his father and his son to control opposition. Amongst these latter-day Tsars he was undeniably most successful at dealing with opposition. However, candidates must also consider whether other rulers dealt with opposition more successfully than Alexander III did. Most candidates are likely to concentrate their alternate arguments on Lenin and Stalin when considering whether Alexander III was the most successful ruler at dealing with opposition. Some candidates may well argue that Lenin was even more successful because he cut a swath through the other parties that aspired to power in 1917 and successfully defended his revolution during the Civil War. He created the world’s first communist state and died with his party securely in power. Other parties were all banned, as were factions within the Communist Party. Stalin defeated all of his rivals during the power struggle with consummate skill and exterminated real and imagined opponents with bloodcurdling efficiency for the next 25 years and his chilling terror may well lead candidates to argue that he, rather than Alexander III, was the most successful ruler at dealing with opposition.

60 Candidates are expected to demonstrate understanding of the issues in each of their selected questions over a period of at least a hundred years (unless an individual question specifies a slightly shorter period.) Candidates are reminded of the synoptic nature of the Unit. Answers are required to demonstrate understanding of the processes of historical continuity, development and change across the full breadth of the period studied. Assessors must be open to alternative approaches. If in doubt, they should consult their Team Leader.

Page 475: History A - The Bicester Schoolthebicesterschool.org.uk/wp-content/uploads/2015/09/Mark... · 2015-09-28 · History A Mark Schemes for the Units January 2009 H106/H506/MS/R/09J

F966/02 Mark Scheme June 2013

23

Question Answer Marks Guidance 13 Candidates who agree with this proposition may consider a range of supporting

evidence. Some candidates may discuss the use of intimidation and terrorism activities by the Ku Klux Klan and the Knights of the White Camelia (a group similar to, and operating at the time of, the KKK), the incidence of lynching (especially in the Reconstruction era and c1890 until c1940). There may also be discussion of the hostility of the Supreme Court (Plessy v Ferguson) and the use of state law to entrench segregation and the denial of civil rights to African Americans in the South following the end of Reconstruction in 1877. There might also be consideration of the Social Darwinian intellectual underpinnings of white racial prejudice, especially in the period from the late 19th century until World War Two. Responses might also discuss the strength and determination of Southern whites to resist the civil rights revolution of the 1950s and 1960s, including the White Citizens’ Councils created in the wake of the 1954 Brown decision by the US Supreme Court and the violence meted out to civil rights campaigners. Some may consider the relatively small numbers of AA politicians at local, state and federal level for much of the period. Other candidates might discuss the continued de facto discrimination in housing, employment, political representation and social integration, and controversies over bussing and affirmative action since the 1960s. There may also be consideration of the Federal Government which was less supportive after 1968, eg Nixon, Reagan, some adverse Supreme Court decisions (1974: Milliken v Bradley, 1978: Bakke v University of California, 1984: Grove City v Bell, 1992: Freeman v Pitts); Conservative appointments to SC.

Candidates should also weigh up the counter argument. They might analyse the support of the Federal Government for civil rights, especially during the Reconstruction period and after the Second World War: examples might include Supreme Court judgements (especially Brown, 1954), the various civil rights acts and the Voting Rights Act, supportive presidents (eg Johnson, Carter). There might also be consideration of the degree of white support for civil rights, especially in the early 1960s, the decline of the acceptability of overt racism, international pressure on the USA to change – the impact of the Cold War, decolonisation and the UN. Some candidates might discuss the greater profile of African Americans in US society in politics and business, the media, music

60 Candidates are expected to demonstrate understanding of the issues in each of their selected questions over a period of at least a hundred years (unless an individual question specifies a slightly shorter period.) Candidates are reminded of the synoptic nature of the Unit. Answers are required to demonstrate understanding of the processes of historical continuity, development and change across the full breadth of the period studied. Assessors must be open to alternative approaches. If in doubt, they should consult their Team Leader.

Page 476: History A - The Bicester Schoolthebicesterschool.org.uk/wp-content/uploads/2015/09/Mark... · 2015-09-28 · History A Mark Schemes for the Units January 2009 H106/H506/MS/R/09J

F966/02 Mark Scheme June 2013

24

Question Answer Marks Guidance and sport, eg Jesse Jackson’s bids for Democratic presidential nominations in

1984 and 1988. Strong answers may also point out that the strength of opposition to civil rights has varied over time, contrasting the periods of progress (Reconstruction and c1954 to c1968) with the period of indifference and hostility (1877 to Second World War) and the contested nature of civil rights since 1968, eg Reagan hostile but Democrat-dominated Congress largely supportive.

Page 477: History A - The Bicester Schoolthebicesterschool.org.uk/wp-content/uploads/2015/09/Mark... · 2015-09-28 · History A Mark Schemes for the Units January 2009 H106/H506/MS/R/09J

F966/02 Mark Scheme June 2013

25

Question Answer Marks Guidance 14 In considering the 1890s, candidates may be aware of a number of negative

developments, such as the background of economic depression following the Panic of 1893, the use of the 1890 Sherman Anti-Trust Act to gain Federal injunctions against strikers, Supreme Court rulings that unions were 'illegal combinations', the use of troops to break strikes, the Homestead steel strike 1892, the failure of Coxey’s ‘Army’ in 1894 to lobby for a Federal programme of public works to provide employment, the violence of the Pullman Strike in 1894, and the continuing divide between white and black workers, between skilled, craft workers and the unskilled – divisions worsened by the growth of immigration from eastern and southern Europe. Some candidates may argue that the New Deal was more important in the development of union and labour rights, and consider some of the following: the alphabet agencies aimed to get the unemployed back to work, trade unions and workers gained the support of the Federal government for the first time, NIRA of 1933 (Clause 7(a) granted workers the right to join labour unions, and obliged employers to recognize unions and bargain with them), the Wagner Act of 1935 (which gave workers the right to join a union and to bargain collectively and created the National Labor Relations Board) and the Fair Labor Standards Act of 1938 (which established a minimum wage). They may also consider the fact that union membership tripled between 1933 and 1939 (perhaps mentioning the creation of the CIO in 1935). There may be consideration of FDR and the fact that most state officials refused to intervene in strikes on the side of manage-ment. Some may point out that the New Deal gains were sustained during WW2 when an unprecedented expansion of American industry to meet the demands of war production gave considerable bargaining power to workers. Better answers may be aware that attitudes to workers’ rights were not completely transformed by the New Deal – the NIRA was declared unconstitutional in 1935, there were serious, and sometimes, violent industrial disputes in 1934 and 1937 and some major employers (such as Ford) resisted recognising unions until the war. Candidates may also consider a variety of other turning points. There may be discussion of the immediate post-war period the New Deal gains were, to some degree, clawed back by Congress – the 1947 Taft-Hartley Act (passed over

60 Candidates are expected to demonstrate understanding of the issues in each of their selected questions over a period of at least a hundred years (unless an individual question specifies a slightly shorter period.) Candidates are reminded of the synoptic nature of the Unit. Answers are required to demonstrate understanding of the processes of historical continuity, development and change across the full breadth of the period studied. Assessors must be open to alternative approaches. If in doubt, they should consult their Team Leader.

Page 478: History A - The Bicester Schoolthebicesterschool.org.uk/wp-content/uploads/2015/09/Mark... · 2015-09-28 · History A Mark Schemes for the Units January 2009 H106/H506/MS/R/09J

F966/02 Mark Scheme June 2013

26

Question Answer Marks Guidance Truman’s veto) allowed states to pass laws guaranteeing people’s right to work (ie workers could ignore their union if it ordered them to go on strike) and banned the ‘closed shop’; the 1959 Landrum-Griffin Act banned secondary picketing. Some may argue that despite the AFL-CIO merger in 1955, structural changes in the post-war US economy (the decline of heavy industry and blue-collar work) gradually eroded union power. Others might suggest that the New Frontier and Great Society programmes of the 1960s might be offered as a positive turning point for workers. Some might argue that defeat of the PATCO strike in 1981 can be seen as a major negative turning point for unions leading to a decline in membership and in the number of strikes and signalling the hostility of the Federal government to organised labour. Better candidates may place this in the context of changes in the US economy with the growth of the service sector and non-unionised, low-paid, part-time and immigrant labour.

Page 479: History A - The Bicester Schoolthebicesterschool.org.uk/wp-content/uploads/2015/09/Mark... · 2015-09-28 · History A Mark Schemes for the Units January 2009 H106/H506/MS/R/09J

F966/02 Mark Scheme June 2013

27

Question Answer Marks Guidance 15 Arguably, women were very little united. Throughout the period they have been

divided on whether they wanted equality with men or to have their differences properly acknowledged. The notion that men and women occupied ‘separate spheres’ entered political discourse in the late 19th century and influenced activists campaigning for temperance and social reform in the Progressive Era. In the 20th century, campaigners wanting to stop the Equal Rights Amendment stressed gender differences. Radical feminists of the late 20th century campaigning against pornography, sexual harassment and for better protection against rape and domestic violence also wanted differences recognised. But equality and the removal of discrimination (over issues such as property rights, divorce, contraception, wages, access to education, the professions and politics) have been the goals of suffrage reform campaigners of the Progressive Era, the ERA, the National Organisation of Women (formed in 1966) and abortion reform. Better candidates will be aware that these campaigns, and campaigners, were not always mutually exclusive and sometimes overlapped. The aims of campaigners were not always united. In the late 19th century and early 20th century the Women’s Christian Temperance Union and other campaigners pressed for a wide range of social reforms whereas suffrage groups wanted the vote. There were also differences over tactics between the various suffrage campaigners: the National Women’s Suffrage Association (founded in 1869 and led by Susan B. Anthony and Elizabeth Cady Stanton) wanted a constitutional amendment, adopted a confrontational approach and campaigned for a wide range of women’s rights, as did the 1913 Congressional Union (which became the National Women’s Party during WW1). The American Women’s Suffrage Association (founded in 1869 and led by Lucy Stone) was prepared to work with men and worked at state and local level, with links to the Republican Party. Although the NWSA and the AWSA merged (as the NAWSA) in 1890, they continued to be divided over whether to campaign at national or local level. Three of the most divisive factors throughout the period have been class, race and religion. Most pressure groups and campaigns have been founded and led by educated, middle class women whose concerns have not much interested

60 Candidates are expected to demonstrate understanding of the issues in each of their selected questions over a period of at least a hundred years (unless an individual question specifies a slightly shorter period.) Candidates are reminded of the synoptic nature of the Unit. Answers are required to demonstrate understanding of the processes of historical continuity, development and change across the full breadth of the period studied. Assessors must be open to alternative approaches. If in doubt, they should consult their Team Leader.

Page 480: History A - The Bicester Schoolthebicesterschool.org.uk/wp-content/uploads/2015/09/Mark... · 2015-09-28 · History A Mark Schemes for the Units January 2009 H106/H506/MS/R/09J

F966/02 Mark Scheme June 2013

28

Question Answer Marks Guidance poor women in low-paid and/or part-time work. Racial issues were particularly divisive in the early part of the period as black women and immigrants were excluded from suffrage groups. White women were not much interested in black anti-lynching campaigns and there were many white women members of the revived KKK. Religion was also a divisive issue. Progressive Era campaigners were usually protestant evangelicals whereas urban immigrant women were often Catholic, socially conservative and less interested in female enfranchisement. The campaign against the ERA led by Phyllis Schlafly from 1972 and the opposition to the 1973 Roe versus Wade judgement on abortion were strongly influenced by religion, though in these cases protestant evangelicals were on the same side as Catholics. Candidates may also wish to argue that the degree of support that women were able to mobilise for their campaigns, such as franchise reform, the ERA and the changes of the modern era suggests that, despite tactical differences, there was considerable unity in favour of change.

Page 481: History A - The Bicester Schoolthebicesterschool.org.uk/wp-content/uploads/2015/09/Mark... · 2015-09-28 · History A Mark Schemes for the Units January 2009 H106/H506/MS/R/09J

F966/02 Mark Scheme June 2013

29

Question Answer Marks Guidance 16 The argument that the mass media were the most important factor is that

governments noted its impact and sought to exploit it politically or moved to include groups affected by it across the period. The emergence of a powerful provincial middle class press in the 1860s, especially nonconformist and artisan, aided the Liberals at the beginning of the period and Gladstone exploited this in his Midlothian campaign. The new populist press of the 1890s aided Balfour and Lord Salisbury, despite their scathing view of its readership, and helped them use imperialism to gain votes (Khaki election during the Boer War). The Mass media had a negative effect on Labour politics and indeed remained its enemy, with the exception of the Daily Herald and later the Daily Mirror, until the 1990s. It could be argued here that the media were not the most important factor in the fortunes of the Labour party. Nonetheless the Guardian moved from Liberalism at the turn of the century to the Labour party after 1945, becoming almost its academic journal. The rise of the tabloid press in the 1960s and its attitude was widely held by governments to be crucial during elections, the ‘Sun’ in particular. Press barons were considered important whether they be Harmsworth, Beaverbrook or Murdoch. Examples could be cited of how it helped the Conservative party. The BBC helped Baldwin’s Conservative government in 1926 when Labour and the Archbishop of Canterbury were refused the chance to broadcast. Candidates could examine some elections across the period to illustrate the importance of the media – 1874; 1880; 1906; 1924 (Campbell Case and the Zinoviev telegram adversely affecting Labour), 1945; 1964; 1974;1979 (Labour isn’t working);1983 etc. Candidates will need to balance their answers with reference to other factors which could be seen as more important – the franchise, polarising policies, party organisation; governmental legislative records; the state of the economy, leadership etc.

60 Candidates are expected to demonstrate understanding of the issues in each of their selected questions over a period of at least a hundred years (unless an individual question specifies a slightly shorter period.) Candidates are reminded of the synoptic nature of the Unit. Answers are required to demonstrate understanding of the processes of historical continuity, development and change across the full breadth of the period studied. Assessors must be open to alternative approaches. If in doubt, they should consult their Team Leader.

Page 482: History A - The Bicester Schoolthebicesterschool.org.uk/wp-content/uploads/2015/09/Mark... · 2015-09-28 · History A Mark Schemes for the Units January 2009 H106/H506/MS/R/09J

F966/02 Mark Scheme June 2013

30

Question Answer Marks Guidance 17 Candidates will need to address the issue of ‘steady decline’. Religious groups

were important at the beginning of the periods but was the process a gradual one from the 1880s onward or was there a turning point coming either in the 1900–1914 period, the war periods (1914–18; 1939–45), the Inter War period or in the 1950s and 1960s? The religious groups – the Anglican Church, Nonconformists (and particular groups within – Methodists, Quakers, and Salvation Army etc) and Catholics – were very active in the period to 1945, either pushing for recognition or resisting the erosion of privilege. Arguably religious affiliation determined political affiliation (Anglicans were Conservatives; Nonconformists were Liberals or later Labour). Nonconformity pushed for civic and civil equality but also a wider political and social agenda. They championed Temperance reform, charity, international forums (later the League of Nations) and used the Liberal party as their vehicle. Religious groups also has a considerable impact on political methods – meetings, campaigns and political oratory were copied from preaching and dissenting practice and through this many gained a ‘political’ experience up to the 1950s. Liberalism and Labour gained much from this but Conservatism gained from the moral tone set by the Church of England (on issues like the Abdication). After 1918 however the Anglican Church and individuals within it could take a non conservative line – on the General Strike and on the Great Depression (the Red Dean). Candidates will need to examine when and why such influence declined – was it sudden, a product of the Great War or the Great Depression or the materialism of the 1950s and 1960s? Religious groups were very prominent in CND in the 1950s, revivalist meetings still had an impact in that decade and Mary Whitehouse’s media campaigns mobilised much middle opinion well into the 1980s. The Churches could still command attention at the end of the period on a range of moral issues for example on the Falklands War or was it a steady decline from the 1880s, demonstrated in falling attendance figures at Church in favour of newly created leisure?

60 Candidates are expected to demonstrate understanding of the issues in each of their selected questions over a period of at least a hundred years (unless an individual question specifies a slightly shorter period.) Candidates are reminded of the synoptic nature of the Unit. Answers are required to demonstrate understanding of the processes of historical continuity, development and change across the full breadth of the period studied. Assessors must be open to alternative approaches. If in doubt, they should consult their Team Leader.

Page 483: History A - The Bicester Schoolthebicesterschool.org.uk/wp-content/uploads/2015/09/Mark... · 2015-09-28 · History A Mark Schemes for the Units January 2009 H106/H506/MS/R/09J

F966/02 Mark Scheme June 2013

31

Question Answer Marks Guidance 18 A variety of views are possible on this issue. Candidates may well argue that

before 1918 governments were resistant or doubted many aspects of mass democracy such as extension of the franchise to the residuum and women, the Secret Ballot at the beginning of the period, effective redistribution before 1885, electoral reform, reform of the Lord’s Veto and to wider issues like education and the intrusion of the mass media. Conservative governments especially opposed many of the above developments before 1918 and both parties sought to restrict the development of a separate Labour party after 1900 (Liberal Trades Disputes Act; opting in on the political levy; the Campbell Case in 1924). Even after 1918 there was resistance to female equality, either electorally (to 1928) or socially (to the 1970s and beyond). In the work place there was little legislation before the 1970s. Mass education, at a secondary level, had to wait until 1944 whilst the Conservatives opposed the idea of a welfare state in 1945. There is more evidence that democracy was helped after 1918 – growing acceptance of the Labour party and movement from 1919 and especially during the 2nd World War; creation of the BBC and in the 1950s ITV. However it is perfectly possible to argue that, contrary to the assertion in the question, governments were willing to pass democratic reform throughout the period and were indeed a major factor in helping democracy emerge. A reference to the 1870s and 1880s would demonstrate this (parliamentary reform via the Ballot Act and the Redistribution and Corrupt Practices Act were willingly embraced by most Liberals, although Gladstone was reluctant on the Ballot in 1872; Local government reform were embraced by both Conservatives and Liberals in the 1880s). There is an argument that democracy was hastened by political competition between the parties, as the Conservatives extended the vote in 1867 to capture the upper working class, the Liberals extended it in 1884 to capture the rural vote and in 1918 and after all parties hoped to take advantage of a democratic vote that included women.

60 Candidates are expected to demonstrate understanding of the issues in each of their selected questions over a period of at least a hundred years (unless an individual question specifies a slightly shorter period.) Candidates are reminded of the synoptic nature of the Unit. Answers are required to demonstrate understanding of the processes of historical continuity, development and change across the full breadth of the period studied. Assessors must be open to alternative approaches. If in doubt, they should consult their Team Leader.

Page 484: History A - The Bicester Schoolthebicesterschool.org.uk/wp-content/uploads/2015/09/Mark... · 2015-09-28 · History A Mark Schemes for the Units January 2009 H106/H506/MS/R/09J

Oxford Cambridge and RSA Examinations is a Company Limited by Guarantee Registered in England Registered Office; 1 Hills Road, Cambridge, CB1 2EU Registered Company Number: 3484466 OCR is an exempt Charity OCR (Oxford Cambridge and RSA Examinations) Head office Telephone: 01223 552552 Facsimile: 01223 552553 © OCR 2013

OCR (Oxford Cambridge and RSA Examinations) 1 Hills Road Cambridge CB1 2EU OCR Customer Contact Centre Education and Learning Telephone: 01223 553998 Facsimile: 01223 552627 Email: [email protected] www.ocr.org.uk For staff training purposes and as part of our quality assurance programme your call may be recorded or monitored